Nursing Fundamentals for NCLEX

Ace your homework & exams now with Quizwiz!

What position do you place the pt in after surgery?

low Fowler's position - 15 to 30 deg (increase the size of the thorax for lung expansion)

What do you do for a hemolytic transfusion reaction?

stop the infusion, infuse NS, obtain urine specimen, monitor for respiratory probs, benadryl, airway management

after cardiac catheterization, the extremity into which the catheter was inserted is kept______ for ___to__hours. The head is elevated no more than ___ degrees.

straight; 4 to 6 hours; 30 degrees

What breath sounds indicate partial obstruction, bronchospasm or laryngospasm?

stridor, wheezing, crowing sound

For the first 24 hours a pt with a right above knee amputation of the right leg is placed in what position?

supine with amputated leg supported on pillows to prevent venous return & decrease edema

What is a priority for autonomic dysreflexia?

sx: profuse HA and sweating in pt with SCI positioning: place pt in sitting position to dec r/f cerebral hem & dec BP

What are possible s/sx of shock?

tachycardia, inc RR, clammy, lightheaded

valsalva maneuver

take a deep breath, exhale, and bear down used when removing chest tubes

type of prevention crisis: reduction of the amount & degree of disability, injury, & damage after a crisis

tertiary prevention

During second stage of labor, what is the priority?

to assess the perineum to see if the baby is crowning...the head will need support and put hand on abd to help control delivery

a nurse is to remove a Nasogastric tube, why does the nurse instruct the pt to take and hold a deep breathe during removal?

to close the epiglottis and allow easy removal through the esophagus into the nose

What is an imp goal for a pt dx w/anorexia?

to gain self-worth and self-acceptance not based on appearance ex: making jewelry w/Occupational therapist which will promote socialization and self acceptance

What should you do if the pt is unable to get out of bed?

turn every 1 to 2 hrs

Site selection for a 6 month old infant

tympanic

Ulcerative Colitis

ulcerative and inflammatory disease of the bowel that results in poor absorption of nutrients, commonly begins in rectum and spreads upward severe diarrhea that may contain blood and mucus

Where is the fundus at 20-22 weeks?

umbilicus

When does an infant feed best?

upon wakening b/c is well rested enlarge nipple for ease feed q3h

A vomiting 18 month old child should be placed?

upright or side lying position to prevent aspiration.

urineanalysis & cystoscopy

urineanalysis contains wbc & rbc and cystoscopy diagnosis cystitis

Anthrax can be contacted through?

the digestive system or abrasions in the skin; inhaled through the lungs

When does atelectasis usually occur after surgery?

1 to 2 days

What is the immediate postop stage?

1 to 4 hours after surgery

Bedpan verses on toilet:

Sitting position increase pressure in abdomen

How often to do you check resp rate & pain after opioid admin?

every 30 min

IV site dressings should be changed______

every 48 to 72 hours or every 2 to 3 days.

Painful sexual intercourse is often a result of decreased levels of which of the following hormones?

Estrogen

Post menopausal, infection, why is this happening now, nurse says:

Estrogen levels, more susceptible

Assessment: Discharge of serosangineous fluid from a previously dry wound. Loops of bowel or other abdo contents through wound. Popping sensation after coughing or turning.

Evisceration

Delayed transfusion signs

Fever, mild jaundice, decreased hematocrit level

In a young child, ages 3 to 6 years, the maximum volume of medication that can be safely injected into the ventral gluteal muscle is?

1.5 mL

Which of the following clients is experiencing an abnormal change in vital signs? A client whose (select all that apply): 1) blood pressure (BP) was 132/80 mm Hg sitting and is 120/60 mm Hg upon standing 2) rectal temperature is 97.9F in the morning and 99.2F in the evening 3) heart rate was 76 before eating and is 60 after eating 4) respiratory rate was 14 when standing and is 22 after walking

1) BP was 132/80 mm Hg sitting and is 120/60 mm Hg upon standing 2) heart rate was 76 before eating and is 60 after eating

Common Preoperative Medications:

1) Benzodiazepines and barbiturates: for sedation and amnesia 2) Anticholinergics: to reduce secretions 3) Opioids: to decrease intraoperative anesthetic requirements and pain 4) Additional drugs include antiemetics, antibiotics, eye drops, and regular prescription drugs

1) difficulty breathing 2) pulse rate less than 60 3) effects both pulse and blood pressure 4) abscence of respirations 5) amount of air moving in and out with each breath 6) body's thermostat 7) blood pressure greater than 120/80 8) pulse rate greater than 100

1) dyspnea 2) bradycardia 3) ANS 4) apnea 5) tidal volume 6) hypothalamus 7) hypertension 8) tachycardia

Determine whether each of the following factors increase or decrease the pulse rate. 1) hyperthyroidism 2) calcium channel blockers 3) hypothermia 4) acute pain 5) blood loss 6) anxiety 7) walking on the treadmill

1) increases 2) decreases 3) decreases 4) increases 5) increases 6) increases 7) increases

An 82 year old man arrives at the emergency department with an oral body temperature of 101F, a pulse rate of 114/min, and respiratory rate of 22/min. He is restless and his skin is warm to the touch. Which of the following are appropriate nursing interventions for this client? (Select all that apply) 1) obtain culture specimens before initiating prescribed anti-microbials 2) restrict fluids 3) allow for adequate rest 4) provide oral care 5) only change bed linens when the client requests it 6) apply an additional blanket if the client feels chilled

1) obtain culture specimens before initiating prescribed anti-microbials 3) allow for adequate rest 4) provide oral care 6) apply an additional blanket of the client feels chilled

Respiratory acidosis is at highest risk for a patient with:

COPD

What does progressive systematic sclerosis cause?

CT dz that causes dysphagia & esophageal reflux b/c of dec motility

Which of the following nursing actions takes priority when working with a patient who has a problem of sexual functioning?

Clarification of the nurse's personal values.

Yellow:

Clean

Home health nurse teacher patient about straight catheter, effective statement by patient:

Clean catheter before and after each use

Serous:

Clear

Serous Sanguineous:

Clear and blood tinged

Diagnosis of diarrhea, day of admission, diet to be ordered:

Clear liquids with Gatorade

Actions by client effective in teaching:

Clients walks alot

foods rich in calcium

Clients with low calcium levels should be encouraged to consume dairy products, seafood, nuts, broccoli, and spinach. Which are all good sources of dietary calcium.

What are two tools that support case management?

Clinical pathways and disease management rationale: Both of these tools support case management by coordination and delivery of high-quality care. Clinical pathways are interdisciplinary plans of care that outline the optimal sequencing and timing of interventions for clients with a particular diagnosis, procedure, or symptom.

Alteplase

Clot buster Alteplase is a clot buster. With patient who has experienced hemorrhagic stroke, there is already bleeding into the brain. A drug like alteplase can worsen the bleeding.

319 Which of following lab results w/verify diagnosis of bacterial meningitis?

Cloudy cerebrospinal fluid w/high protein and low glucose levels. R: A diagnosis of meningitis is made by testing cerebrospinal fluid obtained by lumbar puncture. In bacterial meningitis, findings usually include increased pressure, cloudy CSF, a high protein level, and a low glucose level.

In addition to cyanotic lips and nail beds, nasal flaring, and pursed lips, what additional sign would indicate that a patient is suffering from cardiac or pulmonary difficulty?

Clubbing of fingers and toes.

The Joint Commission publishes a Sentinel event alert every month. Which of the following is the best example of a sentinel event?

Code Pink is called after a newborn is discovered missing from the nursery. rationale: A sentinel event is an unexpected occurrence involving death or loss of limb or function. Examples of sentinel events include serious medication errors, significant drug reactions, surgery performed on the wrong body site, blood transfusion reactions, and infant abductions.

Which of the following is the best position for a male patient during a breast examination?

Sitting with arms hanging at sides.

What is homonymous hemianopsia?

loss of half of visual field; can happen in stroke pt

Galactorrhea in a nonpregnent patient is most commonly caused by?

Presence of a tumor

A patient who has dull, dry, and thin hair is most likely to have a nutritional deficit in which of the following?

Protein

What are the best indicators of pain relief?

non-verbal ques

A patient is admitted with dysentery resulting from Clostridium difficile, or pseudomembranous colitis. To obtain the most helpful information about the cause of the dysentary, a nurse should ask the patient:

"Are you taking any antibiotics?"

Which of the following statements would indicate that a patient taking anti-hypertensive drugs understands the management of hypertension?

"These pills will help control, but not cure, my high blood pressure."

Site selection for an 82 year old man with confusion

oral, tympanic, or axillary

A patient on initial assessment reports "a cough for 3 days and it's getting worst." The nurse says, "Tell me more about your cough." The patient says, "I wish I could, but that is why I'm here. You tell me what's wrong!" Which of these responses would be most appropriate for enhancing communication?

"After 3 days, you're tired of coughing. Have you had a fever?"

Which of the following is an optional element in a measurable outcome?

"Conditions" - only subject, verb, performance criteria, and target time are mandatory in a measurable outcome.

A 82-year-old man tells a nurse that he is having difficulty hearing and that he has "too much ear wax." Because of the patient's age, the nurse should most appropriately ask:

"Did you ever experience impacted ear wax?"

When a patient asks a nurse about the use of therapeutic herbs, which of the following is the most appropriate response?

"Herbs are not regulated and some herbs can have health risks, especially if used with prescription drugs."

When the initial nursing assessment revealed that the patient had not had a bowel movement for 2 days, the student wrote the diagnostic label "constipation." Which of the following comments is she most likely to hear from her instructor?

"Hold on a minute...nursing diagnoses should always be derived from clusters of significant data rather than from a single cue."

When administering a mental status examination, a nurse should suspect depression when the patient responds:

"I don't know."

The nurse evaluates that teaching for the patient with iron deficiency anemia has been effective when the patient states:

"I should take the iron for several months after my blood is normal."

A nurse should realize that additional teaching is needed when a patient with an ileal conduit says:

"I won;t drink too much so the amount of urine is less."

A patient with cancer is experiencing increased pain issues. A plan is developed for adding ibuprofen 600mg BID to the medication regimen of narcotics. The patient asks the nurse why he is now expected to take ibuprofen because he does not have arthritis. What is the most appropriate reply by the nurse?

"Ibuprofen increases the effects of your narcotic, providing better pain relief" Nonsteroidal anti-inflammatory drugs potentiate the effects of opiates and, when in combination, are of particular use in cancer patients because of major contributing factor of pain is cell destruction. Narcotic doses may still need to be increased as the disease is progressive. NSAIDS have an anti-inflammatory effect, but the ability to block prostaglandin synthesis promotes their pain-relieving properties. There is no information to support that they act more slowly or extend pain relief.

A patient responds to the clinic with severe groin pain and a history of kidney stones. The patient's son tells the nurse that, for religious reasons, his father wishes to keep any stone that is passed into the urine filter. The nurse's most appropriate response would be:

"It would be best is we could examine any passed material and then return it to your father."

If a patient who has undergone a transurethral resection tells a nurse that immediately following his discharge his wife is going to drive him to his daughter's house 3 hours away, which of the following is a nurse's most appropriate response?

"Long drives should be avoided for at least 2 weeks. Can an alternative plan be arranged?"

If a 20-year-old patient calls to report that she has found a lump in her breast, what is the nurse's best response?

"Many women have benign lumps and bumps in their breasts. But to be sure, you should come in for an examination with your physician."

What instructions regarding bladder training should be included in the teaching plan for the family of a patient who is incontinent because of a stroke?

"Offer the patient the commode or urinal every 2 hours."

If a nurse is planning discharge instructions for a patient diagnosed as human immunodeficiency virus (HIV) positive, which of the following statements made by the patient would indicate effective teaching?

"The virus can be spread to another person by contact with body fluids."

When assessing chronic pain in the older adult, which question will be most helpful in determining appropriate interventions?

"What treatments have you used and which have been most helful?" Chronic pain may begin insidiously and many remedies may have been tried before seeking treatment.

Which of the following statements made by a patient undergoing drug therapy for hypertension indicates a need for further clarification?

"When my blood pressure becomes normal, I no longer need to take medication."

Three days after undergoing exploratory laparotomy and lysis of adhesions, a patient tells the nurse that his pain is no better than on the first day postop and he fears that he will be unable to return to his work withing the allotted time frame. Which response by the nurse is the most appropriate for the situation?

"You have undergone a major surgery, which is a major stressor to your body. As your body heals, your pain should resolve" Acute pain occurs after surgury and is usually limited and of predictable duration. Increased activity is needed to maintain function, promote healing, and prevent complications of surgery.

Although reported in small numbers, toxic shock syndrome (TSS) can occur with the use of the contraceptive diaphragm. The nurse should instruct the female patient about ways to reduce her risk of TSS. This includes:

"You should always remove your diaphragm 6 to 8 hours after intercourse; do not use the diaphragm during menses; and watch for danger signs of TSS, including a sudden-onset fever of greater than 100 degrees F."

What do you do just before starting surgery?

"time out" make sure oyu have the right patient, right surgical sight

A fourteen year old male has just been admitted to your floor. He has a history of central abdominal pain that has moved to the right iliac fossa region. He also has tenderness over the region and a fever. Which of the following would you most likely suspect? A: Appendicitis B: Acute pancreatitis C: Ulcerative colitis D: Cholecystitis

(A) Appendicitis is most likely indicated in this case.

A nurse if reviewing a patient's chart and notices that the patient suffers from Lyme disease. Which of the following microorganisms is related to this condition? A: Borrelia burgdorferi B: Streptococcus pyrogens C: Bacilus anthracis D: Enterococcus faecalis

(A) Choice B is linked to Rheumatic fever, Choice C is linked to Anthrax, Choice D is linked to Endocarditis.

You are responsible for reviewing the nursing unit's refrigerator. If you found the following drug in the refrigerator it should be removed from the refrigerator's contents? A: Corgard B: Humulin (injection) C: Urokinase D: Epogen (injection)

(A) Corgard could be removed from the refigerator.

Which of the following normal blood therapeutic concentrations is abnormal? A: Phenytoin 10 - 20 mcg/ml B: Quinidine 02 - 06 mcg/ml C: Haloperidol 05 - 20 ng/ml D: Carbamazepine 5 - 25 mcg/ml

(C) The normal ranges of Carbamazepine is 10 - 20 mcg/ml.

When you are taking a patient's history, she tells you she has been depressed and is dealing with an anxiety disorder. Which of the following medications would the patient most likely be taking? A: Elavil B: Calcitonin C: Pergolide D: Verapamil

(A) Elavil is a tricyclic antidepressant.

A patient asks a nurse, "My doctor recommended I increase my intake of folic acid. What type of foods contain folic acids?" A: Green vegetables and liver B: Yellow vegetables and red meat C: Carrots D: Milk

(A) Green vegetables and liver are a great source of folic acid.

A nurse is caring for an adult that has recently been diagnosed with renal failure. Which of the following clinical signs would most likely not be present? A: Hypotension B: Heart failure C: Dizziness D: Memory loss

(A) Hypertension is often related renal failure.

A patient has recently been diagnosed with hypernatremia. Which of the following is not associated with hypernatremia? A: Hypotension B: Tachycardia C: Pitting edema D: Weight gain

(A) Hypotension would be associated with hyponatremia.

A patient's chart indicates a history of meningitis. Which of the following would you not expect to see with this patient if this condition were acute? A: Increased appetite B: Vomiting C: Fever D: Poor tolerance of light

(A) Loss of appetite would be expected.

A nurse is educating a patient about right-sided heart deficits. Which of the following clinical signs is not associated with right-sided heart deficits? A: Orthopnea B: Dependent edema C: Ascites D: Nocturia

(A) Orthopnea is a left- sided heart failure clinical symptom.

A patient has recently experienced a (MI) within the last 4 hours. Which of the following medications would most like be administered? A: Streptokinase B: Atropine C: Acetaminophen D: Coumadin

(A) Streptokinase is a clot busting drug and the best choice in this situation.

A toddler is 16 months old and has been recently admitted into the hospital. According to Erickson which of the following stages is the toddler in? A: Trust vs. mistrust B: Initiative vs. guilt C: Autonomy vs. shame D: Intimacy vs. isolation

(A) Trust vs. Mistrust- 12-18 months old

A 28 year old male has been found wandering around in a confused pattern. The male is sweaty and pale. Which of the following tests is most likely to be performed first? A: Blood sugar check B: CT scan C: Blood cultures D: Arterial blood gases

(A) With a history of diabetes, the first response should be to check blood sugar levels.

A nurse is reviewing a patient's medication during shift change. Which of the following medication would be contraindicated if the patient were pregnant? Note: More than one answer may be correct. A: Coumadin B: Finasteride C: Celebrex D: Catapress E: Habitrol F: Clofazimine

(A) and (B) are both contraindicated with pregnancy.

A second year nursing student has just suffered a needlestick while working with a patient that is positive for AIDS. Which of the following is the most important action that nursing student should take? A: Immediately see a social worker B: Start prophylactic AZT treatment C: Start prophylactic Pentamide treatment D: Seek counseling

(B) AZT treatment is the most critical innervention.

You are taking the history of a 14 year old girl who has a (BMI) of 18. The girl reports inability to eat, induced vomiting and severe constipation. Which of the following would you most likely suspect? A: Multiple sclerosis B: Anorexia nervosa C: Bulimia D: Systemic sclerosis

(B) All of the clinical signs and systems point to a condition of anorexia nervosa.

A 65 year old man has been admitted to the hospital for spinal stenosis surgery. When does the discharge training and planning begin for this patient? A: Following surgery B: Upon admit C: Within 48 hours of discharge D: Preoperative discussion

(B) Discharge education begins upon admit.

A nurse is caring for an adult that has recently been diagnosed with metabolic acidosis. Which of the following clinical signs would most likely not be present? A: Weakness B: Dysrhythmias C: Dry skin D: Malaise

(B) Dysrhythmias are associated with metabolic alkalosis.

A nurse is making rounds taking vital signs. Which of the following vital signs is abnormal? A: 11 year old male - 90 b.p.m, 22 resp/min. , 100/70 mm Hg B: 13 year old female - 105 b.p.m., 22 resp/min., 105/60 mm Hg C: 5 year old male- 102 b.p.m, 24 resp/min., 90/65 mm Hg D: 6 year old female- 100 b.p.m., 26 resp/min., 90/70mm Hg

(B) HR and Respirations are slightly increased. BP is down.

A 24 year old female is admitted to the ER for confusion. This patient has a history of a myeloma diagnosis, constipation, intense abdominal pain, and polyuria. Which of the following would you most likely suspect? A: Diverticulosis B: Hypercalcaemia C: Hypocalcaemia D: Irritable bowel syndrome

(B) Hypercalcaemia can cause polyuria, severe abdominal pain, and confusion.

A child is 5 years old and has been recently admitted into the hospital. According to Erickson which of the following stages is the child in? A: Trust vs. mistrust B: Initiative vs. guilt C: Autonomy vs. shame D: Intimacy vs. isolation

(B) Initiative vs. guilt- 3-6 years old

A patient has recently been diagnosed with polio and has questions about the diagnosis. Which of the following systems is most affected by polio? A: PNS B: CNS C: Urinary system D: Cardiac system

(B) Polio is caused by a virus that attacks the CNS.

A nurse is caring for an infant that has recently been diagnosed with a congenital heart defect. Which of the following clinical signs would most likely be present? A: Slow pulse rate B: Weight gain C: Decreased systolic pressure D: Irregular WBC lab values

(B) Weight gain is associated with CHF and congenital heart deficits.

A mother is inquiring about her child's ability to potty train. Which of the following factors is the most important aspect of toilet training? A: The age of the child B: The child ability to understand instruction. C: The overall mental and physical abilities of the child. D: Frequent attempts with positive reinforcement.

(C) Age is not the greatest factor in potty training. The overall mental and physical abilities of the child is the most important factor.

A nurse is reviewing a patient's medication. Which of the following is considered a potassium sparing dieuretic? A: Esidrix B: Lasix C: Aldactone D: Edecrin

(C) Aldactone (Spironolactone) is considered a potassium sparing diuretic.

A thirty five year old male has been an insulin-dependent diabetic for five years and now is unable to urinate. Which of the following would you most likely suspect? A: Atherosclerosis B: Diabetic nephropathy C: Autonomic neuropathy D: Somatic neuropathy

(C) Autonomic neuropathy can cause inability to urinate.

A 20 year-old female attending college is found unconscious in her dorm room. She has a fever and a noticeable rash. She has just been admitted to the hospital. Which of the following tests is most likely to be performed first? A: Blood sugar check B: CT scan C: Blood cultures D: Arterial blood gases

(C) Blood cultures would be performed to investigate the fever and rash symptoms.

A nurse is reviewing a patient's medication. The patient is taking Digoxin. Which of the following is not an effect of Digoxin? A: Depressed HR B: Increased CO C: Increased venous pressure D: Increased contractility of cardiac muscle

(C) Digoxin decreases venous pressure.

A nurse is caring for an adult that has recently been diagnosed with respiratory alkalosis. Which of the following clinical signs would most likely not be present? A: Anxiety attacks B: Dizziness C: Hyperventilation cyanosis D: Blurred vision

(C) Hyperventilation cyanosis is associated with respiratory acidosis.

Rho gam is most often used to treat____ mothers that have a ____ infant. A: RH positive, RH positive B: RH positive, RH negative C: RH negative, RH positive D: RH negative, RH negative

(C) Rho gam prevents the production of anti-RH antibodies in the mother that has a Rh positive fetus.

A father notifies your clinic that his son's homeroom teacher has just been diagnosed with meningitis and his son spent the day with the teacher in detention yesterday. Which of the following would be the most likely innervention? A: Isolation of the son B: Treatment of the son with Aciclovir C: Treatment of the son with Rifampicin D: Reassure the father

(C) Rifampicin would be used in this case.

A patient is getting discharged from a SNF facility. The patient has a history of severe COPD and PVD. The patient is primarily concerned about their ability to breath easily. Which of the following would be the best instruction for this patient? A: Deep breathing techniques to increase O2 levels. B: Cough regularly and deeply to clear airway passages. C: Cough following bronchodilator utilization D: Decrease CO2 levels by increase oxygen take output during meals.

(C) The bronchodilator will allow a more productive cough.

Which of the following normal blood therapeutic concentrations is abnormal? A: Phenobarbital 10-40 mcg/ml B: Lithium .6 - 1.2 mEq/L C: Digoxin .5 - 1.6 ng/ml D: Valproic acid 40 - 100 mcg/ml

(C) The normal ranges for Digoxin is .7 - 1.4 ng/ml.

Which of the following normal blood therapeutic concentrations is abnormal? A: Digitoxin 09 - 25 mcg/ml B: Vancomycin 05 - 15 mcg/ml C: Primidone 02 - 14 mcg/ml D: Theophylline 10 - 20 mcg/ml

(C) The normal ranges of Primidone is 04 -12 mcg/ml.

A parent calls the pediatric clinic and is frantic about the bottle of cleaning fluid her child drank 20 minutes. Which of the following is the most important instruction the nurse can give the parent? A: This too shall pass. B: Take the child immediately to the ER C: Contact the Poison Control Center quickly D: Give the child syrup of ipecac

(C) The poison control center will have an exact plan of action for this child.

A mother has recently been informed that her child has Down's syndrome. You will be assigned to care for the child at shift change. Which of the following characteristics is not associated with Down's syndrome? A: Simian crease B: Brachycephaly C: Oily skin D: Hypotonicity

(C) The skin would be dry and not oily.

A nurse is reviewing a patient's medication list. The drug Pentoxifylline is present on the list. Which of the following conditions is commonly treated with this medication? A: COPD B: CAD C: PVD D: MS

(C) This drug is a hemorheological agent that helps blood viscosity.

A nurse is administering a shot of Vitamin K to a 30 day-old infant. Which of the following target areas is the most appropriate? A: Gluteus maximus B: Gluteus minimus C: Vastus lateralis D: Vastus medialis

(C) Vastus lateralis is the most appropriate location.

A patient's chart indicates the patient is suffering from Digoxin toxicity. Which of the following clinical signs is not associated with digoxin toxicity? A: Ventricular bigeminy B: Anorexia C: Normal ventricular rhythm D: Nausea

(C) Ventricular rhythm may be premature with Digoxin toxicity.

A 84 year-old male has been loosing mobility and gaining weight over the last 2 months. The patient also has the heater running in his house 24 hours a day, even on warm days. Which of the following tests is most likely to be performed? A: FBC (full blood count) B: ECG (electrocardiogram) C: Thyroid function tests D: CT scan

(C) Weight gain and poor temperature tolerance indicate something may be wrong with the thyroid function.

A patient's chart indicates a history of ketoacidosis. Which of the following would you not expect to see with this patient if this condition were acute? A: Vomiting B: Extreme Thirst C: Weight gain D: Acetone breath smell

(C) Weight loss would be expected.

A fragile 87 year-old female has recently been admitted to the hospital with increased confusion and falls over last 2 weeks. She is also noted to have a mild left hemiparesis. Which of the following tests is most likely to be performed? A: FBC (full blood count) B: ECG (electrocardiogram) C: Thyroid function tests D: CT scan

(D) A CT scan would be performed for further investigation of the hemiparesis.

A thirteen-year old male has a tender lump area in his left groin. His abdomen is distended and he has been vomiting for the past 24 hours. Which of the following would you most like suspect? A: Ulcerative colitis B: Biliary colic C: Acute gastroenteritis D: Strangulated hernia

(D) A hernia is the most likely indicated in this case.

A patient tells you that her urine is starting to look discolored. If you believe this change is due to medication, which of the following patient's medication does not cause urine discoloration? A: Sulfasalazine B: Levodopa C: Phenolphthalein D: Aspirin

(D) All of the others can cause urine discoloration.

A patient has been instructed by the doctor to reduce their intake of Potassium. Which types of foods should not worry about avoiding? A: Bananas B: Tomatoes C: Orange juice D: Apples

(D) All the others are high in potassium.

A patient's chart indicates a history of hyperkalemia. Which of the following would you not expect to see with this patient if this condition were acute? A: Decreased HR B: Paresthesias C: Muscle weakness of the extremities D: Migranes

(D) Answer choices A-C were symptoms of acute hyperkalemia.

A patient has taken an overdose of aspirin. Which of the following should a nurse most closely monitor for during acute management of this patient? A: Onset of pulmonary edema B: Metabolic alkalosis C: Respiratory alkalosis D: Parkinson's disease type symptoms

(D) Aspirin overdose can lead to metabolic acidosis and cause pulmonary edema development.

A patient has been on long-term management for CHF. Which of the following drugs is considered a loop dieuretic that could be used to treat CHF symptoms? A: Ciprofloxacin B: Lepirudin C: Naproxen D: Bumex

(D) Bumex is considered a loop dieuretic.

A nurse if reviewing a patient's chart and notices that the patient suffers from conjunctivitis. Which of the following microorganisms is related to this condition? A: Yersinia pestis B: Helicobacter pyroli C: Vibrio cholera D: Hemophilus aegyptius

(D) Choice A is linked to Plague, Choice B is linked to peptic ulcers, Choice C is linked to Cholera.

A nurse is putting together a presentation on meningitis. Which of the following microorganisms has noted been linked to meningitis in humans? A: S. pneumonia B: H. influenza C: N. meningitis D: Cl. difficile

(D) Cl. difficile has not been linked to meningitis.

Which of the following conditions would a nurse not administer erythromycin? A: Campylobacterial infection B: Legionnaire's disease C: Pneumonia D: Multiple Sclerosis

(D) Erythromycin is used to treat conditions A-C.

A nurse is caring for an adult that has recently been diagnosed with hypokalemia. Which of the following clinical signs would most likely not be present? A: Leg cramps B: Respiratory distress C: Confusion D: Flaccid paralysis

(D) Flaccid paralysis is an indication of Hyperkalemia.

A nurse is caring for an adult that has recently been diagnosed with metabolic alkalosis. Which of the following clinical signs would most likely not be present? A: Vomiting B: Diarrhea C: Agitation D: Hyperventilation

(D) Hyperventilation occurs with metabolic acidosis. Hypoventilation occurs with metabolic alkalosis.

A 34 year old female has recently been diagnosed with an autoimmune disease. She has also recently discovered that she is pregnant. Which of the following is the only immunoglobulin that will provide protection to the fetus in the womb? A: IgA B: IgD C: IgE D: IgG

(D) IgG is the only immunoglobulin that can cross the placental barrier.

A nurse has just started her rounds delivering medication. A new patient on her rounds is a 4 year-old boy who is non-verbal. This child does not have on any identification. What should the nurse do? A: Contact the provider B: Ask the child to write their name on paper. C: Ask a co-worker about the identification of the child. D: Ask the father who is in the room the child's name.

(D) In this case you are able to determine the name of the child by the father's statement, moreover you should not withhold the medication from the child following identification.

A young adult is 20 years old and has been recently admitted into the hospital. According to Erickson which of the following stages is the adult in? A: Trust vs. mistrust B: Initiative vs. guilt C: Autonomy vs. shame D: Intimacy vs. isolation

(D) Intimacy vs. isolation- 18-35 years old

Which of the following is the key risk factor for development of Parkinson's disease dementia? A: History of strokes B: Acute headaches history C: Edward's syndrome D: Use of phenothiazines

(D) Penothiazines are considered a risk factor for Parkinson's disease dementia.

A nurse is administering blood to a patient who has a low hemoglobin count. The patient asks how long to RBC's last in my body? The correct response is. A: The life span of RBC is 45 days. B: The life span of RBC is 60 days. C: The life span of RBC is 90 days. D: The life span of RBC is 120 days.

(D) RBC's last for 120 days in the body.

A patient has recently been diagnosed with hyponatremia. Which of the following is not associated with hyponatremia? A: Muscle twitching B: Anxiety C: Cyanosis D: Sticky mucous membranes

(D) Stick mucuous membranes are associated with hypernatremia.

A nurse is caring for an adult that has recently been diagnosed with respiratory acidosis. Which of the following clinical signs would most likely not be present? A: CO2 Retention B: Dyspnea C: Headaches D: Tachypnea

(D) Tachypnea is associated with respiratory alkalosis.

A new mother has some questions about (PKU). Which of the following statements made by a nurse is not correct regarding PKU? A: A Guthrie test can check the necessary lab values. B: The urine has a high concentration of phenylpyruvic acid C: Mental deficits are often present with PKU. D: The effects of PKU are reversible.

(D) The effects of PKU stay with the infant throughout their life.

A nurse is observing a child's motor, sensory and speech development. The child is 7 months old. Which of the following tasks would generally not be observed? A: Child recognizes tone of voice. B: Child exhibits fear of strangers. C: Child pulls to stand and occasionally bounces. D: Child plays patty-cake and imitates.

(D) These skills generally develop between 10-15 months.

A fifty-year-old blind and deaf patient has been admitted to your floor. As the charge nurse your primary responsibility for this patient is? A: Let others know about the patient's deficits B: Communicate with your supervisor your concerns about the patient's deficits. C: Continuously update the patient on the social environment. D: Provide a secure environment for the patient.

(D) This patient's safety is your primary concern.

A nurse is observing a child's motor, sensory and speech development. The child is 5 months old. Which of the following tasks would generally not be observed? A: Child sits with support. B: Child laughs out loud. C: Child shifts weight side to side in prone. D: Child transfers objects between hands.

(D) Transferring objects between hands is a 8-9 month skill.

A nurse is reviewing a patient's PMH. The history indicates photosensitive reactions to medications. Which of the following drugs has not been associated with photosensitive reactions? Note: More than one answer may be correct. A: Cipro B: Sulfonamide C: Noroxin D: Bactrim E: Accutane F: Nitrodur

(F) All of the others have can cause photosensitivity reactions.

If water breaks and is clear, what are priority nursing actions?

- encourage peeing every 1-2 hr to help fetal head descend -monitor temperature b/c of membrane rupture

The purpose of diagnosing is to identify:

- How an individual, group, or community responds to actual or potential health and life processes - Factors that contribute to or cause health problems (etiologies) - Strengths the patient can draw on to prevent or resolve problems

Which type of nursing delivery models would be appropriate in a rural area that would require fewer numbers of registered nurses? Select all that apply.

-Functional nursing -Team nursing -Patient-focused care rationale: When you have a limited number of RN's, then functional nursing, team nursing, and patient-focused care are useful, because you can deliver quality care by using more ancillary staff. Primary care is usually 24-hour RN coverage, and transition care is where APRNs conduct assessments and—with physicians—design and coordinate patient care and discharge plans.

The American Hospital Association and Congress have identified which of the following as major barriers to the full integration health information technology? Select all that apply.

-Lack of standardization across point-of-care areas -Funding issues -No single set of privacy laws -Lack of a unique client identifier number rationale: Research data support that the use of the computerized physician order system (CPOE), computerized decision support systems, medication administration record (MAR), and bar-codes on client identification bands can limit errors and improve care. Both organizations identify barriers as: Lack of standardization across point-of-care areas—Laboratory data and pharmacy systems need to be integrated with the client's health record. Funding—Information technology is costly, and often the major costs are borne by hospitals rather than shared by other providers, payers, and employers. Privacy laws—A single set of privacy laws is needed to simplify the task of communicating across agencies and local, state, and federal governments, which can make compliance difficult and interfere with client care. Lack of a uniform approach (identifier number) to match the client to his or her record—A single authentication number is needed to reduce safety risks and provide a uniform access to a client's data.

Which of the following are considered nursing practice classification systems? Select all that apply.

-North American Nursing Diagnosis Association International (NANDA-I) Approved List of Diagnostic Labels -Nursing Interventions Classification (NIC) System -Nursing Outcomes Classification (NOC) System -Nursing Management Minimum Data Set (NMMDS) rationale: The ANA approved the establishment of the Nursing Information and Data Set Evaluation Center (NIDSEC) to review, evaluate against defined criteria, and recognize information systems from developers and manufacturers that support documentation of nursing care within automated nursing information systems (NIS) or within computer-based patient record systems (CPR). They recognized the following 13 nursing practice classification systems: North American Nursing Diagnosis Association International (NANDA-I) Approved List of Diagnostic Labels Nursing Interventions Classification System (NIC) Nursing Outcomes Classification System (NOC) Nursing Management Minimum Data Set (NMMDS) Clinical Care Classification (CCC, formerly Home Health Care Classification [HHCC]) Omaha System Patient Care Data Set (PCDS) PeriOperative Nursing Dataset (PNDS) SNOMED CT Nursing Minimum Data Set (NMDS) International Classification of Nursing Practice (ICNP) ABC codes Logical Observation Identifier Names & Codes (LOINC)

Which of the following statements are true about continuous quality improvement? Select all that apply.

-The accountability for quality is vested in the individual nurse -Quality standards must incorporate the expectations of clients and their families.providing direct client care. -Systems within the hospital must be reviewed to determine how care can be enhanced. rationale: This approach emphasizes continually looking for opportunities to improve. CQI looks not only at what the nurse does in the pursuit of quality but also at how the systems of the units in the hospital can be improved to provide better care at lower cost.

What precautions should a pt take when on antianxiety meds?

-do not do acitivites that req operating machinery -can cause drowsiness, confusions, and retrograde amnesia which can be problematic b/c pt can forget that they took the medication

A patient complains of not having had a bowel movement since being admitted 2 days ago for multiple fractures of both lower legs. The patient is on bedrest and has skeletal traction. Which intervention would be the most appropriate nursing action: a) Administer an enema c) Ensure maximum fluid intake (3000 mL/day) d) Perform range of motion exercises to all extremeties

...

A patient is receiving 3% NaCl solution for correction of hypoatremia. During administration of the solution, the most important assessment for the nurse is to monitor is: a) Lung sounds c) Peripheral pulses d) Peripheral edema

...

A patient substained several wounds on the legs caused by a fall. On the day after the injuries, the wounds appear and edematous. The nurse identifies the stage of healing of these wounds as long: a) Inflammatory b) Proliferate d) Remodeling

...

The RN should incorporate which instructions into the teaching plan for a client with a urinary diversion: b) Notify the physician if the stoma is deep pink and shiny c) Strands of blood appear in the urine d) Increase fluid intake

...

The nurse notes that a patient who was admitted with diabetic ketoacidosis has rapid, deep respirations. Which action should the nurse take: a) Notify the patient's health care provider b) Give the prescribed PRN lorazepam (ativan) c) Start the prescribed PRN oxygen at 2 to 4 L/min

...

When the body is subjected to invasion or trauma, the role of Europhiles is to: b) Release histamine into the circulation c) Produce specific antigens d) Phagocytize injurious agents

...

When the nurse assesses dyspnea in a client with congestive heart failure, she assesses for other manifestations of fluid volume excess including: b) Peripheral Edema c) Increased hematocrit level d) decreased urine output

...

Which potential potassium order is safe for the nurse to implement: a) Add 20 mEq of KCL to 1,000 mL of IV fluid b) 10 mEq KCL IV over 1-2 min d) 10 mEq KCL SQ

...

Which problem is most appropriate for the nurse to identify for the client with diarrhea: a) Alteration in skin integrity b) Chronic pain perception d) INeffective coping

...

If a health care provider is planning to transfuse a patient with a unit of packed RBCs, which of the following solutions should the health care provider hang with the transfusion?

0.9% Sodium Chloride

What are 3 IV isotonic solution?

0.9% normal saline, 5% dextrose in water, lactated ringer's solution

What is the distance required to place medications into the rectum of a child?

1 to 2 cm

How long is the stay in an ambulatory care unit?

1 day

Which of the following outcomes are correctly written?

1. During the next 24-hour pariod, the patient's fluid intake will total at least 2,000 mL. 2. At the next visit, 12/23/11, the patient will correctly demonstrate relaxation exercises.

The IV catheter used for a blood transfusion should be at least

18 or 19 gauge

What are some common Nursing Interventions to reduce risk/avoid post-op complications?

1. Turn & reposition the pt to promote circulation and reduce the risk of skin breakdown, especially over boney prominences. Initially position pt in a Lateral recumbant position until arousal from anesthesia, then position pt in Semi or Fowler position to reduce breathing effort. 2. Encourage coughing & deep breathing. This helps clear anestetics from the body, lowers risk of pulmonary/fat emboli, and hypostatic pneumonia associated with secretion buildup in the airways. 3. Encourage use of Incentive Spirometer. 4. Monitor In's and Out's. Hydration and protein rich nutrition promotes healing and provides energy to meet the needs of the pts increased metabolism associated with surgery. 5. Promote early ambulation. Early post-op exercise and ambulation significantly reduce the risk of thromboembolism.

Decerebrate posturing

1. arms extended, legs stiffly extended 2. damage to upper brain stem/ diencephalon, midbrain, or pons

Decorticate posturing

1. arms flexed, legs rotated inward 2. lesion in the cerebral hemisphere/ disruption of the corticospinal tracts

When can a child return to school if they have antibiotic eye drops for viral conjunctivits?

Child should be kept home from school or day care until antibiotic eye drops have been administered for 24 hours.

Signs of septicemia with a blood transfusion in a pt.

Chills, fever, vomiting, diarrhea, hypotension, development of shock.

Five minutes after the client's first postoperative exercise, the client's vital signs have not yet returned to baseline. Which is an appropriate nursing diagnosis? 1.Activity Intolerance. 2.Risk for Activity Intolerance. 3.Impaired Physical Mobility. 4.Risk for Disuse Syndrome.

1.Activity Intolerance. Rationale: Vital signs that do not return to baseline 5 minutes after exercising indicate intolerance of exercise at that time. This is a real problem, not "at risk for," as in option 2. There is no evidence that the client requires assistance (impaired mobility, option 3), or is immobile (disuse syndrome, option 4).

A nurse who is teaching a group of adults ages 20 to 40 years old about safety is going to ensure that which topic is a priority? 1. Automobile crashes 2. Drowning and firearms 3. Falls 4. Suicide and homicide

1.Automobile crashes Rationale: When educating a group of young to middle-aged adults on safety, it is important to instruct them on the leading cause of injuries in this group. The leading cause of injuries in this group is related to automobile use. Option 2 is the leading cause for school-age children. Option 3 is the leading cause for older adults, and option 4 relates to adolescents.

Performance of activities of daily living (ADLs) and active range of motion (ROM) exercises can be accomplished simultaneously as illustrated by which of the following? Select all that apply. 1.Elbow flexion with eating and bathing. 2.Elbow extension with shaving and eating. 3.Wrist hyperextension with writing. 4.Thumb ROM with eating and writing. 5.Hip flexion with walking.

1.Elbow flexion with eating and bathing. 4.Thumb ROM with eating and writing. 5.Hip flexion with walking. Rationale: Eating and bathing will flex the elbow joint, and grasping and manipulating utensils to eat and write will take the thumb through its normal ROM. Walking flexes the hip. Shaving and eating require elbow flexion, not extension (option 2). Writing brings the fingers toward the inner aspect of the forearm, thus flexing the wrist joint (option 3).

Isotonic exercises such as walking are intended to achieve which of the following? Select all that apply. 1.Increase muscle tone and improve circulation. 2.Increase blood pressure. 3.Increase muscle mass and strength. 4.Decrease heart rate and cardiac output. 5.Maintain joint range of motion.

1.Increase muscle tone and improve circulation. 3.Increase muscle mass and strength. 5.Maintain joint range of motion. Rationale: Isotonic exercise increases muscle tone, mass, and strength, maintains joint flexibility, and improves circulation. During isotonic exercise, both heart rate and cardiac output quicken to increase blood flow to all parts of the body (option 4). Little or no change in blood pressure occurs (option 2).

When assessing a client's gait, which does the nurse look for and encourage? 1.The spine rotates, initiating locomotion. 2.Gaze is slightly downward. 3.Toes strike the ground before the heel. 4.Arm on the same side as the swing-through foot moves forward at the same time.

1.The spine rotates, initiating locomotion. Rationale: Normal gait involves a level gaze, an initial rotation beginning in the spine, heel strike with follow-through to the toes, and opposite arm and leg swinging forward.

normal ammonia level

10 to 80 mcg/dL

Residual amount from a NG tube more than ______mL require holding the feeding.

100 mL

If the client has a temperature higher than___ degrees F, the unit of blood should not be hung until ________.

100; the physician is notified.

How often should the pt do incentive spiro?

10x per hour

What is expected after a CBI?

12 hr after could have clots monitor for too much fluid loss b/c could go into shock make sure bag is strait and attached to leg strap

normal platelet count

150,000 to 450,000

A nurse is documenting the removal of a urinary drainage catheter from a patient. If the catheter is removed at 0900, it should be notes that the patient is due to void by:

1700

Where is the fundus at 36 weeks?

right below the ensiform cartilage

the pt w/ tuberculosis usually is not contagious after taking the meds for ___ to ___ consective _____.

2 to 3 weeks

The nurse assesses a client's pedal pulses as having a pulse volume of 1 on a scale of 0-3. Based on this assessment finding, it would be important for the nurse to also assess the: 1) pulse deficit 2) BP 3) apical pulse 4) pulse pressure

2) blood pressure

For the following patient problems, which answer choice lists the problems in order of highest to lowest priority based on Maslow's hierarchy of human needs? 1. Disturbed Body Image 2. Ineffective Airway Clearance 3. Spiritual Distress 4. Impaired Social Interaction

2, 4, 1, 3

The Consensus Formula for fluid requirements in burn victims

2-4mL of Lactated Ringer's solution x kg body weight x percent burn

What is the most common sexually transmitted infection (STD) in the United States?

Chlamydial Infection

Before a female patient begins using oral contraceptives, a nurse should question the patient about a history of:

Cholecystitis

What are sx of peritoneal dialysis probs with inflow?

right shoulder pain slow infusion b/c is going to fast

The pt should remain in bed for at least ____ hours after a parenteral dose of diazepam.

3

Using the Glasgow Coma Scale (GCS), a patient in the deepest coma would have a score of:

3

After the urine begins to flow, the catheter is inserted ___ to ____ after urine begins to flow to provide sufficient space to inflate the balloon

2.5 to 5 cm

a pt with a fiberglass(nonplaster) cast can bear weight on the casted leg within?

20 to 30 minutes

When in a pregnancy is a miscarriage likely to occur?

20-24 weeks

% Oxygen in Room Air

21%

Normal Range for HCO3:

21-28 mEq/L

What is the average length of a NG tube for an adult?

22 to 26 inches

What is the expected funal height for a pt 24-34 weeks?

24-34 cm

For subcutaneous heparin sodium injection, what needle gauge & needle inch be used?

25 to 26 gauge; 3/5 to 5/8 inch needle

A nurse administers a PPD intradermally for a Mantoux test by using a tuberculin syringe with what size gauge and needle?

25 to 27 gauge; 5/8 inch needle

How long shouldn't you eat before local anesthesia?

3 hours

When may pneumonia develop?

3 to 5 days postop

When does wound infection usually occur post op?

3 to 6 days after surgery

Which of the following clients should have an apical pulse taken? A client who is: 1) febrile and has a radial pulse of 100 bpm 2) a runner who has a radial pulse of 62 bpm 3) an infant with no history of cardiac defect 4) an elderly adult who is taking antianxiety medication

3) an infant with no history of cardiac defect

A client who has experienced prolonged exposure to the cold is admitted to the hospital. Which method of taking a temperature would be most appropriate for this client? 1) axillary with an electronic thermometer 2) oral with a glass thermometer 3) rectal with an electronic thermometer 4) tympanic with an infrared thermometer

3) rectal with an electronic thermometer

Whe nurse assesses clients' breath sounds. Which one requires immediate medical attention? A client who has: 1) crackles 2) rhonchi 3) stridor 4) wheezes

3) stridor

The amount of blood ejected from each ventricle with each heartbeat is which of the following? 1) pulse pressure 2) cardiac output 3) stroke volume 4) tidal volume

3) stroke volume

A nurse is caring for a client diagnosed with Chron's disease, who has undergone a barium enema that demonstrated the presence of strictures in the ileum. Based on this finding, the nurse should monitor the client closely for signs of: A. peritonitis B. obstruction C. malaborsorption. D. fluid imbalance.

B. obstruction

At 7 months after back injury and lumbar laminectomy, a patient complains of tenderness at the operative site and appearss depressed and unwell. Other symptoms include depression, fatigue, and sleep disturbances. Which nursing diagnosis is a priority for this patient?

Chronic Pain Chronic pain has vague symptoms and few other physical findings and occurs beyond

Immunizations 15 mo.

DTaP [Diptheria, Tetanus, Pertussis] (4 of 4)

WHat is of particular concern in client who were in the lithotomy position?

thrombophlebitis

The nurse has completed discharge teaching for a client who will be going home on oxygen therapy. Which statement made by the client, would indicate that this client needs further instruction: a) I will replace my cotton blankets with polyester ones b) My son will not be able to smoke when I am around

A) I will replace my cotton blankets with polyester ones

Immunizations 4-6 years

DTaP [Diptheria, Tetanus, Pertussis] (booster) IPV [Inactivated Polio Virus] (4 of 4) MMR [Measles, mumps, and rubella] (2 of 2) Varicella (2 of 2)

Community health nurses conducting health education among populations vulnerable to HIV infection should explain the natural history of the infection, including the fact that HIV infection may go undetected during the primary infection stage because: A) Antibody test results are typically negative. B) Antibody production by the immune system increases. C) Incubation period is prolonged. D) Symptoms include myalgias, sore throats, and rash.

A

Which statement from a client with one weak leg regarding use of crutches when using stairs indicates a need for increased teaching? 1."Going up, the strong leg goes first, then the weaker leg with both crutches." 2."Going down, the weaker leg goes first with both crutches, then the strong leg." 3."The weaker leg always goes first with both crutches." 4."A cane or single crutch may be used instead of both crutches if held on the weaker side."

3."The weaker leg always goes first with both crutches." Rationale: Although the crutches (or cane) are always used along with the weaker leg, the weaker leg should go down the stairs first. The stronger leg can support the body as the weaker leg moves forward. All of the other statements are correct.

When planning to teach health care topics to a group of male adolescents, which topic should the nurse consider a priority? 1.Sports contribute to an adolescent's self-esteem. 2.Sunbathing and tanning beds can be dangerous. 3.Guns are the most frequently used weapon for adolescent suicide. 4. A driver's education course is mandatory for safety.

3.Guns are the most frequently used weapon for adolescent suicide. Rationale: Suicide and homicide are two leading causes of death among teenagers. Adolescent males commit suicide at a higher rate than adolescent females. Options 1 and 2 are true; however, neither would be as high a priority as preventing suicide. Option 4 is not true. A driver's education course does not ensure safe practice.

Blood must be hung within _____

30 minutes

Which of the following represents the normal range of pulse pressure?

30 to 50 mm Hg

Normal Range for: PaCO2

35-45 mm Hg

Bladder Infection, most important to report to Dr:

Flank pain

The shift change while the nursing staff was waiting for the adult children of a deceased client to arrive. The oncoming nurse has never met the family. Which of the following initial greetings is most appropriate: a) I'm very sorry for your loss b) I'll take you in to view the body

A) I'm very sorry for your loss

Which nurse in history is credited with establishing nursing education?

Florence Nightingale

Which of the following clients has indications of orthostatic hypotension? A client whose BP is: 1) 188/68 when standing and 110/72 when lying down 2) 140/80, HR 82 bpm when sitting and 136/76, HR 98 bpm when standing 3) 126/72 lying down and 133/80 when sitting, and reports shortness of breath 4) 146/88 when lying down and 130/78 when standing, and reports feeling dizzy

4) 146/88 when lying down and 130/78 when standing, and reports feeling dizzy

Which of the following clients would have the most difficulty maintaining thermoregulation? 1) young child playing soccer during the summer 2) middle-aged adult snow skiing 3) young adult playing golf on a hot day 4) older adult raking leaves on a cold day

4) older adult raking leaves on a cold day

If the community is where nurses practice and apply the nursing process, and the community is the client in that practice, then nurses will want to analyze and synthesize information about: A) Boundaries, parts, and dynamic processes of the client community. B) Community health status and structure. C) Community problems and problem correlates. D) Role of the nurse and lay advisors in the community partnership.

A

When giving a med by IV bolus, if the med is incompatible with the IV solution, what should the RN do?

Flush the tubing before and after the bolus with infusions of normal saline.

An adrenal crisis is characterized by all of the signs and symptoms except: A. weakness and fatigue B. nausea & vomiting C. hypotension D. sodium & fluid retention

D. Sodium and fluid retention are characteristics of Cushing's Syndrome. Adrenal crisis causes decreased sodium levels and hypotension.

What is the nL range of Hg A1C?

4.5-7.6%

A nurse is teaching a client about active range-of-motion (ROM) exercises. The nurse then watches the client demonstrate these principles. The nurse would evaluate that teaching was successful when the client does which of the following? 1.Exercises past the point of resistance. 2.Performs each exercise one time. 3.Performs each series of exercises once a day. 4.Uses the same sequence during each exercise session.

4.Uses the same sequence during each exercise session. Rationale: When the client performs the movements systematically, using the same sequence during each session, the nurse can evaluate that the teaching was understood and is successful. When performing active ROM the client should exercise to the point of slight resistance, but never past that point of resistance in order to prevent further injury (option 1). The client should perform each exercise at least three times, not just once (option 2). The client should perform each series of exercises twice daily, not just once per day (option 3).

normal white blood cell count

4500 to 11,000

pH of NG aspirate with client on ventilator

5 or higher

Excess Fluid Volume IV fluids

5% dextrose in 0.9% NS hypertonic to pull fluid into intravascular space and excreted by kidneys

bleeding precautions need to be initiated when the platelet count decrease. When the platelets are lower than _____ any small trauma can lead to episodes of prolonged bleeding.

50,000 mm

to prevent reinfection or drug resistant tuberculosis the pt must take the full course of therapy for _____.

6 months or longer

When does wound dehiscence or evisceration usually occur?

6 to 8 days after surgery

How long shouldn't you eat before general anesthesia?

6 to 8 hours

When does urinary retention postop?

6 to 8 hrs after surgery

Addison's disease

HYPOsecretion of adrenal cortex hormones lethargy, fatigue, muscle weakness GI disturbances wt LOSS menstrual changes, impotence in men HYPOglycemia, HYPOnatremia HYPERkalemia, HYPERcalcemia postural hypotension hyperpigmentation of skin (bronzed) with primary disease

Normal Range for pH:

7.35- 7.45

drainage that is more than ____ to ____ mL/hr is considered excessive & requires MD notification

70 to 100

normal clotting time

8 to 15 minutes

At risk for difficulty urination elimination:

80 year old male, frequent urination at night

What is ok level of bili?

<15 at 72 hr = good and does not need phototherapy

LGA (large for gestational age infant)

> 4000 g higher incidence of birth injuries, congenital abnormalities, hypoglycemia

Advantage of Ileal conduit:

> chance of ascending kidney infection

A community-level intervention designed to increase the sense of belonging among older community residents at risk for social isolation was implemented by opening a senior center every other Wednesday at a local church that provided lunch and social programs. At the end of 6 months, the attendees were surveyed to determine their experience with the program, barriers to attendance, expansion of their social networks, and involvement in other community activities. This survey allowed the community health nurse to _______ the program and design program improvements. A) Evaluate the effectiveness of. B) Assess the expansion needs of. C) Identify problems with D) Implement the expansion of.

A

A nursing diagnosis of Increased risk for delayed development, injury, and disease because of inadequate parenting by a primary parent experiencing depression would most likely indicate that the nursing process is being applied at the _______ level of practice and the _______ level of prevention. A) Individual/family + secondary. B) Community + primary. C) Community + secondary. D) Individual/family + primary.

A

A rural community health nurse has made sure that male and female lay advisors are involved in the health department's migrant worker outreach program. The nurse believes this intervention strategy is important because the nurse knows that such individuals can be: A) People who are influential in approving or vetoing new ideas. B) Medical professionals within the migrant community. C) Natural healers within their community. D) Translators to help overcome language barriers.

A

A school nurse notes that 60 children have missed days of high school because of pertussis this past year and this rate has been relatively constant for the past 5 years. The nurse plans to work with the community to increase awareness of the seriousness of this disease for children younger than 6 months of age and to raise and maintain the immunization rates, because in this community the pertussis is: A) Endemic. B) Epidemic. C) Pandemic. D) Sporadic.

A

A state public health region reported 39 cases of meningitis in children 15 years of age and younger to date this year. Seven of those children died. The total population of the region is 780,000, of whom 84,000 are children 15 years old and younger. What is the age-specific meningitis death rate for children age 15 years and younger for this region to date this year? A) 0.08/1000 B) 0.46/1000 C) 1/1000 D) 8/1000

A

A woman comes to the community health center complaining of increasing lower abdominal pain, fever, and abnormal menses for several months. During the assessment, the client indicates that she is aware that her husband has had multiple sex partners in the past two years. Appropriate intervention by the nurse would be to: A) Arrange to have the client referred for medical evaluation for pelvic inflammatory disease (PID) and appropriate intervention and treatment. B) Contact the health department to confirm the spouse's diagnosis of Chlamydia infection to determine the client's exposure, give the client antibiotics, and have her return to the clinic if symptoms worsen. C) Provide sexually transmitted disease (STD) prevention and treatment education and refer the client to the health department for STD screening for gonorrhea and/or Chlamydia infection. D) Supply the client with nonsteroidal antiinflammatory drugs and caution her to call the after-hours call doctor if her symptoms worsen.

A

Although the definitions of evidence-based practice (EBP) in the literature vary widely, their common thread across all health care disciplines is: A) Application of the best available evidence to improve practice. B) Definition of what counts as evidence. C) Reliance on principles of pathophysiology. D) Method of transforming research into practice.

A

An American takes a long-awaited vacation in sunny Mexico, spending days on the beach eating fresh raspberries from a nearby vendor and drinking bottled water. The tourist may be altering: A) Agent-host-environment interaction. B) Circadian rhythms. C) Herd immunity. D) Host resistance.

A

Public health nurses are challenged to respond to public health-related trends of the twenty-first century, which include: A) Racial, ethnic, and economic health disparities; rise of drug-resistant pathogens; unequal access to health care; and violence. B) Violence, availability of health care for all, and increasing life expectancy. C) Health disparities, access issues, and adequate mental health program funding. D) Rise of drug-resistant organisms, increased life expectancy, societal violence, and more effective disease surveillance.

A

Public health nursing practice is guided by the community's priorities as identified by community: A) Assessment. B) Diagnosis. C) Interventions. D) Planning.

A

The World Health Organization (WHO) developed the Five Keys to Safer Food campaign in 2001 to address the problem of foodborne and waterborne diarrheal diseases worldwide. This campaign emphasizes which of the following practices? A) Keep clean, separate raw and cooked, cook thoroughly. B) Never use raw, always cook, buy better. C) Wash, cut, cook, and throw away. D) Wash, cover, and always refrigerate.

A

The clients most at risk of reactivation of latent infections of tuberculosis (TB) are: A) Immunocompromised persons, substance abusers, and those with diabetes. B) Individuals previously treated for TB. C) Long-term cigarette smokers. D) Persons with new-onset asthma or emphysema.

A

The community practice nurse is preparing to initiate a community partnership with a neighborhood watch association to address teenager street vandalism. The nurse is evaluating the community health dimension of process and seeks to determine the community's: A) Commitment to prioritizing and solving health problems. B) Crime rate and school absenteeism rate. C) Educational level. D) Local client-to-provider ratio.

A

The federal-state-local partnership teams with other organizations to develop and implement responses to identified public health concerns because: A) Community health is a shared responsibility. B) Health objectives are defined nationally. C) Population health is the responsibility of the government. D) Public health trends focus on bioterrorism.

A

The public health nurse applies knowledge in working with a local school board coalition to develop a helmet safety campaign in the middle and high schools. This best exemplifies which aspect of public health? A) Core competency. B) Core function. C) Nursing role. D) Standard of practice.

A

Today, an evidence-based nursing practice can best be defined as which of the following? A) Approach to the integration of the best research available, nursing expertise, and the preferences/values of the clients served. B) Concept developed by acute care nurses to ensure the quality of care of hospitalized clients and to challenge managed-care decisions. C) Concept developed in the early twentieth century to help nurses document the scientific basis of their nursing practice. D) Framework supporting the use of traditional research as the only basis for making clinical decisions in practice.

A

When a nurse evaluates the completeness and accuracy of information made available to community residents regarding the impact of rezoning of land parcels for industrial use, the nurse can best be described as: A) Advocating for ethical choices. B) Communicating risk. C) Controlling environmental damage. D) Volunteering for service on state boards.

A

When a situation exists in which there is potential contact with blood or body fluids, health care workers must always perform hand hygiene and wear gloves, masks, protective clothing, and other indicated personal protective barriers. The underlying reason for requiring these practices, known as universal precautions, is that: A) Blood and body fluids of all clients need to be handled as if they were infected. B) Effective infection control surveillance programs are in place. C) Health care settings are reservoirs of infection. D) Health care workers do not effectively use hand hygiene.

A

When confirmed cases of the mumps, a vaccine-preventable disease, emerged on college campuses in fall 2006, public health nurses conducted outreach at campuses and collaborated with student health officials to increase the number of students with full immunization compliance. This is an example of: A) Community-level practice. B) Family-level practice. C) Individual-level practice. D) Systems-level practice.

A

A 63-year-old patient is newly diagnosed with type 2 diabetes. When developing an education plan, the nurse's first action should be to a. assess the patient's perception of what it means to have type 2 diabetes. b. demonstrate how to check glucose using capillary blood glucose monitoring. c. ask the patient's family to participate in the diabetes education program. d. discuss the need for the patient to actively participate in diabetes management.

A Rationale: Before planning education, the nurse should assess the patient's interest in and ability to self-manage the diabetes. After assessing the patient, the other nursing actions may be appropriate, but planning needs to be individualized to each patient.

A patient with type 2 diabetes that is controlled with diet and metformin (Glucophage) also has severe rheumatoid arthritis (RA). During an acute exacerbation of the patient's arthritis, the health care provider prescribes prednisone (Deltasone) to control inflammation. The nurse will anticipate that the patient may a. require administration of insulin while taking prednisone. b. develop acute hypoglycemia during the RA exacerbation. c. have rashes caused by metformin-prednisone interactions. d. need a diet higher in calories while receiving prednisone.

A Rationale: Glucose levels increase when patients are taking CORTICOsteroids, and insulin may be required to control blood glucose. Hypoglycemia is not a complication of RA exacerbation or prednisone use. Rashes are not an adverse effect caused by taking metformin and prednisone simultaneously. The patient is likely to have an increased appetite when taking prednisone, but it will be important to avoid weight gain for the patient with RA.

Intramuscular glucagon is administered to an unresponsive patient for treatment of hypoglycemia. Which action should the nurse take after the patient regains consciousness? a. Give the patient a snack of cheese and crackers. b. Have the patient drink a glass of orange juice or nonfat milk. c. Administer a continuous infusion of 5% dextrose for 24 hours. d. Assess the patient for symptoms of hyperglycemia.

A Rationale: Rebound hypoglycemia can occur after glucagon administration, but having a meal containing complex carbohydrates plus protein and fat will help prevent hypoglycemia. Orange juice and nonfat milk will elevate blood sugar rapidly, but the cheese and crackers will stabilize blood sugar. Administration of glucose intravenously might be used in patients who were unable to take in nutrition orally. The patient should be assessed for symptoms of hypoglycemia after glucagon administration.

Cardiac monitoring is initiated for a patient in diabetic ketoacidosis (DKA). The nurse recognizes that this measure is important to identify a. electrocardiographic (ECG) changes and dysrhythmias related to hypokalemia. b. fluid overload resulting from aggressive fluid replacement. c. the presence of hypovolemic shock related to osmotic diuresis. d. cardiovascular collapse resulting from the effects of hyperglycemia.

A Rationale: The hypokalemia associated with metabolic acidosis can lead to potentially fatal dysrhythmias such as ventricular tachycardia and ventricular fibrillation, which would be detected with ECG monitoring. Fluid overload, hypovolemia, and cardiovascular collapse are possible complications of DKA, but cardiac monitoring would not detect theses.

A patient with type 1 diabetes has been using self-monitoring of blood glucose (SMBG) as part of diabetes management. During evaluation of the patient's technique of SMBG, the nurse identifies a need for additional teaching when the patient a. chooses a puncture site in the center of the finger pad. b. washes the puncture site using soap and water. c. says the result of 130 mg indicates good blood sugar control. d. hangs the arm down for a minute before puncturing the site.

A Rationale: The patient is taught to choose a puncture site at the side of the finger pad. The other patient actions indicate that teaching has been effective.

A patient with cancer of the liver has severe ascites, and the health care provider plans a paracentesis to relieve the fluid pressure on the diaphragm. To prepare the patient for the procedure, the nurse a. asks the patient to empty the bladder. b. positions the patient on the right side. c. obtains informed consent for the procedure. d. assists the patient to lie flat in bed.

A Rationale: The patient should empty the bladder to decrease the risk of bladder perforation during the procedure. The patient would be positioned in Fowler's position and would not be able to lie flat without compromising breathing. The health care provider is responsible for obtaining informed consent.

While hospitalized and recovering from an episode of diabetic ketoacidosis, the patient calls the nurse and reports feeling anxious, nervous, and sweaty. Based on the patient's report, the nurse should a. obtain a glucose reading using a finger stick. b. administer 1 mg glucagon subcutaneously. c. have the patient eat a candy bar. d. have the patient drink 4 ounces of orange juice.

A Rationale: The patient's clinical manifestations are consistent with hypoglycemia and the initial action should be to check the patient's glucose with a finger stick or order a stat blood glucose. If the glucose is low, the patient should ingest a rapid-acting carbohydrate, such as orange juice. Glucagon might be given if the patient's symptoms become worse or if the patient is unconscious. Candy bars contain fat, which would slow down the absorption of sugar and delay the response to treatment.

When assessing the patient experiencing the onset of symptoms of type 1 diabetes, which question should the nurse ask? a. "Have you lost any weight lately?" b. "Do you crave fluids containing sugar?" c. "How long have you felt anorexic?" d. "Is your urine unusually dark-colored?"

A Rationale: Weight loss occurs because the body is no longer able to absorb glucose and starts to break down protein and fat for energy. The patient is thirsty but does not necessarily crave sugar- containing fluids. Increased appetite is a classic symptom of type 1 diabetes. With the classic symptom of polyuria, urine will be very dilute.

A client has recently been diagnosed with Type I diabetes and asks the nurse for help formulating a nutrition plan. Which of the following recommendations would the nurse make to help the client increase calorie consumption to offset absorption problems? a. Eating small meals with two or three snacks may be more helpful in maintaining blood glucose levels than three large meals. b. Eat small meals with two or three snacks throughout the day to keep blood glucose levels steady c. Increase consumption of simple carbohydrates d. Skip meals to help lose weight

A Eating small meals with two or three snacks may be more helpful in maintaining blood glucose levels than three large meals.

A patient in the outpatient clinic has positive serologic testing for anti-HCV. Which action by the nurse is appropriate? a. Schedule the patient for HCV genotype testing. b. Teach the patient that the HCV will resolve in 2 to 4 months. c. Administer immune globulin and the HCV vaccine. d. Instruct the patient on self-administration of -interferon.

A Rationale: Genotyping of HCV has an important role in managing treatment and is done before drug therapy with -interferon or other medications is started. HCV has a high percentage of conversion to the chronic state so the nurse should not teach the patient that the HCV will resolve in 2 to 4 months. Immune globulin or vaccine is not available for HCV.

A 75-year-old patient who is snoring loudly and having periods of apnea several times each night is most likely experiencing:

Sleep Apnea

After the home health nurse has taught a patient and family about how to use glargine and regular insulin safely, which action by the patient indicates that the teaching has been successful? a. The patient disposes of the open insulin vials after 4 weeks. b. The patient draws up the regular insulin in the syringe and then draws up the glargine. c. The patient stores extra vials of both types of insulin in the freezer until needed. d. The patient's family prefills the syringes weekly and stores them in the refrigerator.

A Rationale: Insulin can be stored at room temperature for 4 weeks. Glargine should not be mixed with other insulins or prefilled and stored. Freezing alters the insulin molecule and should not be done.

The nurse identifies the collaborative problem of potential complication: electrolyte imbalance for a patient with severe acute pancreatitis. Assessment findings that alert the nurse to electrolyte imbalances associated with acute pancreatitis include a. muscle twitching and finger numbness. b. paralytic ileus and abdominal distention. c. hypotension. d. hyperglycemia.

A Rationale: Muscle twitching and finger numbness indicate hypocalcemia, a potential complication of acute pancreatitis. The other data indicate other complications of acute pancreatitis but are not indicators of electrolyte imbalance.

A patient using a split mixed-dose insulin regimen asks the nurse about the use of intensive insulin therapy to achieve tighter glucose control. The nurse should teach the patient that a. intensive insulin therapy requires three or more injections a day in addition to an injection of a basal long-acting insulin. b. intensive insulin therapy is indicated only for newly diagnosed type 1 diabetics who have never experienced ketoacidosis. c. studies have shown that intensive insulin therapy is most effective in preventing the macrovascular complications characteristic of type 2 diabetes. d. an insulin pump provides the best glucose control and requires about the same amount of attention as intensive insulin therapy.

A Rationale: Patients using intensive insulin therapy must check their glucose level four to six times daily and administer insulin accordingly. A previous episode of ketoacidosis is not a contraindication for intensive insulin therapy. Intensive insulin therapy is not confined to type 2 diabetics and would prevent microvascular changes as well as macrovascular changes. Intensive insulin therapy and an insulin pump are comparable in glucose control.

A homeless patient with severe anorexia, fatigue, jaundice, and hepatomegaly is diagnosed with viral hepatitis and has just been admitted to the hospital. In planning care for the patient, the nurse assigns the highest priority to the patient outcome of a. maintaining adequate nutrition. b. establishing a stable home environment. c. increasing activity level. d. identifying the source of exposure to hepatitis.

A Rationale: The highest priority outcome is to maintain nutrition because adequate nutrition is needed for hepatocyte regeneration. Finding a home for the patient and identifying the source of the infection would be appropriate activities, but they do not have as high a priority as having adequate nutrition. Although the patient's activity level will be gradually increased, rest is indicated during the acute phase of hepatitis.

A nurse is caring for patients on a med-surg unit. The nurse plans the patient's care and instructs the nursing assistant to assist in repositioning patients every two hours. Which patients are the greatest risk for complications if not repositioned properly?

A 20-year old unconscious patient; A 90-year old frail patient; A 40-year old patient with paraplegia

Amitriptyline (Elavil) is prescribed for a diabetic patient with peripheral neuropathy who has burning foot pain occurring mostly at night. Which information should the nurse include when teaching the patient about the new medication? a. Amitriptyline will help prevent the transmission of pain impulses to the brain. b. Amitriptyline will improve sleep and make you less aware of nighttime pain. c. Amitriptyline will decrease the depression caused by the pain. d. Amitriptyline will correct some of the blood vessel changes that cause pain.

A Rationale: Tricyclic antidepressants decrease the transmission of pain impulses to the spinal cord and brain. Tricyclics also improve sleep quality and are used for depression, but that is not the major purpose for their use in diabetic neuropathy. The blood vessel changes that contribute to neuropathy are not affected by tricyclics.

When a client learned that the symptoms of diabetes were caused by high levels of blood glucose the client decided to stop eating carbohydrates. In this instance, the nurse would be concerned that the client would develop what complication? a. acidosis b. atherosclerosis c. glycosuria d. retinopathy

A When a client's carbohydrate consumption is inadequate ketones are produced from the breakdown of fat. These ketones lower the pH of the blood, potentially causing acidosis that can lead to a diabetic coma.

Which of the following patients is at highest risk of developing dehydration?

A 78-year old patient with dementia

The nurse is instructing the unlicensed assistant on how to care for a client with chest tubes that are connected to water seal drainage. Which of the following instruction would be appropriate for the nurse to give the unlicensed assistant? A) Mark the time and amount of drainage collected in the container B) Raise the collection apparatus to the height of the bed to measure the fluid level. C) Milk the test tubes every 4 hours D) Attach the chest tubes to bed linen to avoid tension of the tubing

A = It is appropriate for an unlicensed assistant to mark the time of measurement and fluid level in the collection container.

A male client is receiving chemotherapy for lung cancer. He asks the nurse how the drug will work. Which of the following is the correct response of the nurse? A) "Chemotherapy affects all rapidly dividing cells." B) "Structure of the DNA is altered." C) "Chemotherapy encourages cancer cells to divide." D) "Cancer cells have susceptible drug toxins."

A = There are many mechanisms of action for chemotherapeutic agents, but most affect the rapidly dividing cells-both cancerous and noncancerous. Cancer cells are characterized by rapid cell division. Chemotherapy slows cell division

The nurse is preparing a discharge plan to a female client with peptic ulcer for the dietary modification she will need to follow at home. Which of the following statements indicates that the client understands the instruction of the nurse? A) "I should not drink alcohol and caffeine." B) "I should eat a bland, soft diet." C) "It is important to eat six small meals a day." D) "I should drink several glasses of milk a day."

A = caffeinated beverages and alcohol should be avoided because they stimulate gastric acid production and irritate gastric mucosa. The client should avoid foods that cause discomfort; however, there is no need to follow a soft, bland diet. Eating six small meals daily is no longer a common treatment for peptic ulcer disease. Milk in large quantities is not recommended because it actually stimulates further production of gastric acids.

A community health nurse is teaching smoking cessation program to a group of healthy adult smokers. What type of prevention activity is this? A) Primary B) Secondary C) Tertiary D) None of the above

A = primary cancer prevention targets healthy individuals and includes steps to avoid factors that might lead to the development of diseases.

D. Consuming a low-carbohydrate, high protein diet and avoiding fasting

A nurse is providing dietary instructions to a male client with hypoglycemia. To control hypoglycemic episodes, the nurse should recommend: A. increasing saturated fat intake and fasting in the afternoon B. Increasing intake of vitamins B and D and taking iron supplements C. Eating a candy bar if light-headedness occurs D. Consuming a low-carbohydrate, high protein diet and avoiding fasting

A 60 year old patient has an abrupt onset of anorexia, nausea and vomiting, hepatomegaly, and abnormal liver function studies. Serologic testing is negative for viral causes of hepatitis. During assessment of the patient, it is most important for the nurse to question the patient regarding A. any prior exposure to people with jaundice B. the use of all prescription and OTC (over the counter) medications C. treatment of chronic diseases with corticosteriods D. exposure to children recently immunized for hepatitis B

A and D assess for exposure to hepatitis. Hepatitis was ruled out this is inappropriate. C is incorrect because corticosteroids do not commonly cause liver disease B is correct because overdose of medications can cause liver disease.

B. A recent injury

A client arrives at the clinic complaining of knee pain. On assessment the nurse notes that the knee area is swollen. The nurse interprets that the client signs and symptoms likely indicates. A. Osteoporosis B. A recent injury C. Rheumatoid Arthritis D. Degenerative Joint Disease

D. Transport

A client has lost a considerable amount of blood and is demonstrating signs of pending shock. Which of the following shock states is this client most likely developing? A. Cardiogenic B. Obstructive C. Hypovolemic D. Transport

B. Transport

A client has lost a considerable amount of blood and is demonstrating signs of pending shock. Which of the following shock states is this client most likely developing? A. Obstructive B. Transport C. Hypovolemic D. Cardiogenic

C. Impaired tissue perfusion

A client has sustained a closed fracture and has just had a cast applied to the affected arm. The client is complaining of intense pain. The nurse elevates the limb, applies an ice bag, and administers analgesic, with little relief. The nurse interprets that this pain may be caused by: A. Infection under the cast B. The anxiety of the patient C. Impaired tissue perfusion D. The recent occurrence of the fracture

B. Dopamine

A client in shock is prescribed an inotropic drug to act on alpha and beta receptors. The nurse will most likely be administering A. Dobutamine B. Dopamine C. Pavulon D. Milrinone

D. It is a fracture in which parts of the bone are broken into small pieces

A client is admitted after an automobile accident, and the X-ray shows she has a comminuted fracture of the right tibia. Which of the following describes a comminuted fracture? A. A fracture has not caused the skin to be disrupted. B. It occurs when the broken sections of the bone are not in alignment with one another. C. The break did not go completely through the bone D. It is a fracture in which parts of the bone are broken into small pieces

D. Hypovolemic

A client is admitted to the ICU with a spinal cord injury from a motor vehicle accident. Which shock is this client most at risk for developing? A. Cardiogenic B. Obstructive C. Transport D. Hypovolemic

D. Limit the drug to 5 days of use to prevent rebound nasal congestion.

A client is prescribed the decongestant oxymetazoline (Afrin) nasal spray. What should the nurse teach the client? A. Take this drug at bedtime as a sleep aid? B. Directly spray away from the nasal septum and gently sniff. C. This drug may be used in maintenance treatment for asthma D. Limit the drug to 5 days of use to prevent rebound nasal congestion.

B. Avoid driving a motor vehicle until stabilized on the drug.

A client tells the nurse that he has started to take an OTC antihistamine, diphenhydramine. In teaching him about side effects, what is most important for the nurse to tell the client? A. Do not take this drug at bedtime to avoid insomnia. B. Avoid driving a motor vehicle until stabilized on the drug. C. Nightmares and nervousness are more likely in an adult. D. Limit use to 1 or 2 puffs/sprays 4-6 times per day to avoid rebound congestion.

The nurse understands that a collection of various information technology applications that provides a centralized repository of information related to client care across distributed locations is called:

A clinical information system (CIS) rationale: A clinical information system (CIS) is a repository that also encodes the status of decisions, actions underway for those decisions, and relevant information that can help in performing those actions. The database could also hold other information about the client, including genetic, environmental, and social contexts. Essentially, the CIS uses the computer to provide and store information and data about a client from departments that are client-focused or departmental-focused.

Which of the following best describes a consent form?

A consent form may be signed by an emancipated minor, and consent may be obtained by fax or phone with appropriate witnesses. Only in cases of underage children or unconscious or mentally incompetent people must a family member be aware.

B. an x-ray showing joint erosion of the wrists.

A diagnostic test result that would be indicative of rheumatoid arthritis would be: A. a bone density scan revealing increased porosity of bone B. an x-ray showing joint erosion of the wrists. C. Rheumatoid factor in the urine D. bacteria in the blood.

What is meant by the term integrated health care delivery system (IHS)?

A large health care system that can provide all needed services under one corporate umbrella rationale: As part of the corporatization of health care, the individual health care organizations are merging into systems to provide all needed services under one corporate umbrella. These are known as integrated health care delivery systems.

C. "You may not be able to use desmopressin nasally if you have nasal discharge or blockage."

A male client with primary diabetes insipidus is ready for discharge on desmopressin (DDAVP). Which instruction should nurse Lina provide? A. "Administer desmopressin while the suspension is cold." B. "Your condition isn't chonic, so you won't need to wear a medical identification bracelet." C. "You may not be able to use desmopressin nasally if you have nasal discharge or blockage." D. "You won't need to monitor your fluid intake and output after you start taking desmopressin."

Which of the following changes in vital signs would most strongly suggest cardiac tamponade?

A narrowing pulse pressure

2. Keep small toys and sharp objects away from the cast 5. Contact the health care provider if the child complains of numbness or tingling in the extremity 6. Elevate the extremity on pillows for the first 24 to 48 hours after casting to prevent swelling.

A nurse prepares a list of home care instructions for the parents of a child who has a plaster cast applied to the left forearm. Choose the instructions that would be included on the list. Select all that apply. 1. use the fingertips to lift the cast while it is drying 2. Keep small toys and sharp objects away from the cast 3. Use a padded ruler or another padded object to scratch the skin under the cast if it itches 4. Place a heating pad on the lower end of the cast and over the fingers, if the fingers feel cold 5. Contact the health care provider if the child complains of numbness or tingling in the extremity 6. Elevate the extremity on pillows for the first 24 to 48 hours after casting to prevent swelling.

Patient-focused care is a recently developed term to describe:

A nursing care delivery system rationale: Patient-focused care is another type of delivery system. The level of productivity is increased through centralization of functions (phlebotomy, diet instruction, respiratory therapy) on the unit under the direction of the RN and cross-training ancillary workers to do more than one function. The patient comes into contact with fewer people under this organizational framework.

C. "Rheumatoid arthritis affects three times as many women as men, and an onset date usually between 20 and 40 years of age."

A nursing student is attempting to understand the difference between osteoarthritis and rheumatoid arthritis. Which of the following statements, if made by the nursing student demonstrates a good understanding of the two disorders? A. "Rheumatoid arthritis (RA) is an acute systemic autoimmune disease that causes inflammation of connective tissue, primarily in the joints." B. "Osteoarthritis is the least common type of arthritis." C. "Rheumatoid arthritis affects three times as many women as men, and an onset date usually between 20 and 40 years of age." D. "Women are affected more than men at an earlier age, but the rate of osteoarthritis (OA) in men exceeds women by the middle adult years."

C. "Rheumatoid arthritis affects three times as many women as men, and an onset date usually between 20 and 40 years of age."

A nursing student is attempting to understand the difference between osteoarthritis and rheumatoid arthritis. Which of the following statements, if made by the nursing student, demonstrates a good understanding of the two disorders? A. " Rheumatoid arthritis (RA) is an acute systemic autoimmune disease that causes inflammation of connective tissue, primarily in the joints." B. "Osteoarthritis is the least common type of arthritis." C. "Rheumatoid arthritis affects three times as many women as men, and an onset date usually between 20 and 40 years of age." D. "Women are affected more than men at an earlier age, but the rate of OA in men exceeds women by the middle adult years."

A. The symptoms may be the result of anemia caused by chemotherapy.

A patient received surgery and chemotherapy for colon cancer, completing therapy 3 months previously, and she is now in remission. At a follow-up appointment, she complains of fatigue following activity and difficulty with concentration at her weekly complaints of fatigue following activity and difficulty with concentration at her weekly bridge games. Which of the following explanations could account for her symptoms? A. The symptoms may be the result of anemia caused by chemotherapy. B. The patient may be immunosuppressed. C. The patient may be depressed. D. The patient may be dehydrated.

The technique used to identify the factors involved in an error is called:

A root cause analysis rationale: When an error is analyzed, the primary causes need to be determined so that a workable and effective solution can be developed. A root cause analysis is such a process designed to investigate and categorize the root cause of the event.

328 Nurse initiating seizure precautions. Which of following items s/nurse place at bedside?

A suction apparatus and oxygen. R: Seizures cause a tightening of all body muscles that is followed by tremors. An obstructed airway and increased oral secretions are major complications during and after the seizure. Suctioning and oxygen is helpful to prevent choking and cyanosis.

A community health nurse is testing the theory of locus of control (LOC). Which of the following client's demonstrates the internal control concept of this theory: a) A client who takes an active role in all health decisions b) A client who allows the primary care provider to make all the decisions c) A client who does not make any decisions without his/her souse's input d) A client who relies on information from the local hospital for his.her health needs

A) A client who takes an active role in all health decisions

On one of the first days working alone, the novice nurse must provide teaching on tracheotomy care to the client as well as the client's spouse. This nurse is not familiar with the teaching aspect. The best action for the nurse is to: a) ASk the nurse mentor to assist with the teaching after reviewing the procedure b) Read the policy and procedure manual before the teaching session c) Do the best the nurse can by remembering what was taught in nursing school d) ASk for a different assignment until the nurse feels comfortable with this one

A) ASk the nurse mentor to assist with the teaching after reviewing the procedure

Which nursing action will be included when the nurse is doing a wet-to-dry dressing change for a patient's Stage III sacrel pressure ulcer: a) Administer the ordered PRN oral opoid 30 min before the dressing change b) Soak the old dressing with sterile saline a few minutes before removing them

A) Administer the ordered PRN oral opiod 30 min before the dressing change

The nurse is reviewing laboratory data for a patient who is receiving total parental nutrition. Which lab value should be immediately brought to the physicians attention: a) BUN of 60 c) Serum glucose 328 d) Potassium of 3.5

A) BUN of 60

Outcome statement is: a) Client will ambulate without a walker by 6 weeks b) Client will ambulate freely in house c) Client will not fall

A) Client will ambulate without walker by 6 weeks

The nursing action most appropriate for a client who has an infection and develops a fever of 99.8' F is to: a) Continue to monitor the patient's temp b) Administer an antipyretic

A) Continue to monitor the patient's temp

A student nurse who claims to be very uncreative and dose not understand why it is necessary to assess and develop new ideas in the clinical area. The best response by the nurse educator is: a) Creativity allows unique solutions to unique problems b) Not all your answers are going to be from your textbook c) Creativity makes nursing fun d) You'll get bored if you don't learn to be creative

A) Creativity allows unique solutions to unique problems

A client has an open wound that is yellow and black. Using the RYB color code, which nursing intervention needs to occur first? a) Debride the area with wet-to-dry dressing b) Apply topical antibiotic ointment

A) Debride the area with wet-to-dry dressing

A client recovering from abdominal surgery refuses analgesia, saying that he is "fine, as long as he dosen't move." Which nursing diagnosis should be a priority: A) Deficient Knowledge (pain control measures) b) Ineffective Health Maintenance

A) Deficient Knowledge (pain control measures)

A nursing activity that is carried out during the evaluation phase of the nursing process is: a) Determining if interventions have been effective in meeting patient's outcome b) Documenting the nursing care plan in the progress notes in the medical record c) Deciding whether the patient's health problems have been completely resolved d) Asking the patient to evaluate whether the nursing care provided was satifactory

A) Determining if interventions have been effective in meeting patient's outcomes

A patient complains of pain during circumfusion of the shoulder when the nurse moves the arm behind the patient which question should the nurse ask: a) Do you have difficulty in putting on a jacket b) Are you able to feed yourself without difficulty

A) Do you have difficulty in putting on a jacket

During a well-child visit, a mother tells the nurse that her 4- year old daughter typically goes to bed at 10:30 pm and awakens each morning at 7 am. She does not take a napin the afternoon. Which is the best response by the nurse: a) encourage the mother to consider putting her daughter to bed between 8 and 9 pm d) Reassure her that her daughter's sleep pattern is normal and that she has outgrown her need for an afternoon nap

A) Encourage the mother to consider putting her daughter to bed between 8 and 9 pm

The mother of a 1 month old infant is concerned because the infant has had vomiting and diarrhea for 2 days. What instructions should the nurse give this infants mother: a) Have the infant be seen by a physician b) Give the infant at least 2 ounces of juice every 2 hours

A) Have the infant be seen by a physician

The telemetry unit nurse is reviewing laboratory results for a patient who is scheduled for an operative procedure later in the day. The nurse notes on the laboratory report that the patient has a serum potassium level of 6.5 mEq/L, indicative of hyperkalemia. The nurse informs the physician of this laboratory result because the nurse recognizes hyperkalemia increases the patient's operative risk for: A. Cardiac problems B. Bleeding with anemia C. Fluid imbalances D. Infection

A) Hyper/hypokalemia increases the patient's risk for cardiac problems. A decrease in the hematocrit and hemoglobin level may indicate the presence of anemia or bleeding. An elevated WBC occurs in the presence of infection. Abnormal urine constituents may indicate infection or fluid imbalances.

The patient has been admitted with complaints of shortness of breath for 2 week duration and has received the nursing diagnosis impaired gas exchange. Which admission laboratory result would support the choice of this diagnosis: a) Increased hematocrit b) Decreased BUN

A) Increased Hematocrit

The nursing process is a dynamic process. This means that it: a) Is ever changing to the client's needs b) Conveys the force or power of the health team

A) Is ever changing in response to the client's needs

The nurse anticipates that osteoposis may result from prolonged immobilization because of: a) Lack of weight bearing, which decreases osteoblastic activity b) Decreased dietary calcium intake

A) Lack of weight bearing, which decreases osteoblastic activity

A patient has the following arterial blood gas (ABG) results: ph 7.32, PAO2 88 mmHg, PaCO@ and HCO3 16 mEqL. The nurse interprets these results as: a) Metabolic acidosis b) Metabolic alkalosis c) Respiratory acidosis d) Respiratory alkalosis

A) Metabolic Acidosis

At SAM, a nurse checks the amount of solution left in a potential nutrition infusion bag for an assigned client. It is a 3000 mL bag with 1000 mL remaining. The solution is running at a rate of 100 mL/hr. The bag was hung the previous day at noon. The nurse plans to change the infusion bag and tubing today at: a) Noon b) 2 pm

A) Noon

A patient with an open abdominal wound has a complete blood cell (CBC) count and differential, which indicate an increase in white blood cells (WBC)s and a shift to the left. The nurse anticipates that the next action will be to: a) Obtain wound cultures b) start antibiotic c) Reddress the wound with wet-to-dry dressing d) Continue to monitor the wound for purulent drainage

A) Obtain wound cultures

An 85 year old client has impaired hearing. When creating the care plan which intervention should have the highest priority: a) Obtaining an amplified telephone b) Teaching the importance of changing his position

A) Obtaining an amplified telephone

Which of these patients in the clinic will the nurse plan to teach about risks associated with obesity: a) Patient who has a BMI of 18 kg/m2 b) Patient with a waist circumference 34 inches (86 cm) d) patient whose waist measures 30 in. (75 cm) and hips measure 34 in. *85 cm)

A) Patient who has a BMI of 18 kg/m2

Upon assessment, a patient reports that he drinks 5-6 bottles of beer every evening after work. Based upon this information, the nurse is aware that the patient may require: A. Larger doses of anesthetic agents and larger doses of postoperative analgesics. B. Larger doses of anesthetic agents and lower doses of postoperative analgesics. C. Lower doses of anesthetic agents and larger doses of postoperative analgesics. D. Lower doses of anesthetics agents and lower doses of postoperative analgesics.

A) Patients with a larger habitual intake of alcohol require larger doses of anesthetic agents and postoperative analgesics, increasing the risk for drug-related complications.

A patient in the hospital has a history of functional urinary incontinence. Which nursing action will be included in the plan of care: a) Place a bedside commode near the patient's bed b) Demonstrate the use of the Crede maneuver to the patient

A) Place a bedside commode near the patient's bed

Assessment: Restlessness. Weak, rapid pulse. Hypotension. Tachypnea. Cool, clammy skin. Reduced urine output.

Hemorrhage Shock

The edges of a patient's appendectomy incision are approximated, and no drainage is noted. The nurse documents on the client's wound record that the incision appears to be healing by: a) Primary intention b) Secondary intention

A) Primary Intention

Which of these nursing actions included in the plan of care for a patient who is receiving intermittent tube feedings through a percutaneous endoscopic gastrostomy )PEG) tube may be delegated to an LPN/LVN: a) Providing skin care to the area around the tube site b) Assessing the patient's nutritional status at least weekly

A) Providing skin care to the area around the tube site

While changing a patient's dressing the nurse notes thick yellow-green drainage on the gauze. How should the nurse document this wounds drainage: a) Purulent b) Serous

A) Purulent

After completing a scheduled every 2-hour turn by turning the patient to the left side, the nurse notices a reddened are over the coccyx. The area blanches when the nurse compresses it with thumb pressure. One hour later, the nurse reassesses the area and finds the redness has disappeared. How should the nurse document this area: a) Reactive hyperemia c) Stage II pressure ulcer d) Stage III pressure ulcer

A) Reactive Hyperemia

The nurse is caring for an 80 year old female nursing home resident who has been admitted to the hospital with pneumonia and is becoming progressively more confused. Her vital signs are: Temp 101' F, Pulse 112, Resp. 28 and BP 100/70. ABG results include pH 7.50, PaCO@ 25 mmHg, and bicarbonate level 18 mEq/L. The nurse interprets these findings to indicate: a) Respiratory acidosis secondary to hypoexmia b) Respiratory acidosis secondary to anxiety

A) Respiratory acidosis secondary to hypoeximia

The nurse is developing a plan of care for a client with a newly created ileostomy. The priority nursing diagnosis for this client is: a) Risk for deficient fluid volume related to excessive fluid loss from ostomy b) Disturbed body image related to presence of ostomy

A) Risk for Deficient Fluid Volume related to Excessive Fluid loss from Ostomy

A patient who has just been started on continuous tube feedings of a full strength commercial formula at 100 mL/hr using a closed system method has six diarrhea stools the first day. What action should the nurse plan to take: a) Slow the infusion rate of the tube feeding b) Check the gastric residual volumes more frequently c) Change the internal feeding system and formula every 8 hrs d) Discontinue administration of water through the feeding tube

A) Slow the infusion rate of the tube feeding

When asked to sign the permission form for surgical removal of a large but noncancerous lesion on her face, the client begins to cry. Which of the following is the most appropriate response: a) Tell me what it means to you to have surgery b) you must be very glad to be having this lesion removed

A) Tell me what it means to you to have surgery

The nurse obtains all of the following assessment data about a patient with deficient fluid volume caused by a massive burn injury. Which of the following assessment data will be of greatest concern: a) The BP is 90/40 mm/Hg c) Oral fluid intake is 100 mL for the last 8 hours d) There is prolonged skin tenting over the sternum

A) The BP is 90/40 mm/Hg

A nurse is practicing the concept of holism to the client. Which of the following is the best example of this: a) The nurse considers how the loss of a client's job will affect the regulation of the client's diabetes b) The nurse makes sure to do a complete teaching regarding pharmacological interventions c) The nurse is careful to follow physician treatments on schedule d) The nurse is able to prioritize the needs of the client assigned according to Maslow's hierarchy

A) The nurse consider's how the loss of a client's job will affect the regulation of the client's diabetes

Which outcome is appropriate for the client problem "ineffective gas exchange" for the client recently diagnosed with COPD: a) The patient demonstrates the correct way to pursed lip breathe b) The client lists three signs/symptoms to report to the Health Care provider

A) The patient demonstrates the correct way to pursed lip breathe

The nurse case manager is concerned about A particular client being discharged from the hospital. Which of the following factors, if present for this client, would alert the nurse to possible problems with treatment adhearance: a) The prescribed therapy is costly and of unknown duration b) The therapy will require no lifestyle changes of the client c) The client has not had difficulty understanding the regimen d) The client's culture is supportive of Western medicine

A) The prescribed therapy is costly and of unknown duration

The nurse is providing teaching to a patient regarding pain control after surgery. The nurse informs the patient that the best time to request pain medication is: A. Before the pain becomes severe. B. When the patient experiences a pain rating of 10 on a 1-to-10 pain scale. C. After the pain becomes severe and relaxation techniques have failed. D. When there is no pain, but it is time for the medication to be administered.

A) The question states that the patient is being instructed on when to "request" pain medication. If a pain medication is ordered PRN, the patient should be instructed to ask for the medication before the pain becomes severe.

The nurse is preparing to send a patient to the operating room for an exploratory laparoscopy. The nurse recognizes that there is no informed consent for the procedure on the patient's chart. The nurse informs the physician who is performing the procedure. The physician asks the nurse to obtain the informed consent signature from the patient. The nurse's best action to the physician's request is to: A. Inform the physician that is his responsibility to obtain the signature. B. Obtain the signature and ask another nurse to co-sign the signature. C. Inform the physician that the nurse manager will need to obtain the signature.

A) The responsibility for securing informed consent from the patient lies with the person who will perform the procedure. The nurse's best action is to inform the physician that it is his responsibility to obtain the signature.

A client who has just been diagnosed with pancreatic cancer is quite upset and verbal. The nurse has the following diagnoses: anxiety related to unfamiliarity of disease process, manifested by restlessness tachycardia. The etiology of this diagnoses is which of the following: a) Unfamiliarity of disease process b) Anxiety c) Restlessness d) Tachycardia

A) Unfamiliarity of disease process

A client has a serum sodium concentration of 160 mEq/L and exhibits generalized weakness and confusion. The nurse should plan to initiate: a) Fluid restrictions c) Monitoring of urine specific gravity d) Seizure precautions

A) fluid restrictions

A client who is started on metformin and glyburide would have initially present with with symptoms? A. Polydipsia, polyuria, and weight loss B. Weight gain, tiredness, and bradycardia C. irritability, diaphoresis and tachycardia D. Diarrhea, abdominal pain and weight loss

A) symptoms of hyperglycemia included polydispia, polyuria, and weightloss. Metformin and sulfonylureas are commonly ordered medications. Weight gain, tiredness, and bradycardia are symptoms of hypothyroidism. Irritability, diaphoresis, and tachycardia are symptoms of hypoglycemia. Symptoms of Crohn's disease include diarrhea, abdominal pain, and weight loss.

Which of the following would be most important for a nurse to do to ensure the accuracy of inspection during assessment? A)Compare bilateral body parts B)Have 20/20 vision C)Focus on selected body systems D)Use touch judiciously

A)Compare bilateral body Parts With inspection, a comparison of bilateral body parts is necessary for recognizing abnormal findings

The nurse is assessing a "PT" to determine the "PT" adjustment to presbycusis. Which is the following indicates successful adaptation to this problem? A)Proper use of a hearing aid B)Denial of a hearing impairment C)Withdraw from social actives D)Reluctance to answer the telephone

A)Proper use of a hearing aid Presbycusis occurs as part of the aging process; its a progressive sensorineural hearing loss. Some "PT" may not adept well to the hearing loss, denying its presence. Others withdraw from social actives and contact with others.

When using an otoscope to assess the tympanic membrane of an adult, the nurse straightens the ear canal by gently pulling the pinna in which direction? A)Up & Back B)Down & Froward C)Away from the examiner D)In any direction

A)Up & Back The ear canal of an adult is straightened by gently pulling the pinna of the ear up and back. In children the ear is pulled back and down

Select all that apply. Advantages of laser surgery include diminished A. bleeding. B. swelling. C. tissue damage. D. postoperative pain. E. postoperative infection.

A, B, C, D, & E (All of the above) Laser surgery offers the benefits of diminished bleeding, swelling, tissue damage, and postoperative pain and infection.

Which of the following are the best argument(s) for supporting community-as-client nursing (select all that apply)? A) Change for the benefit of the community-client must often occur at several levels. B) Changes in the health of individuals will affect the health of their communities. C) The idea of providing health-related care within the community is not new. D) The impact of the environment on health has long been established. E) Direct hands-on nursing care delivered to individuals or families in community settings is important.

A,B,C,D

Factors related to the determinants of health identified in Healthy People 2020 include which of the following (select all that apply)? A) Education and literacy. B) Genetic endowment. C) Gender. D) Culture. E) Social status.

A,B,C,D,E

The role of the nurse who wants to become more active in environmental health could include which of the following (select all that apply)? A) Assessing farmworkers for pesticide exposure and providing pesticide risk education. B) Conducting epidemiologic investigations as a public health nurse. C) Developing corporate policy to protect workers from unsafe levels of toxic agents. D) Organizing the local community to encourage landlords to remove lead-based paint. E) Working as a skilled risk communicator for a local chemical manufacturer.

A,B,C,D,E

The Quad Council of Public Health Nursing identified eight principles that distinguish the public health nursing specialty from other nursing specialties, including which of the following (select all that apply)? A) Collaboration with other professions, organizations, and entities. B) Optimal use of available resources. C) Population-based unit of care. D) Primary obligation to work for the good of individuals and families. E) Engagement with clients as an equal partner.

A,B,C,E

A community health nurse is revising the agency's nursing protocols to incorporate current evidence-based practice (EBP) clinical practice guidelines. Common barriers to EBP implementation that could be faced include which of the following (select all that apply)? A) Disempowerment of nurses in their ability to make clinical decisions. B) Experienced nurses' challenging of the need to change long-accepted practices. C) Lack of knowledge of how to conduct a systematic review of the research literature. D) Unwillingness of clients to accept changes in familiar agency programs. E) Urban agency setting with restricted computer resources.

A,B,D,E

Emerging infectious diseases may arise as a result of factors operating singly or in combination, and these factors may include which of the following (select all that apply)? A) Environmental changes. B) Host behavior. C) Improved surveillance. D) Microbial adaptation. E) Public health infrastructure deterioration.

A,B,D,E

A client diagnosed with human papillomavirus (HPV) infection states, "I'm not concerned, I know the warts disappear after a while." The nurse should counsel the client regarding which of the following? Select all that apply. A) Link between HPV and cervical cancer. B) Status of HPV infection as a reportable disease. C) Need to eliminate the warts. D) Serious complications of HPV infection for men. E) Lack of cure for HPV infection.

A,C,E

A pt has returned from surgery with a tracheostomy tube in place. After about 10 minutes in postoperative recovery, the pt begins to have noisy, increased respirations and an elevated heart rate. What action should the RN take immediately? A. Suction the tracheostomy. B. Readjust the tracheostomy tube and tighten the ties. C. Preform a complete respiratory assessment.

A. Noisy, increased respiration & increased pulse are signs that the pt needs immediate suctioning to clear the airway of secretions. A complete respiratory assessment may then be completed.

The reason pts are sent to a PACU after surgery is: A. to be monitored while recovering from anesthesia. B. to remain near the surgeon immediately after surgery. C. to allow the medical-surgical unit time to prepare for transfer. D. to provide time for the pt to cope with the effects of surgery.

A. Pts are sent to a PACU to be monitored while they're recovering from anesthesia.

The nurse is most likely to collect timely, specific information by asking which of the following questions? A. "Would you describe what you are feeling?" B. "How are you today?" C. "What would you like to talk about?" D. "Where does it hurt?"

A. "Would you describe what you are feeling?" Rationale: This is an open-ended question that will elicit subjective data. The data collected will reflect the client's current health status and human response(s) and should generate specific information that can be used to identify actual and/or potential health problems. Options 2 and 3 are more likely to elicit general, nonspecific information. Option 4 may result in a brief, one-word response or nonverbal gesture indicating the site of the client's pain. A better approach to collect specific information might be, "Describe any pain you are having."

Select all that apply. A nurse is caring for patients on a medical-surgical unit. The nurse plans the patients' care and instructs the nursing assistant to assist in repositioning patients every 2 hours. Which patients are at the greatest risk for complications if not repositioned properly? A. A 20-year-old unconscious patient B. A 90-year-old frail patient C. A 65-year-old patient who is visually impaired D. A 40-year-old patient who has paraplegia

A. A 20-year-old unconscious patient B. A 90-year-old frail patient (&) D. A 40-year-old patient who has paraplegia Patients who are at the greatest risk for complications if not properly repositioned are those who are unconscious, frail, or paralyzed.

A nurse is reviewing the history and physical of a teenager admitted to a hospital with a diagnosis of ulcerative colitis. Based on this diagnosis, which information should the nurse expect to see on this client's medical record? A. Abdominal pain and bloody diarrhea. B. Weight gain and elevated blood glucose. C. Abdominal distention and hypoactive bowel sounds. D. Heartburn and regurgitation.

A. Abdominal pain and bloody diarrhea

In the operating room, a patient tells a circulating nurse that he is going to have the cataract in his left eye removed. If the nurse notes that the consent form indicates that surgery is to be performed on the right eye, what should be the nurse's first action? A. Ask the patient his name. B. Notify the surgeon and anesthesiologist. C. Check to see whether the patient has received any preoperative medications. D. Assume that the patient is a little confused because he is older and has received midazolam intramuscularly.

A. Ask the patient his name. Ensuring proper identification of a patient is a responsibility of all members of the surgical team. In a specialty surgical setting where many patients undergo the same type of surgery each day, such as cataract removal, it is possible that the patient and the record do not match. Nurses do not assume in the care of their patients. The priority is with the nurse identifying the patient and the patient's consent form before the physicians are notified.

The nurse is admitting a patient to the same day surgery unit. The patient tells the nurse that he was so nervous he had to take kava last evening to help him sleep. Which of the following nursing actions would be most appropriate? A. Inform the anesthesiologist of the patient's ingestion of kava. B. Tell the patient that using kava to help sleep was a good idea. C. Tell the patient that the kava should continue to help him relax before surgery. D. Inform the patient about the dangers of taking herbal medicines without consulting his health care provider.

A. Inform the anesthesiologist of the patient's ingestion of kava. Kava may prolong the effects of certain anesthetics. Thus the anesthesiologist needs to be informed of recent ingestion of this herbal supplement.

Immunizations Birth

Hep B (1 of 3) usually before hospital discharge **If mom HBsAg-positive, administer immunoglobulin and vaccine within 12 hours of birth

The nurse decides it would be beneficial to the client to allow the client's infant granddaughter to visit before the client's scheduled heart transplant. Before implementing this intervention the nurse should collaborate with which of the following? Select all that apply. A. Client and Family B. Other nursing staff on the unit C. Security department D. Hospital administration E. This is not a collaborative intervention so no collaboration will be needed prior to implementation

A. Client and Family B. Other nursing staff on the unit Rationale: Collaboration with the client and family will encourage a sense of autonomy and active involvement in the healthcare process for the client. In this case collaboration with other nursing staff will ensure the successful implementation of the planned intervention. There is no real need for collaboration with hospital administration or the security department in this situation although the nurse should be aware of her responsibility to collaborate at those levels when the situation demands it.

The nurse would make which of the following inferences after performing the appropriate client assessment? A. Client is hypotensive B. Respiratory rate of 20 breaths per minute C. Oxygen saturation of 95% D. Client relays anxiety about blood work

A. Client is hypotensive Rationale: An inference is the nurse's judgment or interpretation of cues such as judging a blood pressure to be lower than normal. A cue is any piece of data information that influences a decision. Options 2, 3, and 4 are cues that could lead to inferences.

Which of the following preoperative assessment findings should be reported to a surgeon for preoperative treatment? A. Excessive thirst B. Gradual weight gain C. Overwhelming fatigue D. Recurrent blurred vision

A. Excessive thirst. The classic clinical manifestations of diabetes mellitus are increased frequency of urination (polyuria); increased thirst and fluid intake (polydipsia); and as the disease progresses, weight loss despite increased hunger and food intake (polyphagia). Weakness, fatigue, and recurrent blurred vision are associated with diabetes mellitus but are not considered priority manifestations because of the generalization of these complaints being applied to other disease processes. Weight loss is the cardinal sign related to the depletion of water, glycogen, and triglyceride stores.

The use of percutaneous electrical stimulation as an effective means to control pain is based on which of the following? a. Gate-control theory b. Concept of therapeutic touch c. idea of using distraction d. theory of using heat application

A. Gate-control theory

The nurse notes that the client often sighs and says in a monotone voice, "I'm never going to get over this." When encouraged to participate in care, the client says, "I don't have the energy." The nurse believes these cues are suggestive of which nursing diagnoses? Select all that apply. A. Hopelessness B. Powerlessness C. Interrupted sleep pattern D. Disturbed self esteem E. Self care deficit

A. Hopelessness B. Powerlessness Rationale: Rationale: A nursing diagnosis is a clinical judgment about a response to an actual or potential health problem. This client is manifesting symptoms of both hopelessness and powerlessness. Although the client does report symptoms compatible with fatigue, there is no direct data is given that indicates the client has interrupted sleep patterns (option 3), disturbed self esteem (option 4), or self care deficit (option 5).

The nurse suspects that a client is withholding health-related information out of fear of discovery and possible legal problems. The nurse formulates nursing diagnoses for the client carefully, being concerned about a diagnostic error resulting from which of the following? A. Incomplete data B. Generalize from experience C. Identifying with the client D. Lack of clinical experience

A. Incomplete data Rationale: To collect data accurately, the client must actively participate. Incomplete data can lead to inappropriate nursing diagnosis and planning. The other options are not relevant to the question as presented.

Select all that apply. A nurse is caring for a surgical patient in the preoperative area. The nurse obtains the patient's informed consent for the surgical procedure. Which statements are true regarding informed consent? A. Informed consent must be signed while the patient is free from mind-altering medications. B. Informed consent must be witnessed. C. Informed consent may be withdrawn at any time. D. Informed consent must be signed by patients age 16 and older. E. Informed consent must be obtained by the physician. F. Informed consent must be obtained from the family even in a life-threatening emergency.

A. Informed consent must be signed while the patient is free from mind-altering medications. B. Informed consent must be witnessed. An informed consent must be signed while the patient is free from mind-altering medications and must be witnessed after it has been determined that the patient has received all of the necessary information needed to make an informed decision. An informed consent may be withdrawn at any time before the procedure and must be signed by patients age 18 and older. A parent or guardian's signature is required for minors. The informed consent may be obtained by the physician or the nurse and is not required in the event of a life-threatening emergency.

During which part of the client interview would it be best for the nurse to ask, "What's the weather forecast for today?" A. Introduction B. Body C. Closing D. Orientation

A. Introduction Rationale: Asking about the weather initiates the social or introductory phase of the interview and allows the nurse to begin an assessment of the client's mental status. The goal is to develop rapport with the client at the beginning of the interview. In the body the client responds to the nurse's questions. During the closing the nurse or the client terminates the interview.

Select all that apply. Which of the following best describes a consent form? A. May be signed by an emancipated minor. B. Protects the health care facility but not the physician C. Signifies that the patient understands all aspects of the procedure. D. Signifies that the patient and family have been told about the procedure E. Must be signed by the patient or responsible party at the health care facility, and that consent may not be obtained by phone or fax

A. May be signed by an emancipated minor. (&) C. Signifies that the patient understands all aspects of the procedure. A consent form may be signed by an emancipated minor, and consent may be obtained by fax or phone with appropriate witnesses. Only in the cases of underage children or unconscious or mentally incompetent people must a family member be aware of the procedure. The document protects the surgeon and the health care facility in that it indicates that the patient knows and understands all aspects of the procedure.

While conducting a home visit with a client who had a partial resection of the ileum for Chron's Disease 4 weeks previously, a nurse becomes concerned when the client states: A. My stools float and seem to have fat in them. B. I have gaiend 5 pounds since I left the hospital. C. I am still avoiding milk products. D. I only have 2 formed stools per day.

A. My stools float and seem to have fat in them.

Which of the following nursing interventions should receive highest priority when a patient is admitted to the postanesthesia care unit? A. Positioning the patient B. Observing the operative site C. Checking the postoperative orders D. Receiving report from operating room personnel.

A. Positioning the patient. A patient is received in the postanesthesia care unit on a bed or stretcher. Proper positioning is necessary to ensure airway patency in a sedated, unconscious, or semiconscious patient. Observation of the operative site, receiving report from operating room personnel, and checking postoperative orders are interventions made after proper positioning of the patient.

After instructing the client on crutch walking technique, the nurse should evaluate the client's understanding by using which of the following methods? A. Return demonstration B. Explanation C. Achievement of 90 on written test D. Have client explain produce to the family

A. Return demonstration Rationale: Interpersonal skills are the sum of the activities the nurse uses when communicating with others. Technical/psychomotor skills are "hands-on" skills, which are often procedures and are evaluated by return demonstration. Cognitive skills are the intellectual skills of analysis and problem-solving and are evaluated by tests.

Which of these is a correctly stated outcome goal written by the nurse? A. The client will walk 2 miles daily by March 19 B. The client will understand how to give insulin by discharge C. The client will regain their former state of health by April 1 D. The client achieve desired mobility by May 7

A. The client will walk 2 miles daily by March 19 Rationale: Outcome goals should be SMART, i.e., Specific, Measurable, Appropriate, Realistic, and Timely. Option 1 is the only outcome that has a specific behavior (walks daily), with measurable performance criteria (2 miles), and a time estimate for goal attainment (by March 19).

In caring for a person receiving an opioid analgesic through an epidural catheter, the nursing responsibility of prime importance is A. assessing for respiratory depression. B. establishing a baseline laboratory profile. C. inspecting the catheter insertion site hourly. D. ensuring that the patient remains on strict bed rest.

A. assessing for respiratory depression. Possible side effects of epidural opioids are pruritus, urinary retention, and delayed respiratory depression, occurring 4 to 12 hours after a dose. Establishing a baseline laboratory profile is outside the scope of practice for a nurse. Hourly inspection of the catheter insertion site is an unnecessary nursing intervention. In general, the site is assessed once a shift unless unexpected complications occur. Strict bed rest is not necessary for the patient with an epidural catheter; however, assistance with getting out of bed could be necessary related to effects of the opioid analgesic.

The primary goal of the circulating nurse during preparation of the operating room, transferring and positioning the patient, and assisting the anesthesia team is A. avoiding any type of injury to the patient. B. maintaining a clean environment for the patient. C. providing for patient comfort and sense of well-being. D. preventing breaks in aseptic technique by the sterile members of the team.

A. avoiding any type of injury to the patient. The protection of the patient from injury in the operating room environment is maintained by the circulating nurse by ensuring functioning equipment, preventing falls and injury during transport and transfer, monitoring asepsis, and being with the patient during anesthesia induction.

When administering low-molecular-weight heparin (LMWH) after an operation, a nurse should A. explain that the drug will help prevent clot formation in the legs. B. check the results of the partial thromboplastin time before administration. C. administer the dose with meals to prevent GI irritation and bleeding. D. inform the patient that blood will be drawn every 6 hours to monitor the prothrombin time.

A. explain that the drug will help prevent clot formation in the legs. Unfractionated heparin or LMWH is given as a prophylactic measure for venous thrombosis and pulmonary embolism. These anticoagulants work by inhibiting thrombin-mediated conversion of fibrinogen to fibrin. LMWH is injected subcutaneously with no relationship to meals. It has a more predictable dose response and less risk of bleeding complications. It does not require anticoagulant monitoring and dosage adjustments.

A patient is scheduled for a hemorrhoidectomy at an ambulatory day-surgery center. An advantage of performing surgery at an ambulatory center is a decreased need for A. laboratory tests and perioperative medications. B. preoperative and postoperative teaching by the nurse. C. psychologic support to alleviate fears of pain and discomfort. D. preoperative nursing assessment related to possible risks and complications.

A. laboratory tests and perioperative medications. Ambulatory surgery is usually less expensive and more convenient, generally involving fewer laboratory tests, fewer preoperative and postoperative medications, less psychological stress, and less susceptibility to hospital-acquired infections. However, the nurse is still responsible for assessing, supporting, and teaching the patient undergoing surgery, regardless of where the surgery is performed.

The nurse is planning care for a pediatric client with a fractured and casted left tibia. The nurse anticipates an appropriate goal for this client is to ultimately reach what level of mobility after removal of the cast and rehabilition? A: 5 (Not compromised) B: 3 (Moderately Compromised) C: 2 (Substantially Compromised) D: 1 (Severely Compromised)

A: 5 (Not compromised)

Which of the following principles does the nurse use in selecting interventions for the care plan? A: Actions should address the etiology of the nursing diagnosis B: Always select independent interventions when possible C: There is one best intervention for each goal/outcome D: Interventions should be "doing," not just "monitoring"

A: Actions should address the etiology of the nursing diagnosis

The client with a fractured pelvis requests that family members be allowed to stay overnight in the hospital room. Before determining whether or not this request can be honored, the nurse should consult which of the following? A: Hospital policies B: Standardized care plans C: Orthopedic protocols D: Standards of care

A: Hospital policies

The nurse ranks the following client diagnoses in what priority (from highest to lowest)? A: Ineffective Airway Clearance related to poor cough effort B: Pain related to surgical incision C: Risk for Constipation related to pain medications and decreased activity D: Risk for infection related to surgical incision

A: Ineffective Airway Clearance related to poor cough effort B: Pain related to surgical incision D: Risk for infection related to surgical incision C: Risk for Constipation related to pain medications and decreased activity

The nurse uses a standardized care plan to develop an individualized care plan for each client. Match the nursing action to the appropriate holistic care plan approach or rationale: The nurse updates the care plan on the computer to reflect the current client assessment. A: Ongoing individualized care planning B: Standardized care plan C: Complete individualized plan of care D: Individualized care plan

A: Ongoing individualized care planning

The nurse has identified the nursing diagnosis of Impaired Physical Mobility related to inflammation of knee joint. A short-term goal could be: A: The client will ambulate will crutches by the end of the week B: The client will ambulate C: The client will verbalize his frustration D: The client will stand without assistance by the end of the month

A: The client will ambulate will crutches by the end of the week

Which of the following is likely to occur if the goal statement is poorly written? A: There is no standard against which to compare outcomes B: The nursing diagnoses cannot be prioritized C: Only dependent nursing interventions can be used D: It is difficult to determine which nursing interventions can be delegated

A: There is no standard against which to compare outcomes

Consider the following nursing diagnosis for a client who is on bed rest. Risk for Impaired Skin Integrity related to bed rest. The nursing interventions are derived from the etiologic portion of the nursing diagnosis, which includes: A: Turn and reposition every 2 hours B: Select high-protein foods at each meal C: Proide a daily bath D: Offer a back rub from time to time

A: Turn and reposition every 2 hours

Which of the following nursing diagnoses are correctly written as two-part nursing diagnoses?

ALL: - Ineffective Coping related to inability to maintain marriage. - Defensive Coping related to loss of job and economic security. - Altered Thought Process related to panic state. - Decisional Conflict related to placement of parent in nursing home.

Which of the following guidelines for outcome writing are correct?

ALL: 1. At least one of the outcomes shows a direct resolution of the problem statement in the nursing diagnosis. 2. The patient (and family) values the outcomes. 3. The outcomes are supportive of the total treatment plan. 4. Each outcome is brief and specific (clearly describes one observable, measurable patient behavior/manifestation), is phrased positively, and specifies a time line.

Which of the following are examples of well-stated nursing interventions?

ALL: 1. Offer patient 60 mL water or juice every 2 hours while awake for a total minimum PO intake of 500 mL. 2. Walk with patient to bathroom for toileting every 2 hours while patient is awake.

What is an early indication of hypovolemia?

AMS (confusion) ex: pt w/bowel prep before surgery assess for dec BP, lightheadedness, dizziness, inc HR

Nursing action most helpful decreasing risk of hospital acquired infections, in urinary tract:

Avoid unnecessary catheters

When a public health nurse uses evidence-based interventions to evaluate the effectiveness, accessibility, and quality of population-based services within the community, the nurse is addressing the core public health function of: A) Assessment. B) Assurance. C) Policy development. D) Research.

B

A nurse is caring for a client who has threatened to commit suicide by hanging. The client states, "I'm going to use a knotted shower curtain when no one is around." Which information would determine the nurse's plan of care for this client? A. The more specific the plan is, the more likely the client will attempt suicide. B. Clients who talk about suicide never actually commit it. C. Clients who threaten suicide should be observed every 15 minutes. D. After a brief assessment, the nurse should avoid the topic of suicide.

ANS: A Clients who have specific plans are at greater risk for suicide.

When the association between maternal alcohol use and low birth weight is being studied, the nurse investigator's failure to consider the variable of smoking could introduce bias into the observed association, because smoking has a correlation with both factors. This effect could best be described as: A) Causality. B) Confounding. C) Information bias. D) Selection bias.

B

A client has been brought to the emergency department for signs and symptoms of Chronic Obstructive Pulmonary Disease (COPD). The client has a history of a suicide attempt 1 year ago. Which nursing intervention would take priority in this situation? A. Assessing the client's pulse oximetry and vital signs B. Developing a plan for safety for the client C. Assessing the client for suicidal ideations D. Establishing a trusting nurse-client relationship

ANS: A It is important to prioritize client interventions that assess the symptoms of COPD prior to any other nursing intervention. Physical needs must be prioritized according to Maslow's hierarchy of needs. This client's problems with oxygenation will take priority over assessing for current suicidal ideations.

Enteral feeding is preferred over parenteral nutrition whenever possible because enteral feeding:

Maintains gut function and integrity.

Immunizations 2 mo.

Hep B (2 of 3) btwn 4-8 wks (1-2mo.) RV [Rotavirus/ Rotarix] (1 of 2) DTaP [Diptheria, Tetanus, Pertussis] (1 of 4) Hib [Haemophilus influenzae type b] (1 of 3) PCV [Pneumococcal] (1 of 4) IPV [Inactivated Polio Virus] (1 of 4)

Which nursing intervention strategy is most appropriate to implement initially with a suicidal client? A. Ask a direct question such as, "Do you ever think about killing yourself?" B. Ask client, "Please rate your mood on a scale from 1 to 10." C. Establish a trusting nurse-client relationship. D. Apply the nursing process to the planning of client care.

ANS: A The risk of suicide is greatly increased if the client has suicidal ideations, has developed a plan, and particularly if means exist for the client to execute the plan.

Which of the following are advantages of enteral feedings over parenteral feedings?

Enteral feedings keep the gut in better working order than does parenteral feeding, and they prevent mucosal atrophy. The opportunity for infection is diminished because there is not insertion site, and enteral feedings can be provided at less cost than can parental feedings.

After a teenager reveals that he is gay, the father responds by beating him. The next morning, the teenager is found hanging in his closet. Which paternal emotions should a nurse anticipate? (Select all that apply.) A. Shock and disbelief B. Guilt and remorse C. Anger and resentment D. Bargaining and depression E. Denial and rationalization

ANS: A, B, C Suicide of a family member can induce a whole gamut of feelings in the survivors. Shock, disbelief, guilt, remorse, anger, and resentment are all feelings that may be experienced by this father.

The nurse is educating a pregnant client who has gestational diabetes. Which of the following statements should the nurse make to the client? Select all that apply. a. Cakes, candies, cookies, and regular soft drinks should be avoided. b. Gestational diabetes increases the risk that the mother will develop diabetes later in life. c. Gestational diabetes usually resolves after the baby is born. d. Insulin injections may be necessary. e. The baby will likely be born with diabetes f. The mother should strive to gain no more weight during the pregnancy.

ANS: A, B, C, D Gestational diabetes can occur between the 16th and 28th week of pregnancy. If not responsive to diet and exercise, insulin injections may be necessary. Concentrated sugars should be avoided. Weight gain should continue, but not in excessive amounts. Usually, gestational diabetes disappears after the infant is born. However, diabetes can develop 5 to 10 years after the pregnancy.

A nursing student is developing a study guide related to historical facts about suicide. Which of the following facts should the student include? (Select all that apply.) A. In the Middle Ages, suicide was viewed as a selfish and criminal act. B. During the Roman Empire, suicide was treated by incineration of the body. C. Suicide was an offense in ancient Greece, and a common site burial was denied. D. During the Renaissance, suicide was discussed and viewed more philosophically. E. Old Norse traditionally set a person who committed suicide adrift in the North Sea.

ANS: A, C, D These are true historical facts about suicide and should be included in the student's study guide.

Which is a correctly written, appropriate outcome for a client with a history of suicide attempts who is currently exhibiting symptoms of low self-esteem by isolating self? A. The client will not physically harm self. B. The client will express three positive self-attributes by day 4. C. The client will reveal a suicide plan. D. The client will establish a trusting relationship with the nurse by day 1.

ANS: B Although the client has a history of suicide attempts, the current problem is isolative behaviors based on low self-esteem. Outcomes should be client centered, specific, realistic, measureable, and contain a time frame.

How should the nurse use the JCAHO 2006 National Patient Safety Goals to improve communication among caregivers: a) Review a list of look-a-like sound-a-like drugs used in the organization c) Studying a list of abbreviations that are not to be used throughout the organization d) Use the client's room number as an identifier

C) Studying a list of abbreviations that are not to be used throughout the organization

The client has been close to death for some time and the family asks how the nurse will know when the client has actually died,. Which of the following would be the most accurate response from the nurse: c) When there is no apical pulse d) When the extremities are cool and dark in color

C) When there is no apical pulse

A nurse recently admitted a client to an inpatient unit after a suicide attempt. A health-care provider orders amitriptyline (Elavil) for the client. Which intervention related to this medication should be initiated to maintain this client's safety upon discharge? A. Provide a 6-month supply of Elavil to ensure long-term compliance. B. Provide a 1-week supply of Elavil with refills contingent on follow-up appointments. C. Provide a pill dispenser as a memory aid. D. Provide education regarding the avoidance of foods containing tyramine.

ANS: B The health-care provider should provide a 1-week supply of Elavil with refills contingent on follow-up appointments as an appropriate intervention to maintain the client's safety. Tricyclic antidepressants have a narrow therapeutic range and can be used in overdose to commit suicide. Distributing limited amounts of the medication decreases this potential.

Which statement indicates that the nurse is acting as an advocate for a client who has recently made a suicide attempt? A. "I must observe you continually for 1 hour in order to keep you safe." B. "Let's confer with the treatment team about the triggers to your attempt that we discussed." C. "You must have been very upset to do what you did today." D. "Are you currently thinking about harming yourself?"

ANS: B The nurse is functioning in an advocacy role when collaborating with the client and treatment team to discuss client problems.

A stockbroker commits suicide after being convicted of insider trading. Which information should a nurse share with the grieving family? A. "Keep in mind that your grieving will only last for 1 year." B. "To deal with your grief, try using coping strategies that have worked for you in the past." C. "You need to write a letter to the brokerage firm to express your anger with them." D. "It would be best if you avoid discussing the suicide."

ANS: B The nurse should discuss coping strategies that have been successful in times of stress in the past, and work to reestablish these within the family.

After years of dialysis, an 84-year-old states, "I'm exhausted, depressed, and so over these attempts to keep me alive." Which question should the nurse ask the spouse when preparing a discharge plan of care? A. "Has there been had any appetite or sleep changes?" B. "How often is your spouse left alone?" C. "Has your spouse been following a diet and exercise program consistently?" D. "How would you characterize your relationship with your spouse?"

ANS: B This client has many risk factors for suicide. The client should have increased supervision to decrease likelihood of self-harm.

A nursing instructor is teaching about suicide in the elderly population. Which information should the instructor include? A. Elderly people use less lethal means to commit suicide. B. While the elderly make up less than 13% of the population, they account for 16% of all suicides. C. Suicide is the second leading cause of death in the elderly. D. It is normal for elderly individuals to express a desire to die because they have come to terms with their mortality.

ANS: B This factual information should be included in the nursing instructor's teaching plan. An expressed desire to die is not normal in any age group.

Which client data indicate that a suicidal client is participating in a plan for safety? A. Compliance with antidepressant therapy B. A mood rating of 9/10 C. Disclosing a plan for suicide to staff D. Expressing feelings of hopelessness to nurse

ANS: C A degree of the responsibility for the suicidal client's safety is given to the client. When a client shares with staff a plan for suicide, the client is participating in a plan for safety by communicating thoughts of self-harm that would initiate interventions to prevent suicide.

A client is newly admitted to an inpatient psychiatric unit. Which assessment data are critical in determining an increased risk for suicide? A. Monitoring the client continually for 1 hour after admission B. Encouraging the client to discuss feelings C. Asking the client about any history of suicide attempts D. Removing hazardous materials from the environment

ANS: C A history of suicide attempts places a client at a higher risk for current suicide behaviors. Knowing this specific data will alert the nurse to the client's risk.

A nurse is caring for four clients diagnosed with major depression. When considering the client's belief system, which client would potentially be at highest risk for suicide? A. Roman Catholic B. Protestant C. Atheist D. Muslim

ANS: C Depressed men and women who consider themselves affiliated with a religion are less likely to attempt suicide than their nonreligious counterparts.

When written properly, NOC outcomes and indicators: A: Do not require customization B: Address several nursing diagnoses C: Are broad statements of desired end points D: Reflect both the nurse's and the client's values

D: Reflect both the nurse's and the client's values

A new nursing graduate asks the psychiatric nurse manager how to best classify suicide. Which is the nurse manager's best reply? A. "Suicide is a DSM-IV-TR diagnosis." B. "Suicide is a mental disorder." C. "Suicide is a behavior." D. "Suicide is an antisocial affliction."

ANS: C Suicide is not a diagnosis, disorder, or affliction. It is a behavior.

A terminally ill patient is experiencing chronic pain due to spinal cord tumor and has been admitted on several occasions for pain crises. Which intervention can produce positive outcome for the individual with uncontrolled pain and a short life expectancy?

Analgesic Nerve Blocks Analgesic blocks using neurolytic agents block nerve conductivity and destroys the nerves. Topical anesthesia, local anesthetic agents, and nonnarcotics are not effective for a patient experiencing pain due to cord compression.

A client is newly committed to an inpatient psychiatric unit. Which nursing intervention best lowers this client's risk for suicide? A. Encouraging participation in the milieu to promote hope B. Developing a strong personal relationship with the client C. Observing the client at intervals determined by assessed data D. Encouraging and redirecting the client to concentrate on happier times

ANS: C The nurse should observe the actively suicidal client continuously for the first hour after admission. After a full assessment, the treatment team will determine the observation status of the client. Observation of the client allows the nurse to interrupt any observed suicidal behaviors.

A client with a history of three suicide attempts has been taking fluoxetine (Prozac) for 1 month. The client suddenly presents with a bright affect, rates mood at 9/10, and is much more communicative. Which action should be the nurse's priority at this time? A. Give the client off-unit privileges as positive reinforcement. B. Encourage the client to share mood improvement in group. C. Increase frequency of client observation. D. Request that the psychiatrist reevaluate the current medication protocol.

ANS: C The nurse should be aware that a sudden increase in mood rating and change in affect could indicate that the client is at risk for suicide and client observation should be more frequent. Suicide risk may occur early during treatment with antidepressants. The return of energy may bring about an increased ability to act-out self-destructive behaviors prior to the client attaining the full therapeutic effect of the antidepressant medication.

A client diagnosed with major depressive disorder with psychotic features hears voices commanding self-harm. The client refuses to commit to developing a plan for safety. What should be the nurse's priority intervention at this time? A. Obtaining an order for locked seclusion until client is no longer suicidal B. Conducting 15-minute checks to ensure safety C. Placing the client on one-to-one observation while monitoring suicidal ideations D. Encouraging client to express feelings related to suicide

ANS: C The nurse's priority intervention when a client hears voices commanding self-harm is to place the client on one-to-one observation while continuing to monitor suicidal ideation.

A nurse discovers a client's suicide note that details the time, place, and means to commit suicide. What should be the priority nursing intervention, and the rationale for this action? A. Administering lorazepam (Ativan) prn because the client is angry about the discovery of the note B. Establishing room restrictions because the client's threat is an attempt to manipulate the staff C. Placing this client on one-to-one suicide precautions because the more specific the plan, the more likely the client will attempt suicide D. Calling an emergency treatment team meeting because the client's threat must be addressed

ANS: C The priority nursing action should be to place this client on one-to-one suicide precautions, because the more specific the plan, the more likely the client will attempt suicide. The appropriate nursing diagnosis for this client would be risk for suicide.

Assessing a patient with renal failure for an elevated serum potassium level is best achieved by:

Monitoring the patient's electrocardiogram (ECG) strips.

A nursing student is developing a plan of care for a suicidal client. Which documented intervention should the student implement first? A. Communicate therapeutically. B. Observe the client. C. Provide a hazard-free environment. D. Assess suicide risk.

ANS: D Assessment is the first step of the nursing process to gain needed information to determine further appropriate interventions.

The family of a suicidal client is very supportive and requests more facts related to caring for their family member after discharge. Which information should the nurse provide? A. Address only serious suicide threats to avoid the possibility of secondary gain. B. Promote trust by verbalizing a promise to keep suicide attempt information within the family. C. Offer a private environment to provide needed time alone at least once a day. D. Be available to actively listen, support, and accept feelings.

ANS: D Being available to actively listen, support, and accept feelings increases the potential that a client would confide suicidal ideations to family members.

A nursing instructor is teaching about suicide. Which student statement indicates that learning has occurred? A. "Suicidal threats and gestures should be considered manipulative and/or attention-seeking." B. "Suicide is the act of a psychotic person." C. "All suicidal individuals are mentally ill." D. "50% to 80% of all people who kill themselves have a history of a previous attempt."

ANS: D It is a fact that between 50% and 80% of all people who kill themselves have a history of a previous attempt. All other answer choices are myths about suicide.

The treatment team is making a discharge decision regarding a previously suicidal client. Which client assessment information should a nurse recognize as contributing to the team's decision? A. No previous admissions for major depressive disorder B. Vital signs stable; no psychosis noted C. Able to comply with medication regimen; able to problem-solve life issues D. Able to participate in a plan for safety; family agrees to constant observation

ANS: D Participation in a plan of safety and constant family observation will decrease the risk for self-harm. All other answer choices are not directly focused on suicide prevention and safety.

B. Spine

Ankylosing spondylitis is a chronic, systemic inflammatory disease of the joints that manifests pain and progressive stiffening in what area of the musculoskeletal system? A. Elbows B. Spine C. Ankles D. All joints (polyarticular)

During a one-to-one session with a client, the client states, "Nothing will ever get better," and "Nobody can help me." Which nursing diagnosis is most appropriate for a nurse to assign to this client at this time? A. Powerlessness R/T altered mood AEB client statements B. Risk for injury R/T altered mood AEB client statements C. Risk for suicide R/T altered mood AEB client statements D. Hopelessness R/T altered mood AEB client statements

ANS: D The client's statements indicate the problem of hopelessness. Prior to assigning either risk for injury or risk for suicide a further evaluation of the client's suicidal ideations and intent would be necessary.

In planning care for a suicidal client, which correctly written outcome should be a nurse's first priority? A. The client will not physically harm self. B. The client will express hope for the future by day 3. C. The client will establish a trusting relationship with the nurse. D. The client will remain safe during the hospital stay.

ANS: D The nurse's priority should be that the client will remain safe during the hospital stay. Client safety should always be the nurse's priority. The "A" answer choice is incorrectly written. Correctly written outcomes must be client focused, measurable, realistic, and contain a time frame. Without a time frame, an outcome cannot be correctly evaluated.

A suicidal client says to a nurse, "There's nothing to live for anymore." Which is the most appropriate nursing reply? A. "Why don't you consider doing volunteer work in a homeless shelter." B. "Let's discuss the negative aspects of your life." C. "Things will look better in the morning." D. "It sounds like you are feeling pretty hopeless."

ANS: D This statement verbalizes the client's implied feelings and allows him to validate and explore them.

A nurse has just given a narcotic for pain relief and must now leave the unit. To whom should the nurse delegate the task of evaluating the patient's response to pain medication?

Another RN

What is the best position for a client who is one day post op for a total hip replacement?

Abduction of the legs & have the pt supine or positioned on the nonoperative side

When developing client care plans, the first process the nurse engages in is: A: Establishing client goals B: Selecting nursing interventions C: Writing individualized nursing interventions to be performed D: Setting priorities

D: Setting priorities

The nurse knows that diarrhea is a symptom and not a disease and that treatment generally begins with:

Adequate fluids to hydrate the patient.

Which of the following addresses the type and extent of care a patient wishes?

Advance Directive

Urine output:

Alert physician if below 30 mL/hr

What is the primary work environment of nurses who are case managers?

All types of health care organizations, as well as organizations with health-related functions, hire case managers. rationale: The case managers work in all types of health care institutions. They may also work for insurance or utilization review companies or in health-related areas of various types of large businesses.

"Client will walk to the end of hallway without assistance by Friday" is an example of: A: Nursing intervention B: Rationale C: Long-term goal D: Short-term goal

D: Short-term goal

Purpose of a urinary catheter:

Amount of residual urine

C. lispro

An example of a rapid-acting insulin is: A. regular B. glargine C. lispro D. NPH

A nurse is caring for a surgical patient in the preoperative area. The nurse obtains the patient's informed consent for the surgical procedure. Which statements are true regarding informed consent?

An informed consent must be signed while the patient is free from mind-altering medications and must be witnessed after it has been determined that the patient has received all of the necessary information needed to make an informed decision.

Why would you not want an isotonic IV solution for a pt with congestive heart failure and excess fluid volume?

An isotonic IV solution would worsen the excess fluid volume.

What test should be performed to confirm tube placement of a nasoenteric feeding tube before the start of feedings?

An x-ray study

The nursing intervention classification (NIC): A: Improves cost effectiveness in planning care B: Isolates community input C: Conflicts with nursing management and therapies D: Standardizes and defines the knowledge base for nursing curricula and practice

D: Standardizes and defines the knowledge base for nursing curricula and practice

A mother comes to the emergency department after receiving a phone call informing her that her son was involved in a motor vehicle accident. When she approaches the triage desk, she frantically asks, "How is my son?" Which response by the nurse is best? 1) "He's being examined now; he's awake and talking. We'll take you to see him soon." 2) "Don't worry, I'm sure he'll be fine; we have an excellent trauma team caring for him." 3) "Everything will be okay; please take a seat and I'll check on him for you." 4) "Your son is strong and has youth on his side; I'm sure he'll be fine."

Answer: 1) "He's being examined now; he's awake and talking. We'll take you to see him soon." Rationale: By telling the mother that her son is awake and talking and being examined by the doctor, the nurse provides accurate information and helps reduce the mother's anxiety. Responses such as "Don't worry, everything will be okay" and "I'm sure he'll be fine" offer false reassurance and fail to respect the mother's concern.

A client with a stage 2 pressure ulcer has methicillin-resistant Staphylococcus aureus (MRSA) cultured from the wound. Contact precautions are initiated. Which rule must be observed to follow contact precautions? 1) A clean gown and gloves must be worn when in contact with the client. 2) Everyone who enters the room must wear a N-95 respirator mask. 3) All linen and trash must be marked as contaminated and send to biohazard waste. 4) Place the client in a room with a client with an upper respiratory infection.

Answer: 1) A clean gown and gloves must be worn when in contact with the client. Rationale: A clean gown and gloves must be worn when any contact is anticipated with the client or with contaminated items in the room. A respirator mask is required only with airborne precautions, not contact precautions. All linen must be double-bagged and clearly marked as contaminated. The client should be placed in a private room or in a room with a client with an active infection caused by the same organism and no other infections.

Which of the following is/are an example(s) of a health restoration activity? Select all that apply. 1) Administering an antibiotic every day 2) Teaching the importance of hand washing 3) Assessing a client's surgical incision 4) Advising a woman to get an annual mammogram after age 50 years

Answer: 1) Administering an antibiotic every day 3) Assessing a client's surgical incision Rationale: Health restoration activities help an ill client return to health. This would include taking an antibiotic every day and assessing a client's surgical incision. Hand washing and mammograms both involve healthy people who are trying to prevent illness.

Which of the following is/are an example(s) of theoretical knowledge as defined in this chapter? Select all that apply. 1) Antibiotics are ineffective in treating viral infections. 2) When you take a patient's blood pressure, the patient's arm should be at heart level. 3) In Maslow's framework, physical needs are most basic. 4) When drawing medication out of a vial, inject air into the vial first.

Answer: 1) Antibiotics are ineffective in treating viral infections. 3) In Maslow's framework, physical needs are most basic. Rationale: Theoretical knowledge consists of research findings, facts (e.g., "Antibiotics are ineffective . . ." is a fact), principles, and theories (e.g., "In Maslow's framework . . ." is a statement from a theory). Instructions for taking a blood pressure and withdrawing medications are examples of practical knowledge—what to do and how to do it.

The nurse is working on a unit that uses nursing assessment flow sheets. Which statement best describes this form of charting? Nursing assessment flow sheets: 1) Are comprehensive charting forms that integrate assessments and nursing actions 2) Contain only graphic information, such as I&O, vital signs, and medication administration 3) Are used to record routine aspects of care; they do not contain assessment data 4) Contain vital data collected upon admission, which can be compared with newly collected data

Answer: 1) Are comprehensive charting forms that integrate assessments and nursing actions Rationale: Nursing assessment flow sheets are organized by body systems. The nurse checks the box corresponding to the current assessment findings. Nursing actions, such as wound care, treatments, or IV fluid administration, are also included. Graphic information, such as vital signs, I&O, and routine care, may be found on the graphic record. The admission form contains baseline information.

Critical thinking and the nursing process have which of the following in common? Both: 1) Are important to use in nursing practice 2) Use an ordered series of steps 3) Are patient-specific processes 4) Were developed specifically for nursing

Answer: 1) Are important to use in nursing practice Rationale: Nurses make many decisions: some require using the nursing process, whereas others are not client related but require critical thinking. The nursing process has specific steps; critical thinking does not. Neither is linear. Critical thinking applies to any discipline.

Which pain management task can the nurse safely delegate to nursing assistive personnel? 1) Asking about pain during vital signs 2) Evaluating the effectiveness of pain medication 3) Developing a plan of care involving nonpharmacologic interventions 4) Administering over-the-counter pain medications

Answer: 1) Asking about pain during vital signs Rationale: The nurse can delegate the task of asking about pain when nursing assistive personnel (NAP) obtain vital signs. The NAP must be instructed to report findings to the nurse without delay. The nurse should evaluate the effectiveness of pain medications and develop the plan of care. Administering over-the-counter and prescription medications is the responsibility of the registered nurse or licensed practical nurse.

The charge nurse asks the nursing assistive personnel (NAP) to give a bag bath to a patient with end-stage chronic obstructive pulmonary disease. How should the NAP proceed? 1) Bathe the patient's entire body using 8 to 10 washcloths. 2) Assist the patient to a chair and provide bathing supplies. 3) Saturate a towel and blanket in a plastic bag, and then bathe the patient. 4) Assist the patient to the bathtub and provide a bath chair.

Answer: 1) Bathe the patient's entire body using 8 to 10 washcloths. Rationale: A towel bath is a modification of the bed bath in which the NAP places a large towel and a bath blanket into a plastic bag, saturates them with a commercially prepared mixture of moisturizer, nonrinse cleaning agent, and water; warms in them in a microwave, and then uses them to bathe the patient. A bag bath is a modification of the towel bath, in which the NAP uses 8 to 10 washcloths instead of a towel or blanket. Each part of the patient's body is bathed with a fresh cloth. A bag bath is not given in a chair or in the tub.

Which of the following clients is experiencing an abnormal change in vital signs? A client whose (select all that apply): 1) Blood pressure (BP) was 132/80 mm Hg sitting and is 120/60 mm Hg upon standing 2) Rectal temperature is 97.9°F in the morning and 99.2°F in the evening 3) Heart rate was 76 before eating and is 60 after eating 4) Respiratory rate was 14 when standing and is 22 after walking

Answer: 1) Blood pressure (BP) was 132/80 mm Hg sitting and is 120/60 mm Hg upon standing 3) Heart rate was 76 before eating and is 60 after eating Rationale: The BP change is abnormal; a BP change greater than 10 mm Hg may indicate postural hypotension. The change in heart rate is abnormal; heart rate usually increases slightly after eating rather than decreasing. The temperatures are within normal range for the rectal route, and temperature increases throughout the day. It is normal to have an increased respiratory rate after exercise.

A patient is admitted to the medical surgical floor with a kidney infection. The nurse introduces herself to the patient and begins her admission assessment. Which goal is most appropriate for this phase of the nurse-patient relationship? The patient will be able to: 1) Describe how to operate the bed and call for the nurse. 2) Discuss communication patterns and roles within the family. 3) Openly express his concerns about the hospitalization. 4) State expectations related to discharge.

Answer: 1) Describe how to operate the bed and call for the nurse. Rationale: This is the orientation phase of the relationship. The orientation phase begins when the nurse introduces herself to the patient and begins to gather data. In this phase, the nurse and patient are getting to know each other. As part of the orientation phase, the nurse will orient the patient to the hospital room and routines. In the preinteraction phase, the nurse gathers information about the patient before she meets him. Discussion of personal information, particularly if sensitive or complex, is suitable for the working phase of the nurse-patient interaction. The patient expressing feelings and concerns also occurs during the working phase. During the working phase, care is communicated, thoughts and feelings are expressed, and honest verbal and nonverbal communication occurs. Stating expectations related to discharge is most appropriate for the termination phase—the conclusion of the relationship.

Which factor in the patient's past medical history dictates that the nurse exercise caution when administering acetaminophen (Tylenol)? 1) Hepatitis B 2) Occasional alcohol use 3) Allergy to aspirin 4) Gastric irritation with bleeding

Answer: 1) Hepatitis B Rationale: Even in recommended doses, acetaminophen can cause severe hepatotoxicity in patients with liver disease, such as hepatitis B. Patients who consume alcohol regularly should also use acetaminophen cautiously. Those allergic to aspirin or other nonsteroidal anti-inflammatory drugs (NSAIDs) can use acetaminophen safely. Acetaminophen rarely causes gastrointestinal (GI) problems; therefore, it can be used for those with a history of gastric irritation and bleeding.

What is the most basic reason that self-knowledge is important for nurses? Because it helps the nurse to: 1) Identify personal biases that may affect his thinking and actions 2) Identify the most effective interventions for a patient 3) Communicate more efficiently with colleagues, patients, and families 4) Learn and remember new procedures and techniques

Answer: 1) Identify personal biases that may affect his thinking and actions Rationale: The most basic reason is that self-knowledge directly affects the nurse's thinking and the actions he chooses. Indirectly, thinking is involved in identifying effective interventions, communicating, and learning procedures. However, because identifying personal biases affects all the other nursing actions, it is the most basic reason.

How are critical thinking skills and critical thinking attitudes similar? Both are: 1) Influences on the nurse's problem solving and decision making 2) Like feelings rather than cognitive activities 3) Cognitive activities rather than feelings 4) Applicable in all aspects of a person's life

Answer: 1) Influences on the nurse's problem solving and decision making Rationale: Cognitive skills are used in complex thinking processes, such as problem solving and decision making. Critical thinking attitudes determine how a person uses her cognitive skills. Critical thinking attitudes are traits of the mind, such as independent thinking, intellectual curiosity, intellectual humility, and fair-mindedness, to name a few. Critical thinking skills refer to the cognitive activities used in complex thinking processes. A few examples of these skills involve recognizing the need for more information, recognizing gaps in one's own knowledge, and separating relevant from irrelevant data. Critical thinking, which consists of intellectual skills and attitudes, can be used in all aspects of life.

What is the leading cause of unintentional death for the entire U.S. population? 1) Motor vehicle accidents 2) Poisoning 3) Choking 4) Falls

Answer: 1) Motor vehicle accidents Rationale: The leading causes of unintentional death for the total population, in this order, are automobile accidents, poisoning, falls, and drowning.

Which factor(s) in the patient's past medical history place(s) him at risk for falling? Select all that apply. 1) Orthostatic hypotension 2) Appendectomy 3) Dizziness 4) Hyperthyroidism

Answer: 1) Orthostatic hypotension 3) Dizziness Rationale: Orthostatic hypotension, cognitive impairment, difficulty with walking or balance, weakness, dizziness, and drowsiness from certain medications place the patient at risk for falling. A history of right appendectomy and hyperthyroidism do not place that patient at risk for falling.

The nurse is orienting a new nurse to the unit and reviews source-oriented charting. Which statement by the nurse best describes source-oriented charting? Source-oriented charting: 1) Separates the health record according to discipline 2) Organizes documentation around the patient's problems 3) Highlights the patient's concerns, problems, and strengths 4) Is designed to streamline documentation

Answer: 1) Separates the health record according to discipline Rationale: In source-oriented charting, each discipline documents findings in a separately labeled section of the chart. Problem-oriented charting organizes notes around the patient's problems. Focus® charting highlights the patient's concerns, problems, and strengths. Charting by exception is a unique charting system designed to streamline documentation.

The nurse is assessing a patient admitted to the hospital with rectal bleeding. The patient had a hip replacement 2 weeks ago. Which position should the nurse avoid when examining this patient's rectal area? 1) Sims' 2) Supine 3) Dorsal recumbent 4) Semi-Fowler's

Answer: 1) Sims' Rationale: Sims' position is typically used to examine the rectal area. However, the position should be avoided if the patient has undergone hip replacement surgery The patient with a hip replacement can assume the supine, dorsal recumbent, or semi-Fowler's positions without causing harm to the joint. Supine position is lying on the back facing upward. The patient in dorsal recumbent is on his back with knees flexed and soles of feet flat on the bed. In semi-Fowler's position, the patient is supine with the head of the bed elevated and legs slightly elevated.

The nurse is assessing vital signs for a patient just admitted to the hospital. Ideally, and if there are no contraindications, how should the nurse position the patient for this portion of the admission assessment? 1) Sitting upright 2) Lying flat on the back with knees flexed 3) Lying flat on the back with arms and legs fully extended 4) Side-lying with the knees flexed

Answer: 1) Sitting upright Rationale: If the patient is able, the nurse should have the patient sit upright to obtain vital signs in order to allow the nurse to easily access the anterior and posterior chest for auscultation of heart and breath sounds. It allows for full lung expansion and is the preferred position for measuring blood pressure. Additionally, patients might be more comfortable and feel less vulnerable when sitting upright (rather than lying down on the back) and can have direct eye contact with the examiner. However, other positions can be suitable when the patient's physical condition restricts the comfort or ability of the patient to sit upright.

During a presentation at a nursing staff meeting, the unit manager speaks very slowly with a monotone. She uses medical and technical terminology to convey her message. Dressed in business attire, the manager stands erect and smiles occasionally while speaking. Which elements of her approach are likely to cause the staff to lose interest in what she has to say? Select all answers that apply. 1) Slow speech 2) Monotone 3) Occasional smile 4) Formal dress

Answer: 1) Slow speech 2) Monotone Rationale: Speaking slowly with a monotone can contribute to reduced attention as the listener can think faster than the speaker is speaking, and the monotone voice has an almost hypnotizing effect. Smiling improves personal interest and connection between the speaker and listener so should not cause a loss of interest. Wearing formal business attire would not directly detract from listeners' engagement in the speaker's message unless it was unusual enough to distract listeners; nothing in the situation above indicates that is so.

The nurse is recording assessment data. She writes, "The patient seems worried about his surgery. Other than that, he had a good night." Which errors did the nurse make? Select all that apply. 1) Used a vague generality 2) Did not use the patient's exact words 3) Used a "waffle" word (e.g., appears) 4) Recorded an inference rather than a cue

Answer: 1) Used a vague generality 3) Used a "waffle" word (e.g., appears) 4) Recorded an inference rather than a cue Rationale: The nurse recorded a vague generality: "he has had a good night." The nurse did not use the patient's exact words, but she did not quote the patient at all, so that is not one of her errors. The nurse used the "waffle" word, "seems" worried instead of documenting what the patient said or did to lead her to that conclusion. The nurse recorded these inferences: worried and had a good night.

A client requires protective isolation. Which client can be safely paired with this client in a client-care assignment? One 1) admitted with unstable diabetes mellitus. 2) who underwent surgical repair of a perforated bowel. 3) with a stage 3 sacral pressure ulcer. 4) admitted with a urinary tract infection.

Answer: 1) admitted with unstable diabetes mellitus. Rationale: The client with unstable diabetes mellitus can safely be paired in a client-care assignment because the client is free from infection. Perforation of the bowel exposes the client to infection requiring antibiotic therapy during the postoperative period. Therefore, this client should not be paired with a client in protective isolation. A client in protective isolation should not be paired with a client who has an open wound, such as a stage 3 pressure ulcer, or with a client who has a urinary tract infection.

What is the rationale for hand washing? Hand washing is expected to remove: 1) transient flora from the skin. 2) resident flora from the skin. 3) all microorganisms from the skin. 4) media for bacterial growth.

Answer: 1) transient flora from the skin. Rationale: There are two types of normal flora: transient and resident. Transient flora are normal flora that a person picks up by coming in contact with objects or another person (e.g., when you touch a soiled dressing). You can remove these with hand washing. Resident flora live deep in skin layers where they live and multiply harmlessly. They are permanent inhabitants of the skin and cannot usually be removed with routine hand washing. Removing all microorganisms from the skin (sterilization) is not possible without damaging the skin tissues. To live and thrive in humans, microbes must be able to use the body's precise balance of food, moisture, nutrients, electrolytes, pH, temperature, and light. Food, water, and soil that provide these conditions may serve as nonliving reservoirs. Hand washing does little to make the skin uninhabitable for microorganisms, except perhaps briefly when an antiseptic agent is used for cleansing.

Which action should the nurse take when preparing patient-controlled analgesia for a postoperative patient? 1) Caution the patient to limit the number of times he presses the dosing button. 2) Ask another nurse to double-check the setup before patient use. 3) Instruct the patient to administer a dose only when experiencing pain. 4) Provide clear, simple instructions for dosing if the patient is cognitively impaired.

Answer: 2) Ask another nurse to double-check the setup before patient use. Rationale: As a safeguard to reduce the risk for dosing errors, the nurse should request another nurse to double-check the setup before patient use. The nurse should reassure the patient that the pump has a lockout feature that prevents him from overdosing even if he continues to push the dose administration button. The nurse should also instruct the patient to administer a dose before potentially painful activities, such as walking. Patient-controlled analgesia is contraindicated for those who are cognitively impaired.

Which change in hygiene practices may be necessary as the patient ages? 1) Brushing teeth twice a day 2) Bathing every other day 3) Decreasing moisturizer use 4) Increasing soap use

Answer: 2) Bathing every other day Rationale: As a person ages, sebaceous glands become less active, causing skin to dry. Older people may find it necessary to bathe every 2 days, increase the use of moisturizers, and decrease soap use to prevent further drying of skin. Older adults should brush their teeth after every meal and at bedtime to prevent tooth decay. It is recommended that people of all ages brush their teeth at least twice a day, so that option does not represent a change in an older adult's hygiene practices.

The nurse assesses the client's pedal pulses as having a pulse volume of 1 on a scale of 0 to 3. Based on this assessment finding, it would be important for the nurse to also assess the: 1) Pulse deficit 2) Blood pressure 3) Apical pulse 4) Pulse pressure

Answer: 2) Blood pressure Rationale: If the leg pulses are weak, the nurse should assess the blood pressure in order to further explore the reason for the low pulse volume. If the blood pressure is low, then a low pulse volume would be expected. The pulse deficit is the difference between the apical and radial pulse. The apical pulse would not be helpful to assess peripheral circulation. The pulse pressure is the difference between the systolic and diastolic pressures.

For all body systems except the abdomen, what is the preferred order for the nurse to perform the following examination techniques? A. Palpation B. Auscultation C. Inspection D. Percussion 1) D, B, A, C 2) C, A, D, B 3) B, C, D, A 4) A, B, C, D

Answer: 2) C, A, D, B Rationale: Inspection begins immediately as the nurse meets the patient, as she observes the patient's appearance and behavior. Observational data are not intrusive to the patient. When performing assessment techniques involving physical touch, the behavior, posture, demeanor, and responses might be altered. Palpation, percussion, and auscultation should be performed in that order, except when performing an abdominal assessment. During abdominal assessment, auscultation should be performed before palpation and percussion to prevent altering bowel sounds.

How should the nurse modify the examination for a 7-year-old child? 1) Ask the parents to leave the room before the examination. 2) Demonstrate equipment before using it. 3) Allow the child to help with the examination. 4) Perform invasive procedures (e.g., otoscopic) last.

Answer: 2) Demonstrate equipment before using it. Rationale: The nurse should modify his examination by demonstrating equipment before using it to examine a school-age child. The nurse should make sure parents are not present during the physical examination of an adolescent, but they usually help younger children feel more secure. The nurse should allow a preschooler to help with the examination when possible, but not usually a school-age child. Toddlers are often fearful of invasive procedures, so those should be performed last in this age group. It is best to perform invasive procedures last for all age groups; therefore, this does not represent a modification.

In which step of the nursing process does the nurse analyze data and identify client problems? 1) Assessment 2) Diagnosis 3) Planning outcomes 4) Evaluation

Answer: 2) Diagnosis Rationale: In the assessment phase, the nurse gathers data from many sources for analysis in the diagnosis phase. In the diagnosis phase, the nurse identifies the client's health status. In the planning outcomes phase, the nurse formulates goals and outcomes. In the evaluation phase, which occurs after implementing interventions, the nurse gathers data about the client's responses to nursing care to determine whether client outcomes were met.

A client exhibits all of the following during a physical assessment. Which of these is considered a primary defense against infection? 1) Fever 2) Intact skin 3) Inflammation 4) Lethargy

Answer: 2) Intact skin Rationale: Intact skin is considered a primary defense against infection. Fever, the inflammatory response, and phagocytosis (a process of killing pathogens) are considered secondary defenses against infection.

For a morbidly obese patient, which intervention should the nurse choose to counteract the pressure created by the skin folds? 1) Cover the mattress with a sheepskin. 2) Keep the linens wrinkle free. 3) Separate the skin folds with towels. 4) Apply petrolatum barrier creams.

Answer: 2) Keep the linens wrinkle free. Rationale: Separating the skin folds with towels relieves the pressure of skin rubbing on skin. Sheepskins are not recommended for use at all. Petrolatum barrier creams are used to minimize moisture caused by incontinence.

Which of the following incidents requires the nurse to complete an occurrence report? 1) Medication given 30 minutes after scheduled dose time 2) Patient's dentures lost after transfer 3) Worn electrical cord discovered on an IV infusion pump 4) Prescription without the route of administration

Answer: 2) Patient's dentures lost after transfer Rationale: You would need to complete an occurrence report if you suspect your patient's personal items to be lost or stolen. A medication can be administered within a half-hour of the administration time without an error in administration; therefore, an occurrence report is not necessary. The worn electrical cord should be taken out of use and reported to the biomedical department. The nurse should seek clarification if the provider's order is missing information; an occurrence report is not necessary.

Which situation requires intrapersonal communication? 1) Staff meetings 2) Positive self-talk 3) Shift report 4) Wound care committee meeting

Answer: 2) Positive self-talk Rationale: The nurse engaging in positive self-talk is using intrapersonal communication—conscious internal dialogue. Staff meetings, shift report, and a committee meeting are all examples of group or interpersonal communication.

The nurse is preparing to admit a patient from the emergency department. The transferring nurse reports that the patient with chronic lung disease has a 30+ year history of tobacco use. The nurse used to smoke a pack of cigarettes a day at one time and worked very hard to quit smoking. She immediately thinks to herself, "I know I tend to feel negatively about people who use tobacco, especially when they have a serious lung condition; I figure if I can stop smoking, they should be able to. I must remember how physically and psychologically difficult that is, and be very careful not to let be judgmental of this patient." This best illustrates: 1) Theoretical knowledge 2) Self-knowledge 3) Using reliable resources 4) Use of the nursing process

Answer: 2) Self-knowledge Rationale: Personal knowledge (2) is self-understanding—awareness of one's beliefs, values, biases, and so on. That best describes the nurse's awareness that her bias can affect her patient care. Theoretical knowledge consists of information, facts, principles, and theories in nursing and related disciplines; it consists of research findings and rationally constructed explanations of phenomena. Using reliable resources is a critical thinking skill. The nursing process is a problem-solving process consisting of the steps of assessing, diagnosing, planning outcomes, planning interventions, implementing, and evaluating. The nurse has not yet met this patient, so she could not have begun the nursing process.

The nurse must examine a patient who is weak and unable to sit unaided or to get out of bed. How should she position the patient to begin and perform most of the physical examination? 1) Dorsal recumbent 2) Semi-Fowler's 3) Lithotomy 4) Sims'

Answer: 2) Semi-Fowler's Rationale: If a patient is unable to sit up, the nurse should place him lying flat on his back, with the head of the bed elevated. Dorsal recumbent position is used for abdominal assessment if the patient has abdominal or pelvic pain. The patient in dorsal recumbent is on his back with knees flexed and soles of feet flat on the bed. Lithotomy position is used for female pelvic examination. It is similar to dorsal recumbent position, except that the patient's legs are well separated and thighs are acutely flexed. Feet are usually placed in stirrups. Fold sheet or bath blanket crosswise over thighs and legs so that genital area is easily exposed. Keep patient covered as much as possible. The patient in Sim's position is on left side with right knee flexed against abdomen and left knee slightly flexed. Left arm is behind body; right arm is placed comfortably. Sims' position is used to examine the rectal area. In semi-Fowler's position, the patient is supine with the head of the bed elevated and legs slightly elevated.

Which of the following is an example of data that should be validated? 1) The urinalysis report indicates there are white blood cells in the urine. 2) The client states she feels feverish; you measure the oral temperature at 98°F. 3) The client has clear breath sounds; you count a respiratory rate of 18. 4) The chest x-ray report indicates the client has pneumonia in the right lower lobe.

Answer: 2) The client states she feels feverish; you measure the oral temperature at 98°F. Rationale: Validation should be done when subjective and objective data do not make sense. For instance, it is inconsistent data when the patient feels feverish and you obtain a normal temperature. The other distractors do not offer conflicting data. Validation is not usually necessary for laboratory test results.

A local church organizes a group for people who are having difficulty coping with the death of a loved one. Which type of group has been organized? 1) Work-related social support group 2) Therapy group 3) Task group 4) Community committee

Answer: 2) Therapy group Rationale: Therapy groups are designed to help individual members cope with issues, such as the death of a spouse, divorce, or motherhood. Work-related social support groups help members of a profession cope with work-associated stress. Task groups meet to accomplish a specified task. Community-based committees meet to discuss community issues.

Arrange the steps of the nursing process in the sequence in which they generally occur. A. Assessment B. Evaluation C. Planning outcomes D. Planning interventions E. Diagnosis 1) E, B, A, D, C 2) A, B, C, D, E 3) A, E, C, D, B 4) D, A, B, E, C

Answer: 3) A, E, C, D, B Rationale: Logically, the steps are assessment, diagnosis, planning outcomes, planning interventions, and evaluation. Keep in mind that steps are not always performed in this order, depending on the patient's needs, and that steps overlap.

Which of the following clients should have an apical pulse taken? A client who is: 1) Febrile and has a radial pulse of 100 bpm 2) A runner who has a radial pulse of 62 bpm 3) An infant with no history of cardiac defect 4) An elderly adult who is taking antianxiety medication

Answer: 3) An infant with no history of cardiac defect Rationale: An apical pulse should be taken if the radial pulse is weak and/or irregular, if the rate is <60 or >100, if the patient is on cardiac medications, or when assessing children up to 3 years. It is difficult to palpate a peripheral pulse on infants and young children.

While teaching a health promotion group of adults, the nurse notices one person who is clutching his throat with both hands. What should the nurse do first? 1) Call 9-1-1. 2) Encourage the person to cough vigorously. 3) Ask, "Are you choking?" 4) Give five back blows.

Answer: 3) Ask, "Are you choking?" Rationale: Clutching the throat is the universal sign of choking. The first action when you suspect airway obstruction is to ask, "Are you choking?" If the person indicates "yes," or if the person cannot cough, speak, or breathe, that indicates choking. You must first be certain the person is choking because you can cause harm when you perform the choking maneuver. You would not call 9-1-1, encourage coughing, or give five back blows until you first establish that the person is choking. The client appears to be giving the universal sign for choking, but the nurse must validate the client's meaning before acting.

Which nonsteroidal anti-inflammatory drug might be administered to inhibit platelet aggregation in a patient at risk for thrombophlebitis? 1) Ibuprofen (Motrin) 2) Celecoxib (Celebrex) 3) Aspirin (Ecotrin) 4) Indomethacin (Indocin)

Answer: 3) Aspirin (Ecotrin) Rationale: Aspirin is a unique NSAID that inhibits platelet aggregation. Low-dose aspirin therapy is commonly administered to decrease the risk of thrombophlebitis, myocardial infarction, and stroke. Ibuprofen, celecoxib, and indomethacin are NSAIDs, but they do not inhibit platelet aggregation.

Which action should the nurse take before administering morphine 4.0 mg intravenously to a patient complaining of incisional pain? 1) Assess the patient's incision. 2) Clarify the order with the prescriber. 3) Assess the patient's respiratory status. 4) Monitor the patient's heart rate.

Answer: 3) Assess the patient's respiratory status. Rationale: Before administering an opioid analgesic, such as morphine, the nurse should assess the patient's respiratory status because opioid analgesics can cause respiratory depression. It is not necessary to clarify the order with the physician because morphine 4 mg IV is an appropriate dose. It is not necessary to monitor the patient's heart rate.

The nurse should use the diaphragm of the stethoscope to auscultate which of the following? 1) Heart murmurs 2) Jugular venous hums 3) Bowel sounds 4) Carotid bruits

Answer: 3) Bowel sounds Rationale: The bell of the stethoscope should be used to hear low-pitched sounds, such as murmurs, bruits, and jugular hums. The diaphragm should be used to hear high-pitched sounds that normally occur in the heart, lungs, and abdomen.

A 73-year-old patient who sustained a right hip fracture in a fall requests pain medication from the nurse. Based on his injury, which type of pain is this patient most likely experiencing? 1) Phantom 2) Visceral 3) Deep somatic 4) Referred

Answer: 3) Deep somatic Rationale: Deep somatic pain originates in ligaments, tendons, nerves, blood vessels, and bones. Therefore, a hip fracture causes deep somatic pain. Phantom pain is pain that is perceived to originate from a part that was removed during surgery. Visceral pain is caused by deep internal pain receptors and commonly occurs in the abdominal cavity, cranium, and thorax. Referred pain occurs in an area that is distant to the original site.

A client who is receiving epidural analgesia complains of nausea and loss of motor function in his legs. The nurse obtains his blood pressure and notes a drop in his blood pressure from the previous reading. Which complication is the patient most likely experiencing? 1) Infection at the catheter insertion site 2) Side effect of the epidural analgesic 3) Epidural catheter migration 4) Spinal cord damage

Answer: 3) Epidural catheter migration Rationale: The patient is exhibiting signs of epidural catheter migration, which include nausea, a decrease in blood pressure, and loss of motor function without an identifiable cause. Signs of infection at the catheter site include redness, swelling, and drainage. Loss of motor function is not a typical side effect associated with epidural analgesics. These are common signs of catheter migration, not spinal cord damage.

A woman of Orthodox Jewish faith who underwent a hysterectomy for cancer is being cared for on the surgical floor. Which healthcare team member(s) could be assigned to bathe this patient? Choose all correct answers. 1) Male nursing assistant 2) Male licensed practical nurse 3) Female graduate nurse 4) Female registered nurse

Answer: 3) Female graduate nurse 4) Female registered nurse Rationale: Orthodox Judaism prohibits personal care being provided by a member of the opposite sex. The patient who underwent a hysterectomy is female; therefore, out of respect for her religious beliefs, she should not be bathed by the male licensed practical nurse or nursing assistant.

A patient is admitted with shortness of breath, so the nurse immediately listens to his breath sounds. Which type of assessment is the nurse performing? 1) Ongoing assessment 2) Comprehensive physical assessment 3) Focused physical assessment 4) Psychosocial assessment

Answer: 3) Focused physical assessment Rationale: The nurse is performing a focused physical assessment, which is done to obtain data about an identified problem, in this case shortness of breath. An ongoing assessment is performed as needed, after the initial data are collected, preferably with each patient contact. A comprehensive physical assessment includes an interview and a complete examination of each body system. A psychosocial assessment examines both psychological and social factors affecting the patient. The nurse conducting a psychosocial assessment would gather information about stressors, lifestyle, emotional health, social influences, coping patterns, communication, and personal responses to health and illness, to name a few aspects.

The charge nurse on the medical surgical floor assigns vital signs to the nursing assistive personnel (NAP) and medication administration to the licensed vocational nurse (LVN). Which nursing model of care is this floor following? 1) Team nursing 2) Case method nursing 3) Functional nursing 4) Primary nursing

Answer: 3) Functional nursing Rationale: With team nursing, an RN or LVN is paired with a NAP. The pair is then assigned to render care for a group of patients. In case method nursing, one nurse cares for one patient during her entire shift. Private duty nursing is an example of this care model. This medical surgical floor is following the functional nursing model of care, in which care is partitioned and assigned to a staff member with the appropriate skills. For example, the NAP is assigned vital signs, and the LVN is assigned medication administration. When the primary nursing model is utilized, one nurse manages care for a group of patients 24 hours a day, even though others provide care during part of the day.

A mother who breastfeeds her child passes on which antibody through breast milk? 1) IgA 2) IgE 3) IgG 4) IgM

Answer: 3) IgG Rationale: The antibody IgG is passed to the child through the mother's breast milk during breastfeeding. IgA, IgE, and IgM are produced by the child's body after exposure to an antigen.

The client asks the nurse why an electronic health record (EHR) system is being used. Which response by the nurse indicates an understanding of the rationale for an EHR system? 1) It includes organizational reports of unusual occurrences that are not part of the client's record. 2) This type of system consists of combined documentation and daily care plans. 3) It improves interdisciplinary collaboration that improves efficiency in procedures. 4) This type of system tracks medication administration and usage over 24 hours.

Answer: 3) It improves interdisciplinary collaboration that improves efficiency in procedures. Rationale: The EHR has several benefits for use, including improving interdisciplinary collaboration and making procedures more accurate and efficient. An occurrence report is an organizational record of an unusual occurrence or accident that is not a part of the client's record. Integrated plans of care (IPOC) are a combined charting and care plan format. A medication administration record (MAR) is used to document medications administered and their usage.

The nurse wishes to identify nursing diagnoses for a patient. She can best do this by using a data collection form organized according to (select all that apply): 1) A body systems model 2) A head-to-toe framework 3) Maslow's hierarchy of needs 4) Gordon's functional health patterns

Answer: 3) Maslow's hierarchy of needs 4) Gordon's functional health patterns Rationale: Nursing models produce a holistic database that is useful in identifying nursing rather than medical diagnoses. Body systems and head-to-toe are not nursing models, and they are not holistic; they focus on identifying physiological needs or disease. Maslow's hierarchy is not a nursing model, but it is holistic, so it is acceptable for identifying nursing diagnoses. Gordon's functional health patterns are a nursing model.

When the nurse completes the patient's admission nursing database, the patient reports that he does not have any allergies. Which acceptable medical abbreviation can the nurse use to document this finding? 1) NA 2) NDA 3) NKA 4) NPO

Answer: 3) NKA Rationale: The nurse can use the medical abbreviation NKA, which means no known allergies, to document this finding. NA is an abbreviation for not applicable. NDA is an abbreviation for no known drug allergies. NPO is an abbreviation that means nothing by mouth.

Which of the following aspects of nursing is essential to defining it as both a profession and a discipline? 1) Established standards of care 2) Professional organizations 3) Practice supported by scientific research 4) Activities determined by a scope of practice

Answer: 3) Practice supported by scientific research Rationale: The American Nurses Association (ANA) has developed standards of care, but they are unrelated to defining nursing as a profession or discipline. Having professional organizations is not included in accepted characteristics of either a profession or a discipline. A profession must have knowledge that is based on technical and scientific knowledge. The theoretical knowledge of a discipline must be based on research, so both are scientifically based. Having a scope of practice is not included in accepted characteristics of either a profession or a discipline.

A client who has experienced prolonged exposure to the cold is admitted to the hospital. Which method of taking a temperature would be most appropriate for this client? 1) Axillary with an electronic thermometer 2) Oral with a glass thermometer 3) Rectal with an electronic thermometer 4) Tympanic with an infrared thermometer

Answer: 3) Rectal with an electronic thermometer Rationale: The rectal route is the most accurate for assessing core temperature, especially when it is critical to get an accurate temperature. Therefore, in this situation it is preferred. Temperature is a particularly relevant data point for this client with hypothermia as it indicates the patient's baseline status and response to treatment. The electronic thermometer is safer than glass and is relatively accurate. Mercury thermometers are no longer used in the hospital setting. The accuracy of tympanic thermometers is debatable.

The nurse is teaching a child and family about firearm safety. The nurse should instruct the child to take which step first if he sees a gun at a friend's house? 1) Leave the area. 2) Do not touch the gun. 3) Stop where he is. 4) Tell an adult.

Answer: 3) Stop where he is. Rationale: The child should be instructed to stop where he is. This allows him to think about the next steps he has memorized. Next, he should avoid touching the gun, leave the area, and immediately go tell an adult.

The nurse assesses clients' breath sounds. Which one requires immediate medical attention? A client who has: 1) Crackles 2) Rhonchi 3) Stridor 4) Wheezes

Answer: 3) Stridor Rationale: Stridor is a sign of respiratory distress, possibly airway obstruction. Crackles and rhonchi indicate fluid in the lung; wheezes are caused by narrowing of the airway. Crackles, rhonchi, and wheezes indicate respiratory illness and are potentially serious but do not necessarily indicate respiratory distress that requires immediate medical attention.

Which of the following clients has indications of orthostatic hypotension? A client whose blood pressure is: 1) 118/68 when standing and 110/72 when lying down 2) 140/80, HR 82 bpm when sitting and 136/76, HR 98 bpm when standing 3) 126/72 lying down and 133/80 when sitting, and reports shortness of breath 4) 146/88 when lying down and 130/78 when standing, and reports feeling dizzy

Answer: 4) 146/88 when lying down and 130/78 when standing, and reports feeling dizzy Rationale: Orthostatic hypotension is a drop of 10 mm Hg or more in blood pressure upon moving to a standing position, with complaints of feeling dizzy and/or faint.

The nurse suspects that a patient is being physically abused at home. What is the best environment in which to discuss the possibility of abusive events? 1) The patient's shared semiprivate room 2) The hallway outside the patient's room 3) An empty corner at the nurse's station 4) A conference room at the end of the hall

Answer: 4) A conference room at the end of the hall Rationale: The best environment in which to discuss sensitive matters is a quiet room where conversation can occur in private, particularly when the space is nonthreatening. The patient might be distracted if conversation takes place in a room where others (e.g., patients and visitors) are present. The hallway outside the patient's room and the nurses' station are public areas and should not be used for private conversation.

The nurse administers codeine sulfate 30 mg orally to a patient who underwent craniotomy 3 days ago for a brain tumor. How soon after administration should the nurse reassess the patient's pain? 1) Immediately 2) In 10 minutes 3) In 15 minutes 4) In 60 minutes

Answer: 4) In 60 minutes Rationale: Codeine administered by the oral route reaches peak concentration in 60 minutes; therefore, the nurse should reassess the patient's pain 60 minutes after administration. The nurse should reassess pain after 10 minutes when administering codeine by the intramuscular or subcutaneous routes. Drugs administered by the intravenous (IV) route are effective almost immediately; however, codeine is not recommended for IV administration.

What should parents do to promote child safety in the home? 1) Attach the baby's pacifier to a ribbon so that it does not fall on the floor. 2) Give a 2-year-old whole grapes instead of popcorn for a snack. 3) Store firearms unloaded and out of sight in a location too high for the child to reach. 4) Install window guards; never leave a window wide open.

Answer: 4) Install window guards; never leave a window wide open. Rationale: To prevent falls, install window guards and never leave a window wide open. A ribbon can become entangled around a small child's neck, causing asphyxiation. Young children can easily choke on a grape. Firearms should be unloaded, but stored in a locked cabinet. Children are curious and like to explore and climb. It would not be too difficult for a child to find a firearm stored, for example, on a high closet shelf.

At the end of the shift, the nurse realizes that she forgot to document a dressing change that she performed for a patient. Which action should the nurse take? 1) Complete an occurrence report before leaving. 2) Do nothing; the next nurse will document it was done. 3) Write the note of the dressing change into an earlier note. 4) Make a late entry as an addition to the narrative notes.

Answer: 4) Make a late entry as an addition to the narrative notes. Rationale: If the nurse fails to make an important entry while charting, she should make a late entry as an addition to the narrative notes. An occurrence report is not necessary in this case. If documentation is omitted, there is no legal verification that the procedure was performed. It is illegal to add to a chart entry that was previously documented. The nurse can only document care directly performed or observed. Therefore, the nurse on the incoming shift would not record the wound change as performed.

Which of the following clients would have the most difficulty maintaining thermoregulation? 1) Young child playing soccer during the summer 2) Middle-aged adult snow skiing 3) Young adult playing golf on a hot day 4) Older adult raking leaves on a cold day

Answer: 4) Older adult raking leaves on a cold day Rationale: Older adults have more difficulty maintaining body heat because of their slower metabolism, loss of subcutaneous fat, and decreased vasomotor control.

The nurse notes a lesion that appears to be caused by tissue compression on the right hip of a patient who suffered a stroke 5 days ago. How should the nurse document this finding? 1) Maceration 2) Abrasion 3) Excoriation 4) Pressure ulcer

Answer: 4) Pressure ulcer Rationale: The nurse should document a lesion caused by tissue compression and inadequate perfusion as a pressure ulcer. Abrasion, a rubbing away of the epidermal layer of skin, is commonly caused by shearing forces that occur when a patient moves or is moved in bed. Maceration is a softening of skin from prolonged moisture. Excoriation is a loss of the superficial layers of the skin caused by the digestive enzymes in feces.

A patient is agitated and continues to try to get out of bed. The nurse tries unsuccessfully to reorient him. What should the nurse do next? 1) Apply a vest restraint. 2) Move the patient to a quieter room. 3) Ask another nurse to care for the patient. 4) Provide comfort measures.

Answer: 4) Provide comfort measures. Rationale: Patients sometimes become agitated because they are uncomfortable or in pain. Providing comfort measures may decrease agitation. If the patient continues to be agitated, the nurse should encourage a family member or friend to sit with the patient. Applying a physical restraint should be kept as a last resort for use only when less restrictive measures fail. The patient should be placed in a room near the nurses' station so he can be checked frequently if there is no one available to provide one-on-one supervision. A quieter room would probably not help.

Which action demonstrates a break in sterile technique? 1) Remaining 1 foot away from nonsterile areas 2) Placing sterile items on the sterile field 3) Avoiding the border of the sterile drape 4) Reaching 1 foot over the sterile field

Answer: 4) Reaching 1 foot over the sterile field Rationale: Reaching over the sterile field while wearing sterile garb breaks sterile technique. While observing sterile technique, healthcare workers should remain 1 foot away from nonsterile areas while wearing sterile garb, place sterile items needed for the procedure on the sterile drape, and avoid coming in contact with the 1-inch border of the sterile drape.

In the United States, the first programs for training nurses were affiliated with: 1) The military 2) General hospitals 3) Civil service 4) Religious orders

Answer: 4) Religious orders Rationale: When the Civil War broke out, the Army used nurses who had already been trained in religious orders. Although the Army did provide some training, it occurred later than in the religious orders. Although nurses were trained in hospitals, the training and the hospitals were affiliated with religious orders. Civil service was not mentioned in Chapter 1 and was not a factor in the early 1800s. Nursing started with religious orders. The Hindu faith was the first to write about nursing. In the United States, all training for nurses was affiliated with religious orders until after the Civil War.

Which of the following is an example of appropriate behavior when conducting a client interview? 1) Recording all the information on the agency-approved form during the interview 2) Asking the client, "Why did you think it was necessary to seek health care at this time?" 3) Using precise medical terminology when asking the client questions 4) Sitting, facing the client in a chair at the client's bedside, using active listening

Answer: 4) Sitting, facing the client in a chair at the client's bedside, using active listening Rationale: Active listening should be used during an interview. The nurse should face the patient, have relaxed posture, and keep eye contact. Asking "why" may make the client defensive. Note-taking interferes with eye contact. The client may not understand medical terminology or health care jargon.

A patient who suffered a stroke has difficulty swallowing. Which healthcare team member should be consulted to assess the patient's risk for aspiration? 1) Respiratory therapist 2) Occupational therapist 3) Dentist 4) Speech therapist

Answer: 4) Speech therapist Rationale: Respiratory therapists provide care for patients with respiratory disorders. Occupational therapists help patients regain function and independence. Dentists diagnose and treat dental disorders. Speech and language therapists provide assistance to clients experiencing swallowing and speech disturbances. They assess the risk for aspiration and recommend a treatment plan to reduce the risk.

Which organization's standards require that all patients be assessed specifically for pain? 1) American Nurses Association (ANA) 2) State nurse practice acts 3) National Council of State Boards of Nursing (NCSBN) 4) The Joint Commission

Answer: 4) The Joint Commission Rationale: The Joint Commission has developed assessment standards, including that all clients be assessed for pain. The ANA has developed standards for clinical practice, including those for assessment, but not specifically for pain. State nurse practice acts regulate nursing practice in individual states. The NCSBN asserts that the scope of nursing includes a comprehensive assessment but does not specifically include pain.

The nurse calculates a body mass index (BMI) of 18 for a young adult woman who comes to the physician's office for a college physical. This patient is considered: 1) Obese 2) Overweight 3) Average 4) Underweight

Answer: 4) Underweight Rationale: For adults, BMI should range between 20 and 25; BMI less than 20 is considered underweight; BMI 25 to 29.9 is overweight; and BMI greater than 30 is considered obese.

A 75-year-old patient who is 5 feet 7 inches tall and weighs 170 pounds is admitted with dehydration. A nursing diagnosis of Risk for Impaired Skin Integrity is identified for this patient. Which factor places the client at Risk for Impaired Skin Integrity?

Answer: Dehydration Rationale: Dehydration places the patient at risk for impaired skin integrity. Dehydration, caused by fluid volume deficit, causes the skin to become dry and crack easily, impairing skin integrity. People who are very thin or very obese are more likely to experience impaired skin integrity. This patient is of normal height and weight; therefore, his body stature does not place him at risk. There is nothing to suggest that this patient has an impaired nutritional status.

In which phase of the nursing process does the nurse decide whether her actions have successfully treated the client's health problem?

Answer: Evaluation Rationale: In the assessment phase, the nurse gathers data from many sources for analysis in the diagnosis phase. In the diagnosis phase, the nurse identifies the client's health status. In the planning outcomes phase, the nurse and client decide on goals they want to achieve. In the intervention planning phase, the nurse identifies specific interventions to help achieve the identified goal. During the implementation phase, the nurse carries out the interventions or delegates them to other health care team members. During the evaluation phase, the nurse judges whether her actions have been successful in treating or preventing the identified client health problem.

A patient is admitted to the hospital with a diagnosis of primary hyperparathyroidism. A nurse checking the patient's lab results would expect which of the following changes in laboratory findings? A. Elevated serum calcium. B. Low serum parathyroid hormone (PTH). C. Elevated serum vitamin D. D. Low urine calcium.

Answer: A The parathyroid glands regulate the calcium level in the blood. In hyperparathyroidism, the serum calcium level will be elevated. Parathyroid hormone levels may be high or normal but not low. The body will lower the level of vitamin D in an attempt to lower calcium. Urine calcium may be elevated, with calcium spilling over from elevated serum levels. This may cause renal stones.

When obtaining a health history from a patient with acute pancreatitis, the nurse asks the patient specifically about a history of A. smoking B. alcohol use C. diabetes mellitus D. high-fat dietary intake

Answer: B pancreatitis is associated with alcoholism

A nurse motivating a patient in a smoking cessation program should suggest:

Attempting to rid the area of tobacco odor.

Outcome/ Goal, urinary pattern alteration related to enlarged prostate:

Avoid Bladder Distension

usually uses fleet enema, response by nurse:

Are you taking any Vitamin supplements

For a patient with cirrosis, ascites, 4_ pitting edema of the feet and legs, and increased risk for impaired skin integrity, an appropriate nursing intervention is to:

Arrange for a special pressure-relieving mattress.

What countries are endemic to TB?

Asia, Latin America, Carribbean, Middle East, Africa screen pt from these areas

In the OR, a patient tells a circulating nurse that he is going to have the cataract in his left eye removed. If the nurse notes that the consent form indicates that surgery is to be performed on the right eye, what should the nurse's first action?

Ask the patient his name.

What would be the most effective way for a nurse to validate "informed consent"?

Ask the patient what he or she understands regarding the procedure.

When caring for a patient with a suspected viral infection, which medication order would the nurse question?

Aspirin ASA may pose a risk for people of any age when administered to those with viral infections. Adults have experienced Reye's syndrome-like manifestations.

87 year old dehydration, incontinent urine, nursing action best for care:

Assist patient to restroom every 2 hours

A patient with COPD is receiving oxygen at 2L/min. While in the supine position for a bath, the patient complains of shortness of breath. What is the most appropriate first nursing action?

Assist the patient to the fowlers position.

The primary purpose of a soft, high-fiber diet immediately following a MI it to:

Create a high-bulk, soft stool to minimize Vasalva maneuver.

A 6-year-old is brought to the emergency department with a full-body rash and fever. During the nursing assessment, which of the following findings would be most relevant to recognizing the case as potential smallpox rather than varicella? A) Fever has responded to acetaminophen, and the child is playful when temperature is not elevated. B) Fever of 101° F was present for several days before the rash appeared. C) Low-grade fever (100° F or less) has been present ever since the rash became obvious. D) Rash is primarily on the trunk of the body.

B

A business executive develops flu-like symptoms 1 day after returning by air from a trans-Atlantic 2-day conference that involved lengthy meetings into the evening. The scenario best illustrates the interaction of: A) Host and agent. B) Host, agent, and environment. C) Risk and causality. D) Morbidity and disease.

B

A client comes to the local clinic with acute symptoms of fever, nausea, lack of appetite, malaise, and abdominal discomfort. During the course of the assessment, the nurse determines that the client is a health care aide working at a daycare center. These facts are important because: A) Acute hepatitis B is self-limiting. B) Hepatitis A outbreaks commonly occur in facilities where staff change diapers. C) Hepatitis C is a "silent stalker." D) Individuals with chronic liver disease are at greater risk for hepatitis A.

B

A college health nurse is working with students, faculty, and staff to improve environmental air quality. To address the primary cause of air pollution on campus, the nurse plans a precautionary intervention. Which of the following interventions best demonstrates an appropriate approach? A) Encourage the use of electric cars and scooters on campus. B) Increase the use of bicycles, foot-powered scooters, rollerblades, and walking as the primary mode of transportation on campus. C) Make the entire campus a no-smoking zone. D) Establish a policy to reduce electricity consumption in university buildings by raising the thermostat to 78 degrees in the summer and lowering the thermostat to 70 degrees in the winter.

B

A community health nurse manager has integrated exposure history elements into the assessment practices of the health department that are relevant to the urban industrial community served. This strategy indicates that the nurse manager is aware of the relationship between: A) Community strengths and weaknesses. B) Environment and human health/disease. C) Toxicology studies conducted by the Environmental Protection Agency and the environment. D) Federal and state environmental regulations.

B

A community-oriented nurse introduces a community partnership group to the Healthy People 2020 information access objective to use electronic personal health management tools. This is an example of: A) Meta-analysis of research evidence. B) Primary prevention using evidence-based practice (EBP). C) Secondary prevention using EBP. D) Tertiary prevention using EBP.

B

A community-oriented nurse seeks to implement evidence-based practice (EBP) in the community clinic's programs. The best model for the nurse to apply is: A) Action research and review. B) Community development. C) Community research utilization. D) EBP.

B

A nurse is assigned to teach clients sexually transmitted disease (STD) prevention information. The nurse updates her teaching plan to incorporate new guidelines from the Centers from Disease Control and Prevention (CDC). She includes which of the following as updated information during her next teaching session? A) Always use spermicides with condoms to reduce the risk of contracting chlamydia or gonorrhea. B) Condoms can be effective in preventing infections transmitted by fluids from mucosal surfaces but are not always effective in preventing infections transmitted by skin-to-skin contact. C) Condoms should not be used during oral sex, because they are not effective in preventing transmission of infection. D) When genital ulcers are present, condoms should be used to prevent the spread of infection.

B

A nurse is concerned about the accuracy of the purified protein derivative (tuberculin) test in screening individuals with tuberculosis exposure for followup chest radiography. The nurse's concern is related to which aspect of the test's validity? A) Reliability. B) Sensitivity. C) Specificity. D) Variability.

B

A nurse teaches an asthmatic client to recognize and avoid exposure to asthma triggers and assists the client's family in implementing specific protection strategies in the home, such as removing carpets and avoiding pets. This nurse's activities can best be described as: A) Comprehensive assessment. B) Primary prevention. C) Secondary prevention. D) Treatment intervention.

B

A public health nurse in the local health department assists the community in identifying the health need priorities and the services that can best meet these needs in a cost-effective manner. This is an example of the tertiary prevention public health nursing function of: A) Case finding. B) Case management. C) Collaboration. D) Provision of direct services.

B

A state public health region reported 39 cases of meningitis in children 15 years of age and younger to date this year. Seven of those children died. The total population of the region is 780,000, of whom 84,000 are children age 15 years old and younger. Only four cases of meningitis were reported in the public health region during the previous year. No other public health region in the state has an incidence of meningitis that is higher than expected for that region. Based on the information given, the relative frequency of meningitis in the region at this time can best be described as: A) Endemic. B) Epidemic. C) Pandemic. D) Sporadic.

B

Community-oriented nurses use evidence-based practice (EBP) most effectively when they: A) Base care on nationally accepted clinical guidelines, informing clients and community groups that the accepted standards of care need to be universally applied. B) Base care on nationally accepted clinical guidelines, involve clients in individual care decisions, and include community input when applying evidence in practice. C) Make client care decisions using the latest nursing research findings. D) Work with physicians to design client care guidelines for community clinics.

B

In an effort to address West Nile virus, a community increased livestock immunization, began a vector control program, and initiated a community campaign to eliminate standing water reservoirs. This best exemplifies communicable disease control through: A) Health education. B) Multisystem approach. C) Improved public health infrastructure. D) Reduction of environmental hazards.

B

Many behaviors place any individual-regardless of age, gender, ethnicity, or other characteristics-at greater risk for sexually transmitted diseases (STDs). The nurse should include primary prevention interventions in all client encounters through the discussion of: A) Partner notification. B) Safer sex. C) Standard precautions. D) STD testing.

B

Randomized controlled trials are often inappropriate for evaluating many public health interventions. The most common approach to establishing evidence in public health is the use of: A) Blinded studies. B) Case-control studies. C) Expert opinion. D) Research synthesis.

B

The Superfund Amendments and Reauthorization Act (SARA) increased the involvement of the states and their citizens in the cleanup of toxic waste sites and stressed the importance of permanent remedies and innovative treatment technologies. Another important aspect of this federal legislation was that it: A) Provided for the appointment of state emergency response commissions. B) Increased focus on the human health problems related to hazardous waste sites. C) Established a new safety standard of reasonable certainty of no harm that is to be applied to all pesticides used on food. D) Reduced the amount of pollution by mandating cost-effective changes in production, operation, and raw materials use.

B

The community planning board's evaluation of a community intervention (child immunization campaign) carried out by the health department determined that some progress was made toward the desired outcome (target rate of childhood immunization), but the degree of progress achieved was not sufficient to offset the initial effort in terms of cost and time to launch the campaign. The community determined that the rate gain was not adequate when compared with that achieved through similar initiatives in other communities, which obtained better results by using more efficient strategies. The budget for this program was cut. This community decision best exemplifies which statement about evaluation? A) Evaluation should start in the planning phase of the nursing process. B) Evaluation can have unintended consequences. C) Effectiveness is the only true measure of worthiness. D) The power to design, judge, or institute change is important.

B

The monitoring and public reporting of air quality in a local community to assist individuals with asthma or other respiratory conditions best illustrates the application of: A) Compliance and enforcement. B) Environmental epidemiology. C) Secondary prevention. D) Toxicology.

B

The public health nurse ensures that a local community coalition for improving school lunches takes the time to listen to each stakeholder's view, develops a common validated language for discussing the initiative, and shares the credit for the success of the initiative. The public health nurse is adhering to the principles of: A) Collaboration. B) Partnership. C) Public health care. D) Public health nursing.

B

The public health nurse serves as a bridge between at-risk populations and the community's health care resources. This role is based on the nurse's responsibility to: A) Collect and analyze data on public health programs. B) Ensure that all populations have access to affordable, quality health care. C) Monitor and assess critical health status indicators. D) Provide evidence-based use of resources.

B

The client arrives in the emergency room with a penetrating eye injury from wood chips that occurred while cutting wood. The nurse assesses the eye and notes a piece of wood protruding form the eye. What is the initial nursing action? a) apply an eye patch b) perform visual acuity tests c) irrigate the eye with sterile saline d) remove the piece of wood using a sterile eye clamp

B - If the laceration is the result of a penetrating injury, an object may be noted protruding from the eye. This object must never be removed except by the ophthalmologist because it may be holding ocular structures in place. Application of an eye patch or irrigation of the eye may disrupt the foreign body and cause further tearing of the cornea.

A newly diagnosed type 1 diabetic patient likes to run 3 miles several mornings a week. Which teaching will the nurse implement about exercise for this patient? a. "You should not take the morning NPH insulin before you run." b. "Plan to eat breakfast about an hour before your run." c. "Afternoon running is less likely to cause hypoglycemia." d. "You may want to run a little farther if your glucose is very high."

B Rationale: Blood sugar increases after meals, so this will be the best time to exercise. NPH insulin will not peak until mid-afternoon and is safe to take before a morning run. Running can be done in either the morning or afternoon. If the glucose is very elevated, the patient should postpone the run.

A patient with type 1 diabetes who uses glargine (Lantus) and lispro (Humalog) insulin develops a sore throat, cough, and fever. When the patient calls the clinic to report the symptoms and a blood glucose level of 210 mg/dl, the nurse advises the patient to a. use only the lispro insulin until the symptoms of infection are resolved. b. monitor blood glucose every 4 hours and notify the clinic if it continues to rise. c. decrease intake of carbohydrates until glycosylated hemoglobin is less than 7%. d. limit intake to non-calorie-containing liquids until the glucose is within the usual range.

B Rationale: Infection and other stressors increase blood glucose levels and the patient will need to test blood glucose frequently, treat elevations appropriately with insulin, and call the health care provider if glucose levels continue to be elevated. Discontinuing the glargine will contribute to hyperglycemia and may lead to DKA. Decreasing carbohydrate or caloric intake is not appropriate as the patient will need more calories when ill. Glycosylated hemoglobins are not used to test for short-term alterations in blood glucose.

A patient with cirrhosis has a massive hemorrhage from esophageal varices. In planning care for the patient, the nurse gives the highest priority to the goal of a. controlling bleeding. b. maintenance of the airway. c. maintenance of fluid volume. d. relieving the patient's anxiety.

B Rationale: Maintaining gas exchange has the highest priority because oxygenation is essential for life. The airway is compromised by the bleeding in the esophagus and aspiration easily occurs. The other goals would also be important for this patient, but they are not as high a priority as airway maintenance.

A patient with cirrhosis has 4+ pitting edema of the feet and legs and massive ascites. The data indicate that it is most important for the nurse to monitor the patient's a. temperature. b. albumin level. c. hemoglobin. d. activity level.

B Rationale: The low oncotic pressure caused by hypoalbuminemia is a major pathophysiologic factor in the development of ascites and edema. The other parameters should also be monitored, but they are not contributing factors to the patient's current symptoms.

A patient with type 2 diabetes has sensory neuropathy of the feet and legs and peripheral vascular disease evidenced by decreased peripheral pulses and dependent rubor. The nurse teaches the patient that a. the feet should be soaked in warm water on a daily basis. b. flat-soled leather shoes are the best choice to protect the feet from injury. c. heating pads should always be set at a very low temperature. d. over-the-counter (OTC) callus remover may be used to remove callus and prevent pressure.

B Rationale: The patient is taught to avoid high heels and that leather shoes are preferred. The feet should be washed, but not soaked, in warm water daily. Heating pad use should be avoided. Commercial callus and corn removers should be avoided; the patient should see a specialist to treat these problems.

A patient is admitted with an abrupt onset of jaundice, nausea and vomiting, hepatomegaly, and abnormal liver function studies. Serologic testing is negative for viral causes of hepatitis. Which question by the nurse is most appropriate? a. "Have you been around anyone with jaundice?" b. "Do you use any prescription or over-the-counter (OTC) drugs?" c. "Are you taking corticosteroids for any reason?" d. "Is there any history of IV drug use?"

B Rationale: The patient's symptoms, lack of antibodies for hepatitis, and the ABRUPT onset of symptoms suggest toxic hepatitis, which can be caused by commonly used OTC drugs such as acetaminophen (Tylenol). Exposure to a jaundiced individual and a history of IV drug use are risk factors for VIRAL hepatitis. Corticosteroid use does not cause the symptoms listed.

The nurse identifies a nursing diagnosis of risk for impaired skin integrity for a patient with cirrhosis who has ascites and 4+ pitting edema of the feet and legs. An appropriate nursing intervention for this problem is to a. restrict dietary protein intake. b. arrange for a pressure-relieving mattress. c. perform passive range of motion QID. d. turn the patient every 4 hours.

B Rationale: The pressure-relieving mattress will decrease the risk for skin breakdown for this patient. Dietary protein intake may be increased in patients with ascites to improve oncotic pressure. Turning the patient every 4 hours will not be adequate to maintain skin integrity. Passive range of motion will not take pressure off areas like the sacrum that are vulnerable to breakdown.

The client sustains a contusion of the eyeball following a traumatic injury with a blunt object. Which intervention is initiated immediately? a) notify the physician b) apply ice to the affected eye c) irrigate the eye with cool water d) accompany the client to the emergency room

B Treatment for a contusion begins at the time of injury. Ice is applied immediately. The client then should be seen by a physician and receive a thorough eye examination to rule out the presence of other eye injuries.

A nurse is performing an initial post op assessment on a client following upper GI surgery. The client has a NG tube to low, intermittent suction. To best assess the client for the presence of bowel sounds, the nurse should: A. place the stethoscope to the left of the umbilicus. B. turn off the nasogastric suction. C. use the bell of the stethoscope. D. turn the suction on the NG tube to continuous.

B. turn off the nasogastric suction.

When obtaining a health history from a patient with acute pancreatitis, the nurse asks the patient specifically about a history of a. cigarette smoking. b. alcohol use. c. diabetes mellitus. d. high-protein diet.

B Rationale: Alcohol use is one of the most common risk factors for pancreatitis in the United States. Cigarette smoking, diabetes, and high-protein diets are not risk factors.

A diabetic patient is started on intensive insulin therapy. The nurse will plan to teach the patient about mealtime coverage using _____ insulin. a. NPH b. lispro c. detemir d. glargine

B Rationale: Rapid or short acting insulin is used for mealtime coverage for patients receiving intensive insulin therapy. NPH, glargine, or detemir will be used as the basal insulin.

A 32-year-old patient has early alcoholic cirrhosis diagnosed by a liver biopsy. When planning patient teaching, the priority information for the nurse to include is the need for a. vitamin B supplements. b. abstinence from alcohol. c. maintenance of a nutritious diet. d. long-term, low-dose corticosteroids.

B Rationale: The disease progression can be stopped or reversed by alcohol abstinence. The other interventions may be used when cirrhosis becomes more severe to decrease symptoms or complications, but the priority for this patient is to stop the progression of the disease.

A patient with type 2 diabetes is scheduled for an outpatient coronary arteriogram. Which information obtained by the nurse when admitting the patient indicates a need for a change in the patient's regimen? a. The patient's most recent hemoglobin A1C was 6%. b. The patient takes metformin (Glucophage) every morning. c. The patient uses captopril (Capoten) for hypertension. d. The patient's admission blood glucose is 128 mg/dl.

B Rationale: To avoid lactic acidosis, metformin should not be used for 48 hours after IV contrast media are administered. The other patient data indicate that the patient is managing the diabetes appropriately.

Mr. Xenobia's chronic cancer pain has recently increased and he asks the home health nurse what can be done. In relationship to his long-acting morphine, which of the following is an appropriate response by the nurse? a. "if you take more morphine, it will not change your pain effect" b. "I'll call the physician and ask for an increased dose" c. "the amount you are taking now is all I can give you" d. "I'm worried if we increase your dose that you will stop breathing"

B "I'll call the physician and ask for an increased dose" Rationale: there is no ceiling on the analgesic effect of opioid narcotics. Patients develop a tolerance to the effects, which often necessitates an increase in the dose.

A client will be receiving general anesthesia. The nurse reviews the laboratory result of the client and found out that the serum potassium level is 5.8 mEq/L. What should be the nurse's initial response? A) Send the client to surgery B) Notify the anesthesiologist C) Call the surgeon D) Send the client to surgery

B = the nurse should notify the anesthesiologist because a serum potassium level of 5.8 mEq/L places the client at risk for dysrhythmias when under general anesthesia.

As a young adult single mother of a second-grade child has to make a decision regarding the teacher for her child will have in third grade and asks the nurse for advice: All other variables being equal which choice is best: a) A woman with 35 year old of teaching experience b) A man who is 40 years old

B) A man who is 40 years old

During a routine physical, an 11 year old tells the nurse that many students in school are "doing it". How should the nurse respond to this statement: a) Tell the client to talk with parents about sexual matters b) ASk what "doing it" means to the client

B) ASk what "doing it" means to the client

Which of the following nursing diagnosis pertains to a client's learning needs: b) Altered health maintenance related to knowledge deficit: catheter care d) Anxiety related to wife's illness

B) Altered health maintenance related to knowledge deficit: catheter care

A client is hospitialized with numerous acute health problems. According to Maslow's Basic needs model, which nursing diagnosis would take the highest priority: a) Risk for injury related to unsteady gait b) Altered nutrition, less than body requirements related to inability to absorb nutrients c) Self-care deficit related to weakness and debilitation d) Powerlessness related to chronic disease state

B) Altered nutrition, less than body requirements related to inability to absorb nutrients

The most appropriate manner in which to state an intervention directed towards assisting a client with ambulation is: a) Assist patient with ambulation b) Ambulate with client, using gait belt, two times daily for 15 minutes

B) Ambulate with client, using gait belt, two times daily for 15 minutes

Which statement best reflects the nurse's assessment of the fifth vital sign: a) Do you have any complaints b) Are you experiencing any discomfort right now

B) Are you experiencing any discomfort right now

The nurse primarily uses the nursing process in the care of patient's: a) To explain nursing interventions to other health care professionals b) As a problem solving tool to identify and treat patient's health care problems c) As a scientific based process of diagnosing the patient's health care problems d) To establish nursing theory that incorporates the biopsychosocial nature of humans

B) As a problem solving tool to identify and treat patient's health care needs

A client tells the nurse that she does not want to get into the tub for a morning bath. The client has not been bathed for several days. What should the nurse do? a) Assign UAP the task of giving the client's bath b) ASk the client the usual way bathing occurs at home c) Skipping the patient's bath and documenting "refused" is not following at client-centered approach d) Tell the client that a bath is needed and ignore the client's comment

B) Ask the client the usual way bathing occurs at home

The daughter of an 80 year old man is aphastoc after suffering a cerebrovascular accident (stroke) express concern that their father is "always" exposing and playing with himself and his catheter. While they are in the room. Upon assessment the nurse finds the patient pulling on and rubbing his penis. What is the nurse's priority action: b) Assess the client's penis for irritations from the catheter c) ASk the client to keep his linens at waist level when he has visions

B) Assess the client's penis for irritations from the catheter

One of the client's assigned to the nurse's care is to receive a medication that the nurse is not familiar with and is not not listed in the drug reference manual. The best action of the nurse is to: a) Follow the physician's order as written and give the medication b) Call the pharmacy and do further investigating before administering the medication c) Ask the client about this medication d) Call the physician and ask what the medication is and what it is used for

B) Call the pharmacy and do further investigating before administering the medication

After the nurse implements diet instructions for a patient with heart disease the patient can explain the information but fails to make recommended dietary changes. The nurse's evaluations that: a) Learning did not occur because the patient's behavior did not change b) Choosing not to follow the diet is the behaviors that resulted from learning c) The nursing responsibility for helping the patient make dietary changes has been fulfilled d) The teaching methods were ineffective in helping the patient learn the dietary information

B) Choosing not to follow the diet is the behaviors that resulted from learning

A pediatric nurse is preparing a child for cleft palate repair surgery. The nurse recognizes that this type of surgery is categorized as: A. Transplantation surgery B. Constructive surgery C. Palliative surgery D. Reconstructive surgery

B) Cleft palate repair considered constructive surgery because the goal is to restore function in congenital anomalies. Reconstructive surgery serves to restore function to traumatized or malfunctioning tissues and includes plastic surgery or skin grafting. Transplant surgery replaces organs or structures that are diseased or malfunctioning, such as a liver or kidney transplant. Palliative surgery is not a curative and seeks to relieve or reduce the intensity of an illness, such as debridement or necrotic tissue.

When the nurse is evaluating the fluid balance for a patient admitted for hypervolemia associated with multiple draining wounds, the most accurate assessment to include is: a) Skin turgor b) Daily weight c) Presence of edema

B) Daily Weight

Which are the following are normal physiological changes that occur during non- REM sleep: b) Decrease in pulse d) drop in basal metabolic rate

B) Decrease in pulse

A 43 year old is diagnosed with type 2 diabetes mellitus after being admitted to the hospital with an infected foot wound. When applying principles of adult learning, which teaching strategy by the nurse is most likely to be effective: a) Discuss the importance of blood glucose control in maintenance of long term health b) Demonstrate the correct method for cleaning and redressing the wound to the patient c) Assure the patient that the nurse is an expert on management of diabetes complications

B) Demonstrate the correct method for cleaning and redressing the wound to the patient

When preparing to teach an 82 year old Hispanic patient who lives with an adult daughter ways to improve nutrition, which action should the nurse take first: a) Ask the daughter about the patient's food preference b) Determine who shops for groceries and prepare meals

B) Determine who shops for groceries and prepare meals

Upon entering the room, the client is found crying along with the client's spouse. The nurse decides to sit with both of them, offering presence and listening to their fears instead of the planned education. This is an example of which of the following: B) Determining the nurse's needs for assistance c) Supervision delegated care d) Reassuring the client

B) Determining the nurse's needs for assistance

Two days after surgery for an Ileal conduit, the patient will not look at the stoma or participate in care. The patient insists that no one but the ostomy nurse specialists care for the stoma. The nurse identifies a nursing diagnosis of: a) Anxiety related to effects of procedure on lifestyle b) Disturbed body image related to change in body function

B) Disturbed body image related to change in body function

Nurses must use critical thinking in their day-to-day-practice, especially in circumstances surrounding client care and wise use of resources. In which of the following situations would critical thinking be most beneficial: a) Administering IV push medications to critically ill patients b) Educating a home health patient about treatment options c) teaching a new parent car seat safety d) Assisting an orthopedic client with the proper use of crutches

B) Educating a home health patient about treatment options

Which nursing intervention would be the most beneficial in preparing the patient psychologically for ileostomy surgery: a) Include the patient's family in preoperative teaching sessions b) Encourage the patient to express his or her concerns and to ask questions regarding the management of the ileostomy

B) Encourage the patient to express his or her concerns and to ask questions regarding the management of the ileostomy

An older patient receiving intravenous fluids at 175 mL/HR is demonstrating crackles, shortness of breath, and distended neck veins. The nurse recognizes these findings as being which complication of intravenous fluid therapy: b) Fluid volume excess c) Pulmonary embolism

B) Fluid volume excess

Which statement indicates the client needs a sensory aid in the home: a) I tripped over that throw rug again b) I can't hear the doorbell

B) I can't hear the doorbell

The nurse is caring for a client who uses cathartics frequently. Which statement made by the client indicates an understanding of the discharge teaching: a) In the future I will eat a banana every time I take the medication b) I don't have to have a bowel movement every day

B) I don't have to have a bowel movement every day

A patient who is suspected of experiencing respiratory distress from a left-sided pneumothorax should be positioned: a) On the right side b) In semi-fowler's position

B) In the Semi-Fowler's Position

The nurse teaching a 32 year old man with renal failure about the path physiologic mechanism of acid-base balance recognize that the instructions have been understood when the client says: a) I lose too much acid through my kidneys b) My breathing increases to correct imbalances

B) My breathing increases to correct imbalances

What is wrong with the following outcome? Client will be able to climb one flight of stairs without shortness of breath: a) Nothing is wrong b) No target time is given

B) No target time is given

Which of the following may be left in place when a patient is sent to the operating room? A. Wig B. Hearing aid C. Engagement ring D. Well-fitting dentures

B. Hearing aid If a patient is wearing a hearing aid, the perioperative nurse should be notified. Leaving the hearing aid in place enhances communication in the operating room. The nurse should make certain to record that the appliance is in place. Wigs, engagement rings, and dentures are not necessary items to facilitate quality patient care in the operating room.

A patient returns to the clinic with recurrent dysuria after being treated with trimethoprium and sulfamethoxazole (Bactrim) for 3 days. Which action will the nurse plan to take: a) remind the patient about the need to drink 1000 mL of fluids daily b) Obtain a midstream urine specimen for culture and sensitivity testing

B) Obtain a midstream urine specimen for culture and sensitivity testing

When assessing a "PT" eye, which instrument would the nurse use to visualize the retina? A) Otoscope B) Ophthalmoscope C) Stethoscope D) Tuning Fork

B) Ophtalmoscope Only the ophtalmoscope is used to assess the internal eye.

How should the nurse position a client who is complaining of dyspnea: a) A high fowler's position with two pillows behind the head b) Orthopneic position across the over bed table

B) Orthopneic position across the over bed table

When the client has arrived at the nursing unit from surgery, the nurse is most likely to give priority to which of the following assessments? a) pain tolerance b) Pain intensity

B) Pain Intensity

A client has been having pain without any clear pathology for cause. The most appropriately written nursing diagnoses for this client would be which of the following: a) Pain due to unknown factors b) Pain related to unknown etiology c) Pain caused by psychosomatic condition d) Pain manifested by client's report

B) Pain related to unknown etiology

A client has joined a fitness club and is working with the nurse to design a program for weight reduction and increased muscle tone. The client has tried exercise in the past with success, but has not been participating in a program for some time. In order to assess the potential for success with this client, the nurse should evaluate which of the behavior- specific conditions: a) Interpersonal influences b) Perceived benefits of action c) Situational influences

B) Perceived benefits of action

What is primary function of a family? a) Provide everything each member wants b) Provide an environment that supports growth of individuals

B) Provide an environment that supports growth of individuals

The newly admitted client has contractures of both lower extremities. What nursing intervention should be included in the client's plan of care: a) Weight-bearing activities to stimulate joint relaxation b) Range of motion exercises to prevent worsening of contractures c) Exercises to strengthen flexor muscles

B) Range of motion exercises to prevent worsening of contractures

Because of significant concerns about financial problems a middle-aged client complains of difficulty sleeping. Which outcome would be the most appropriate for the nursing care plan? By day 5, the client will: b) Report falling asleep within 20 to 30 minutes c) Have a plan to pay all bills

B) Report falling asleep within 20 to 30 minutes

Which nursing diagnoses would the nurse use for a client prone to falls: a) Deficient knowledge b) Risk for Injury c) Risk for disuse syndrome d) Risk for suffocation

B) Risk for Injury

A client is attending classes on building positive relationships with significant others as well as learning skills to be open minded and respectful to those whose opinions are different. This client is focusing on which component of wellness: a) Physical b) Social c) Emotional d) Environment

B) Social

Which action can the nurse delegate to nursing assistive personnel (NAP) who help with treatment of a patient admitted with tuberculosis and placed on airborne precautions: a) Teach the patient about how to use tissues to dispose of respiratory secretions b) Stock the patients room with all necessary personal protective equipment c) Interview the patient to obtain the names of family members and close contacts d) tell the patient's family members the reason for the use of airborne precautions

B) Stock the patient's room with all the necessary personal protective equipment

The nurse is organizing a wellness project to educate teenagers about keeping their bodies healthy. Which information about diet and exercise should be included: a) Diet is the most important predictor of health b) The most important factors for maintaining health are diet and activity

B) The most important factors for maintaining health are diet and activity

A patient with poor circulation to the feet requires teaching about foot care. Which learning goal should the nurse include in the teaching plan? a) The nurse will demonstrate the proper technique for trimming toenails b) The patient will list three ways to protect the feet from injury by discharge d) The patient will understand the rationale for proper foot care after instructions

B) The patient will list three ways to protect the feet from injury by discharge

The nurse is preparing written handouts to be used as part of the standardized teaching plan for patient's who have been recently diagnosed with diabetes. Which of the following statements would be appropriate to include in the handouts: a) Polyphagia, polydipsia, and polyuria are common symptoms of Diabetes mellitus b) The use of the right foods can help in keeping blood glucose at a near-normal level c) Some diabetes control blood glucose with oral medications or nutritional interventions d) Diabetes mellitus is characterized by chronic hyperglycemia and the associated symptoms

B) The use of the right foods can help in keeping blood glucose at a near-normal level

Which factor reduces the risk of electrical hazards: a) two-pronged electrical plugs b) Three-prolonged electrical plugs

B) Three-prolonged electrical plugs

The nurse notes that the tube fed client has shallow breathing and dusky color. The feeding is running at the prescribed rate. What is the nurses priority action: a) Place the client in high fowler's position b) Turn off tube feeding d) Assess the patient's bowel sounds

B) Turn off the tube feeding

The nursing diagnosis Risk for Impaired Skin Integrity related to sensory-perception disturbance would best fit a client who: a) Cut a foot by stepping on broken glass b) Uses a wheelchair due to paraplegia

B) Uses a wheelchair due to paraplegia

An oxygen delivery system is prescribed for a client with chronic obstructive pulmonary disease (COPD) to deliver a precise oxygen concentration. Which of the following types of oxygen delivery systems would the nurse anticipate to be prescribed: a) Face tent b) Venture Mask

B) Venture Mask

When reviewing both the client's problem list against the various identified nursing diagnoses, both of which include client and family input, the nurse is utilizing of the following processes to minimize diagnostic error: a) Understanding what is normal vs. what is not normal b) Verifying c) Consulting resources d) Basing diagnoses on patterns

B) Verifying

A patient with frequent urinary tract infections ask the nurse how she can prevent the reoccurence. The nurse should teach the client to: a) Douche after intercourse b) Void every three hours

B) Void every three hours

The nurse is caring for the patient with clostridum difficile. Which intervention should the nurse implement to prevent nosocomial spread to other clients: a) Wash hands with betadine for 2 min after giving care b) Wear nonsterile gloves when handling GI excretions

B) Wear nonsterile gloves when handling GI excretions

To assess a patient's readiness to learn before planning, teaching activities, which question should the nurse ask: a) What kind of work and leisure activities do you do b) What information do you think you need right now c) Do you have any religious beliefs that are inconsistent with the treatment

B) What information do you think you need right now

During an initial interview the client makes this statement, I'm really not that sick or in pain right now. The nurses best response is: a) It's ok to be worried surgery is a big step b) What kind of questions do you have about your surgery c) I think these are things you should be asking your doctor d) have you had surgery before

B) What kind of questions do you have about your surgery

A client is admitted to the hospital after vomiting for 3 days. Which arterial blood gas results would the nurse expect to find in this patient: a) pH 7.30, PaCO2 50, HCO3 27 b) pH 7.47, PaCO2 43, HCO3 28 c) pH 7.43, PaCO2 50, HCO3 28

B) pH 7.47, PaCO2 43, HCO3 28

The home care nurse is planning to visit a "PT" with Meniere's disease. The nurse reviews the physician prescriptions and expects to note that which of the following dietary measures is prescribed? A)A low-fiber diet with decreased fluids B)A low-sodium diet and fluid restriction C)A low-fat diet with restriction of citrus fruit D)A low-Carbohydrate diet and the elimination of red meats

B)A low-sodium diet and fluid restriction Dietary changes such as salt and fluid restriction that reduce the amount of endolymphatic fluid are sometimes prescribed for "PT" with MD

The operating room is aware that which of the following patients are at a greater risk related to a surgical procedure? A. 34 yr old female B. 83 yr old female C. 48 yr old male D. 8 yr old male

B)Infants and older adults are at greatest risk from surgery than are children and young or middle-aged adults. Physiologic changes associated with aging increase the surgical risk for older patients.

The home-care nurse visit an elderly "PT" with arthritis. The "PT" complains of difficulty instilling glaucoma eye drops because of shaking hands caused by the arthritis. Which instruction should the nurse plan to provide to the "PT" to alleviate this problem? A)Tilt the head back to instill the eye drops B)Lie down on the bed or sofa to instill the eye drops C)A family member will have to instill the eye drops D)Keep the eye drops in the refrigerator so that they will thicken and be easier to instill

B)Lie down on the bed or sofa to instill the eye drops Older "PT" with arthritis or shacking hands have difficulty instilling their own eye drops. The "PT" is told to sit or lie down because tilting the head back can lead to a loss of balance. *Eye drop SHOULD NOT be places in refrigerator unless specifically prescribed

A patient returning to the floor after orthopedic surgery is complaining of nausea. The nurse is aware that an appropriate intervention is to: A. Hold all medications. B. Avoid strong smelling foods. C. Avoid oral hygiene until the nausea subsides. D. Provide clear liquids with a straw.

B)Nursing care for a patient with nausea includes avoiding strong smelling foods. Providing oral hygiene, administering prescribed medications (especially if medications ordered are anti-nausea/antiemetics), and avoid the use of a straw.

The nurse is planning on instructed a "PT" with chronic vertigo about safety measures to prevent exacerbation of symptoms or injury. The nurse plans to teach the "PT" that it is important to. A)Turn head slowly when spoken to B)Remove throw rugs and clutter in the home. C)Drive at times when the "PT" doesn't feel dizzy D)Walk into the bedroom and lie down when vertigo is experienced

B)Remove throw rugs and clutter in the home The "PT" should maintain a home that is in a clutter-free state and have throw rugs removed, because the effort of trying to regain balance after slipping could trigger the onset of vertigo

The nurse is providing instructions to a "PT" and family regarding home-care after left eye cataract removal. The nurse tells the "PT" and family which of the following about positioning during the postoperative period? A)Sleep only on the left side B)Sleep only on the right side or back C)Bend below the waist as often as you are able D)Lower the head between the knees three times a day

B)Sleep only on the right side or back After cataract surgery the "PT" should not sleep on the operative side to prevent the development of edema. The "PT" should also avoid bending below the level of the waist or lowering the head, because these actions will increase interocular pressure

Wet-to-Dry dressings for mechanical debridement of a wound should:

Be mostly moist, not wet, when aplied

The nurse is developing a teaching plan for a client who will undergo a stapedectomy for treatment of otosclerosis. Which point should the plan include? a. ringing in the ears is common after surgery b. vertigo and dizziness are common after surgery c. hearing should return immediately after surgery d. excessive drainage is common after surgery

B. Vertigo is the most frequent complication of stapedectomy. The patient should move slowly to avoid triggering or worsening vertigo and should ask for assistance with ambulation. Ringing in the ears rarely follows this surgery and should be reported to the doctor if it does. Hearing typically decreases after surrey bc of ear packing and tissue swelling, but commonly returns over the next 2-6 weeks. Usually, post-op drainage and pain are minimal Excessive drainage should be reported.

The nurse should avoid asking the client which of the following leading questions during a client interview? A. "What medication do you take at home?" B. "You are really excited about the plastic surgery, aren't you?" C. "Were you aware I've has this same type of surgery?" D. "What would you like to talk about?"

B. "You are really excited about the plastic surgery, aren't you?" Rationale: A leading question directs the client's answer. The phrasing of the question indicates an expected answer. The client may be influenced by the nurse's expectations and may give inaccurate responses. This process can result in an error in diagnostic reasoning.

Immunizations 6 mo.

Hep B (3 of 3) DTaP [Diptheria, Tetanus, Pertussis] (3 of 4) PCV [Pneumococcal] (3 of 4) IPV [Inactivated Polio Virus] (3 of 4) Begin yearly Influenza vaccination

The nurse would do which of the following activities during the diagnosing phase of the nursing process? Select all that apply. A. Collect and organize client information B. Analyze data C. Identify problems, risk, and client strengths D. Develop nursing diagnoses E. Develop client goals

B. Analyze data C. Identify problems, risk, and client strengths D. Develop nursing diagnoses Rationale: The diagnosing phase of the nursing process involves data analysis, which leads to identification of problems, risks, and strengths and the development of nursing diagnoses. Collecting and organizing client data is done in the assessment phase of the nursing process. Goal setting occurs during the planning phase.

Which of the following descriptors is most appropriate to use when stating the "problem" part of a nursing diagnosis? A. Grimacing B. Anxiety C. Oxygenation saturation 93% D. Output 500 mL in 8 hours

B. Anxiety Rationale: The problem part of a nursing diagnosis should state the client's response to a life process, event, or stressor. These are categorized as nursing diagnoses. The incorrect options are cues the nurse would use to formulate the nursing diagnostic statement.

The functional health pattern assessment data states: "Eats three meals a day and is of normal weight for height." The nurse should draw which of the following conclusions about this data? Select all that apply. A. Client has an actual health problem B. Client has a wellness diagnosis C. Collaborative health problem needs to be written D. Possible nursing diagnosis exists E. Specific questions about the diet should be asked next

B. Client has a wellness diagnosis E. Specific questions about the diet should be asked next Rationale: The description indicates a healthy pattern of nutrition for the client. A wellness diagnosis might be stated as: "Potential for enhanced nutrition." An actual health problem is a client problem that is currently present. The nurse should also do a diet assessment to determine the quality of the food eaten during meals. These actions by the nurse are within the scope of independent nursing practice and are not collaborative in nature.

The client reports nausea and constipation. Which of the following would be the priority nursing action? A. Collect a stool sample B. Complete an abnormal assessment C. Administer an anti-nausea medication D. Notify the physician

B. Complete an Abdominal assessment Rationale: Assessment involves the systematic collection of data about an individual upon which all subsequent phases of the nursing process are built. In response to a client's complaint, a nurse assesses a specific body system to obtain data that will help the nurse make a nursing diagnosis and plan the client's care. The other options reflect interventions, which are not timely unless there is first a complete assessment.

HepA immunization

HepA Series (second dose administered 6-18 mo after the first) min age: 12 mo. 2-3 year recommended

The client is being discharged to a long-term care (LTC) facility. The nurse is preparing a progress note to communicate to the LTC staff the client's outcome goals that were met and those that were not. To do this effectively, the nurse should: A. Formulate post-discharge nursing diagnoses B. Draw conclusion about resolution of current client problems C. Assess the client for baseline data to be used at the LTC facility D. Plan the care that is needed in the LTC facility

B. Draw conclusion about resolution of current client problems Rationale: Terminal evaluation is done to determine the client's condition at the time of discharge. This evaluation is best reflected in option 2 because it focuses on which goals were achieved and which were not. Ongoing evaluation is done while or immediately after implementing a nursing intervention. Intermittent evaluation is performed at specified intervals, such as twice a week. Items related to care post-discharge (options 2, 3, and 4) should be done on admission to the LTC facility.

Which of the following things must the nurse working with diabetic clients keep in mind about Hyperglycemic Hyperosmolar Nonketotic Syndrome (HHNS)? A. This syndrome occurs mainly in people with Type I Diabetes B. It has a higher mortality rate than Diabetic Ketoacidosis C. The client with HHNS is in a state of overhydration D. This condition develops very rapidly

B. It has a higher mortality rate than Diabetic Ketoacidosis HHNS occurs only in people with Type II Diabetes. It is a medical emergency and has a higher mortality rate than Diabetic Ketoacidosis. This condition develops very slowly over hours or days.

The nurse would be alerted to the occurrence of malignant hyperthermia when the patient demonstrates: A. Hypocapnia B. Muscle rigidity C. Decreased body temperature D. Confusion upon arousal from anesthesia

B. Muscle rigidity Malignant hyperthermia is a metabolic disease characterized by hyperthermia with rigidity of skeletal muscles occurring secondary to exposure to certain anesthetic agents in susceptible patients. Hypoxemia, hypercarbia, and dysrhythmias may also be seen with this disorder.

The nurse is assessing the confused client, in trying to determine the client's level of pain, the nurse should: a. be aware that confused clients don't feel as much pain due to their confusion b. observe the client carefully for changes in behavior or vital signs c. ask the client's family how much pain the client normally has d. use only pain scales that feature numbers or "faces" the client can point to.

B. Observe the client carefully for changes in behavior or vital signs Rationale: the nurse should observe the confused client for nonverbal cues to pain

If a 77-year-old patient who is NPO after surgery has dry oral mucous membranes, which of the following is the most appropriate nursing intervention? A. Increase oral fluid intake. B. Perform oral hygiene frequently. C. Swab the inside of the mouth with petroleum. D. Increase the rate of IV fluid administration.

B. Perform oral hygiene frequently. Frequent oral hygiene will help alleviate discomfort for a patient who is NPO. IV fluid rate is prescribed by the physician. Petroleum is always inappropriate intraorally. Oral fluid intake is contraindicated in a patient who is NPO.

The nurse would do which of the following during the implementation phase of the nursing process when working with a hospitalized adult? A. Formulate a nursing diagnosis of impaired gas exchange B. Record in the medical record the distance a client ambulate in the hall C. Write individualized nursing orders in the care plan D. Compare client responses to the desired outcomes for pain relief

B. Record in the medical record the distance a client ambulate in the hall Rationale: The implementation phase of the nursing process involves carrying out or delegating the nursing interventions and recording nursing activities and client responses in the medical records. Option 1 represents diagnosing. Option 3 represents planning. Option 4 represents evaluation.

Which of the following preoperative assessment findings should be reported to a surgeon for preoperative treatment? A. Serum sodium level of 140 mEq/L B. Serum potassium level of 3 mEq/L C. Hb concentration of 13.5 mg/dl D. Partial thromboplastin time of 25 seconds

B. Serum potassium level of 3 mEq/L. Electrolyte imbalances increase operative risk. Preoperative laboratory results should be checked to see whether they are within the normal range. The normal potassium level is 3.5 to 5.0 mEq/L. A low serum potassium level puts the patient at risk for cardiac dysrhythmias. A serum sodium level of 140 mEq/L is considered a normal value. An Hb concentration of 13.5 mg/dl is considered a normal value by most laboratory standards and does not interfere with operative decisions. A partial thromboplastin time of 25 seconds is a normal value and conducive to proceeding with a surgical procedure.

The nursing diagnosis is Risk for impaired skin integrity related to immobility and pressure secondary to pain and presence of a cast. Which of the following desired outcomes should the nurse include in the care plan? A. Client will be able to turn self by day 3 B. Skin will remain intact and without redness during hospital stay C. Client will state pain relieved within 30 minutes after medication D. Pressure will be prevented by repositioning client every 2 hours

B. Skin will remain intact and without redness during hospital stay Rationale: The human response/label is what needs to change (Risk for impaired skin integrity). The label suggests the outcomes. In this case, "skin will remain intact" is the desired outcome for a client at risk for impaired skin integrity. Option 1 addresses immobility. Option 3 addresses pain. Option 4 is an intervention.

Which of the following is most likely to be effective in meeting a patient's teaching/learning needs preoperatively? A. Teaching only the patient B. Teaching the patient and family C. Using brief verbal instructions D. Using only written instructions

B. Teaching the patient and family. A nurse should determine learning needs preoperatively and teach both the patient and the family before surgery. Using only written instructions does not provide the opportunity for evaluation for learning. Brief verbal instructions are often forgotten. Teaching only the patient limits learning preoperatively because the patient can be anxious and not receptive to new information.

Which desired outcome written by the nurse is correctly written and measurable? A. Client will have a normal bowel pattern by April 2 B. The client will lose 4 lbs. within next 2 weeks C. The nurse will provide skin care at least 3 times each day D. The client will breathe better after resting for 10 minutes

B. The client will lose 4 lbs. within next 2 weeks Rationale: An outcome statement must describe the observable client behavior that should occur in response to the nursing interventions. It consists of a subject, action verb, conditions under which the behavior is to be performed, and the level at which the client will perform the desired behavior. Each of the incorrect options lacks one of these required elements. Option 1 is not measurable. Option 3 is a nursing goal rather than a client goal. Option 4 does not include the level at which the behavior should be performed.

A pt has cerebral edema, what is an inappropriate IV solution to use for this pt ?

Hypotonic solutions like 0.45% normal saline

A patient has the following preoperative medication order: morphine 10 mg with atropine 0.4 mg IM. The nurse informs the patient that this injection will A. decrease nausea and vomiting during and after surgery. B. decrease oral and respiratory secretions, thereby drying the mouth. C. decrease anxiety and produce amnesia of the preoperative period. D. induce sleep, so the patient will not be aware during transport to the operating room.

B. decrease oral and respiratory secretions, thereby drying the mouth. Atropine, an anticholinergic medication, is frequently used preoperatively to decrease oral and respiratory secretions during surgery, and the addition of morphine will help to relieve discomfort during the preoperative procedures. Antiemetics decrease nausea and vomiting during and after surgery, and scopolamine and some benzodiazepines induce amnesia. An actual sleep state is rarely induced by preoperative medications unless an anesthetic agent is administered before the patient is transported to the operating room.

Conscious sedation is being considered for a patient undergoing a cervical dilation and endometrial biopsy in the health care provider's office. The patient asks the nurse, "What is this conscious sedation?" The nurse's response is based on the knowledge that conscious sedation A. can be administered only by anesthesiologists or nurse anesthetists. B. enables the patient to respond to commands and accept painful procedures. C. is so safe that it can be administered by nurses with direction from health care providers. D. should never be used outside of the OR because of the risk of serious complications.

B. enables the patient to respond to commands and accept painful procedures. Conscious sedation is a moderate sedation that allows the patient to manage his or her own airway and respond to commands, and yet the patient can emotionally and physically accept painful procedures. Drugs are used to provide analgesia, relieve anxiety and/or provide amnesia. It can be administered by personnel other than anesthesiologists, but nurses should be specially trained in the techniques of conscious sedation to carry out this procedure because of the high risk of complications resulting in clinical emergencies.

The home health care nurse is visiting a client with a nursing diagnosis of Activity Intolerance related to the effects of inflammation secondary to rheumatoid arthritis. While planning care with the client, the nurse will: (select all that apply) A: Perform procedures required by the client B: Identify resources to help the client accomplish tasks that require a great deal of energy C: Prioritize care needs D: Set a desired outcome to demonstrate energy conservation E: Assess the client's ability to take care of herself

B: Identify resources to help the client accomplish tasks that require a great deal of energy C: Prioritize care needs D: Set a desired outcome to demonstrate energy conservation

The nurse lists the different types of planning that nurses perform, and includes: A: The care plan, interventions, and outcomes B: Initial planning, ongoing planning, and discharge planning C: Interventions, outcomes, and evaluation D: Collaboration with health officials, management of care, and implementation

B: Initial planning, ongoing planning, and discharge planning

The nurse reviews which level of the Nursing Interventions Classification (NIC) taxonomy for interventions? A: Classes B: Level 3 C: Level 1 D: Level 2

B: Level 3

"The client will ambulate 20 yards without assistance in 8 weeks." The nurse recognizes this is an example of a: A: Nursing intervention B: Long-term goal C: Short-term goal D: Rationale

B: Long-term goal

The nurse assesses a postoperative client with an abdominal wound and finds the client drowsy when not aroused. The client's pain is ranked 2 on a scale of 0 to 10, vital signs are within preoperative range, extremities are warm with good pulses but very dry skin. The client declines oral fluids due to nausea, and reports no bowel movements in the past 2 days. Hip dressing is dry with drains intact. Which element is most likely to be considered of high priority for a change in the current care plan? A: Pain B: Nausea C: Constipation D: Potential for wound infection

B: Nausea

A taxonomy of nursing outcome statements was developed to describe measureable states, behaviors, or perceptions to respond to which part of the nursing process? A: Nursing outcomes B: Nursing interventions C: Nuring assessments D: Nursing goals

B: Nursing interventions

Outcomes and goals should not only be compatible with the nurse and client, but also with: A: The family B: Other health care workers assigned to the client's care C: The classification system used D: Facility policy

B: Other health care workers assigned to the client's care

In the long-term care setting, it is important for the nurse to: A: Revise care plans to reflect standardized protocols B: Revise care plans to reflect goal achievement C: Carry out dependent nursing actions D: Have an informal care plan

B: Revise care plans to reflect goal achievement

The nurse is planning the client's care. One of the activities performed by the nurse during this phase is: A: Analyzing data B: Selecting nursing interventions C: Determining the nurse's need for assistance D: Reassessing the client

B: Selecting nursing interventions

The nurse uses a standardized care plan to develop an individualized care plan for each client. Match the nursing action to the appropriate holistic care plan approach or rationale: The nurse utilizes the agency's "Brain Tumor" care plan. A: Ongoing individualized care planning B: Standardized care plan C: Complete individualized plan of care D: Individualized care plan

B: Standardized care plan

When setting priorities for delivering care, the nurse considers which of the following processes first? A: The client's home setting B: The most important disease process that the client is experiencing C: The client's ability to pay D: The client's ethnic background

B: The most important disease process that the client is experiencing

The nurse recognizes which of the following as a benefit of using a standardized care plan? A: No individualization is needed B: The nurse chooses from a list of interventions C: They are much shorter than the nurse-authored care plans D: They have been approved by accrediting agencies

B: The nurse chooses from a list of interventions

The best reason for the development of the nursing outcomes classification (NOC) is that: A: Measuring outcomes makes it easier for clients to reach their goals B: The use of NOC will enable nursing data to be analyzed to help improve nursing practice C: NOC outcomes identify the specific behavior to be measured D: A classification system is helpful in writing care plans

B: The use of NOC will enable nursing data to be analyzed to help improve nursing practice

Before a meal, a nurse should test the swallowing ability of a patient with dysphagia by assessing the patient's:

Before feeding a patient with dysphagia, the nurse should assess the patient's gag reflex, level of consciousness, ability to produce a cough, and ability to swallow voluntary to decrease the risk of aspiration.

When does the nurse assess vital signs during a blood transfusion?

Before the transfusion for baseline vitals and again after the first 15 minutes.

Characteristics of clinical pathways include all of the following except:

Being a nurse-centered orientation rationale: Clinical pathways help reduce expensive variations in care. Clinical pathways are interdisciplinary plans of care that outline the optimal sequencing and timing of interventions for clients with a particular diagnosis, procedure, or symptom.

D. Dry nasal mucosa

Belcomethasone (Beconase) has been prescribed for a client with allergic rhinitis. The nurse teaches the client that which is the most common side effect from continuous use? A. Dizziness B. Rinorrhea C. Hallucinations D. Dry nasal mucosa

327 Which of following is indicative of a potential complicaiton assoc. w/tonic-clonic seiure?

Blood on pillow. R: The complications assoc. w/seizures include airway compromise, extremity and teeth injuries, and tongue lacerations. Night seizures can cause child to bite down on tongue.

45 year old had no interest in sex, not in 16 years, nurse interprets:

Both parents share lack of desire there is no problem

324. Mother arrives at ER w/5yr. old and states child fell off bunk bed. A head injury is suspected, and nurse is assesing the child continuously for signs of ICP. Which of following is late sign of increased ICP in child?

Bradycardia R: Late signs of increased ICP include a significant decrease in level of consciousness, bradycrdia, and fixed and dilated pupils. A bulging fontanel and dilated scalp veins are early s/of ICP and w/be noted in an infant rather than 5 yr. old child. Nausea is early sign of ICP.

Which of the following is an example of an affective outcome?

By 6/12/12, the patient will verbalize valuing health sufficiently to practice new health behaviors to prevent recurrence of leg ulcer (affective outcomes describe changes in patient values, beliefs, and attitudes)

A client newly diagnosed with human papillomavirus (HPV) infection, herpes simplex virus 2 (HSV-2) infection, and syphilis asks, "Okay, so how do I get rid of all this stuff?" In developing a plan of care, the nurse recognizes that it is essential to address: A) Correct use of condoms to prevent transmission of all sexually transmitted diseases (STDs). B) Cures for each of the STDs identified. C) Risk of skin-to-skin contact in transmitting the identified STDs. D) Safety of sexual contact in the absence of lesions.

C

A local health department in the Midwest reports cases of certain diseases to the state health department for inclusion in the National Notifiable Diseases Surveillance System (NNDSS). From the perspective of community-level disease monitoring, which of the following 3-year trends in incidence rates for hepatitis A would be of local, state, and national interest? A) 1998 = 2/100,000; 1999 = 3/100,000; 2000 = 1/100,000 B) 1998 = 4/100,000; 1999 = 8/100,000; 2000 = 6/100,000 C) 1998 = 12/100,000; 1999 = 8/100,000; 2000 = 31/100,000 D) 1998 = 16/100,000; 1999 = 24/100,000; 2000 = 9/100,000

C

A rural public health nurse is in the first phase of a community assessment to determine the community health status characteristics of the local county. This initial data gathering should most likely begin with which agency? A) County public health department. B) National Centers for Disease Control and Prevention. C) State vital statistics bureau. D) U.S. Census Bureau.

C

A state public health region reported 39 cases of meningitis in children 15 years of age and younger to date this year. Seven of those children died. The total population of the region is 780,000, of whom 84,000 are children age 15 years old and younger. What is the prevalence proportion of meningitis in this region thus far in the current year? A) 4.1/100,000 B) 5/100,000 C) 46/100,000 D) 50/100,000

C

After consulting with the health department director, a public health nurse collaborates with a housing advocate service and legal counsel on behalf of the nurse's clients who live in substandard housing under fear of eviction. The nurse is applying the _______ component of the nursing process to a _______ level of practice. A) Evaluation + systems. B) Assessment + community. C) Implementation + systems. D) Diagnosis + community.

C

Although infectious disease epidemics are still the major cause of death worldwide, they have subsided in the United States because of improvements in nutrition and sanitation, the discovery of antibiotics, and the development of vaccines. Infectious diseases have not vanished, however, and remain a continuing cause of concern. Healthy People 2020 has a number of objectives aimed at reducing these illnesses because of the morbidity, mortality, and costs associated with infectious diseases. One such costly disease trend related to an increase in the performance of invasive diagnostic and surgical procedures, the use of broad-spectrum antibiotics, and treatment with immunosuppressive drugs is the rise of: A) Escherichia coli 0157:H7. B) Multisyndrome effect. C) Hospital acquired infections. D) Severe acute respiratory syndrome.

C

An example of secondary prevention of infectious disease is: A) Malaria chemoprophylaxis. B) Pneumocystis carinii pneumonia chemoprophylaxis for people with AIDS. C) Quarantine. D) Restaurant inspections.

C

Collaboration is an intervention that would be located where in the Intervention Wheel? A) Red wedge at the individual/family level of practice. B) Blue wedge at the community level of practice. C) Orange wedge at the community level of practice. D) Green wedge at the systems level of practice.

C

Examples of the application of evidence-based practice (EBP) to improve public health nursing can be found in research projects designed to test the effectiveness of public health nursing interventions related to the core functions and essential services of public health. These projects are associated with: A) Agency on Healthcare Research and Quality. B) Cochrane Database of Systematic Reviews. C) The Intervention Wheel. D) U.S. Preventive Services Task Force.

C

If the two major goals of Healthy People 2020 are to be achieved, collaboration is essential for public health nursing practice, and collaboration with existing groups at the local level is encouraged for which of the following reasons? A) The federal government is ultimately responsible for the health status of the nation. B) The public demands that the government protect the people. C) Public health departments do not have the resources to accomplish these goals independently. D) State health agencies must take a universal approach to achieving objectives.

C

Rapid changes in public health are providing a challenge to public health nurses because there is neither time nor staff to provide nurses with the on-the-job training needed to acquire the core public health competencies required of the public health nurse. This resulted in revisions to the American Nurses Association (ANA)'s Scope and Standards of Public Health Nursing Practice in 2005 that established: A) Core public health functions as the competency framework. B) Minnesota Department of Health's intervention wheel as the practice competencies. C) Standards for baccalaureate- and master's-prepared public health nurses. D) Quad Council principles as the primary framework for practice.

C

State public health agency responsibilities include: A) Conducting community health assessments. B) Enforcing public health codes. C) Monitoring health status. D) Providing expertise that facilitates evidence-based practice.

C

The most important aspect of the nursing community assessment phase can best be described as: A) Analyzing and synthesizing data. B) Collecting and gathering data. C) Formulating a community nursing diagnosis. D) Identifying problem correlates.

C

The three components of the Intervention Wheel are: A) Communities, systems, and individuals/families. B) Interventions, color wedges, and levels of practice. C) Population base, levels of practice, and public health interventions. D) Populations at risk, populations of interest, and levels of practice.

C

When nurses work with communities, "best practices"-the application of the best available evidence to improve practice-must also be: A) Accessible and diverse. B) Competent and compliant. C) Culturally and financially appropriate. D) Reasonable and deliverable in a timely fashion.

C

Which action by the community-oriented nurse best illustrates a partnership for health? A) Assisting a school nurse in conducting vision screening of elementary school children. B) Developing a volunteer program for teaching parenting skills. C) Helping a group of citizens concerned about potential environmental hazards collect relevant health data and develop needed interventions. D) Informing a neighborhood council that smoking is its major community health problem.

C

Which of the following article titles include(s) an example of epidemiologic distribution and determinants (select all that apply)? A) Can Operating Room Nurses Measurably Reduce Patient Anxiety? B) Characteristics of Patients Newly Diagnosed with Tuberculosis C) Comparison of Postinsecticide Exposure Incidence of Atopic Dermatitis in Migrant Farmworkers and Land-Owning Farmers in Southwestern Utah D) Postpartum Nurses' Reaction to Rotating Shifts Compared with Assigned Stable Shift

C

A diagnosis of hyperglycemic hyperosmolar nonketotic coma (HHNC) is made for a patient with type 2 diabetes who is brought to the emergency department in an unresponsive state. The nurse will anticipate the need to a. administer glargine (Lantus) insulin. b. initiate oxygen by nasal cannula. c. insert a large-bore IV catheter. d. give 50% dextrose as a bolus.

C Rationale: HHNC is initially treated with large volumes of IV fluids to correct hypovolemia. Regular insulin is administered, not a long-acting insulin. There is no indication that the patient requires oxygen. Dextrose solutions will increase the patient's blood glucose and would be contraindicated.

. A patient with cirrhosis who is being treated with spironolactone (Aldactone) and furosemide (Lasix) has a serum sodium level of 135 mEq/L (135 mmol/L) and serum potassium 3.2 mEq/L (3.2 mmol/L). Before notifying the health care provider, the nurse should a. administer the furosemide and withhold the spironolactone. b. give both drugs as scheduled. c. administer the spironolactone. d. withhold both drugs until talking with the health care provider.

C Rationale: Spironolactone is a potassium-sparing diuretic and will help to increase the patient's potassium level. The nurse does not need to talk with the doctor before giving the spironolactone, although the health care provider should be notified about the low potassium value. The furosemide will further decrease the patient's potassium level and should be held until the nurse talks with the health care provider.

A patient with severe cirrhosis has an episode of bleeding esophageal varices. To detect possible complications of the bleeding episode, it is most important for the nurse to monitor a. prothrombin time. b. bilirubin levels. c. ammonia levels. d. potassium levels.

C Rationale: The blood in the GI tract will be absorbed as protein and may result in an increase in ammonia level since the liver cannot metabolize protein well. The prothrombin time, bilirubin, and potassium levels should also be monitored, but these will not be affected by the bleeding episode.

The doctor is interested in how well a client has controlled their blood glucose since their last visit. What lab values could the nurse evaluate to determine how well the client has controlled their blood glucose over the past three months?

C HbgA1c is a blood test used to determine how well blood glucose has been controlled for the last three months.

The client tells the nurse that the client really misses having sugar with tea in the morning. What is an alternative that the nurse could advise them to help sweeten their drink. a. Oatrim c. sucralose b. Olestra d. tannin

C Aspartame is the generic name for a sweetener composed of two amino acids, phenylalanine and aspartic acid. Olestra and Oatrim are fat replacers and tannin is an acid found in some foods such as tea.

A patient hospitalized with possible acute pancreatitis has severe abdominal pain and nausea and vomiting. The nurse would expect the diagnosis to be confirmed with laboratory testing that reveals elevated serum a. calcium. b. bilirubin. c. amylase. d. potassium.

C Rationale: Amylase is elevated early in acute pancreatitis. Changes in bilirubin, calcium, and potassium levels are not diagnostic for pancreatitis.

The client is admitted to a comprehensive rehabilitation center for continuing care,following a motor vehicle crash. While the admitting nurse will develop the initial care who will be involved with the ongoing planning of this client's care: a) The admitting nurse continues to assume that responsibility b) All nurses who work with the client c) Everybody involved in the client's care d) The client and the client's support system

C) Everybody involved in the client's care

When assessing the neurologic status of a patient with a diagnosis of hepatic encephalopathy, the nurse asks the patient to a. stand on one foot. b. ambulate with the eyes closed. c. extend both arms. d. perform the Valsalva maneuver.

C Rationale: Extending the arms allows the nurse to check for asterixis, a classic sign of hepatic encephalopathy. The other tests might also be done as part of the neurologic assessment but would not be diagnostic for hepatic encephalopathy.

A type 1 diabetic patient who was admitted with severe hypoglycemia and treated tells the nurse, "I did not have any of the usual symptoms of hypoglycemia." Which question by the nurse will help identify a possible reason for the patient's hypoglycemic unawareness? a. "Do you use any calcium-channel blocking drugs for blood pressure?" b. "Have you observed any recent skin changes?" c. "Do you notice any bloating feeling after eating?" d. "Have you noticed any painful new ulcerations or sores on your feet?"

C Rationale: Hypoglycemic unawareness is caused by autonomic neuropathy, which would also cause delayed gastric emptying. Calcium-channel blockers are not associated with hypoglycemic unawareness, although -adrenergic blockers can prevent patients from having symptoms of hypoglycemia. Skin changes can occur with diabetes, but these are not associated with autonomic neuropathy. If the patient can feel painful areas on the feet, neuropathy has not occurred.

A patient who has just been diagnosed with type 2 diabetes is 5 ft 4 in (160 cm) tall and weighs 182 pounds (82 kg). A nursing diagnosis of imbalanced nutrition: more than body requirements is developed. Which patient outcome is most important for this patient? a. The patient will have a diet and exercise plan that results in weight loss. b. The patient will state the reasons for eliminating simple sugars in the diet. c. The patient will have a glycosylated hemoglobin level of less than 7%. d. The patient will choose a diet that distributes calories throughout the day.

C Rationale: The complications of diabetes are related to elevated blood glucose, and the most important patient outcome is the reduction of glucose to near-normal levels. The other outcomes are also appropriate but are not as high in priority.

A diabetic patient is admitted with ketoacidosis and the health care provider writes all of the following orders. Which order should the nurse implement first? a. Start an infusion of regular insulin at 50 U/hr. b. Give sodium bicarbonate 50 mEq IV push. c. Infuse 1 liter of normal saline per hour. d. Administer regular IV insulin 30 U.

C Rationale: The most urgent patient problem is the hypovolemia associated with DKA, and the priority is to infuse IV fluids. The other actions can be accomplished after the infusion of normal saline is initiated.

A patient with recurring heartburn receives a new prescription for esomeprazole (Nexium). In teaching the patient about this medication, the nurse explains that this drug a. reduces the reflux of gastric acid by increasing the rate of gastric emptying. b. coats and protects the lining of the stomach and esophagus from gastric acid. c. treats gastroesophageal reflux disease by decreasing stomach acid production. d. neutralizes stomach acid and provides relief of symptoms in a few minutes.

C Rationale: The proton pump inhibitors decrease the rate of gastric acid secretion. Promotility drugs such as metoclopramide (Reglan) increase the rate of gastric emptying. Cryoprotective medications such as sucralfate (Carafate) protect the stomach. Antacids neutralize stomach acid and work rapidly.

A client has disabling attacks of vertigo. The nurse suspects that the client has Meniere's disease. The nurse is aware that the diet of the client must be modified. Which of the following is the best diet for the client? A) High protein B) Low Carbohydrates C) Low Sodium D) Low Fat

C = A low sodium diet is frequently an effective mechanism for reducing the frequency and severity of the disease episodes. About three-quarters of clients with Meniere's disease respond to treatment with a low salt diet.

A 78-year-old does not want to eat lunch and complains that the food that is serve does not taste good. Consistent with knowledge about age-related changes to taste, the nurse may find that the client is more willing to eat. A) Greasy foods B) Sour foods C) Sweet foods D) Salty foods.

C = the older adults' taste buds retain their sensitivity to carbohydrates. In addition, carbohydrates. Tend to be food items that are easy to chew. Older adults lose their sensitivity to sour and salty foods. Older adults may find greasy foods harder to digest and therefore may avoid them; however, preference for greasy foods is not related to changes in taste associated with age.

The nurse uses the PLISSIT format in helping client's who have sexual dysfunction. Which action by the nurse best reflects the "P" section of this format: a) ASk the physician for permission to discuss sexual topics with the client c) Acknowledge the clients spoken and unspoken sexual concerns when providing care

C) Acknowledge the clients spoken and unspoken sexual concerns when providing care

The client being admitted from the ED is diagnosed with a fecal impaction. Which nursing intervention should be implemented: c) Administer an oil retention enema d) Prepare for an UGI X-ray

C) Administer an oil retention enema

The nurse is caring for a patient diagnosed with pneumonia who is having shortness of breath and difficulty breathing. Which intervention should the nurse implement first: a) Take the client's vital signs b) Check the client's pulse oximetry c) Administer oxygen via nasal cannula

C) Administer oxygen via nasal cannula

A patient is receiving tube feedings through a percutaneous endoscopic gastrostomy (PEG). Which action will the nurse include in the plan of care: a) Keep the patient positioned on the left side b) Obtain a daily x-ray to verify tube placement c) Check the gastric residual volume every 4 to 6 hours

C) Check the gastric residual volume every 4 to 6 hours

The client experienced female circumcision as a puberty ritual while living in Africa as a child. What condition should the nurse monitor the client as an adult: c) Chronic urinary tract infection d) Tendency for postpartum hemorrhage

C) Chronic Urinary Tract Infection

A client with acute pancreatitus has an abnormally low serum calcium level. During a bath the nurse cleans the client's face with a cloth, and the lips, nose, and side of the face. When documenting this information the nurse would state that the patient's facial twitching indicates the presence of: c) Chrostek's sign d) Bell's palsey

C) Chrostek's Sign

An 80 year old client is transferred to a long term care facility. On the second night, he becomes confused and agitated. What is the most appropriate nursing diagnosis? c) Disturbed Sensory Perception d) Disturbed Thought Process

C) Disturbed Sensory Perception

The client who is bed-bound complains of abdominal pain. Bowel sounds are present. What action should be taken by the nurse: b) Palpate for bladder fullness c) Inspect the sacrel area for edema d) Use the PRN order to medicate the client with an antacid

C) Inspect the sacrel area for edema

The nurse is caring for an 80 year old patient with the medical diagnosis of heart failure. The patient has edema, orthopnea, and confusion. Which nursing diagnosis is most appropriate for this client: c) Excess fluid volume related to retension of fluids as evidence by edema and orthopnea d) Excess fluid volume related to cognitive heart failure as evidence by edema and confusion

C) Excess fluid volume related to retension of fluids as evidence by edema and orthopnea

A 76 year old patient has an open surgical wound on the abdomen that contains a creamy exudate and small areas of deep granulation tissue. The nurse documents the wound as a: a) Red wound b) Yellow wound c) Full thickness wound

C) Full thickness wound

Which question should the nurse ask when assessing a patient who has a history of benign prostatic hyperplasia (BPH): c) Has there been a decrease in the force of your urinary stream d) Have you been experiencing any difficulty in achieving an erection

C) Has there been a decrease in the force of your urinary stream

While assisting the client with a bath, the nurse encourages full range of motion in all the client's joints. Which activity would best support range of motion in the hand and arm: b) Move the wash basin farther toward the foot of the bed so the client must reach c) Have the client brush their hair and teeth d) Move each of the patient's hand and arm joints through passive range of motion

C) Have the client brush their hair and teeth

The nurse obtains this information when assessing a 74 year old patient in the outpatient clinic. Which finding os of the highest priority when the nurse is planning care for the patient: c) History of recent loss of balance and fall d) Complaint of left hip aching when jogging

C) History of recent loss of balance and fall

Wanting to know more about the client's pain experience, the nurse continues to explore different questioning techniques. Which of the following is the best example of an open-ended question for this situation: a) Is your pain worse at night b) What brought you to the clinic c) How has the pain impacted your life d) You're feeling down about having pain, aren't you

C) How has the pain impacted your life

Which statement made by a post menopausal client, would the nurse evaluate as indicating the need for further assessment: a) For some reason, I have more sexual desire than ever c) I am so glad that I don't need to worry about sex anymore d) Sex certainly takes longer that it used to, but im getting used to that

C) I am so glad that I don't need to worry about sex anymore

When learning how to implement the nursing process into a plan of care for a client, the student nurse realizes the part of the purpose of the nursing process is to: a) Deliver care to a client in an organized way b) Implement a plan that is close to the medical model c) Identify client needs and deliver care to meet those needs d) Make sure that standardized care is available to clients

C) Identify client needs and deliver care to meet those needs

The 45 year old client reports that she has no interest in sex and that she and her husband have not had intercourse in 16 years. How does the nurse interpret this assessment data: c) If both partners share the same lack of desire there is often not a problem d) This situation is so unnatural that some dysfunction is present

C) If both partners share the same lack of desire there is often not a problem

The PACU has received a semiconscious patient from the operating room and reviews the chart for orders related to positioning of the patient. There are no specific orders on the chart related to specific orders for the patient's position. In this situation, in what position will the nurse place the patient? A. Trendelenburg position B. Prone position C. Side-lying position D. Supine position

C) If the patient is not fully conscious, place the patient in the side-lying position, unless there is an ordered position on the patient's chart.

The client has a documented Stage III pressure ulcer on the right hip. What NANDA nursing diagnosis problem statement is most appropriate for use with this client: c) Impaired tissue integrity d) Risk for Injury

C) Impaired Tissue Integrity

A patient with a stroke is paralyzed on the left side of the body and has developed a pressure ulcer on the left hip. The best nursing diagnoses for this patient is: a) Impaired physical mobility related to left-sided paralysis b) Risk for impaired tissue integrity related to left-sided weakness c) Impaired skin integrity related to altered circulation and pressure d) Ineffective tissue perfusion related to inability to move independently

C) Impaired skin integrity related to altered circulation and pressure

A nurse is providing a back rub to a client just after administering a pain medication, with the hope that these two actions will help decrease the client's pain. Which phase of the nursing process is this nurse implementing: a) Assessment b) Diagnosis c) Implementation d) Evaluation

C) Implementation

Upon assessment the nurse notes that the client is dyspneic; has bibasilar crackles, and tires easily upon exertion. Which nursing diagnosis is best supported by these assessment details: b) Anxiety c) Ineffective airway clearance d) Impaired gas exchange

C) Ineffective Airway Clearance

A client just had a baby following a long labor and difficult delivery. Which of the following nursing diagnoses is formulated correctly: a) Constipation, due to tissue trauma, manifested by no bowel movements for two days b) Risk for infection, because of new incision, related to episiotomy c) Ineffective breast feeding, related to lack of motivation, secondary to exhaustion d) Altered urinary elimination, secondary to childbirth

C) Ineffective breast feeding, related to lack of motivation, secondary to exhaustion

An example of correctly written nursing diagnoses statement is: a) Altered tissue perfussion related to heart failure b) Risk for impaired tissue integrity related to sacrel redness c) Ineffective coping related to response to biopsy test results d) Altered urinary elimination related to urinary tract infection

C) Ineffective coping related to response to biopsy test results

Which nursing intervention should be applied to a client with a nursing diagnosis of Risk for Skin Integrity impairment related to immobility: a) Encourage client to eat at least 40% of meals b) Restrict fluid intake c) Keep lines dry and wrinkle free

C) Keep linens dry and wrinkle free

A client is exhibiting signs and symptoms of acute confusion/delirium. Which strategy should the nurse implement to promote a therapeutic environment: c) keep the room organized and clean d) Use restraints for client safety

C) Keep the room organized and clean

Which information noted by the nurse when caring for a patient with a bladder infection is most important to report to the health care provider: c) Left-sided flank pain d) Temp 100.1" F

C) Left-Sided Flank Pain

The nurse is preparing a patient for a colonoscopy. The nurse is familiar with the colonoscopy procedures at the hospital and is aware that which type of anesthesia os commonly used for his procedure? A. Spinal anesthesia B. Nerve block C. Conscious sedation D. Epidural anesthesia

C) Moderate sedation/analgesia is also known as conscious sedation or procedural sedation and is used for short-term and minimally invasive procedures such as a colonoscopy.

A client who describes his pain as 6 on a scale of 1 to 10 is classified as having which of the following: c) Moderate to severe pain d) Very severe pain

C) Moderate to severe pain

Formulating nursing diagnoses and client strengths is a joint function of: c) Nurse and client d) Physician and client

C) Nurse and client

The nurse is collecting information from a client's family. The client is confused and not able to contribute to the conversation. The spouse's states, "This is not normal behavior". The nurse documents this is which of the following: a) inference b) Subjective data c) Objective data d) Secondary subjective

C) Objective data

When assessing the musculoskeletal system the nurse's initial action will usually be to: b) Have the patient move the extremities against resistance c) Observe the patient's body build and muscle configuration

C) Observe the patient's body build and muscle configuration

All of the following nursing actions are included in the plan of care for the patient who is malnourished. Which action is appropriate for the nurse to delegate to nursing assistive personnel (NAP): c) Offer the patient the prescribed nutritional supplement between meals d) Assess the patient's strength while ambulating the patient in the room

C) Offer the patient the prescribed nutritional supplement between meals

Which behavior is characteristic of someone who is coping well with stress: c) Sets aside 30 min a day to exercise d) has no hobbies

C) Sets aside 30 min a day to exercise

The aspect of an older adult's history indicating a risk, for developing hyperatremia is that the client: c) Takes an over the counter antacid d) Has had frequent urinary tract infections

C) Takes an over the counter antacid

In planning preoperative teaching for a patient undergoing a Roux-en y gastric bypass as treatment for morbid obesity the nurse places the highest priority on: b) Discussing the necessary postoperative modifications in lifestyle c) Teaching the patient proper coughing and deep breathing techniques

C) Teaching the patient proper coughing and deep breathing techniques

The nurse assesses an open area over a patient's greater trochanted that is approximately 10 cm in diameter. The tissue around the area is edematous and feels boggy. The edges of the wound cup in toward the center. Which additional findings would indicate to the nurse that this is a Stage IV pressure ulcer: b) The crater extends into the subcutaneous tissue c) The joint capsule of the hip is visable

C) The joint capsule of the hip is visable

Which information obtained during the nurse assessment of the patient's nutritional- metabolic pattern may indicate the risk for musculoskeletal problems: c) The patient is 5 ft. 2 inches and weighs 180 lbs. d) The patient prefers whole milk to nonfat milk

C) The patient is 5 ft. 2 inches and weighs 180 lbs.

The nurse is writing the plan of care for a client who is confined to bed. Which intervention should be included to help reduce the effects of shearing forces on the client's skin: b) Coat the patient's back and buttocks with baby powder after bathing c) Use a turn sheet lifted by two staff member to move the client in bed

C) Use a turn sheet lifted by two staff member to move the client in the bed

Prior to finalizing a family orientated nursing care plan and implementing interventions, it is essential for the nurse to perform which of the following: a) Meet with all family members simultaneously c) establish a trusting relationship with the family as a group

C) establish a trusting relationship with the family as a group

As a component of a head to toe assessment, Nurse G, is preparing to assess convergence of the "PT" eyes. How should Nure G. conduct this assessment? A)Ask the "PT" to read a Snellen chart from a distance of 20 feet B)Ask the "PT" to look ahead while slowly bringing a penlight in from the side and to the "PT" pupil. C)Ask the patient to follow her finger as she slowly moves it towards the "PT" nose D)Ask the "PT" to hold his head stationary while following a pencil from left to right

C)Ask the patient to follow her finger as she slowly moves it towards the "PT" nose Eye convergence is assessed by holding your fingers 6" to 8" from the "PT" noise and asking the "PT" to follow it as it moves closer. Eyes should cross

A "PT" visual acuity is assessed as 20/40 in both eyes using the Snellen Chart. The nurse interprets this finding as which of the following? A)The "PT" can see twice as well as normal B)The "PT" has double vision C)The "PT" has less then normal vision D)The "PT" has normal vision

C)The "PT" has less then normal vision Normal vision 20/20. A finding of 20/40 would mean that the "PT" has less then normal vision

Which statement should be stressed while giving instructions after adrenalectomy? A. Stop taking medication when pts physical appearance improves. B. Pt should take steroids on an empty stomach. C. Pt should take the prescribed medication as directed.

C. The pt should take prescribed medication as directed. Sudden withdrawl of steroids can precipitate adrenal crisis.

Which statement about diabetes mellitus is false? A. Type 2 diabetes commonly occurs in adults <40 yr. old. B. Type 1 diabetes usually occurs before age 30. C. Type 1 diabetes is treatable with exercise, meal planning, and antidiabetic drugs. D. An increasing number of adolescents are being diagnosed with type 2 diabetes.

C. Type 1 diabetes is treated with insulin and dietary management.

Patients may experience which problem 24-48 hrs post-op as a result of anesthetics? A. colitis B. Stomatitis C. Paralytic ileus D. Gastrocolic reflux

C. After surgery, pts are clients are at risk for paralytic ileus as a result of anesthesia.

Risk factors for type 2 diabetes include all of the following except: a. Advanced age b. Obesity c. Smoking d. Physical inactivity

C: smoking Additional risk factors for type 2 diabetes are a family history of diabetes, impaired glucose metabolism, history of gestational diabetes, and race/ethnicity. African-Americans, Hispanics/Latinos, Asian Americans, Native Hawaiians, Pacific Islanders, and Native Americans are at greater risk of developing diabetes than whites.

Before admitting a patient to the operating room, the nurse recognizes that which of the following must be attached to the chart of all patients? A. A functional status evaluation B. Renal and liver function tests C. A physical examination report D. An electrocardiogram

C. A physical examination report. It is essential to have a physical examination report attached to the chart of a patient going for surgery. This document explains in detail the overall status of the patient for the surgeon and other members of the surgical team.

What would be the most effective way for a nurse to validate "informed consent"? A. Ask the family whether the patient understands the procedure. B. Check the chart for a completed and signed consent form. C. Ask the patient what he or she understands regarding the procedure. D. Determine from the physician what was discussed with the patient.

C. Ask the patient what he or she understands regarding the procedure. Informed consent in the health care setting is a process whereby a patient is informed of the risks, benefits, and alternatives of a certain procedure, and then gives consent for it to be done. The piece of paper is simply evidence that the informed consent process has been done.

A 73 year old patient who sustained a right hip fracture in a fall requests pain medication from the nurse. Based on his injury, which type of pain is this patient most likely experiencing? a. phantom b. visceral c. deep somatic d. referred

C. Deep somatic Rationale: Deep somatic pain originates in ligaments, tendons, nerves, blood vessels, and bones. Therefore, a hip fracture causes deep somatic pain. Phantom pain is a pain that is perceived to originate from a part that was removed during surgery. Visceral pain is caused by deep internal pain receptions and commonly occurs in the abdominal cavity, cranium, and thorax. Referred pain occurs in an area that is distant to the original site.

A client who complains of nausea and seems anxious is admitted to the nursing unit. The nurse should take which of the following actions regarding completion of the admission interview? A. Help the client to get settled and do the interview the next morning when the client is rested B. Do the interview immediately, directing the majority of the questions to the client's spouse C. Do the interview as soon as some uninterrupted time is available in order to address the client's concerns D. Ask the charge nurse to interview the client while the admitting nurse calls the doctor for anti-nausea and anti-anxiety medication

C. Do the interview as soon as some uninterrupted time is available in order to address the client's concerns Rationale: To collect data accurately, the client must participate. Attending to the client's immediate personal needs before expecting the client to focus on the interview will maximize the accuracy of the data collected. Data should be collected shortly after admission. The best source of data is the client. The management of the client's anxiety is the responsibility of the nurse conducting the interview and initiating the relationship.

As the nurse is preparing a patient for surgery, the patient refuses to remove a wedding ring. Which of the following is the most appropriate action by the nurse? A. Note the presence of the ring in the nurse's notes of the chart. B. Insist the patient remove the ring. C. Explain that the hospital will not be responsible for the ring. D. Tape the ring securely to the finger.

C. Explain that the hospital will not be responsible for the ring. It is customary policy to tape a patient's wedding band to the finger and make a notation on the preoperative checklist that the ring is taped in place.

A desired outcome for a client immobilized in a long leg cast reads; Client will state three signs of impaired circulation prior to discharge. When the nurse evaluates the client's progress, the client is able to state that numbness and tingling are signs of impaired circulation. What would be an appropriate evaluation statement for the nurse to write? A. Client understands the signs of impaired circulation B. Goal met: Client cited numbness and tingling as sign of impaired circulation C. Goal not met: Client able to name only two signs of impaired circulation D. Goal not met: Client unable to describe signs of impaired circulation

C. Goal not met: Client able to name only two signs of impaired circulation Rationale: The goal has not been met because the client states only two out of three signs of impaired circulation. By comparing the data with the expected outcomes, the nurse judges that while there has been progress toward the goal, it has not been completely met. The care plan may need to be revised or more effective teaching strategies may need to be implemented to achieve the goal.

When establishing a plan for pain control, what question would the nurse first ask the patient? a. "How long have you been having this pain?" b. "What measures relieve your pain?" c. "How does the presence of this pain affect your life?" d. "Aren't you tired of being in pain?"

C. How does the presence of this pain affect your life? Rationale: Before developing a plan for controlling a patient's pain, the nurse must elicit information about the patient's perception of his pain.

The nurse informs the physical therapy department that the client is too weak to use a walker and needs to be transported by wheelchair. Which step of the nursing process is the nurse engaged in at this time? A. Assessment B. Planning C. Implementation D. Evaluation

C. Implementation Rationale: The nurse is responsible for coordinating the plan of care with other disciplines to ensure the client's safety. This action represents the implementation phase of the nursing process. Data gathering occurs during assessment. Goal setting occurs during planning. Determining attainment of client goals occurs as part of evaluation.

Which of the following should be included in the plan of care for a patient who had spinal anesthesia? A. Elevating the head of the bed to decrease nausea B. Elevating the patient's feet to increase blood pressure C. Instructing the patient to remain flat in bed for 6 hours D. Administering oxygen to reduce hypoxia produced by spinal anesthesia

C. Instructing the patient to remain flat in bed for 6 hours. In addition to interventions designed to replace fluids and indirectly replace lost spinal fluid after administration of spinal anesthesia, the patient is instructed to lie flat for 6 to 8 hours. Elevating the head of the bed after spinal anesthesia can precipitate "spinal headache" or nausea related to losses of cerebrospinal fluid or changes in ICP. Elevating the patient's feet or administering oxygen are not necessary interventions unless the patient becomes hypotensive or hypoxic.

The nurse overhears an unlicensed assistive person (UAP) who has just been accepted to nursing school say to a client, "You must be so pleased with your progress." The nurse later explains to the UAP that this is an example of what type of question? A. Close-ended question B. Open-ended question C. Leading question D. Neutral question

C. Leading question Rationale: A leading question is asked in a way that suggests the type of answer that is expected. This can result in inaccurate data collection. A closed-ended question generally requires only a "yes" or "no" or short factual answer. Open-ended questions encourage clients to elaborate on their thoughts and feelings. Neutral questions do not influence the client's answer.

The nurse is working with an overweight client who has a high-stress job and smokes. This client has just received a diagnosis of Type II Diabetes and has just been started on an oral hypoglycemic agent. Which of the following goals for the client which if met, would be most likely to lead to an improvement in insulin efficiency to the point the client would no longer require oral hypoglycemic agents? a. Comply with medication regimen 100% for 6 months b. Quit the use of any tobacco products by the end of three months c. Lose a pound a week until weight is in normal range for height and exercise 30 minutes daily d. Practice relaxation techniques for at least five minutes five times a day for at least five months

C. Lose a pound a week until weight is in normal range for height and exercise 30 minutes daily When type II diabetics lose weight through diet and exercise they sometimes have an improvement in insulin efficiency sufficient to the degree they no longer require oral hypoglycemic agents.

Five minutes after receiving a preoperative sedative medication by IV injection, a patient asks the nurse to get up to go to the bathroom to urinate. Which of the following is the most appropriate action for the nurse to take? A.) Assist patient to bathroom and stay next to door to assist patient back to bed when done. B. Allow patient to go to the bathroom since the onset of the medication will be more than 5 minutes. C. Offer the patient to use the urinal/bedpan after explaining the need to maintain safety. D. Ask patient to hold the urine for a short period of time since a urinary catheter will be placed in the operating room.

C. Offer the patient to use the urinal/bedpan after explaining the need to maintain safety. The prime issue after administration of either sedative or opioid analgesic medications is safety. Because the medications affect the central nervous system, the patient is at risk for falls and should not be allowed out of bed, even with assistance.

Which of the following is most appropriate after administration of preoperative medications? A. Confirming that the patient has voided B. Monitoring vital signs every 15 minutes C. Placing the patient in bed with the rails up D. Transporting the patient immediately to the operating room

C. Placing the patient in bed with the rails up. After administration of preoperative medications, a nurse should instruct a patient not to get up without assistance because medications can cause drowsiness or dizziness. Confirming that the patient has recently voided should be done before preoperative medications are administered. Monitoring vital signs every 15 minutes is not a necessary intervention unless prescribed by the physician. Transporting the patient immediately to the operating room is not necessary unless the patient is called for.

.The nurse is caring for a hospitalized client with a diagnosis of ulcerative colitis. Which finding, if noted on assessment of the client, would the nurse report to the physician? A. Hypotension B. Bloody diarrhea C. Rebound tenderness D. A hemoglobin level of 12 mg/dL

C. Rebound tenderness

While performing preoperative teaching, the patient asks when she needs to stop drinking water before the surgery. Based on the most recent practice guidelines established by the American Society of Anesthesiologists, the nurse tells the patient that: A. She must be NPO after breakfast. B. She needs to be NPO after midnight. C. She can drink clear liquids up to 2 hours before surgery. D. She can drink clear liquids up until she is taken to the OR.

C. She can drink clear liquids up to 2 hours before surgery. Practice guidelines for preoperative fasting state the minimum fasting period for clear liquids is 2 hours. Evidenced-based practice no longer supports the long-standing practice of requiring patients to be NPO after midnight.

A nurse is preparing to remove a nasogartric tube from a client. The nurse should instruct the client to do which of the following just before the nurse removes the tube? A. Exhale B. Inhale and exhale quickly C. Take and hold a deep breath D. Perform a Valsalva maneuver

C. Take and hold a deep breath

The nurse needs to validate which of the following statements pertaining to an assigned client? A. The client has a hard, raised, red lesion on his right hand. B. A weight of 185 lbs. is recorded in the chart C. The client reported an infected toe D. The client's blood pressure is 124/70. It was 118/68 yesterday.

C. The client reported an infected tow Rationale: Validation is the process of confirming that data are actual and factual. Data that can be measured can be accepted as factual, as in options 1, 3 and 4. The nurse should assess the client's toe to validate the statement.

A patient with cirrhosis is being treated with spironolactone (Aldactone) tid and furosemide (Lasix) bid. The patient's most recent laboratory results indicate a serum sodium of 134 mEq/L (134 mmol/L) and a serum potassium of 3.2 mEq/L (3.2 mmol/L). Before notifying the physician, the nurse should A. administer only the furosemide B. administer both drugs as ordered C. administer only the spironolactone D. Withhold the furosemide and spironolactone

C. administer only the spironolactone The potassium level is dangerously low. Lasix is potassium depleting, while spironolactone is potassium sparing. You would hold the Lasix and call the physician. This is a good NCLEX question that integrates this course with pharmacology.

In the absence of postoperative vomiting, GI suctioning, and wound drainage, the physiologic responses to the stress of surgery are most likely to cause A. diuresis. B. hyperkalemia. C. fluid overload. D. impaired blood coagulation.

C. fluid overload. Secretion and release of aldosterone and cortisol from the adrenal gland and ADH from the posterior pituitary as a result of the stress response cause fluid retention during the first 2 to 5 days postoperatively, and fluid overload is possible during this time. Aldosterone causes renal potassium loss with possible hypokalemia, and blood coagulation is enhanced by cortisol.

It is most important for the nurse to understand the various ways in which pain is classified. a. so that he can document the client's pain using accurate terms. b. so that he can be clear in his communication with the physician. c. so that he can develop an effective pain management plan. d. So that he can educate the client thoroughly

C. so that he can develop an effective pain management plan Rationale: different modalities are used in the treatment/management of pain and are often based on how the pain is classified.

Upon entering the room of a patient who has just returned from surgery for total laryngectomy and radical neck dissection, a nurse should recognize a need for intervention when finding A. a gastrostomy tube that is clamped. B. the patient coughing blood-tinged secretions from the tracheostomy. C. the patient positioned in a lateral position with the head of the bed flat. D. 200 ml of serosanguineous drainage in the patient's portable drainage device.

C. the patient positioned in a lateral position with the head of the bed flat. After total laryngectomy and radical neck dissection, a patient should be placed in a semi-Fowler's position to decrease edema and limit tension on the suture line.

The nurse understands that, in order to individualize client care, decisions are made during the planning phase to: A: Set goals for multiple clients B: Address all of the client's disease processes C: Address problems that need individualized approaches D: Address interventions that can be delegated

C: Address problems that need individualized approaches

A thirty five year old male has been an insulin-dependent diabetic for five years and now is unable to urinate. Which of the following would you most likely suspect? A: Atherosclerosis B: Diabetic nephropathy C: Autonomic neuropathy D: Somatic neuropathy

C: Autonomic neuropathy

The nurse uses a standardized care plan to develop an individualized care plan for each client. Match the nursing action to the appropriate holistic care plan approach or rationale: The nurse integrates dependent and independent nursing function. A: Ongoing individualized care planning B: Standardized care plan C: Complete individualized plan of care D: Individualized care plan

C: Complete individualized plan of care

The nurse selects the nursing diagnosis of Risk for Impaired Skin Integrity related to immobility, dry skin, and surgical incision. Which of the following represents a properly stated outcome/goal? The client will: A: Turn in bed q2h B: Report the importance of applying lotion to skin daily C: Have intact skin during hospitalization D: Use a pressure-reducing mattress

C: Have intact skin during hospitalization

The nurse, when developing a care plan, uses the nursing outcome classification to: A: Put interventions into action B: Set priorities C: Measure desired outcomes and evaluate client progress D: Diagnose a client's problem

C: Measure desired outcomes and evaluate client progress

Which of the following are appropriate nursing guidelines when prioritizing a client's care? A: The items from client identifies as a priority should always be addressed first B: Resolve all high-priority items before addressing lower-priority items C: Priorities change as the client responds to therapies D: Rank nursing diagnoses by degree of importance E: Priorities may need to be shifted based on available resources

C: Priorities change as the client responds to therapies D: Rank nursing diagnoses by degree of importance

The student nurse is developing a nursing care plan and performs which of the following during the planning phase? A: Reassesses the client B: Determines the need for assistance C: Selects nursing interventions D: Analyze data

C: Selects nursing interventions

Place the following activities of planning in the correct order of their use. A: Establish goals/outcomes B: Write the care plan C: Set priorities D: Choose interventions

C: Set priorities A: Establish goals/outcomes D: Choose interventions B: Write the care plan

Consider the following nursing diagnosis: Imbalanced Nutrition: Less Than Body Requirements related to inability to feed self. What is an example of a short-term goal for this client? A: The client will acquire competence in managing cookware designed for handicapped clients B: The client will choose one correct menu C: The client will eat 50% of his meals by Friday with the use of modified eating utensils to feed self with minimal assistance D: The client will demonstrate safe preparation techniques

C: The client will eat 50% of his meals by Friday with the use of modified eating utensils to feed self with minimal assistance

The care plan includes a nursing intervention "4/2/11 Measure client's fluid intake and output. F. Jenkins, RN." What element of a proper nursing intervention has been omitted? A: Action verb B: Content C: Time D: None

C: Time

If a patient has collapsed and cannot be aroused by asking loudly, "Are you ok?" what action should be taken next?

Call for help, if assistance is available, have that person get help.

Colostomy stoma, dark blue in color:

Call physician

Rapid transfusion of cool blood places the pt at risk for what?

Cardiac dysrhythmias

A patient who has undergone gastroscopy is fully awake and asks he nurse for something to drink. After confirming that liquids are allowed on the physician order sheet, which of the following nursing actions should the nurse do before bringing a beverage to the patient?

Check for the return of gag and swallow reflexes.

Which of the following nursing interventions would indicate correct technique when providing NG feeding to an unresponsive patient?

Check residual volume every 4 hours.

Patients who are experiencing nausea or vomiting may find it easiest to tolerate:

Cold foods without odor.

The Joint Commission mandates the use of continuous quality improvement and measurement of specific quality outcomes, including clients admitted with:

Community-acquired pneumonia and congestive heart failure rationale: TJC mandates outcome measures for clients admitted with a diagnosis of acute MI, congestive heart failure, community-acquired pneumonia, surgical infection prophylaxis, pregnancy-related conditions, and deep vein thrombosis.

To determine the significance of a blood pressure that reads 148/100, it is first necessary for the nurse to do which of the following?

Compare this reading to standards.

Stop intercourse before orgasm to not get pregnant;

Conceiving isn't related to orgasm

When admitting a postop patient to the surgical unit, which nursing action is a priority?

Conduct Pain Assessment Assessment is a constant ongoing task for the postop patient

1st BM since surgery, hard/dry formed stool:

Constipation

Because patients older than 80 with HTN may have an auscultatory gap in their Korotkoff sounds, it is most important when taking their blood pressure measurement to:

Continue to listen until the cuff is deflated.

320 Child diagnosed w/meningococcal meningitis. Which of following isolation techniques is appropriate?

Isolation precautions for at least 24 hr. after initiation of antibiotics. R: Meningococcal meningitis is transmitted primarily by droplet infection. Isolation is begun and maintained for at least 24 hr. after antibiotics are given.

Sodium chloride 0.9% (normal saline) is what type of solution?

Isotonic

52 year old man for physical exam, teach patient about:

Practice/Prostate exams

When evaluating an Internet site, the nurse understands that:

Credibility, accuracy, and reasonableness of the information should be considered rationale: The credibility of the information that you obtain from Internet searches should be challenged. It is important to be skeptical, because not all information that is posted is accurate. Information should be evaluated and reevaluated on a regular basis. It is important to remember that Internet information is dynamic and fluid in nature, as compared with printed information.

The nurse is caring for a client after a lung lobectomy. The nurse notes fluctuating water levels in the water-seal chamber of the client's chest tube. What action should the nurse take? A. Do nothing, but continue to monitor the client. B. Call the physician immediately. C. Check the chest tube for a loose connection. D. Add more water to the water-seal chamber

Correct answer: A Fluctuation in the water-seal chamber is a normal finding that occurs as the client breathes. No action is required except for continued monitoring of the client. The nurse doesn't need to notify the physician. Continuous bubbling in the water-seal chamber indicates an air leak in the chest tube system, such as from a loose connection in the chest tube tubing. The water-seal chamber should be filled initially to the 2 cm line, and no more water should be added.

A client is admitted to the medical-surgical floor with a diagnosis of acute pancreatitis. His blood pressure is 136/76 mm Hg, pulse 96 beats/minute, respirations 22 breaths/minute, temperature 99°F (38.3°C), and he has been experiencing severe vomiting for 24 hours. His past medical history reveals hyperlipidemia and alcohol abuse. The physician prescribes a nasogastric (NG) tube for the client. Which of the following is the primary purpose for insertion of the NG tube? A. Empty the stomach of fluids and gas to relieve vomiting. B. Prevent spasms at the sphincter of Oddi. C. Prevent air from forming in the small and large intestines. D. Remove bile from the gallbladder.

Correct answer: A An NG tube is no longer routinely inserted to treat pancreatitis, but if the client has protracted vomiting, the NG tube is inserted to drain fluids and gas and relieve vomiting. An NG tube doesn't prevent spasms at the sphincter of Oddi (a valve in the duodenum that controls the flow of digestive enzymes) or prevent air from forming in the small and large intestine. The common bile duct connects to the pancreas and the gall bladder, and a T tube rather than an NG tube would be used to collect bile drainage from the common bile duct.

A client with cirrhosis of the liver develops ascites. Which of the following orders would the nurse expect? A. Restrict fluid to 1000 mL per day. B. Ambulate 100 ft. three times per day. C. High-sodium diet. D. Maalox 30 ml P.O. BID.

Correct answer: A Fluid restriction is a primary treatment for ascites. Restricting fluids decreases the amount of fluid present in the body, thereby decreasing the fluid that accumulates in the peritoneal space. A high sodium diet would increase fluid retention. Physical activities are usually restricted until ascites is relieved. Loop diuretics (such as furosemide) are usually ordered, and Maalox® (a bismuth subsalicylate) may interfere with the action of the diuretics.

The nurse is doing teaching with the family of a client with liver failure. Which of the following foods should the nurse advise them to limit in the client's diet? A. Meats and beans. B. Butter and gravies. C. Potatoes and pasta. D. Cakes and pastries.

Correct answer: A Meats and beans are high-protein foods and are restricted with liver failure. In liver failure, the liver is unable to metabolize protein adequately, causing protein by-products to build up in the body rather than be excreted. This causes problems such as hepatic encephalopathy (neurologic syndrome that develops as a result of rising blood ammonia levels). Although other nutrients, such as fat and carbohydrates, may be regulated, it's most important to limit protein in the diet of the client with liver failure.

A client who recently underwent cranial surgery develops syndrome of inappropriate antidiuretic hormone (SIADH). Which of the following symptoms should the nurse anticipate? A. Edema and weight gain. B. Excessive urinary output. C. Fluid loss and dehydration. D. Low urine specific gravity.

Correct answer: A Syndrome of inappropriate antidiuretic hormone (SIADH) results in an abnormally high release of antidiuretic hormone, which causes water retention as serum sodium levels fall, leading to edema and weight gain. Because of fluid retention, urine output is low. Fluid is restricted to prevent fluid overload rather than replaced. As the urine becomes more concentrated, the specific gravity increases. Other symptoms include nausea, vomiting, seizures, altered mentation, and coma. SIADH is most common with diseases of the hypothalamus but can also occur with heart failure, Guillain-Barré syndrome, meningitis, encephalitis, head trauma, or brain tumors. It may also be triggered by medications.

A client with type 2 diabetes has a hemoglobin A1C level of 8.8 after 6 months of oral therapy with metformin (Glucophage®). The client tells the nurse that she often forgets to take her medication and doesn't really follow her diet. Which of the following is the nurse's best first response? A. "If you don't get control of your blood sugar, you'll need to take insulin." B. "It can be hard to get used to having a disease like diabetes. What are some of the things you find challenging about it?" C. "Uncontrolled diabetes can lead to eye problems and kidneys problems." D. "Many people have diabetes."

Correct answer: B Acknowledging that the client is going through changes and allowing her to express her concerns will help the nurse assess her needs. Hemoglobin AIC shows the average blood glucose levels over a 3-month period. Diabetes should maintain the AIC <7%. Lecturing, threatening and comparing the clients to others belittles the client and discourages discussion, but the patient must be provided adequate information in order to make informed decisions about self-care.

The nurse is caring for a client with cirrhosis of the liver. The client has developed ascites and requires a paracentesis. Which of the following symptoms is associated with ascites and should be relieved by the paracentesis? A. Pruritus. B. Dyspnea. C. Jaundice. D. Peripheral neuropathy.

Correct answer: B Ascites (fluid buildup in the abdomen) puts pressure on the diaphragm, resulting in difficulty breathing and dyspnea. Paracentesis (surgical puncture of the abdominal cavity to aspirate fluid) is done to remove fluid from the abdominal cavity and thus reduce pressure on the diaphragm in order to relieve the dyspnea. Pruritus, jaundice, and peripheral neuropathy are signs of cirrhosis that aren't relieved or treated by paracentesis.

The nurse is writing the teaching plan for a client undergoing a radioactive iodine uptake test to study thyroid function. Which of the following instructions should the nurse include? A. "You need to stay at least 4 feet (1.2 m) away from other people after the test because you'll be radioactive." B. "You need to lie very still on a stretcher that is placed in a long tube for the scan" C. "Don't take any iodine or thyroid medication before the test." D. "Schedule the bone scans before your radioactive iodine uptake test."

Correct answer: C Medications such as iodine, contrast media, and antithyroid and thyroid drugs can affect the test results and should be withheld by the client for a week or longer, as directed by the physician. During a radioactive iodine uptake test, the client receives radioactive iodine by mouth or I.V. in small doses and doesn't require isolation. During magnetic resonance imaging--not radioactive iodine uptake testing--a client needs to lie still inside a long tube. Any test, such as a bone scan, that requires iodine contrast media should be scheduled after the radioactive iodine uptake test because the iodinated contrast medium can decrease uptake.

A 37-year-old forklift operator presents with shakiness, sweating, anxiety, and palpitations and tells the nurse he has type 1 diabetes mellitus. Which of the follow actions should the nurse do first? A. Inject 1 mg of glucagon subcutaneously. B. Administer 50 mL of 50% glucose I.V. C. Give 4 to 6 oz (118 to 177 mL) of orange juice. D. Give the client four to six glucose tablets.

Correct answer: C Because the client is awake and complaining of symptoms, the nurse should first give him 15 grams of carbohydrate to treat hypoglycemia. This could be 4 to 6 oz of fruit juice, five to six hard candies such as Lifesavers, or 1 tablespoon of sugar. When a client has worsening symptoms of hypoglycemia or is unconscious, treatment includes 1 mg of glucagon subcutaneously or intramuscularly, or 50 mL of 50% glucose I.V. The nurse may also give two to three glucose tablets for a hypoglycemic reaction.

If a patient with an esophagogastric tube suddenly experiences acute respiratory distress, a nurse's first action should be to:

Cut the balloon ports and remove the tube.

The nurse is teaching a client newly diagnosed with type 1 diabetes how to self-administer subcutaneous insulin injections. How does the nurse best evaluate the effectiveness of her teaching? A. Have the client repeat the steps back to the nurse. B. Give the client a written test on self-administration of insulin. C. Ask the client to write out the steps for self-administration of insulin injections. D. Ask the client to give a return demonstration of self-administration of insulin.

Correct answer: D Asking the client to give a return demonstration of his injection technique is the best way to assess whether the client can perform the procedure. It also gives the nurse the opportunity to provide feedback. Asking the client to recite the steps, pass a written test, or write out the steps shows the nurse whether the client is able to recall the steps but doesn't show that he has the necessary motor skills or the ability to perform the procedure.

A community-oriented nurse leader is working with a community partnership to improve access to services for the underserved by planning an expansion of the local community health clinic. This nursing intervention strategy is focused on which of the following community health dimensions? A) Environment. B) Health status. C) Process. D) Structure.

D

The intervention used to influence the knowledge, attitudes, values, beliefs, behaviors, and practices of the population of interest is referred to as: A) Advocacy. B) Coalition building. C) Consultation. D) Social marketing.

D

Circulatory overload signs.

Cough, Dyspnea, chest pain, wheezing, tachycardia, & hypertension

Red:

Cover

A nurse identifies higher-than-normal levels of lead when screening a 3-year-old child. The nurse works with the local health department to put together a team to address the environmental issues responsible for the child's abnormal lead level. Team members should include the following specialists: A) Epidemiologist, pediatric specialist, and sanitarian. B) Laboratory specialist, contractor whose bid for lead reduction work is the lowest, and public health lead reduction specialist. C) Public health sanitarian, pediatric generalist, and plumbing inspector. D) Specially trained housing inspector, pediatric specialist, lead-based paint intervention team, and laboratory specialists to test the child's home and the surrounding neighborhood.

D

A population-level tertiary prevention intervention typically carried out by nurses caring for those with communicable disease in the community is: A) HIV test results counseling. B) Needle exchange. C) Partner notification. D) Instruction in standard precautions.

D

A school nurse is teaching a class of sophomores about the relationship between the risk of sexually transmitted disease (STD) and risk-taking behaviors. A key point to include is: A) All STDs are easily preventable with consistent condom use. B) Once a young woman is pregnant, she is no longer at risk for most STDs. C) STDs are most likely to be transmitted during a student's initial sexual encounter. D) Use of alcohol and drugs makes a student more likely to make decisions that result in exposure to and infection with STDs.

D

An occupational health nurse practitioner's physical assessment of a factory worker identifies an acute-onset pruritic dermatitis extending over the face, hands, neck, and forearms. The nurse's priorities should be to: A) Contact factory senior management, educate workers about their exposure, and clean the area. B) Contact the Occupational Safety and Health Administration immediately and remove the offending chemical in the work environment. C) Immediately evacuate the worker's nearby workspace and treat the worker and other exposed workers. D) Treat the client and obtain a comprehensive exposure history; if an onsite environmental exposure is suspected as the cause, screen other at-risk workers and ensure that the environmental risk is identified and eliminated.

D

Campaigns to decrease the inequitable burden of environmental risks on the poor and people of color in the United States strive to apply the ethical principle of: A) Societal justice. B) Nonmaleficence. C) Compliance and enforcement of the Environmental Protection Agency Regulatory Act. D) Environmental justice.

D

Case fatality from breast cancer has decreased in recent years, although the incidence of breast cancer has increased. Descriptive epidemiology would use the component of time to explain this change in terms of: A) Cyclical patterns. B) Even-related clusters. C) Point epidemic. D) Secular trends.

D

John Snow played a critical role in the development of modern disease surveillance when he: A) Devised a more statistically valid method of analyzing epidemiologic data. B) Discovered causal agents for anthrax, tuberculosis, and cholera. C) Tracked the incidence of tuberculosis in the tenements of New York City. D) Used geographic mapping to demonstrate the connection between water supply and cholera.

D

Promotion of the creation of immunization registries that combine immunization information from different sources into a single electronic record to provide official immunization records for schools, daycare centers, health departments, and clinics is a goal of: A) Community-level practice. B) Family-level practice. C) Individual-level practice. D) Systems-level practice.

D

When applying the nursing process to environmental health, the nurse would: A) Conduct an assessment focused on the client's presenting problem. B) Coordinate interventions with the primary care provider of record. C) Examine criteria that are limited to the client's immediate responses. D) Include outcome measures that involve mitigation and elimination of the contributing factors.

D

Which community attribute is an indicator of a community's health status? A) Mean educational level. B) Location of health facilities within the community. C) Ratio of police to citizens. D) Suicide rate.

D

The client arrives in the emergency room after sustaining a chemical eye injury from a splash of battery acid. The initial nursing action is to: a) begin visual acuity testing b) cover the eye with a pressure patch c) swab the eye with antibiotic ointment d) irrigate the eye with sterile normal saline

D Emergency care following a chemical burn to the eye includes irrigating the eye immediately with sterile normal saline or ocular irrigating solution. In the emergency department, the irrigation should be maintained for at least 10 minutes. Following this emergency treatment, visual acuity is assessed. Options B and C are not a component of initial care.

During a teaching session, the nurse tells the client that 50% to 60% of daily calories should come from carbohydrates. What should the nurse say about the types of carbohydrates that can be eaten? a. Simple carbohydrates are absorbed more rapidly than complex carbohydrates. b. Simple sugars cause a rapid spike in glucose levels and should be avoided c. Simple sugars should never be consumed by someone with diabetes. d. Try to limit simple sugars to between 10% and 20% of daily calories.

D It is recommended that carbohydrates provide 50% to 60% of the daily calories. Approximately 40% to 50% should be from complex carbohydrates. The remaining 10% to 20% of carbohydrates could be from simple sugars. Research provides no evidence that carbohydrates from simple sugars are digested and absorbed more rapidly than are complex carbohydrates, and they do not appear to affect blood sugar control.

A hospitalized diabetic patient receives 12 U of regular insulin mixed with 34 U of NPH insulin at 7:00 AM. The patient is away from the nursing unit for diagnostic testing at noon, when lunch trays are distributed. The most appropriate action by the nurse is to a. save the lunch tray to be provided upon the patient's return to the unit. b. call the diagnostic testing area and ask that a 5% dextrose IV be started. c. ensure that the patient drinks a glass of milk or orange juice at noon in the diagnostic testing area. d. request that the patient be returned to the unit to eat lunch if testing will not be completed promptly.

D Rationale: Consistency for mealtimes assists with regulation of blood glucose, so the best option is for the patient to have lunch at the usual time. Waiting to eat until after the procedure is likely to cause hypoglycemia. Administration of an IV solution is unnecessarily invasive for the patient. A glass of milk or juice will keep the patient from becoming hypoglycemic but will cause a rapid rise in blood glucose because of the rapid absorption of the simple carbohydrate in these items.

The nurse is assessing a patient with gastroesophageal reflux disease (GERD) who is experiencing increasing discomfort. Which patient statement indicates that additional patient education about GERD is needed? a. "I take antacids between meals and at bedtime each night." b. "I quit smoking several years ago, but I still chew a lot of gum." c. "I sleep with the head of the bed elevated on 4-inch blocks." d. "I eat small meals throughout the day and have a bedtime snack."

D Rationale: GERD is exacerbated by eating late at night, and the nurse should plan to teach the patient to avoid eating at bedtime. The other patient actions are appropriate to control symptoms of GERD.

A patient recovering from DKA asks the nurse how acidosis occurs. The best response by the nurse is that a. insufficient insulin leads to cellular starvation, and as cells rupture they release organic acids into the blood. b. when an insulin deficit causes hyperglycemia, then proteins are deaminated by the liver, causing acidic by-products. c. excess glucose in the blood is metabolized by the liver into acetone, which is acidic. d. an insulin deficit promotes metabolism of fat stores, which produces large amounts of acidic ketones.

D Rationale: Ketoacidosis is caused by the breakdown of fat stores when glucose is not available for intracellular metabolism. The other responses are inaccurate.

A patient with type 1 diabetes has received diet instruction as part of the treatment plan. The nurse determines a need for additional instruction when the patient says, a. "I may have an occasional alcoholic drink if I include it in my meal plan." b. "I will need a bedtime snack because I take an evening dose of NPH insulin." c. "I will eat meals as scheduled, even if I am not hungry, to prevent hypoglycemia." d. "I may eat whatever I want, as long as I use enough insulin to cover the calories."

D Rationale: Most patients with type 1 diabetes need to plan diet choices very carefully. Patients who are using intensified insulin therapy have considerable flexibility in diet choices but still should restrict dietary intake of items such as fat, protein, and alcohol. The other patient statements are correct and indicate good understanding of the diet instruction.

The nurse teaches the diabetic patient who rides a bicycle to work every day to administer morning insulin into the a. thigh. b. buttock. c. arm. d. abdomen.

D Rationale: Patients should be taught not to administer insulin into a site that will be exercised because exercise will increase the rate of absorption. The thigh, buttock, and arm are all exercised by riding a bicycle.

A program of weight loss and exercise is recommended for a patient with impaired fasting glucose (IFG). When teaching the patient about the reason for these lifestyle changes, the nurse will tell the patient that a. the high insulin levels associated with this syndrome damage the lining of blood vessels, leading to vascular disease. b. although the fasting plasma glucose levels do not currently indicate diabetes, the glycosylated hemoglobin will be elevated. c. the liver is producing excessive glucose, which will eventually exhaust the ability of the pancreas to produce insulin, and exercise will normalize glucose production. d. the onset of diabetes and the associated cardiovascular risks can be delayed or prevented by weight loss and exercise.

D Rationale: The patient with IFG is at risk for developing type 2 diabetes, but this risk can be decreased with lifestyle changes. Glycosylated hemoglobin levels will not be elevated in IFG and the Hb A1C test is not included in prediabetes testing. Elevated insulin levels do not cause the damage to blood vessels that can occur with IFG. The liver does not produce increased levels of glucose in IFG

A patient screened for diabetes at a clinic has a fasting plasma glucose level of 120 mg/dl (6.7 mmol/L). The nurse will plan to teach the patient about a. use of low doses of regular insulin. b. self-monitoring of blood glucose. c. oral hypoglycemic medications. d. maintenance of a healthy weight.

D Rationale: The patient's impaired fasting glucose indicates pre-diabetes and the patient should be counseled about LIFESTYLE CHANGES to prevent the development of type 2 diabetes. The patient with prediabetes does not require insulin or the oral hypoglycemics for glucose control and does not need to self-monitor blood glucose.

A health care provider who has not been immunized for hepatitis B is exposed to the hepatitis B virus (HBV) through a needle stick from an infected patient. The infection control nurse informs the individual that treatment for the exposure should include a. baseline hepatitis B antibody testing now and in 2 months. b. active immunization with hepatitis B vaccine. c. hepatitis B immune globulin (HBIG) injection. d. both the hepatitis B vaccine and HBIG injection.

D Rationale: The recommended treatment for exposure to hepatitis B in unvaccinated individuals is to receive both HBIG and the hepatitis B vaccine, which would provide temporary passive immunity and promote active immunity. Antibody testing may also be done, but this would not provide protection from the exposure.

The nurse is caring for a client with a diagnosis of detached retina. Which assessment sign would indicate that bleeding has occurred as a result of the retinal detachment? a) total loss of vision b) a reddened conjunctiva c) a sudden sharp pain in the eye d) complaints of a burst of black spots or floaters

D D Complaints of a sudden burst of black spots or floaters indicates that bleeding has occurred as a result of the detachment. Options A, B, and C are not signs of bleeding.

31. A 64-year-old patient with newly diagnosed acute myelogenous leukemia (AML) who is undergoing induction therapy with chemotherapeutic agents tells the nurse, "I feel so sick that I don't know if the treatment is worth completing." The nurse's best response to the patient is a. "I know you feel really ill right now, but after this therapy your disease will go into a remission and you will feel normal again." b. "Induction therapy is very aggressive and causes the most side effects, so when this phase is completed you won't feel so ill." c. "Your type of leukemia has an 80% survival rate if aggressive therapy is started, so the effects of treatment will be worth it to you." d. "The chemotherapy is difficult, but it is necessary to put the disease into remission and give you time to make choices about your life.

D Rationale: AML is very aggressive, and survival after diagnosis is short without treatment. Induction therapy is followed by more chemotherapy, so the nurse should not tell the patient that he or she will feel normal or not so ill. The survival with AML is not 80%.

Which of the following is the most common surgical procedure for chronic otitis media? A) Myringotomy B) Ossiculoplasty C) Mastoidectomy D) Tympanoplasty

D Tympanoplasty involves surgical reconstruction as the tympanic membrane and is done to re-establish middle ear function, close perforation, prevent recurrent infections. - sew the tympanic membrane back up!

A female client with breast cancer is currently receiving radiation therapy for treatment. The client is complaining of apathy, hard to concentrate on something, and feeling tired despite of having time to rest and more sleep. These complains suggest symptoms of: A) Hypocalcemia B) radiation pneumonitis C) advanced breast cancer D) fatigue

D = Fatigue is a common complaint of individuals receiving medication therapy.

After the first three dose of Paroxetine (Paxil) 20 mg, the client complains that the medication upsets his stomach. Which of the following instructions would the nurse give to the client? A) "Take the medication with 4 ounces of orange juice." B) "Take the medication an hour before breakfast." C) "Take the medication at bedtime." D) "Take the medication with some foods.

D = Nausea and gastrointestinal upset is a common but usually temporary side effects of Paroxetine (Paxil). Therefore, the nurse would instruct the client to take the medication with food to minimize nausea and stomach upset.

The nurse is removing the client's staples from an abdominal when the client cough continuously and the incision splits open exposing the intestines. Which of the following is the immediate nursing action of the nurse? A) Call the surgeon to come to the client's room immediately B) Have all visitors and family member leave the room C) Press the emergency alarm to call the resuscitation team D) Cover the abdominal organs with sterile dressing moistened with sterile normal saline.

D = When a wound eviscerates, the nurse should cover the open area with sterile dressing moistened with sterile normal saline and then cover it with a dry dressing. The surgeon should then be notified to take the client back to the operating room to close the incision under general anesthesia.

Which of the following signs and symptoms would indicate that a client has benign prostatic hypertrophy (BPH)? A) Hematuria B) Flank pain C) Impotence D) Difficulty starting the urinary stream

D = the symptoms of BPH are related to obstruction as a result of an enlarged prostate. Difficulty in starting the urinary stream is a common symptom, along with dribbling, hesitancy and urinary retention.

Which client is at greatest risk for experiencing sensory overload: c) A 16 year old listening to loud music d) An 80 year old client admitted for emergency surgery

D) An 80 year old client admitted for emergency surgery

Upon admission for an appendectomy, the patient provides the nurse with a document that specifies instructions his healthcare team should follow in the event he is unable to communicate these wishes postoperatively. This document is best known as: A. An informed consent B. An insurance card C. A Patient's Bill of Rights D. An advance directive

D) An advance directive, a legal document, allows the patient to specify instructions for his or her healthcare treatment should he or she be unable to communicate these wishes postoperatively. The advance directive allows the patient to discuss his or her wishes with the family members in advance of the surgery. Two common forms of advance directives include living wills and durable powers of attorney for healthcare.

A 52 year old man is scheduled for an annual physical exam. The nurse will plan to teach the patient about: c) Normal decreases in testosterone level d) Annual prostate specific antigen testing (PSA)

D) Annual prostate specific antigen testing (PSA)

Immediate surgery is planned for a patient with acute abdominal pain. The question used by the nurse that will elicit the most complete information about the patient's coping-stress tolerance pattern is: b) What do you think caused this abdominal pain c) How do you feel about yourself and your hospitalization d) Are there other major problems that are a concern right now

D) Are there other major problems that are a concern right now

The nurse assess a surgical patient in the morning of the first postoperative day and notes redness and warmth around the incision. Which action by the nurse is most appropriate: a) Obtain wound cultures b) document the assessment d) Assess the wound every 2 hours

D) Assess the wound every 2 hours

A 78 year old who has been admitted to the hospital with dehydration is confused and incontinent of urine. Which nursing action will be best to include in the plan of care: c) Insert an indwelling catheter until the symptoms have resolved d) Assist the patient to the bathroom every 2 hours during the day

D) Assist the patient to the bathroom every 2 hours during the day

In discussing diet modifications the nurse encourages a client with cellulitus and severe inflammation to include: c) Pretzels d) Citrus fruit

D) Citrus Fruit

During the assessing component of the nursing process, the primary reason for interviewing the client is to: c) Provide emotional therapy d) Collect data

D) Collect Data

The patient's teaching plan includes this goal, "The patient will select 2 gram sodium diet from the hospital menu for the next three days". Which evaluation method will be best for the nurse to use. When determining whether teaching was effective: a) Check the sodium content of the patient's menu choices over the next three days c) Have the patient list favorite foods that are high in sodium and foods that could be substituted for these favorites d) Compare the patient's sodium intake over the next three days with the sodium intake before the teaching was implemented

D) Compare the patient's sodium intake over the next three days with the sodium intake before the teaching was implemented

A client reports to the nurse that she has been taking barbiturate sleeping pills every night for several months and now wishes to stop taking them. Which statement is the most appropriate advice for the nurse to provide the client? c) Discontinue taking the pills d) Continue taking pills and discuss tapering the dose with the primary care provider

D) Continue taking the pills and discuss tapering the dose with the primary care provider

A 72 year old who has benign prostatic hyperplasia is admitted to the hospital with chills, fever, and vomiting. Which finding by the nurse will be most helpful in determining whether the patient has an upper tract infection (UTI): c) Foul smelling urine d) Costovertebral tenderness

D) Costovertebral Tenderness

The nurse observes nursing assistive personnel (NAP) taking the following actions when caring for a patient with a retention catheter. Which action requires that the nurse intervene: c) Using an alcohol based hand cleaner before performing catheter care d) Disconnecting the catheter from the drainage tube to obtain a specimen

D) Disconnecting the catheter from the drainage tube to obtain a specimen

A client has a history of sleep apnea. Which is the most appropriate question for the nurse to ask? c) Have you had chest pain with or without activity d) Do you have difficulty with daytime sleepiness

D) Do you have difficulty with daytime sleeping

When assessing a 64 year old woman, the nurse notes that the patient has lost 1 inch in height since the previous visit 2 years ago. The nurse will plan to teach the patient about: c) Magnetic reasonable imaging (MRI) d) Dual energy x-ray absorption (OEXA)

D) Dual Energy X-ray Absorption (OXEA)

When providing care using evidence-based practice, the nurses uses: a) Clinical judgement based on experience c) Evidence-based guidelines in addition to clinical expertise d) Evaluation of data showing that the patient outcomes are met

D) Evaluation of data showing that the patient outcomes are met

A patient who has been admitted to the hospital for surgery tells the nurse, 'I do not feel right about leaving my children with my neighbor", which action should the nurse take next: a) Reassure the patient that these feelings are common for parents b) Have the patient call the children to ensure that they are doing well c) Call the neighbor to determine whether adequate childcare is being provided d) Gather more data about the patient's feeling about the child-care arrangements

D) Gather more data about the patient's feeling about the child-care arrangements

A patient is taking a potassium-wasting diurectic for treatment of hypertension. The nurse will teach the patient to report symptoms of adverse effects such as: a) personality change b) Frequent loose stools c) Facial muscle spasms d) Generalized weakness

D) Generalized weakness

A patient who has a wound infection after major surgery has only been taking in about 50% to 75% of the ordered meals and states, "Nothing on the menu really appeals to me." Which action by the nurse will be most effective in improving the patient's oral intake: a) Make a referral to the dietician d) Have family members bring in favorite foods from home

D) Have family members bring in favorite foods from home

A nursing student is learning the application of the nursing process to client care. When questioned by the student about the reason for implementing a nursing diagnosis, the nurse's professor responds: "The nursing diagnosis statement: a) Describes client problems that nurses are licensed to treat c) Includes the disease the client has during the treatment of care d) Helps standardize care for all clients

D) Helps standardized care for all clients

The nurse has admitted a patient with a new diagnoses of pneumonia and explained to the patient that together they will plan the patient's care and set goals for discharge. The patient says, "How is that different from what the doctor does?" Which response by the nurse is most appropriate: c) Nurses perform many of the procedures done by physicians, but nurses are here in the hospital for a longer time than doctors d) In addition to caring for you while you are sick, the nurses will assist you to develop an individualized plan to maintain your health

D) In addition to caring for you while you are sick, the nurses will assist you to develop an individualized plan to maintain your health

During an admission nursing assessment, a client with diabetes describes his leg pain as a "dull, burning sensation." The nurse recognizes this description to be characteristic of which type of pain: c) Visceral d) Neuropathic

D) Neuropathic

When the nurse is planning for the physical examination of an alert 86 year old patient. Adaptions to the examination technique should include: a) Speaking slowly when directing the patient b) Avoiding the use of touch as much as possible c) Using slightly more pressure for palpation of the liver d) Organizing the sequence to minimize position changes

D) Organizing the sequence to minimize position changes

When assessing the patient who has a lower urinary infections (UTI), the nurse will initially ask about: c) Poor urine output d) Pain with urination

D) Pain with urination

The patient has been experiencing difficulty and straining when expelling feces. Which intervention should the nurse discuss with the client: c) Encourage the client to use a cathartic laxative on a daily basis d) Place the client on a high fiber diet

D) Place the client on a high fiber diet

The nurse is caring for a client diagnosed with early osteoporosis. Which intervention is most applicable for this client: a) Institute an exercise plan that includes weight-bearing activities b) Protect the client's bones with strict bed rest d) Provide the client with assisted range of motion exercising twice daily

D) Provide the client with assisted range of motion exercising twice daily

A patient who has required prolonged mechanical ventilation has the following arterial blood gas results: pH 7.48, PaCO@ 32 mmttg, and HCO 25 mEq/L. The nurse interprets these results as: a) Metabolic acidosis b) Metabolic alkalosis c) Respiratory acidosis d) Respiratory alkalosis

D) Respiratory Alkalosis

A patient who is having difficulty breathing is admitted to the hospital. The best approach for the nurse to use to obtain a complete health history is to: a) Obtain subjective data about the patient's family membrane b) Omit subjective data collection and obtain the physical examination c) Use the health care provider's medical history to obtain subjective data d) Schedule several short sessions with the patient to gather subjective data

D) Schedule several short sessions with the patient to gather subjective data

The nurse is doing bowel and bladder retraining for the client with oaraplegia. Which of the following is NOT a factor for the nurse to consider: c) Fluid intake d) Sexual Function

D) Sexual Function

A nurse in instructing a hospitalized client with a diagnosis of emphysemia about measures that will enhance the effectiveness of breathing during dyspneic periods. Which of the following position will the nurse instruct the client to assume: c) Sitting in a recliner chair d) Sitting on the side of the bed and leaning on an over bed table

D) Sitting on the side of the bed and leaning on an over bed table

A patient returns to the surgical nursing unit following a vertical banded gastroplasty with a nasogastric tube to low, intermittent suction and a patient controlled analgesia (PCA) machine for pain control. Which nursing action should be included in the postoperative plan of care: b) Offer sips of sweetened liquids at frequent intervals c) remind the patient that PCA use may slow the return of bowel functions d) Support the surgical incision during patient coughing and turning in bed

D) Suport the surgical incision during patient coughing and turning in bed

A patient with protein calorie malnutrition who has had abdominal surgery is receiving potential nutrition (PN). Which assessment information obtained by the nurse is the best indicator that the patient is receiving adequate nutrition: a) Blood glucose is 110 m/dL b) Serum albumin level is 3.5 mg/dL d) Surgical incision is healing normally

D) Surgical incision is healing normally

When admitting a patient who has just Arrived on the medical unit with severe abdominal pain, what should the nurse do first: A) Complete only basic demographics data before addressing the patient's abdominal pain b) Medicate the patient for the abdominal pain before attending to the health history and examination c) Inform the patient that the abdominal pain will be treated as soon as the health history is completed d) Take the initial vital signs and then deal with the abdominal pain before completing the health history

D) Take the initial vital signs and then deal with the abdominal pain before completing the health history

While the nurse is assessing a 62 year old man, the patient says he does not respond to sexual stimulation the way he did when he was younger. The nurse's best response to the patient's comment is: c) Erectile dysfunction is a common problem with older man d) Tell me more about how your sexual response has changed

D) Tell me more about how your sexual response has changed

Which would be an expected outcome for a client with the following nursing diagnoses self-care deficit related to congnitive impairment: a) The client will be able to name the staff that works on the day shift b) The client will eliminate safety hazards in her environment c) The nurse will stress the importance of adequate fluid intake d) The client with supervision will brush her teeth

D) The client with supervision will brush her teeth

The nurse has formulated a diagnosis of Activity Intolerance related to Decreased Airway Capacity for chronic asthma. In looking at the client's coping skills, the nurse realizes that the patient has a vast knowledge about the disease and what exacerbates symptoms in particular situations. The nurse will utilize this information because: a) Strengths can be an aid to mobilizing health and the healing process c) It will be easier for the nurse to educate the client about other interventions d) The nurse wont have to spend time going over the pathology of the client's disease

D) The nurse wont have to spend time going over the pathology of the client's disease

The nurse has just received change-of-shift report about the following four patients which patient will the nurse assess first: a) The patient who has multiple black wounds on the feet and ankles b) The newly admitted patient with a stage IV pressure ulcer on the coccyx c) The patient who needs to be medicated with multiple analgesics before a scheduled dressing change d) The patient who has been receiving immunosuppressants medications and has a temp of 102' F

D) The patient who has been receiving immunosuppressants medications and has a temp of 102' F

The nurse is performing an admission assessment on a 20 year old college student who is being admitted for electrolyte disorders of unknown etiology. Which assessment is most important to report to the health care provider: c) The patient has history of weight fluctuations d) The patient's serum potassium level is 2.9 mEq/L

D) The patient's serum potassium level is 2.9 mEq/L

Nurses often utilize systems theory to assess family units. Which example illustrates a family unit that does NOT meet the criteria of a well-functioning system? c) Each member's personal boundaries are well defined d) The primary activities of each member focus on personal purposes

D) The primary activities of each member focus pn personal purposes

While admitting a patient to the medical unit, the nurse learns that the patient does not read well. This information will guide the nurse in determining: a) The degree of patient motivation and readiness to learn b) What information the patient will be able to understand c) That the family must be included in the teaching process d) Which instructional strategies should be used in teaching

D) Which instructional strategies should be used in teaching

The nurse is developing a weight loss plan for a 21 year old patient who is morbidly obese. Which statement by the nurse is most likely to help the patient in loosing weight on the planned 1000 calorie diet: c) Most of the weight that you lose during the first weeks of dieting is water weight rather than fat d) You are likely to start to notice changes in how you feel with just a few weeks of diet and exercise

D) You are likely to start to notice changes in how you feel with just a few weeks of diet and exercise

Upon assessment of a "PT" with myasthenia gravis, the nurse observes drooping of the upper eyelids. What is this finding known as? A)Ectropion B)Entropion C)Miosis D)Ptosis

D)Ptosis Drooping of the upper eyelids and is an abnormal finding

A nurse is observing a nursing assistant talking to a "PT" who is hearing impaired. The nurse would intervene if which of the following is preformed by the nursing assistant during communication with the "PT"? A)The nursing assistant is speaking in a normal tone B)The nursing assistant is speaking clearly to the "PT" C)The nursing assistant is facing the "PT" when speaking D)The nursing assistant is speaking directly into the impaired ear.

D)The nursing assistant is speaking directly into the impaired ear. When communicating with a hearing impaired "PT"the nurse should speak in a normal tone and should not shout. The nurse should talk directly to the "PT" while speaking clearly . If the "PT" does not seem to understand what was said the nurse can state it differently or move closer to the "PT"

A nurse has explained her intention to conduct Weber's test and Rinne's test. Which is the following pieces of equipment will the nurse require? A)Ophthalmoscope B)Otoscope C) Snellen Chart D)Tuning Fork

D)Tuning Fork Weber's test and Rinne's test are performed in order to assess sound conduction; both require a tuning fork

The nurse is caring for patients in the student health center. A patient confides to the nurse that the patient's boyfriend informed her that he tested positive for Hepatitis B. Which of the following responses by the nurse is BEST? a. "That must have been a real shock to you" b. "You should be tested for Hepatitis B" c. "You'll receive the Hepatitis B immune globulin HBIG d. "Have you had unprotected sex with your boyfriend"

D. Hepatitis B is transmitted through parenteral drug abuse and sexual contact. Determine exposure before implementing.

A nurse is assessing a pt with a closed chest tube drainage system connected to suction. Which finding would require additional evaluation in the post-operative period? A. 75ml of bright red drainage in the system. B. A column of water 20cm high in the suction control chamber. C. Constant bubbling in the water seal chamber.

D. Constant bubbling in the water seal chamber is indicative of an air leak. The nurse should assess the entire system to the pt to find the sourse of he leak. The leak may be with in the pts chest or at the insertion site. If it is, notify physician. This could cause the lung to collapse due to a buildup of air pressure within the plural cavity.

After being admitted directly to the surgery unit, a 75-year-old client who had elective surgery to replace an arthritic hip was discharged from the postanesthesia recovery unit. The client has been on the orthopedic floor for several hours. Which type of planning will be least useful during the first shift on the orthopedic unit? A: Initial B: Ongoing C: Discharge D: Strategic

D: Strategic

When taking a health history, the nurse screens for manifestations suggestive of diabetes type I. Which of the following manifestations are considered the primary manifestations of diabetes type I and would be most suggestive of diabetes type I and require follow-up investigation? a. Excessive intake of calories, rapid weight gain, and difficulty losing weight b. Poor circulation, wound healing, and leg ulcers, c. Lack of energy, weight gain, and depression d. An increase in three areas: thirst, intake of fluids, and hunger

D. An increase in three areas: thirst, intake of fluids, and hunger The primary manifestations of diabetes type I are polyuria (increased urine output), polydipsia (increased thirst), polyphagia (increased hunger).

Parathyroid levels related to calcium levels

DIRECT correlation increase= increase

While a nurse is caring for a patient who is scheduled to have surgery in 2 hours, the patient states, "My doctor was here and told me a lot of stuff I didn't understand and then I signed a paper for her." To fulfill the role of advocate, what is the best nursing action? A. Reassure the patient that the surgery will go as planned. B. Explain the surgery and possible outcomes to the patient. C. Complete her first priority, the preoperative teaching plan. D. Call the physician to return and clarify information for the patient.

D. Call the physician to return and clarify information for the patient. Examples of nursing advocacy include questioning doctors' orders, promoting patient comfort, and supporting patient decisions regarding health care choices.

Which of the following items of subjective client data would be documented in the medical record by the nurse? A. Client's face is pale B. Cervical lymph nodes are palpable C. Nursing assistant reports client refused lunch D. Client feel nauseated

D. Client feel nauseated Rationale: Subjective data includes the client's sensations, feelings, and perception of health status. Subjective data can only be verified by the affected person. Options 1, 2, and 3 represent objective data that can be detected by the nurse or measured against an accepted norm.

A client on the nursing unit is terminally ill but remains alert and oriented. Three days after admission, the nurse observes signs of depression. The client states, "I'm tired of being sick. I wish I could end it all." What is the most accurate and informative way to record this data in a nursing progress note? A. Client appears to be depressed, possibly suicidal B. Client reports being tired of being ill and wants to die C. Client does not want to live any longer and is tired of being ill D. Client states, "I'm tired of being sick. I wish I could end it all."

D. Client states, "I'm tired of being sick. I wish I could end it all." Rationale: Subjective data includes thoughts, beliefs, feelings, perceptions, and sensations that are apparent only to the person affected and cannot be measured, seen, or felt by the nurse. This information should be documented using the client's exact words in quotes. The other options indicate that the nurse has drawn the conclusion that the client no longer wishes to live. From the data provided, the cues do not support this assumption. A more complete assessment should be conducted to determine if the client is suicidal.

The nurse would write which of the following outcome statements for a client starting an exercise program? A. Client will walk quickly three times a day B. Client will be able to walk a mile C. Client will have no alteration in breathing during the walk D. Client will progress to walking a 20-minute mile in one month

D. Client will progress to walking a 20-minute mile in one month Rationale: Outcome statements must be written in behavioral terms and identify specific, measurable client behaviors. They are stated in terms of the client with an action verb that, under identified conditions, will achieve the desired behavior. They should also be realistic and achievable.

The nurse is assessing a client 24 hours following a cholecystectomy. The nurse noted that the T tube has drained 750 mL of green-brown drainage since the surgery. Which nursing intervention is appropriate? A. Clamp the T tube B. Irrigate the T tube C. Notify the physician D. Document the findings

D. Document the findings

The nurse evaluates the client's progress and determines that one of the nursing diagnoses on the client's care plan has been resolved. How should the nurse document this so that it is best communicated to the healthcare team? A. Use Liquid PaperTM to "white out" the resolve diagnosis on the care plan B. Recopy the care plan without the resolve diagnosis C. Write a nursing process not indicating that the outcome goals have been achieved D. Draw a single line through the diagnosis on the care plan and write the nurse's initials and date

D. Draw a single line through the diagnosis on the care plan and write the nurse's initials and date Rationale: To discontinue a diagnosis once it has been resolved, cross it off with a single line or highlight it, then write initials and date. Some agency forms may require the nurse to put date and initials in a "Date Resolved" column. Using Liquid PaperTM is not a legal way to amend client records. Outcome goals that have been met and nursing diagnoses that have been resolved should be documented on the care plan. A progress note should also be written, but a single note may not be read by all health team members.

For the nursing diagnostic statement, Self-care deficit: feeding related to bilateral fractured wrists in casts, what is the major related factor or risk factor identified by the nurse? A. Discomfort B. Deficit C. Feeding D. Fractured wrists

D. Fractured Wrists Rationale: The etiology or related factors of a nursing diagnostic statement define one or more probable causes of the problem and allow the nurse to individualize the client's care. In this case, the fracture is the cause of the client's feeding problem.

Should tubing for a blood transfusion be macrodrip or microdrip?

Macrodrip

The rehabilitation nurse wishes to make the following entry into a client's plan of care: "Client will reestablish a pattern of daily bowel movements without straining within two months." The nurse would write this statement under which section of the plan of care? A. Nursing diagnosis/problem list B. Nursing orders C. Short-term goals D. Long-term goals

D. Long-term goals Rationale: Long-term goals describe changes in client behavior expected over a time frame greater than one week. They are usually designed to restore normal functioning in a problem area and are helpful to other healthcare workers who care for the client, often in a variety of settings.

A nurse explains to a student that the nursing process is a dynamic process. Which of the following actions by the nurse best demonstrates this concept during the work shift? A. Nurse and client agree upon health care goals for the client B. Nurse reviews the client's history on the medical record C. Nurse explains to the client the purpose of each administered medication D. Nurse rapidly reset priorities for client care based on a change in the client's condition

D. Nurse rapidly reset priorities for client care based on a change in the client's condition Rationale: The nursing process is characterized by unique properties that enable it to respond to the changing health status of the client. Options 1, 2, and 3 are appropriate nursing care measures, but do not demonstrate the dynamic nature of the nursing process.

While assisting a client from bed to chair, the nurse observes that the client looks pale and is beginning to perspire heavily. The nurse would then do which of the following activities as a reassessment? A. Help client into the chair but more quickly B. Document client's vital signs taken just prior to moving the client C. Help client back to bed immediately D. Observe client's skin color and take another set of vital signs

D. Observe client's skin color and take another set of vital signs Rationale: Assessment is ongoing throughout the nurse-client relationship. During re-assessment, the nurse collects additional data to help evaluate the status of problems or identify new problems. Options 1, 2, and 3 are interventions.

Mr. Mitchell and Mr. Farrell have both had their gallbladders removed laparascopically. Mr. Mitchell is rating his pain at a 5 on a 10-pain scale and states he does not require pain medication. Mr. Farrell is rating his pain a 5 on a 10-pain scale and is demanding something stronger for his pain. This is an example of a difference in which of the following? a. Surgeon's skill b Patient's pain thresholds c. Patient's personalities d. Patient's pain tolerance

D. Patient's pain tolerance Rationale: Both patients would perceive the surgical incision to be painful at about the same point. Mr. Mitchell is able to tolerate his pain when it is rated at a 5, whereas Mr. Farrell is not. The patient's personality is a factor that affects pain tolerance.

A nurse has requested and gotten permission to observe a surgical procedure of interest in the hospital in which the nurse is employed. While the patient is being draped, the nurse notices that a break in sterile technique occurs. Which of the following actions on the nurse's part is most appropriate? A. Tell the surgeon before an incision is made. B. Tell the circulating nurse at the end of the surgery. C. Say nothing because someone else is likely to notice also. D. Point out the observation immediately to the personnel involved.

D. Point out the observation immediately to the personnel involved. Any break in sterile technique in the operating room should be immediately pointed out and remedied.

You are doing some teaching with a client who is starting on a sulfonylurea antidiabetic agent. The client mentions that he usually has a couple of beers each night and takes an aspirin each day to prevent heart attack and/or strokes. Which of the following responses would be best on the part of the nurse? a. As long as you only drink two beers and take one aspirin, this should not be a problem b. The aspirin is alright but you need to give up drinking any alcoholic beverages c. Aspirin and alcohol will cause the stomach to bleed more when on a sulfonylurea drug d. Taking alcohol and/or aspirin with a sulfonylurea drug can cause development of hypoglycemia

D. Taking alcohol and/or aspirin with a sulfonylurea drug can cause development of hypoglycemia Alcohol and/or aspirin taken with a sulfonylurea can cause development of hypoglycemia.

A physician is performing a sterile procedure at a patient's bedside. Near the end of the procedure, the nurse thinks that the physician has contaminated a sterile glove and the sterile field. The nurse should A. report the physician for violating surgical asepsis and endangering the patient. B. not say anything, because the nurse is not sure that the gloves and field were contaminated. C. ask the physician whether the contaminated glove and the sterile field have been contaminated. D. point out the possible break in surgical asepsis and provide another set of sterile gloves and fresh sterile field.

D. point out the possible break in surgical asepsis and provide another set of sterile gloves and fresh sterile field. It is the responsibility of the nurse to point out any possible break in surgical asepsis when others are unaware that they have contaminated the field. Reporting the physician is not indicated, nor does it protect the patient. Asking the physician may lead to infection if the physician is unaware of the break in technique that the nurse believes may have happened. Saying nothing does not protect the patient and is negligence on the part of the nurse.

To prevent airway obstruction in the postoperative patient who is unconscious or semiconscious, the nurse A. encourages deep breathing. B. elevates the head of the bed. C. administers oxygen per mask. D. positions the patient in a side-lying position.

D. positions the patient in a side-lying position. An unconscious or semiconscious patient should be placed in a lateral position to protect the airway from obstruction by the tongue. Deep breathing and elevation of the head of the bed are implemented to facilitate gas exchange when the patient is responsive. Oxygen administration is often used, but the patient must first have a patent airway.

A RN overhears a LPN talking with a client who is being prepared for a total colectomy with the creation of an ileoanal reservoir for ulcerative colitis. To decrease the client's anxiety, the RN should intervene to clarify the information given by the LPN when the LPN is heard saying: A. this surgery will prevent you from developing colon cancer. B. after this surgery you will no longer have ulcerative colitis. C. when you return from surgery you will not be able to eat solid food for several days. D. you will have an ileostomy when you return from the surgery.

D. you will have an ileostomy when you return from the surgery.

An external insulin pump is prescribed for a client with diabetes mellitus and the client asks the nurse about the functioning of the pump. The nurse bases the response on the information that the pump: a) is timed to release programmed doses of regular or NPH insulin into the bloodstream at specific intervals b) continuously infuses small amounts of NPH insulin into the bloodstream while regularly monitoring blood glucose levels c) is surgically attached to the pancreas and infuses regular insulin into the pancreas, which in turn releases the insulin into the bloodstream d) gives a small continuously dose of regular insulin subcutaneously, and the client can self-administer a bolus with an additional dose form the pump before each meal

D.) gives a small continuously dose of regular insulin subcutaneously, and the client can self-administer a bolus with an additional dose form the pump before each meal An insulin pump provides a small continuous dose of regular insulin subcutaneously throughout the day and night, and the client can self-administer a bolus with an additional dose from the pump before each meal as needed. Regular insulin is used in an insulin pump. An external pump is not attached surgically to the pancreas.

Nurses use a standardized care plan as: A: A means to address all of the client's disease processes B: A guide in determining if the client is able to assist with the care planning C: A guide for developing nursing diagnoses D: A guide for developing goals and interventions

D: A guide for developing goals and interventions

When writing a nursing goal and desired outcome, the nurse is aware that goals should: A: Be prioritized B: Be taken from a standardized list C: Have physician input D: Be realistic for the client

D: Be realistic for the client

Assessment of a client who is 2 days after surgery reveals a dressing that is dry and intact, temperature 100.2 degrees F, pulse 90, and respiratory rate of 36. The client requests additional juice or water due to a very dry mouth, and says he is feeling weak and having pain with ambulation. The nurse's highest priority finding that indicates that the plan of care should be changed is: A: Elevated temperature B: Dry mouth C: Pain D: Elevated respiratory rate

D: Elevated respiratory rate

Prediabetes is associated with all of the following except: a. Increased risk of developing type 2 diabetes b. Impaired glucose tolerance c. Increased risk of heart disease and stroke d. Increased risk of developing type 1 diabetes

D: Increased risk of developing type 1 diabetes Persons with elevated glucose levels that do not yet meet the criteria for diabetes are considered to have prediabetes and are at increased risk of developing type 2 diabetes. Weight loss and increasing physical activity can help people with prediabetes prevent or postpone the onset of type 2 diabetes.

The nurse uses a standardized care plan to develop an individualized care plan for each client. Match the nursing action to the appropriate holistic care plan approach or rationale: The nurse includes family and client preoperative teaching in the client's care plan. A: Ongoing individualized care planning B: Standardized care plan C: Complete individualized plan of care D: Individualized care plan

D: Individualized care plan

Which of the following accurately explain how the nurse chooses a nursing intervention? A: Interventions are nurse-initiated activities only B: Most interventions are part of the nurse's dependent role C: Interventions are chosen to alleviate or reduce the impact of the client's medical diagnosis D: Interventions focus on the etiology of the nursing diagnosis

D: Interventions focus on the etiology of the nursing diagnosis

When writing a care plan, a nurse may use what model or theory to assist in prioritizing? A: Orem's model B: Roy's model C: The most important disease process that the client is experiencing D: Maslow's hierarchy of needs

D: Maslow's hierarchy of needs

The nurse chooses nursing interventions from the nursing intervention classification based on: A: The nursing process B: Nursing assessments C: Nursing outcomes classification D: Nursing diagnosis

D: Nursing diagnosis

The nurse finds the nursing interventions of touch in the Nursing Interventions Classification (NIC) by looking up the: A: Problem list B: Client's specific need C: Medical diagnosis D: Nursing diagnosis

D: Nursing diagnosis

The nurse, who is caring for a client with a nursing diagnosis of Ineffective Airway Clearance, instructs the postoperative client on turning, coughing, and deep breathing every 2 hours. What is the relationship of nursing interventions to problem status? A: Observation interventions B: Health promotion interventions C: Treatment interventions D: Prevention interventions

D: Prevention interventions

Black:

Debride

Nurse ask when history of BPH:

Decrease in force of urinary stream

The dietary modification that a nurse should suggest to a patient with premenstrual syndrome (PMS) is to:

Decrease intake of salt

A home health nurse is performing an initial assessment on an elderly patient. Which physiologic changes should the nurse anticipate as being "normal" for an elderly patient?

Decreased Salivary Production; Delayed Wound Healing; Loss of Visual Acuity

Which of the following may indicate pain in a 76-year-old patient with communication impairments?

Decreased appetite, tearing of the eyes, and increased social isolation may indicate pain in an older patient with communication problems.

How does managed care affect the economics of health care delivery?

Decreases duplicate processes and redundant facilities rationale: The corporatization of health care amalgamates the smaller health care operations into the larger organizations to decrease duplicate and redundant health care provisions. For example, one hospital would provide a high level of cardiac care, while another hospital would focus on oncology.

Assessment: Increased drainage. Opened wound edges. Appearance of underlying tissues through the wound.

Dehiscence.

A nurse is performing an assessment of a patient who underwent surgical hernia repair for 8 hours ago. If the nurse notes that the patient has lower abdominal distention accompanies by dullness to percussion over the distended area, the priority nursing action is to:

Determine the last time the patient voided.

A nurse establishes the presence of a tensionpneumothorax when assessment findings reveal a(n):

Deviation of the trachea toward the side opposite the pneumothorax,

Client with TB (home instructions)

Dietary: increase intake of protein, iron, and Vit C considered not contagious after 2-3 weeks of treatment can return to work after 3 negative sputum cultures transmitted by droplet nuclei should wear mask in public medication therapy usually lasts 6-12 months

The primary goal of the continuous quality improvement process of Six Sigma is to:

Diminish misuse of processes rationale: The primary goal of Six Sigma is to increase profits by improving standard operating procedures, reducing errors, and decreasing misuse of the system.

NAP taking action to get urinary specimen, action requires nurse intervene:

Disconnecting catheter drainage tube

A 72 year old patient is hospitalized after a fall at home, is restless, has elevated blood pressure, and moans with turns. When the nurse asks, the patient denies being in pain. What initial interventions should the nurse employ?

Discuss the symptoms and explain how medication will increase comfort and increase healing Older adults are hesitant to express pain becasue they may fear being labeled as a complainer

A nurse should obtain an air mattress for a patient at risk for impaired skin integrity because it will:

Distribute body weight over a larger area.

Two days after surgery, patient wont participate with care and will only let ostomy nurse provide care:

Disturbed Body Image

The primary role of The Joint Commission (TJC) is:

Ensuring medical facilities meet client safety guidelines rationale: The Joint Commission is the primary accrediting body for health care institutions. Its standards directly address client safety issues. Magnet status is approved by the American Nurses Association. TJC does not lobby Medicare/Medicaid issues. The CDC would be the agency that maintains standards regarding infection control for hospital compliance.

While reviewing the medication list for an older client with a history of heart failure, diabetes, and hypertension, which medication might cause concern?

Dolobid 250mg Salicylate salts containing mg or na should be avoided in clients whom excessive amounts of these electrolytes might be harmful

Which of the following reduces the discomfort of dysphagia?

Drinking fluids and retching or vomiting are effective methods of clearing the bolus of food from the esophagus. Coughing, lying down, and massaging the abdomen have no effect.

Drug Use in Pre-Op setting: Benzodiazepines and barbiturates

Drug used in pre-op setting for sedation and amnesia

Drug Use in Pre-Op setting: Opioids

Drug used in pre-op setting to decrease intraoperative anesthetic requirements and pain.

Drug Use in Pre-Op setting: Anticholinergics

Drug used in pre-op setting to reduce secretions.

Fetal Circulation (shunts)

Ductus Venosus: feeds placental blood in through the hepatic portal vein Ductus Arteriousus: between aorta and left pulmonary artery Foramen Ovale: between right and left atrium

If a patient states, "It's hard for me to breath and I feel short-winded all the time." What is the most appropriate terminology to be applied in documenting this assessment by a nurse?

Dyspnea

C. A patient with abdominal and chest pain following a large, spicy meal.

Emergency department triage is an important nursing function. A nurse working the evening shift is presented with four patients at the same time. Which of the following patients should be assigned the highest priority? A. A patient with low-grade fever, headache, and myalgias for the past 72 hours B. A patient who is unable to bear weight on the left foot, with swelling and bruising following a running accident. C. A patient with abdominal and chest pain following a large, spicy meal. D. A child with a one-inch bleeding laceration on the chin but otherwise well after falling while jumping on his bed.

Nursing intervention, preventing UTI:

Empty Bladder Completely

Prevention of UTI when patient states:

Empty bladder every 3/4 hrs. throughout day

Conscious sedation is being considered for a patient undergoing a cervical dilation and endometrial biopsy in the health care provider's office. The patient asks the nurse, "What is this conscious sedation?" The nurse's response is based on the knowledge that conscious sedation:

Enables the patient to respond to commands and accept painful procedures.

If a patient newly diagnosed with colon cancer has become withdrawn from family members, which of the following strategies should the nurse use to assist the patient at this time?

Encourage the patient to identify fears and verbalize feelings.

Adult masturbating, nurse should:

Excuse me and leave the room

The primary barrier to the implementation of quality improvement processes is:

Expense rationale: A primary barrier to implementing effective quality improvement programs is the cost. The cost of providing health care has greatly increased over the past few decades. However, through quality improvement measures, overall health care costs can be reduced.

A patient found unresponsive now opens eyes when spoken to, answers simple questions when asked, and usually sleeps when left unattended. Using the Glasgow Coma Scale, the nurse should document the assessment findings using which of the following categories?

Eye opening and best verbal response

A 93-year old patient is likely to demonstrate hopelessness by

Failing to follow medical recommendations

The preparation of a nursing care plan (NCP) for a resident in a long-term care facility should involve input from:

Family Members; Social Worker; The Resident; and all licensed staff

Which task could a staff nurse delegate to a CNA?

Feeding a stroke patient who has minimal dysphagia.

client at greater risk for bladder infection:

Female not taking estrogen therapy- females at greater risk

A nurse would speculate that the a pt who has been started on an IV blood transfusion for 30 minutes has bacteremia due to what clinical manifestation?

Fever

A nurse teaching a patient the potential complications of osteoporosis should include information on:

Fractures of the hip, waist, and spine.

A delivery of nursing care organized around tasks is known as:

Functional nursing rationale: Aides and vocational nurses/licensed practical nurses are allowed to perform functions such as administration of medication and treatments. This functional kind of nursing results in a fragmented, impersonal kind of care.

Which of the following are traits that predispose people to cardiovascular disease and are difficult to modify?

Genetic traits that are difficult to modify and that may lead to cardiovascular disease include obesity, diabetes, hypertension, and high cholesterol, as well as family history of cardiovascular disease.

5 year old son playing naked with another boy:

Get boys interested in another activity

Clinic with recurrent dysuria, and taking Bactrum, nurse should:

Get culture

A 75-year-old patient who has vision and hearing problems has a history of striking out at caregivers. Which of the following is the most appropriate nursing intervention?

Get the patient's attention and consent before starting care.

Which of the following should be avoided when trying to assist an older patient who has dysphagia to eat?

Give large bites to stimulate swallow reflex.

The nurse is called to a patients room who complains of pain 9/10 and requests pain medication. He is laughing, watching football, and is in conversation with a visitor. Based on the assessment, what intervention should the nurse employ?

Give the total dose of pain medication Pain is a multidimensional phenomenon that is difficult to define. It is personal and subjective and is whatever the patient says it is.

Patients with DM and chronic renal failure who are treated peritoneal dialysis may have difficulty controlling diabetes because:

Glucose is absorbed from the dialysate.

A pt is neutropenic, what is the pt likely to be infused with?

Granulocyte or white blood cell transfusion

Which of the following foods is high in iron?

Green leafy vegetables

Plan of care, chronic constipation, which foods nurse should emphasize:

Greens

Which of the following contributes to peptic ulcer formation?

H. pylori infection

Cushing's disease/ syndrome

HYPERsecretion of glucocorticoids generalized muscle wasting and weakness moon face, buffalo hump truncal obesity with thin extremeties; supraclavical fat pads; weight GAIN hirsutism (masculine characteristics in female) HYPERglycemia, HYPERnatremia HYPOkalemia, HYPOcalcemia hypertension fragile skin, easily bruises reddish-purple striae on abdomen and upper thighs

A RN discovers that an infusion of peripheral parenteral nutrition is empty, and a replacement bag is not ready yet. What should the RN do next while waiting for the PPN bag?

Hang an IV infusion of 10% dextrose in water.

The nurse suctions the patient's ET when a patient:

Has coarse rhonchi over central airways.

Why is a hypotonic solution like 0.45% normal saline an inappropriate IV solution for a pt with cerebral edema?

Has the potential to cause cellular swelling and cerebral edema.

Colonoscopy, nurse action:

Have them sign informed consent

What are examples of regulatory agencies that will affect health care policy with regard to informatics?

Health Insurance Portability and Accountability Act (HIPAA) rationale: The Joint Commission (TJC) and the Health Insurance Portability and Accountability Act (HIPAA) are two regulatory and governmental agencies instituting health care policy.

When providing care to a dying patient, a nurse should remember that which of the following is the longest-lasting sense?

Hearing

Which of the following may be left in place when a patient is sent to the operating room?

Hearing aid

APGAR score

Heart Rate: absent (0); < 100 (1); > 100 (2) Respiratory Effort: absent (0); slow/ irregular/weak cry (1); crying lustily (2) Muscle Tone: limp or hypotonic (0); some extremity flexion (1); active, moving, and well-flexed (2) Irritability/ Reflexes/ Response to Bulb-Suctioning: no response (0); grimace (1); cough, sneeze, or vigorous cry (2) Color: cyanotic/ pale (0); acrocyanotic (1); pink (2)

A hospital employee fails to properly dispose of a sharp used on a patient. The employee is stuck with an infected needle. A nurse in the ED assesses the hospital employee knowing that the employee is at risk for contracting numerous illnesses from the needle stick. Which types of hepatitis could be contracted from the needle stick?

Hepatitis A, B, C and D may all be transmitted parenterally. Hepatitis E is transmitted only via fecal-oral route..

Immunizations 12 mo.

Hib [Haemophilus influenzae type b] (3 of 3) PCV [Pneumococcal] (4 of 4) MMR [Measles, mumps, and rubella] (1 of 2) Varicella (1 of 2)

Which of the following is not a positive risk factor for coronary heart disease?

High-density lipid cholesterol >60 mg/dL

In renal failure, protein intake should be approximately 1 to 1.4g per kg body weight. What is the best source of this protein?

High-quality proteins

For an intermittent IV infusion that is piggybacked to the primary IV line, the bag for the intermittent infusion is placed_____ than the primary solution.

Higher

Which of the following is an appropriate intervention when caring for an 83-year-old patient as death nears?

Hold the patient's hand and state, "You're not alone."

Preterm infants

Hyperbilirubin, retinopathy, necrotizing enterocolitis

A patient is admitted to the hospital with fever, chills, a productive cough with rusty sputum, and pleuritic chest pain. Pneumococcal pneumonia is suspected. An appropriate nursing diagnosis for the patient based on the patient's manifestations is:

Hyperthermia related to acute infectious process.

5% dextrose in 0.45% sodium chloride is what type of IV solution?

Hypertonic

If a plasma expander is not available, what IV solution would you use to treat hypovolemia?

Hypertonic solution

What IV solution is the fluid of choice for a pt with excess fluid volume?

Hypertonic solution of 5% dextrose in 0.9% normal saline

If a patient with type 1 DM runs 3 miles, falls asleep on the sofa, and forgets to eat the next meal, which of the following may be experienced?

Hypoglycemia

329 Nurse providing instructions to adolescent who has a history of seizures and is taking anticonvulsant medication. Which of following statements indicates that client understands the instructions?

I cant' drink alcohol while I am taking my medication. R: Alcohol will lower seizure threshold and s/be avoided. Adolescents can obtain a driver's license in most states when they have been seizure free for 1 year. Anticonvulsants cause acne and oily skin. If anticonvulsant med is missed, doc s/be notified.

Which nursing organization was the first international organization of professional women? ICN, ANA, NLN, or NSNA

ICN

Relationship between K and Na

INVERSE relationship increase serum potassium = decreased (increased excretion of) sodium

What routes of medications are acceptable for laboring pt?

IV b/c PO meds are affected by the dec in GI motility

During the outcome ID and planning step of the nursing process, the nurse works in partnership with the patient and family to do which of the following?

Identify expected patient outcomes, select evidence-based nursing interventions, and communicate the plan of nursing care.

During the define phase of the define-measure-analyze-improve-control (DMAIC) process, the team members:

Identify the stakeholders rationale: In the define phase, a charter is developed; goals, team leaders, membership, and team roles and responsibilities are identified; and the stakeholders affected by the process are identified.

B. Dyspnea

If a nurse is monitoring for signs of intestinal obstruction, the patient would be monitored for all but one of the following: A. Constipation B. Dyspnea C. Vomiting D. Abdominal distention

A. Metformin (Glucophage)

If a patient scheduled for cardiac catheterization has chest pain and a history of Type II Diabetes Mellitus, which medication would need to be held 48 hours prior to the procedure? A. Metformin (Glucophage) B. Regular insulin C. Repaglinide (Prandin) D. Glipizide (Glucotrol)

A non-english speaking hispanic client is moaning and appears to be in pain. How does the nurse intervene to faciliatate adequate pain management?

If an interpreter is available, explain that pain is related to illness and by treating the pain healing will promote wellness Moaning and crying are used to alleviate the pain rather than communicate a need for intervention. If the patient understands that pain is related to illness there is a higher likelihood that the patient will accept treatment.

Immunoglobulins (antibodies)

IgA: high concentrations in the mucous membranes and secretions of respiratory passages and GI tract (saliva, tears) IgG: most abundant, found in all body fluids, extra vascular spaces, placental transfer IgM: found mainly in blood and lymph, first to fight new infection IgE: associated with allergic reactions and parasitic diseases (halminth worms) IgD: minute amounts in blood, least understood

Etiology of heart failure after MI is related to the:

Impairment of the cardiac function of the ventricle.

The most common reason elective cosmetic surgery is requested by patients is to:

Improve self-image

C. control the heart rate and maintain cardiac output

In caring for a client with atrial flutter, which of the following goals would have priority? A. Reduce the ventricular rate to below 100 beats per minute B. Identify and treat the underlying cause C. control the heart rate and maintain cardiac output D. Increase the heart rate

A. High blood glucose damages capillaries.

In educating a client about Type II Diabetes, what would be a proper explanation for poor wound healing? A. High blood glucose damages capillaries. B. Swings in blood sugar prevent proper clotting. C. The pancreas fails to secrete the proper chemicals D. Ketosis prevents proper healing

Low-residue diet to prevent constipation instruct client to:

Increase fluid intake

Women experiencing menopause should:

Increase fluid intake

Hypertension is related to:

Increased BP is related to increased atherosclerosis, stroke, nephropathy, increased peripheral vascular disease, aneurysm, and heart failure.

Which statement is true about the case management model of nursing care?

It is based on previously defined client outcomes. rationale: Clinical pathways are a tool used to support case management and to define client outcomes.. These are interdisciplinary plans of care that outline the optimal sequencing and timing of interventions for clients with a particular diagnosis, procedure, or symptom to achieve a defined client outcome.

The patient with end-stage renal disease (ESRD) tells the nurse that she hates the thought of being ties to the machine nut is glad to start dialysis because she will be able to eat and drink what she wants. Based on this information, the nurse identifies the nursing diagnosis of:

Ineffective management of therapeutic regimen related to lack of knowledge of treatment.

What is the main complication for pt who has parenteral nutrition?

Infection

Which of the following should be included in a community discussion regarding the transmission of anthrax?

Infection is acquired when spores enter the host.

When performing a musculoskeletal assessment on a patient who complains of muscle weakness with cramping, the nurse should assess for:

Ingestion of a potassium-wasting diuretic.

Anticoagulant therapy is used in the treatment of thromoembolic disease because anticoagulants can:

Inhibit the synthesis of clotting factors.

When a patient complains of nausea after receiving the first dose of morphine for pain, the nurse should:

Instruct the patient that with continued use, nausea lessens.

Insulin: NPH

Intermediate onset: 2 hr peak: 6-8hr duration: 12-16hr

A resting HR of 55 bmp is a normal finding in a patient who:

Is an athlete

A Romberg's test is negative is a patient:

Is unable to hold the arms out at shoulder level without pronation.

What is the definition of nursing informatics?

It is the activities focused on collecting, processing, and analyzing nursing data electronically to support nursing practice and knowledge. rationale: The ANA has defined informatics as "the activities involved in identifying, naming, organizing, grouping, collecting, processing, analyzing, storing, retrieving or managing data and information."

With regard to nursing informatics, what is meant by general systems theory?

It organizes interdependent parts that, when working together, can produce a product that none used alone could produce. rationale: Nursing informatics uses a theoretical foundation. This theory organizes interdependent parts that, when working together, can produce a product that none of the interdependent parts used alone could produce.

Which of the following best describes the concept of evidence-based practice?

It provides for the use of recent research in current nursing practice. rationale: The IOM reported that it took 17 years for research to be reflected into daily practice. Evidence based practice is the use of the current best evidence to provide care and make decisions regarding client care.

80 year old worried no BM every day, nurse should say:

It's normal to go 2x a week

The person known for emphasizing the Pareto principle (80/20 rule) leading to the idea of total quality management is:

Joseph Juran rationale: Joseph Juran is one of the forefathers of quality initiatives. He stressed the meaning of the Pareto principle and how it applies to improving quality in all organizations.

To improve the quality of sleep for a confused patient, the nurse should plan to:

Keep patient awake during the day.

To ensure the correct amount of oxygen delivery for a patient receiving 35% oxygen via Venturi mask, it is most important that the nurse:

Keep the air-entrainment ports clean and unobstructed

Most important for nurse, on sexual problems:

Know how you feel first

Isoenzymes affected after acute MI

LDH1 and LDH2 ... begin to elevate 24 hrs post MI and peak 48-72 hours after...return to normal 7-14 days later

321Which of folowing represents a primary characteristic of autism?

Lack of social interaction and awareness. R: Autism is a severe developmental disorder that begins during infancy or toddlerhood. Primary char. is lack of social interaction and awareness. Social behaviors in autism include a lack of or an abnormal imitation of others' actions and a lack of or abnormal social play. Additional characteristics include a lack of or impaired verbal communication and markedly abnormal nonverbal communication.

Compared with pain from an MI, angina usually:

Lasts less than 15 minutes

Assessing UTI labs, finding immediate intervention:

Left shift of WBCs

Client statements indicates Pyridium is effective:

Less bladder pain/burning

A patient is being assessed for the possible UTI. Before sending a urinalysis specimen to the laboratory, a nurse collects a small amount of urine to perform a dipstick test. The nurse anticipate a positive reading for:

Leukocytes

Dilantin- WbC

Leukopenia is a serious adverse effect of phenytoin and would require discontinuation of the medication. T

When unit staffing includes UAP, which of the following is characteristic?

Licensed personnel are accountable for the tasks delegated to the UAP.

When teaching self-care of genital tract infections, a nurse should instruct a female to:

Limit time spent in damp exercise clothes and limit exposure to bath salts or bubble bath.

Stress incontinence, nutrition therapy, need more teaching:

Limit total intake of fluids

If the family of a patient is highly emotional and critical of the nursing care, which of the following is appropriate?

Listen to concerns and provide reassurance.

Insulin: Lantus

Long acting onset: 1 hour peak: none duration: 24+ hour

Which order would the nurse question when caring for a postop patient receiving epidural morphine infusion?

Lovenox 40mg SC BID Molecular weight heparins have been linked to spinal hematoma in clients with epidurals.

During initial assessment, a nurse should record which of the following manifestations of respiratory distress?

Manifestations of respiratory distress include tachypnea, grunting and panting on respiration, central cyanosis, use of accessory muscles, and flaring nares.

38 year old female, boyfriend wants to have sex after smoking marijuana, nurse says:

Marijuana enhances sexual functioning

Which of the following degrees prepares a nurse for advanced practice as a clinical specialist or nurse practitioner? LPN,ADN,BSN,or Masters

Master's

An 80 year old patient who is recovering from a hip fracture with surgical nailing is becoming increasingly confused and unable to participate in care, and has experienced several periods of urinary incontinence. Which orders might the nurse suspect of contributing to the patient's sypmptoms?

Meperidine 25mg Meperidine causes confusion and delerium in the older adult and should be used caustiously in patients with altered renal function.

Which of the following methods reduce food intake?

Methods of modifying circumstances include adding a breakfast mealtime, eating nothing after evening meal, setting the utensil down between bites, and eating only at the table.

Two days after undergoing surgery, a patient refuses to get out of bed. What information can the nurse provide that may increase compliance with the treatment plan?

Movement can cause breakthrough pain. We can give you medication to control the pain and help you to increase your activity

Which of the following best describes therapeutic touch?

Movement of the hands over the body without physical touch.

The causes of obesity are largely:

Multifactorial

If a patient with arthritis develops iron-deficiency anemia, a nurse should ask about the patient's use of:

NSAIDs

Pre-Op patients should be screened for possible critical allergies?

Pre-Op pts should be screened for critical allergies: Latex, Iodine, and allergies to anesthesia that may result in Malignant Hyperthermia.

Role Ambiguity: Don't know if I'm ready to be a mom:

Negative self-esteem

A 28 year old quadriplegic complains of burning pain in his lower legs. What type of pain should the nurse suspect?

Neuropathic Pain Nociceptive/neuropathic pain is due to damage to nerve cells or changes in the processing of pain

Which was teh first state to identify diagnosing as part of the legal domain of professional nursing?

New York

Do you aspirate with a heparin injection?

No

Which group is responsible for the promotion and organization of activities to continue the development, classification, and scientific testing of nursing diagnoses?

North American Nursing Diagnosis Association (NANDA)

A. Hold the feeding

Nurse Oliver checks for residual before administering a bolus tube feeding to a client with a nasogastric tube and obtains a residual amount of 150 mL. What is appropriate action for the nurse to take? A. Hold the feeding B. Reinstall the amount of continue with administering the feeding C. Elevate the client's head at least 45 degrees and administer the feeding D. Discard the residual amount and proceed with administering the feeding

Altered health maintenance is an example of which of the following types of problems?

Nursing Problem, because it describes a problem that can be treated by nurses within the scope of independent nursing practice.

317 Nurse collecting data about child w/diagnosis of seizures. Nurse checks for causes of the seizure activity by:

Obtaining a history regarding factors that may precipitate seiure activity. R: Fever and infections increase the body's metabilic rate; this can cause seizure activity among children who are less than 5 yrs. old. Dehydrations and electrolyte imbalance can also contribute to the occurrence of a seizure. Falls can cause head injuries, which would increase intracranial pressure or cerebral edema. Some medications could cause seizures.

When assessing a patient's respiratory status, which of the following non-respiratory data are most important for the nurse to obtain?

Occupation and hobbies

Scan amount of urine 100-500 mL a day:

Oliguria > 500 mL, low urine output

When helping Mr. Price turn in bed, the nurse notices that his heels are reddened and plans to place him on precautions for skin breakdown. This is an example of what type of planning?

Ongoing planning, which is problem oriented and has its purpose keeping the plan up to date as new actual or potential problems are Identified.

C. Loss of bone density

Osteoporosis can be defined as: A. Loss of bone matrix B. New, weaker bone growth C. Loss of bone density D. Increased phagocytic activity

PQRSTU pneumonic for pain assessment

P: precipitating factors Q: quality of pain R: region or radiation of pain S: severity of the pain T: timing of the pain U: how the pain affects yoU

When is PN usually prescribed for clients?

PN = parenteral nutrition malnourished, protein or metabolic deficiencies from underlying disease, cannont ingest food

Drug administration would be considered palliative in which of the following situations?

Pain management for a patient with terminal cancer

323 Nurse assisiting w/data collection from infant diagnosed w/hydrocephalus. If infant's level of consciousness diminishes, a priority nursing intervention is:

Palpating the anterior fontanel R: A full or bulging anterior fontanel indicates an increase in CSF collection in the cerebral ventricle.

What information about pain must the nurse understand when designing a plan of care to manage pain?

Past experience with pain effects the way current pain is perceived Past experience affects the way current pain is perceived, the impact of pain experiences is not predictable, anxiety influences an individuals response to pain, and no matter what the experience is, one never becomes accustomed to pain.

In preparing the preoperative teaching plan for a patient who is to undergo a total laryngectomy, a nurse should give highest priority to the:

Patient's not being able to speak normally again.

Many complementary and alternative therapies share which of the following concepts?

Patients are capable of decision making and should be a part of the health care team.

Mild anxiety:

Perception and learning is enhanced

Rectal surgery, aptient urinates 50 mL of urine, every 30-60 kin, nurse should:

Perform a bladder scan

325 Child diagnosed w/Reye's syndrome. Nurse understands major symptoms assoc. w/Reye's is:

Pesistent vomiting. R: Persistent vomiting is a major symptom that is associated w/ICP. Reye's syndrome is related to a history of viral infections, and hypoglycemia is a symptom of this disease.

A nurse preparing to assess a patient for the presence of clubbing should instruct the patient to:

Place the nails of the ring fingers together.

A school nurse is teaching a class of junior-high students about the effects of smoking. This edu. program will meet which of the aims of nursing?

Preventing Illness

Indwelling catheter securing:

Preventing trauma from external structures

Why her husband has condom catheter verses catheter inside:

Prevents Infection

If the edges of a patient's appendectomy incision are approximated, and no drainage is noted, a nurse should document that the inspection appears to be healing by:

Primary Intention

A client is admitted to the labor and delivery unit where she is assigned a registered nurse who will manage her care for the duration of her hospitalization. What type of nursing care delivery model is represented in this nursing unit?

Primary care rationale: This is an example of primary care nursing, in which the RN is the principal or primary person to manage and coordinate the client's care around the clock. Patient-focused care is an interdisciplinary approach to client care. Functional care is the provision of care by nursing and nursing ancillary staff rather than by an all-nursing staff as seen in primary care nursing. Case management is a model of care delivery in which an RN case manager coordinates and collaborates with other health services when a client has complicated health care needs, but the case manager may not be involved in the daily care activities of the client.

A system in which a nurse plans and directs the care of clients over a 24-hour period is known as:

Primary nursing rationale: In primary nursing the fragmentation between shifts and nurses is eliminated because one nurse is accountable for planning the care of the client around the clock.

Promotes normal defecation:

Privacy

Contributes to constipation:

Prolonged use of laxatives

When administering an IM injection in the gluteal muscle, the RN places the pt in which position as the best one to relax the muscle?

Prone with leg toe in position

The nurse understands that the computer-based patient record (CPR) was developed to:

Provide an instrument with which to obtain clinical information, transcribe data, and track the care of the client in a variety of settings rationale: The CPR basically refers to the same thing as the electronic medical record (EMR), the electronic patient record (EPR), and the electronic health record (EHR). It is important to note that the CPR is not the clinical information system (CIS), but instead is a complex computer system that captures, records, processes, and communicates client data. The CPR includes all information about an individual's lifetime health status and health care and is maintained electronically. The nurse needs basic computer skills to work with the CPR.

Which of the following is the most appropriate nursing intervention for a patient with total urinary incontinence?

Provide normal fluid intake and establish a toilet schedule.

The nurse understands that computer monitoring on a telemetry unit:

Provides streamlining of data that allows nurses to respond to client changes quickly rationale: The continuous flow of data from a computer allows nurses to respond to client changes very quickly. Timely nursing responses and actions are crucial in leading to positive outcomes when caring for critically ill clients. Computer monitoring is not a substitute for client observation. The number of nurses required to care for clients is not reduced because of computer monitoring, nor does it improve documentation, but rather provides a means to document data.

326 Nurse assists w/preparing a nursing care plan for a child w/has Reye's syndrome. Which of the following is priority nursing intervention?

Providing a quiet atmosphere w/dimmed lights. R: The major elements of care are to maintain effective cerebral perfusion and to control intracranial pressure. Decreasing stimuli in the environment w/decrease the stress on the cerebral tissue and the neuron responses. Cerebral edema is a progressive part of this disease process. The child s/be in a head-elevated position to decrease the progression of the cerebral edema and to promote the drainage of cerebrospinal fluid.

A nurse would best assist a patient to use strategies for primary prevention of STDs by:

Providing risk-reduction counseling

While inspecting a 69-year-old patient, if a nurse notes that the left upper eyelid droops, covering more of the iris than the right eyelid, this should be recorded as:

Ptosis on the left

What does a hypertonic solution do?

Pulls fluid into the intravascular space and kidneys will then excrete the fluid.

Assessment: Sudden SOB. Sudden angina pain. Cyanosis. Tachycardia. Drop in BP.

Pulmonary Embolism

Purulent:

Pus

88 year old, distended bladder, hyperplasia, agitated, confusion, intervention used 1st:

Put in catheter

Who are the largest group of healthcare providers in the US?

RN's

Immunizations 4 mo.

RV [Rotavirus/ Rotarix] (2 of 2) DTaP [Diptheria, Tetanus, Pertussis] (2 of 4) Hib [Haemophilus influenzae type b] (2 of 3) PCV [Pneumococcal] (2 of 4) IPV [Inactivated Polio Virus] (2 of 4)

Insulin: Lispro

Rapid release onset: 15 min peak: 60-90 min duration: 3-4 hr

Healing Touch:

Realign energy flow

A nurse teaching a patient with narcolepsy should include information regarding:

Recommendation to take regular naps.

Using light pressure with the index and middle fingers, the nurse cannot palpate any of the patient's superficial lymph nodes. The nurse:

Record this finding as normal.

Sanguineous:

Red

What is REEDA?

Redness Erythema Ecchymosis Drainage Approximation of the wound edges

A clinical judgment that an individual, family, or community is more vulnerable to develop the problem than others in the same or similar situation is what type of nursing diagnosis?

Risk

A nurse should warm a patient with type 1 DM who has developed proteinuria that this finding is significant because:

Renal failure will most likely develop in 5 to 10 years.

If a 77-year-old patient states, "My religion does not permit me to bathe today." The most appropriate nursing action is to:

Request a clarification of the patient's beliefs and arrange a plan for bathing.

Common causes/ risk factors: Atelectasis

Respiratory complication that may be the result of a blocked airway, diminished surfactant, or mucus plug. Recent general anesthesia, shallow breathing, respiratory muscle weakness and immobility are common risk factors.

Definition: Atelectasis

Respiratory complication when the alveoli within the lung becomes deflated, resulting in a complete or partial collapse of a lung.

Which of the following are long-term complications of DM?

Retinopathy, nephropathy, and neuropathy.

When providing perineal care for an uncircumcised male patient, the nurse should:

Retract the foreskin, then clean the glans, being sure to replace it at the end of the procedure.

Liver biopsy:

Right side position post procedure to prevent patient from bleeding.

Nuring Diagnosis appropriate for client with indwelling catheter, bag is on the floor:

Risk for Infection

Omega-3 fatty acids interfere with the blood clotting process, thereby lowering the:

Risk of heart disease

phenochromocytoma

SEVERE HYPERTENSION ...

A client has just returned to his room after undergoing exploratory abdominal surgery. The nurse notes watery red drainage on his dressing. The nurse will describe the drainage as:

Sanguineous

Which of the following should be emphasized when teaching a patient intermittent self-catheterization?

Self-catheterization does not require gloves, only handwashing; catheterization should be performed at least every 3 to 4 hours; and 300 to 400 mL of urine should be removed with each procedure. The catheter is washed with warm water and a mild soap and stored in a plastic sandwich bag.

Thyroidectomy:

Semi Fowler and avoid hyperflexion and hyperextension of the neck

An infection of a central venus catheter can lead to?

Septicemia

Sexual history on admission:

Sexual history varies on case by case basis

Insulin: Regular

Short acting onset: 30 min peak: 2-3 hr duration: 4-8 hr

Which of the following are indications for the implementation of dialysis?

Significant volume overload, uncontrolled hyperkalemia and acidosis, increasing BUN and creatine concentrations, altered CNS function, and pericarditits are indications for dialysis.

Which of the following is of greatest concern when performing a digital rectal examination to determine the presence of fecal impaction?

Slowing heart rate

A pt is to recieve a unit of packed red blood cells. What IV solution should the nurse hang with the blood product?

Sodium chloride 0.9% (normal saline)

UTIs in women are eight times more common than in men because women:

Spermicides decrease vaginal pH; The urethra is shorter than it is in the man; Perfumed hygiene products may enter the urethra; The urethra lies closer to the anus than in men.

A home health nurse is working with a patient who quit his job after injury- life has no meaning and lonely- everyone has left him- what is this a sign of:

Spiritual Distress

While caring for a patient with respiratory disease, a nurse observes that the oxygen saturation drops from 94% to 85% when the patient ambulates. The nurse should determine that:

Supplemental oxygen should be used when the patient exercises.

What type of authority regulates the practice of nursing?

State nurse practice acts

If a patient is having stools that are foul smelling and that float on water, a nurse should document that the patient is having:

Steatorrhea

To evaluate thoroughly an older patient's memory, it is helpful to use reminiscence strategies because they:

Stimulate memory chain through associations.

What should a RN do if the pt has developed a blood transfusion reaction?

Stop the transfusion, start an infusion of normal saline using new tubing connected to the hub of the IV insertion site.

Which of the following strategies may help prevent relapse in a patient who uses psychoactive drugs?

Strategies to help prevent relapse include building the patient's coping skills, identifying indications of drug craving and elements that will interfere with compliance in a self-help group and helping the patient design healthful exercise and nutrition.

A patient with no history of heart disease is seen in the clinic for periodic episodes of tachycardia with regular rhythm. When obtaining the patient's history, the nurse should question the patient regarding the increase of:

Stress

What is the amount of blood pumped with each heartbeat?

Stroke Volume

318 Child has a basilar skull fracture. Which of following physician orders s/nurse question?

Suction via nasotracheal route as needed. R: Nasotracheal suctioning conraindicated in child w/basilar skull fracture. Because of nature of injury, suction catheter may be introduced into brain. Child may require urinary catheter for accurate monitoring of I & O. Fluids restricted to prevent fluid overload. IV line maintained to administer fluids or meds if necessary.

A patient is experiencing a HR of 200 bpm. If the ECG pattern demonstrates absent P waves and normal and consistent QRS complexes and duration, a nurse should interpret these findings as indictive of:

Supra-ventricular tachycardia

322 Which assessment finding w/indicate possibility of sexual abuse of child?

Swelling of genitals. R: The most likely findings among children who have been sexually abused include difficulty walking or sitting; torn, stained, or bloody underclothing; pain,swelling, or itching of genitals; and bruises, bleeding, or lacerations in genital or anal area.

Which of the following clinical manifestations indicates activity intolerance in a patient with heart failure who is undergoing a progressive increases in activity?

Systolic blood pressure change from 136 to 96 mm Hg

tumor

T (tumor) 0-4 signifies tumors increasing size. N (regional lymph nodes) 0-3 signifies increasing involvement of lymph nodes. M (metastasis) 0 signifies no metastasis and 1 signifies distal metastasis.

Which of the following nursing diagnoses are correctly written as three-part nursing diagnoses?

TWO: -Imbalanced Nutrition: Less Than Body Requirements related to inadequate caloric intake while striving to excel in gymnastics as evidenced by 20-pound weight loss since beginning the gymnastic program, and greatly less than ideal body weight when compared to standard weight height charts. - Spiritual Distress related to inability to accept diagnosis of terminal illness as evidenced by multiple comments such as "How could God do this to me?", I don't deserve this", etc.

To become credentialed as a Certified Professional in Healthcare Quality, you must:

Take an exam rationale: To become a CPHQ, you must take and pass an examination. Only about 75% of those who test actually become certified.

Bactrum BID, E.Coli, UTI:

Take antibiotic for full amount time, finish them all

Antiviral Medications

Taking antiretroviral medications such as indinavir on a rigid time schedule is essential for effective treatment of HIV infection and to avoid development of drug resistant-strains of the virus.

62 year old female- laugh/cough/ leakage of urine, intervention of care plan:

Teach Kegal Exercises

Which of the following is most likely to be effective in meeting a patient's teaching/learning needs preoperatively?

Teaching the patient and family

62 year old man, dosen't respond to sexual stimulation, nurse response:

Tell me more

B. Pain in the femur especially on weight bearing

The LPN/LVN is caring for cliet with osteosarcoma. Which symptoms would the nurse expect? A. Swelling in the area of the clavicle B. Pain in the femur especially on weight bearing C. Difficulty using either hand D. A stiff back

Dehydration may be a problem in older adults because:

The ability of their kidneys to concentrate urine decreases.

How can energy-based therapies be described or defined?

The balance and return of a person's energy field to its optimal flow to facilitate healing.

In the surgical setting, where is the center of the sterile field?

The center of the sterile field is the site of the surgical incision.

C. Acute pharyngitis

The client complains of a sore throat and has been told it is due to beta-hemolytic streptococcal infection. The nurse realizes this condition this condition is called what? A. Acute rhinitis B. Acute sinusitis C. Acute pharyngitis D. Acute rhinorrhea

Which of the following are clinical manifestations of tension pneumothorax?

The indicators of tension pneumothorax are asymmetrical chest wall movement, severe hypotension, subcutaneous emphysema in the neck and upper chest, and progressive cyanosis.

B. Lispro

The insulin that has the most rapid onset of action would be: A. Lente B. Lispro C. Ultralente D. Humulin N

Mr. Price tells the nurse he fears becoming "hooked on drugs" and consequently waits until his pain becomes unbearable before requesting is PRN analgesic. The nurse plans to be more attentive to Mr. Price and to assess his needs for pain management more closely. Which of the following consequences of informal planning ought to be a major concern for the nurse?

The lack of a coordinated plan known by everyone will result in uneven pain management.

What are some common RN interventions to prevent/ minimize paralytic ileus?

The nurse can prevent/minimize paralytic ileus after surgery with pt positioning and early ambulation. Evidence of bowel function returning to normal includes auscultation of bowel sounds and passing of flatus and stool.

C. blurred vision

The nurse is admitting a patient demonstrating hyperglycemic hyperosmolar non-ketotic syndrome. Which of the following will the nurse assess is in this patient? A. Abdominal pain B. Nausea and vomiting C. blurred vision D. Deep and rapid respirations

D. Complete activities that can be stopped immediately if needed

The nurse is caring for a client with rheumatoid arthitis. Which of the following teaching points should the nurse include in discharge teaching for this client? A. instruct the client that pain during exercise means they are working out properly B. Exercise the weakest joints to avoid misuse C. Search for activities that require a firm grip D. Complete activities that can be stopped immediately if needed

A. Cake, candies, cookies, and regular soft drinks should be avoided. B. Gestational diabetes increases the risk that the mother will develop diabetes later in life. C. Gestational diabetes usually resolves after the baby is born D. Insulin injections may be necessary

The nurse is educating a pregnant client who has gestational diabetes. Which of the following statements should the nurse make to the client? Select all that apply. A. Cake, candies, cookies, and regular soft drinks should be avoided. B. Gestational diabetes increases the risk that the mother will develop diabetes later in life. C. Gestational diabetes usually resolves after the baby is born D. Insulin injections may be necessary E. The baby will likely be born with diabetes F. The mother should strive to gain no more weight during the pregnancy

A. Cakes, candies, cookies, and regular soft drinks should be avoided. B. Gestational diabetes increases the risk that the mother will develop diabetes later in life. C. Gestational diabetes usually resolves after the baby is born. D. Insulin injections may be necessary.

The nurse is educating a pregnant client who has gestational diabetes. Which of the following statements should the nurse make to the client? Select all that apply. A. Cakes, candies, cookies, and regular soft drinks should be avoided. B. Gestational diabetes increases the risk that the mother will develop diabetes later in life. C. Gestational diabetes usually resolves after the baby is born. D. Insulin injections may be necessary. E. The baby will likely be born with diabetes F. The mother should strive to gain no more weight during the pregnancy.

B. Renal

The nurse is trying to determine the client's current stage of pending shock. Which of the following shock stages is this client most likely developing? A. Pulmonary B. Renal C. Endocrine D. Cardiovascular

A. Renal

The nurse is trying to determine the client's current stage of shock. Which of the following body systems will begin the renin-angiotensin-aldosterone cycle? A. Renal B. Pulmonary C. Endocrine D. Cardiovascular

A nurse is planning caring for patients in a long-term care facility. The nurse knows that aging patients are at additional risk for which diseases?

The nurse knows that aging adults are at additional risk for anorexia, depression, coronary artery disease, and type 2 diabetes.

B. Scaley, inflamed rash on shoulders, neck, and face

The nurse monitors a patient to have Systemic Lupus Erythematosus. Which of the following symptoms is characteristic of this diagnosis? A. Increased T-cell count B. Scaley, inflamed rash on shoulders, neck, and face C. Swelling of the extremities D. Decreased erythrocyte sedimentation rate (ESR)

D. respiratory rate decreases

The nurse recognizes cardiogenic shock as manifested by all of the classic signs but: A. decreased urine output B. rapid pulse becomes weaker C. hypotension D. respiratory rate decreases

A hospice nurse is caring for the family of a patient who has just died. Which interventions should the nurse implement in caring for the family?

The nurse should allow the family time alone with the body to allow for grieving and closure and use periods of silence during the conversation. The nurse does not have to talk but can emotionally support the family simply by being present.

Which factor regarding older adults and medication is important for the nurse to understand?

The older adult is more likely to experience drug interactions than the general public

What is a realistic outcome for the patient who is terminally ill with bone cancer and is experiencing uncontrolled pain?

The patient experiences improved quality of life

From which of the following are nursing outcomes derived?

The problem statement of the nursing diagnosis

If a nurse is assessing a patient whose recent blood gas determination indicated a pH of 7.32 and respiration's are measured at 32 breath/min, which of the following is the most appropriate nursing assessment?

The rapid breathing is an attempt to compensate for the low pH.

The continuous quality improvement (CQI) committee has performed a retrospective chart audit to investigate whether outcomes recorded in each nursing care plan are client-centered and written in behavioral terms. The expected standard is 98% compliance. The sample size was 200. Results showed that 180 charts met the standard. What assessment can be made?

The standard was not met. An action plan should be developed. rationale: A threshold, or cutoff point, is determined for each indicator. This example represents a 90% compliance rate, but the threshold was set at 98%.

The cane is kept on the weak or strong side?

The strong side

Which statement is true about client classification systems?

The systems provide historical data of the usage of nursing time, which is helpful when developing the department budget. rationale: A client classification or acuity system is used in many acute care hospitals to estimate the intensity of nursing care required to meet client needs. It is not a formula for unit staffing.

You are caring for a 72 year old patient with advanced cancer who complains of increased pain and tactile sensitivity over the last several weeks. Which non pharmacological alternative could be added to her plan of care to enhance her comfort?

Therapeutic Touch Therapeutic touch is thought to realign aberrant energy fields through passing hands over the energy fields without actually touching the body and promoting comfort.

Which statement is true about client classification systems?

They provide historical data of the usage of nursing time, which is helpful when developing the department budget. rationale: A client classification or acuity system is used in many acute care hospitals to estimate the intensity of nursing care required to meet client needs.

----

Three months after surgery and chemotherapy the patient is likely to be feeling the after-effects, which often include anemia because of bone-marrow suppression. There is no evidence that the patient is immunosuppressed, and fatigue is not a typical symptom of immunosuppression. The information given does not indicate that depression or dehydration is a cause of her symptoms.

Assessment: Vein inflammation. Aching or cramping pain. Vein feels hard and cord-like and is tender to touch. Elevated temperature.

Thrombophlebitis

In palliative care, a patient's quality of life is improved because:

Through the work of an interdisciplinary team, palliative care provides physical, psychological, and spiritual support, plus management of clinical manifestations.

330 Nurse developing a plan of care for a child who is at risk for seizures. Select all of interventions that aply if child has a seizure.

Time the seizure Stay w/the child. Move furniture away from the child. R: During seizure, child placed on his or her side in a lateral position. This type of positioning will prevent aspiration, because saliva will drain out of corner of child's mouth. Child is not restrained, because this could cause injury. Nurse w/loosen clothing around the child;s neck and ensure a patent airway. Nothing is placed into child's mouth during a seizure because this action may cause injury to the child's mouth, gums, or teeth. Nurse w/stay w/child to reduce risk of injury and allow for observation and timing of seizure.

When assessing a patient with arterial insufficiency, the nurse should expect:

Tissue atrophy

To which of the following do standard precautions apply?

To all patients receiving care.

What is the importance of using classification systems for nursing nomenclature to describe nursing practice?

To assist the quality control team to survey and gather data that will reflect the acuity classification of clients rationale: Nursing nomenclatures offer a recognized systematic classification and consistent method of describing nursing practice. Without a common language, data cannot be aggregated into a useful language.

What is the purpose of the ANA's Scope and Standards of Practice?

To define the activities that are special and unique to nursing

Which of the following is an appropriate task for a nurse to delegate to a nursing assistant?

Toileting a patient on bladder-training program

Characteristic normal urine:

Transparent

Can you use essential oils for Asthma: True or False

True

Series of "small successes is positive way to help clients True or False

True

Weight loss and exercise are main components of the treatment for:

Type 2 DM

An adult patient with a body mass index (BMI) of 18 kg/m2 would be considered:

Underweight

An odor of ammonia on a patient's breath may signify:

Uremia

Homosexual, worried about him, nurse should consider the factor of saying:

What constitutes, varies among religion

Pyridium:

Urine will turn florescent orange

The lesions exhibited by a patient who has herpes simplex should be documented as:

Vesicles

A patient with diverticula of the esophagus should avoid:

Vigorous exercise after eating.

Virul Lode Testing- Effectivness of Antivirul Medications

Viral load testing measures the amount of HIV genetic material in the blood, so a decrease in viral load indicates that the ART is effective.

The nurse suspects the possibility of sepsis in the burn patient based on changes in:

Vital Signs

Vitamin deficiency in Pernicious anemia

Vitamin B12 deficiency Schilling test

What does Questran deplete?

Vitamins A, D, E, K so get these by drinking milk and eating green leafies

Dehydration may occur secondary to:

Vomiting and diarrhea

What historic event in the 20th century led to an increased emphasis on nursing and broadened the role of nurses?

WW2

Hyperkalemia signs.

Weakness, paresthesias, abdominal cramps, diarrhea,& dysrhythmias.

A nurse is working on a respiratory care unit where many of the patients are affected by asthma. Which of the following actions by the nurse would most likely increase respiratory difficulty for the patients?

Wearing perfume to work

o2 rate greater than 4

When an oxygen flow rate is greater than 4 L/min, the mucous membranes can be dried out. The best treatment is to add humidification to the oxygen delivery system.

The terms diagnose and diagnosis have legal implications. They imply that there is a specific problem that requires management by a qualified expert. Which of the following statements is false?

When nurses diagnose a medical problem, they are just as accountable as physicians for detecting, identifying, and managing the signs and symptoms of disease.

Trach care precautions

When performing tracheostomy care, a sterile field is set up and sterile technique is used

C. Having a consistent HDL level above 40 mg/dl

Which of the following is not a risk factor for the development of diabetes mellitus? A. Age over 45 years B. Overweight wight a waist/hip ratio>1 C. Having a consistent HDL level above 40 mg/dl D. Maintaining a sedentary level

B. Bone remodeling

Which of the following is the major mechanism for maintaining calcium balance in the body? A. Appropriate body alignment B. Bone remodeling C. Active and Passive exercises D. Sarcopenia

B. Bone remodeling

Which of the following is the major mechanism for maintaining calcium balance in the body? A. Appropriate body alignment B. Bone remodeling C. Active and passive exercises D. Sarcopenia

B. To prevent contractures

Which of the following is the primary reason for splinting the hands and wrists of client with rheumatoid arthritis? A. to improve the strength of the hands and wrists. B. To prevent contractures C. To relieve muscle spasm D. To relieve pain

D. A client who has just returned from the operating room (OR)

Which of the following patients with musculoskeletal problems should the nurse observe first? A. A client who needs dressing changes done, because the dressing is wet. B. A patient who tells the nurse that he needs to be suctioned. C. A client with a fracture of the ankle requesting pain medication. D. A client who has just returned from the operating room (OR)

B. African American Woman

Which of the following persons would most likely be diagnosed with diabetes mellitus? A 44-year-old: A. Caucasian Woman B. African American Woman C. Asian Woman D. Hispanic Male

D. Extracapsular fractures cause acute blood loss from the vascular cancellous bone surfaces, but rarely cause avascular necrosis.

Which of the following statements is accurate about hip fractures? A. Intracellular fractures include interochanteric and subtrochateric involvement. B. Intracapsular fractures cause acute blood loss from vascular cancellous bone surfaces, causing avascular necrosis. C. Extracapsular fractures cause blood loss from the vascular cancellous bone surfaces, resulting in avascular necrosis. D. Extracapsular fractures cause acute blood loss from the vascular cancellous bone surfaces, but rarely cause avascular necrosis.

A patient with colon cancer is being managed with OxyContin 30mg PO BID and Oxycodone 5mg PO q4h PRN for breakthrough pain. The patiens wife voices her concern that the patient is becoming addicted to the medication and questions whether milder nonnarcotic medications could be used. What is the most appropriate response by the nurse?

With the diagnosis of cancer, there is a need to use regular and strong mediaction for pain control to provide a better quality of life Persistent pain can be managed using long acting medications and narcotics when the condition warrants their use. Addiction is not an issue for the patien with chronic cancer pain. Amount and types of meds are adjusted according to patient status

To reduce the risk of aspiration, patients who are receiving enteral tube feedings should be positioned:

With the head of the bed elevated 30 to 45 degrees.

women preventing UTIs

Women should avoid irritating substances such as bubble bath, nylon underwear, and scented toilet tissue to prevent urinary tract infections.

D. are likely to experience it with subsequent pregnancies.

Women who develop gestational diabetes: A. will probably remain diabetic following delivery of their babies. B. give birth to babies with low birth weights who are hypoglycemic. C. are primarily treated with any of the available oral agents. D. are likely to experience it with subsequent pregnancies.

Assessment: fever and chills. Warm, tender, painful, inflamed incision site. Elevated WBC count. Edematous skin at the incision and tight skin sutures.

Wound infection

People in crisis - can they work through crisis if someone works with them

Yes

A. Cardiac output

You are caring for a patient in shock. Which parameter would you want to monitor to determine if treatment is effective? A. Cardiac output B. Respiratory Rate C. Peripheral pulse rate D. Central venous pressure

How do you administer parenteral iron?

Z track in the gluteal muscle

What type of room is necessary for a client with an internal radiation implant?

a private room with a private bath

Your patient is diagnosed with congestive heart failure. The pulse is thready, weak, and easily obliterated by light pressure. What is the quality of this pulse? a) 1+ b) 2+ c) 3+ d) 4+

a) 1+

A college student was referred to the campus health service because of difficulty staying awake in class. What should be included in the nurse's assessment? Select all that apply: a) Amount of sleep he usually obtains during the week and on weekends b) How much alcohol he usually consumes c) Onset and duration of symptoms d) Whether or not his classes are boring e) What medications including herbal remedies, he is taking

a) Amount of sleep he usually obtains during the week and on weekends c) Onset and duration of symptoms e) What medications, including herbal remedies, he is taking

The nurse is performing a dressing change for a client and notices that there is a new area of skin breakdown near the site of the dressing. On closer examination, it appears to be caused from the tape used to secure the dressing. This would be an example of which phase of the nursing process: a) Assessment b) Diagnoses c) Implementation d) Evaluation

a) Assessment

Coping with stress differs from adaption to stress in that: a) coping is a more immediate, short term response to stress b) Coping is a later response to stress

a) Coping is a more immediate, short term response to stress

To reduce shearing force for a bedridden client. It is most important for the nurse to: A) Put bed in high Fowler's position b) Pull the client up in at least once an hour

a) Put bed in high Fowler's position

9. The (Ditropan) 5 mg by mouth three times a day after discharge. Which information would you be sure to teach this patient prior to discharge?

a. "Drinkfluidsorusehardcandywhenyouexperienceadrymouth." b. "Be sure to notify your physician if you experience a heart rate of less than 60 per minute." c. "If necessary, your physician can increase your dose up to 40 mg per day." d. "Youshouldtakethismedicationwithmealstoavoidstomachulcers." ANSWER A - Oxybutynin chloride (Ditropan) is an anticholinergic agent, which often causes an extremely dry mouth. The maximum dose is 20 mg per day. This drug can cause tachycardia as a side effect, but does not cause bradycardia. Oxybutynin chloride should be taken between meals because food interferes with absorption of the drug.

20.The nursing assistant reports to you that the patient with ARF has had a urine output of 350 mL for the past 24 hours after receiving furosemide 40 mg IV push. The nursing assistant asks you how this can happen. What is your best response?

a. "During the oliguric phase of acute renal failure, patients often do not respond well to either fluid challenges or diuretics." b. 'There must be some sort of error. Someone must have failed to record the urine output." c. "The patient with acute renal failure retains sodium and water, counteracting the action of the furosemide." d. "The gradual accumulation of nitrogenous waste products results in the retention of water and sodium." During the oliguric phase of acute renal failure, patients' urine output is greatly reduced. Fluid boluses and diuretics do not work well. This phase usually lasts from 8 to 15 days.

19.You are reviewing your client's understanding of the post-operative stapedectomy instructions that you gave several days ago. Which comment concerns you the most?

a. "I'mgoingtotakeswimminglessonsinacoupleofmonths." b. "Ihavetotakealongoverseasflightinseveralweeks." c. "I can't wait to get back to my regular weightlifting class." d. "Ihavebeencoughingalotwithmymouthopen." 19.ANSWER C - Heavy lifting should be strictly avoided after stapedectomy for at least 3 weeks. Water in the ear and air travel should be avoided for at least 1 week. Coughing and sneezing should be performed with the mouth open to prevent increased pressure.

23.The patient with a diagnosis of ARF had a urine output of 1560 mL for the past 8 hours. The LPN/LVN who is caring for this patient under your supervision asks how a patient with renal failure can have such a large urine output. What is your best response?

a. "The patient's renal failure was due to hypovolemia and we have administered IV fluids to correct the problem." b. "Acute renal failure patients go through a diuretic phase when their kidneys begin to recover and may put out up to 10 L of urine per day." c. "With that much urine output, there must have been a mistake made when the patient was diagnosed." d. "An increase in urine output like this is an indicator that the patient is entering the recovery phase of acute renal failure." Patients with acute renal failure usually go though a diuretic phase. The diuresis can result in an output of up to 10 L/day of dilute urine. During this phase, it is important to monitor for electrolyte and fluid imbalances.

17.The patient has been diagnosed with renal cell carcinoma (adenocarcinoma of the kidney). You are orienting a new nurse to the unit, who asks why this patient is not receiving chemotherapy. What is your best response?

a. "The prognosis for this form of cancer is very poor and we will be providing only comfort measures." b. "Chemotherapy has been shown to have only limited effectiveness against this type of cancer." c. "Research has shown that the most effective means of treating this form of cancer is with radiation therapy." d. "Radiofrequency ablation is a minimally invasive procedure that is the way to treat renal cell carcinoma." Chemotherapy has limited effectiveness against renal cell carcinoma.

6. A 67-year-old client with incomplete bladder emptying caused by BPH has a new prescription for tamsulosin (Flomax). Which statement about tamsulosin is most important to include when teaching this client?

a. "Thismedicationwillimproveyoursymptomsbyshrinkingtheprostate." b. "The force of your urinary stream will probably increase." c. "Your blood pressure will decrease as a result of taking this medication." d. "Youshouldavoidmakingsuddenchangesinposition." Answer D - Because tamsulosin blocks alpha receptors in the peripheral arterial system, the most significant side effects are orthostatic hypotension and dizziness. To avoid falls, it is important that the client change position slowly. The other information is also accurate and may be included in client teaching, but is not as important as decreasing the risk for falls

6. You are supervising a new graduate RN who is orienting to the unit. The new RN asks why the patient with uncomplicated cystitis is being discharged with orders for ciprofloxacin 250 mg twice a day for only 3 days. What is your best response?

a. "Weshouldcheckwiththephysicianasthepatientshouldtakethisdrug for 10 to 14 days." b. "A 3-day course of ciprofloxacin is not the appropriate treatment for a patient with uncomplicated cystitis." c. "Research has shown that with a 3-day course of ciprofloxacin, there is increased patient adherence to the plan of care." ANSWER C - For uncomplicated cystitis, a 3-day course of antibiotics is an effective treatment, and research has shown that patients are more complaint with shorter antibiotic courses. Seven-day courses of antibiotics are appropriate for complicated cystitis, and 10-14 day courses are prescribed for uncomplicated pyelonephritis.

12.You are supervising an orienting nurse who is discharging a patient admitted with kidney stones post lithotripsy. Which statement by the nurse requires that you intervene?

a. "You should finish all of your antibiotics to make sure that you don't get a urinary tract infection." b. "Remember to drink at least 3 liters of fluids every day to prevent another stone from forming." c. "Report any signs of bruising to your physician immediately as this indicates bleeding." d. "You can return to work in 2 days to 6 weeks, depending on what your physician prescribes." 12.ANSWER C - Bruising is to be expected after lithotripsy. It may be quite extensive and take several weeks to resolve. All of the other statements are accurate for the patient after lithotripsy.

The nurse assisting in the admission of a client with diabetic ketoacidosis will anticipate the physician ordering which of the following types of intravenous solution if the client cannot take fluids orally? a. 0.45% normal saline solution b. Lactated Ringer's solution c. 0.9 normal saline solution d. 5% dextrose in water (D5W)

a. 0.45% normal saline solution

10.After arriving for your shift in the emergency department (ED), you receive change-of-shift report about all of these clients. Which one do you need to assess first?

a. A 19-year-old client with scrotal swelling and severe pain that has not decreased with elevation of the scrotum b. A 25-year-old client who has a painless indurated lesion on the glans penis c. A 44-year-old client with an elevated temperature, chills, and back pain associated with recurrent prostatitis d. A77-year-oldclientwithabdominalpainandacutebladderdistention 10.Answer A - This client has symptoms of testicular torsion, an emergency which needs immediate assessment and intervention, since it can lead to testicular ischemia and necrosis within a few hours

21.Persons at risk are the greater target population for cancer screening programs. Which asymptomatic patient(s) needs extra encouragement to participate in cancer screening? (Choose all that apply).

a. A 19-year-old white-American female who is sexually inactive for a Pap smear b. A35-year-oldwhite-Americanfemaleforanannualmammogram c. A 45-year-old African-American male for an annual prostate-specific antigen d. A49year-oldAfrican-Americanmaleforanannualfecaloccultbloodtest 21.ANSWER A, C - After age 18, females should annual Pap smears, regardless of sexual activity. African-American males should begin prostate-specific antigen testing at age 45. Annual mammograms are recommended for women over the age of 40. Annual fecal occult blood testing is recommended starting at age 50.

15.You have just received change-of-shift report about your assigned clients. In what order will you assess these clients?

a. A 22-year-old client who has questions about how to care for the drains placed in her breast reconstruction incision b. An anxious 44-year-old client who is scheduled to be discharged today after having a total vaginal hysterectomy c. A 69-year-old client who is complaining of level 5 pain (0 - 10 scale) after having a perineal prostatectomy 2 days ago d. A usually oriented 78-year-old client who has new-onset confusion after having a bilateral orchiectomy the previous day _____, _____, _____, _____ 15.Answer D, C, B, A -

7. You are acting as preceptor for a new graduate RN during her second week of orientation. For which client(s) would you assign the nursing care to the new RN under your supervision? (Choose all that apply.)

a. A 38-year-old client with moderate persistent asthma awaiting discharge. b. A63-year-oldclientwithtracheostomyneedingtrachcareeveryshift c. A 56-year-old client with lung cancer just returned from left lower tracheostomy d. A49-year-oldclientnewdiagnosisofesophagealcancer

It is important to have the appropriate cuff size when taking the blood pressure. WHat error may result from a cuff that is too large or too small? a. an incorrect reading b. injury to the patient c. prolonged pressure on the arm d. loss of Korotkoff sounds

a. A blood pressure cuff that is not the right size may cuase an incorrect reading.

A patient has a blood pressure reading of 130/90 mm Hg when visiting a clinic. What would the nurse recomment to the patient? a. floow-up measurements of blood pressuer b. immediate treatement by a physician c. nothing, becuase the nurse considers this reading is due to anxiety d. a change in dietary intake

a. A single blood pressure reading that is mildly elevated is not significant, but the measurment should be taken again over time to determine if hypertension is a problem. The nurse would recommend a return visit to the clinic for a recheck.

4. You are doing a sterile dressing change for a client with infected deep partial-thickness burns of the chest and abdomen. List the steps of the care pan in the order each should be accomplished.

a. Applysilversulfadiazine(Silvadene)ointment. b. Obtainaerobicandanaerobicwoundcultures. c. Administer morphine sulfate 10 mg IV. d. Debridewoundofescharusinggauzesponges. _____, _____, _____, _____ ANSWER C, D, B, A -

3. Which client(s) would be best to assign to an experienced nurse in an ambulatory eye surgery center? (Choose all that apply.)

a. Aclientwhoneedspost-operativeinstructionsforcataractsurgery b. Aclientwhoneedsaneye-padandametalshieldapplied c. A client who needs home health referral for dressing changes and eye drops d. Aclientwhoneedsteachingaboutself-administrationofeyedrops ANSWERS A and C - Post-operative instructions and home health referrals should be done by an experienced nurse who can give specific details and specialized information for follow-up eye care. The principles of eye pad and shield application and of teaching the administration of eye drops are basic procedures that should be familiar to all nurses.

22.Which client(s) are appropriate to assign to the LPN/LVN, who will function under the supervision of the RN or team leader? (Choose all that apply.)

a. Aclientwhoneedspre-opteachingforuseofaPCApump b. A client with a leg cast who needs neurologic checks and PRN hydrocodone c. A client post-op toe amputation with diabetic neuropathic pain d. Aclientwithterminalcancerandseverepainwhoisrefusingmedication 22.ANSWER B, C - The clients with the cast and the toe amputation are stable clients and need ongoing assessment and pain management that are within the scope of practice for an LPN/LVN under the supervision of an RN. The RN should take responsibility for pre-operative teaching, and the terminal cancer needs a comprehensive assessment to determine the reason for refusal of medication.

11.The patient has urolithiasis and is passing the stones into the lower urinary tract. What is the priority nursing diagnosis for the patient at this time?

a. AcutePain b. RiskforInfection c. Risk for Injury d. FearofRecurrentStones 11.ANSWER A - When patients with urolithiasis pass stones, the stones can cause excruciating pain for up to 24-36 hours

11.A 79-year-old client who has just returned to the surgical unit following a TURP complains of acute abdominal pain caused by bladder spasms. All of these orders are listed on the client's chart. In what order will you accomplish these actions?

a. Administer acetaminophen/oxycodone 325 mg/5 mg (Percocet) 2 tablets. b. Irrigateretentioncatheterwith30-50mLofsterilenormalsaline. c. Infuse 500 mL of 5% dextrose in lactated Ringer's solution over 2 hours. d. Encourageclient'soralfluidintaketoatleast2500-3000mLdaily. 11.Answer B, A, C, D - Bladder spasms are usually caused by the presence of clots obstructing the catheter, so irrigation should be the first action taken. Administration of analgesics may help to reduce spasm. Administration of a bolus of IV fluids is commonly used in the immediate post-operative period to help maintain fluid intake and increase urinary flow. Oral fluid intake should be encouraged once you are sure that the client is not nauseated and has adequate bowel tones.

Which of the following factors are risks for the development of diabetes mellitus? (Select all that apply.) a. Age over 45 years b. Overweight with a waist/hip ratio >1 c. Having a consistent HDL level above 40 mg/dl d. Maintaining a sedentary lifestyle

a. Age over 45 years b. Overweight with a waist/hip ratio >1 d. Maintaining a sedentary lifestyle Aging results in reduced ability of beta cells to respond with insulin effectively. Overweight with waist/hip ratio increase is part of the metabolic syndrome of DM II. There is an increase in atherosclerosis with DM due to the metabolic syndrome and sedentary lifestyle.

3. A client with diabetic neuropathy reports a burning, electrical-type in the lower extremities that is not responding to NSAIDs. You anticipate that the physician will order which adjuvant medication for this type of pain?

a. Amitriptyline(Elavil) b. Corticosteroids c. Methylphenidate (Ritalin) d. Lorazepam(Ativan)

13.You have just received the morning report from the night shift nurses. List the order of priority for assessing and caring for these patients.

a. Apatientwhodevelopedtumorlysissyndromearound5:00AM b. A patient with frequent reports of break-through pain over the past 24 hours c. A patient scheduled for exploratory laparotomy this morning d. Apatientwithanticipatorynauseaandvomitingforthepast24hours _____, _____, _____, _____ A, C, B, D - Tumor lysis syndrome is an emergency of electrolyte imbalances and potential renal failure. A patient scheduled for surgery should be assessed and prepared for surgery. A patient with breakthrough pain needs assessment and the physician may need to be contacted for a change of dose or medication. Anticipatory nausea and vomiting has a psychogenic component that requires assessment, teaching, reassurance, and antiemetics.

9. You interventions for the patient with CTS. Which action requires that you intervene immediately?

a. Arrangethepatient'slunchtrayandcutthemeat. b. Providewarmwaterandassistthepatientwithabath. c. Replace the patient's splint in hyperextension position. d. Remindthepatientnottoliftveryheavyobjects. ANSWER C - When a patient with CTS has a splint used for immobilization of the wrist, it is placed either in the neutral position or in slight extension

8. You are preparing to give an enteral feeding through a nasogastric tube. Place the steps in the correct order.

a. Assessforbowelsounds. b. AuscultatetubeplacementandcheckpH. c. Flush the tube with water. d. Reflushthetubewithwater. e. Administerthefeeding. f. Check for residual volume. _____, _____, _____, _____, _____, _____ ANSWERS A, B, F, C, E & D -

15.You are preparing to insert an intermittent catheter into a male patient to assess for post-void residual urine. Place the following steps I correct order.

a. Assistpatienttothebathroomandaskhimtoattempttovoid. b. Retracttheforeskinandholdthepenisat60to90-degreeangle. c. Open the catheterization kit and put on sterile gloves. d. Lubricatethecatheterandinsertitthroughthemeatusofthepenis. e. Positionthepatientsupineinbedorwithheadslightlyelevated. f. Drain all urine present in the bladder into the container. g. Cleansetheglanspenisstartingatthemeatusandworkingoutward. h. Remove the catheter, clean the penis, and measure the amount of urine returned. _____, _____, _____, _____, _____, _____, _____, _____ 15.ANSWER A, E, C, B, G, D, F, H -

1. The experienced LPN/LVN, under the supervision of the team leader RN, is providing nursing care for a client with a respiratory problem. Which of the following actions are appropriate to the scope of practice for an experienced LPN/LVN? (Choose all that apply.)

a. Auscultatebreathsounds b. AdministerMDI(multidose)inhaler)medications c. Complete in-depth admission assessment d. Checkoxygensaturationusingpulseoximetry e. Initiatenursingcareplan f. Evaluate client's technique for using MDIs A, B, D - The experienced LPN/LVN is capable of gathering data and observations including breath sounds and pulse oximetry. Administering medications, such as MDIs, is within the scope of practice for the LPN/LVN. Independently completing the admission assessment, initiating the nursing care plan, and evaluating a client's abilities require additional education and skills.

10.You are working in the ED caring for a patient who was just admitted with left anterior chest pain, possible unstable angina or myocardial infarction. Which nursing activity will you accomplish first?

a. Auscultateheartsounds. b. Administersublingualnitroglycerin. c. Insert an IV catheter. d. Obtainabriefpatienthealthhistory. The priority for a patient with unstable angina or myocardial infarction is treatment of pain.

2. You are creating a teaching plan for a patient with newly diagnosed migraine headaches. Which key items should be included in the teaching plan? (Choose all that apply).

a. Avoidfoodsthatcontaintyramine,suchasalcoholandagedcheese. b. AvoiddrugssuchasTagamet,nitroglycerinandNifedipine. c. Abortive therapy is aimed at eliminating the pain during the aura. d. Apotentialsideeffectofmedicationsisreboundheadache. e. Complementarytherapiessuchasrelaxationmaybehelpful. f. Continue taking estrogen as prescribed by your physician.

Blood sugar is well controlled when Hemoglobin A1C is: a. Below 7% b. Between 12%-15% c. Less than 180 mg/dL d. Between 90 and 130 mg/dL

a. Below 7% A1c measures the percentage of hemoglobin that is glycated and determines average blood glucose during the 2 to 3 months prior to testing. Used as a diagnostic tool, A1C levels of 6.5% or higher on two tests indicate diabetes. A1C of 6% to 6.5% is considered prediabetes.

17.You are working in the PACU caring for a 32-year-old client who has just arrived after having a dilation and curettage (D & C) to evaluate infertility. Which assessment data are of most concern?

a. Bloodpressure162/90 b. Perinealpadsaturatedafterfirst30minutes c. O2 saturation 91% - 95% d. Sharp,continuouslevel8/10abdominalpain 17.Answer D - Cramping or aching abdominal pain is common after D & c; however, sharp, continuous pain may indicate uterine perforation, which would requires immediate notification of the physician. The other data indicate a need for ongoing assessment or interventions. Transient blood pressure elevation may occur due to the stress response after surgery. Bleeding following the procedure is expected, but should decrease over the first 2 hours. And while the oxygen saturation is not at an unsafe level, interventions to improve the saturation should be accomplished

17.You assess a patient who has just returned to the recovery area after having a coronary arteriogram. Which of these data is of most concern?

a. Bloodpressureis144/78. b. Pedalpulsesarepalpableat+1. c. Left groin has a 3-cm ecchymotic area. d. Apicalpulseis122andregular. The most common complication after coronary arteriogram is hemorrhage and the earliest indication of hemorrhage is an increase in heart rate.

26.Three days after having a pelvic exenteration procedure, a client suddenly complains of a "giving" sensation along her abdominal incision. You check under the dressing and find that the wound edges are open and loops of intestines are protruding. Which action should you take first?

a. Call the client's surgeon and report that wound evisceration has occurred. b. Coverthewoundwithsaline-soakeddressings. c. Don sterile gloves and gently replace the intestine back in the wound. d. Checktheclient'sbloodpressureandheartrate 26.Answer D - Wound dehiscence or evisceration may cause shock, so the first action should be to assess the client's blood pressure and heart rate. The next action should be to ensure that the abdominal contents remain moist by covering the wound and loops of intestine with dressings soaked with sterile normal saline.

26.You are ambulating a cardiac surgery patient who has telemetry cardiac monitoring when another staff member tells you that thepatient has developed a supraventricular tachycardia with a rate of 146 beats per minute. In which order will you take these actions?

a. Callthepatient'sphysician. b. Havethepatientsitdown. c. Check the patient's blood pressure. d. Administeroxygenbynasalcannula.

11.As charge nurse you assign the nursing care of a patient who has just returned form open carpal tunnel release surgery to an experienced LPN/LVN, who will perform under the supervision of an RN. Which of the following instructions will you provide for the LPN/LVN? (Choose all that apply.)

a. Checkthepatient'svitalsignsevery15minutesinthefirsthour. b. Checkthedressingfordrainageandtightness. c. Elevate the patient's hand above the heart. d. Thepatientwillnolongerneedpainmedication. e. Checktheneurovascularstatusofthefingerseveryhour. 11.ANSWERS A, B, C & E - Post-operatively, patients with OCTR surgery have pain and numbness. Their discomfort may last for weeks to months. All of the other directions are appropriate to the post-operative care for this patient. It is important or monitor for drainage, tightness, and neurovascular changes. Raising the hand/wrist above the heart reduces the swelling form surgery, and this is often done for several days

7. While working on the surgical unit, you are assigned to care for a client who has just returned to the surgical unit after a TURP. You assess the client and obtain these data. Which finding will require the most immediate action?

a. Client'sbloodpressurereadingis153/88. b. Client's catheter is draining bright red blood. c. Client is not wearing anti-embolism hose. d. Clientiscomplainingofabdominalcramping. Answer B - Hemorrhage is a major complication after TURP and should be reported to the surgeon immediately.

Patients with acute renal failure usually go though a diuretic phase. The diuresis can result in an output of up to 10 L/day of dilute urine. During this phase, it is important to monitor for electrolyte and fluid imbalances.

a. Clienttellsyouthathealwayshastroublestartinghisurinarystream. b. Client'schartshowsanelevatedprostate-specificantigen(PSA)level. c. Client is restless and his bladder is palpable above the symphysis pubis. d. Client says he has not voided since having a glass of juice 4 hours ago. Answer C - A palpable bladder and restlessness are indicators of bladder distention, which would require action (such as insertion of a catheter) in order to empty the bladder.

17.The LPN/LVN, under your supervision, is providing nursing care for a patient with GBS. What observation would you instruct the LPN/LVN to report immediately?

a. Complaintsofnumbnessandtingling b. Facialweaknessanddifficultyspeaking c. Rapid heart rate of 102 beats per minute d. Shallowrespirationsanddecreasedbreathsounds The priority interventions for the patient with GBS are aimed at maintaining adequate respiratory function.

17.You are caring for an obese post-operative client who underwent surgery for bowel resection. As the client is moving in bed, he states, "Something popped open." Upon examination you note wound evisceration. Place the following steps in order for handling this complication.

a. Covertheintestinewithsterilemoistenedgauze b. Staycalmandstaywiththeclient. c. Monitor the vital signs especially BP and pulse. d. Haveacolleaguegathersuppliesandcontactthephysician. e. Puttheclientintosemi-Fowler'swithkneesslightlyflexed. f. Prepare the client for surgery as ordered. _____, _____, _____, _____, _____, _____ 17.ANSWERS B, D, E, A, C & F

17.An 18-year-old college student with an exacerbation of systemic lupus erythematosus (SLE) has been receiving prednisone (Deltasone) 20 mg daily for 4 hours. Which of these medical orders should you question?

a. Discontinueprednisoneaftertoday'sdose. b. Administerfirstdoseofvaricellavaccine. c. Check patient's C-reactive protein (CRP). d. GiveIbuprofen(Advil)800mgevery6hours. The varicella (chickenpox) vaccine is a live-virus vaccine and should not be administered to patients who are receiving immunosuppressive medications such as prednisone.

25.You are administering vancomycin (Vancocin) 500 mg IV to a client with PID when you notice that the client's neck and face are becoming flushed. Which action should you take next?

a. Discontinuethevancomycin. b. Slowtherateofthemedicationinfusion. c. Obtain an order for an antihistamine. d. Checktheclient'stemperature. 25.Answer B - "Red man syndrome" occurs when vancomycin is infused too quickly. Because the client needs the medication to treat PID, the vancomycin should not be discontinued. Antihistamines may help decrease the flushing but vancomycin should be administered over at least 60 minutes

In addition to standard precautions, the nurse caring for a patient with rubella would plan to implement what type of precautions? a. droplet precautions b. airborne precautions c. Contact precautions d. universal precautions

a. rubella is an illness transmitted by large-particle droplets and requires droplet precautions in addition to standard precautions.

14.A client admitted to the ED complains of itching of the trunk and groin. You note the presence of multiple reddened wheals on the chest, back, and groin. Which question is most appropriate to ask next?

a. Doyouhaveafamilyhistoryofeczema? b. Haveyoubeenusingsunscreenregularly? c. How do you usually manage stress? d. Areyoutakinganynewmedications? 14.ANSWER D - Wheals are frequently associated with allergic reactions, so asking about exposure to new medications is the most appropriate question for this client.

22.The client with acute respiratory distress syndrome (ARDS) is receiving oxygen by non-rebreather mask, but arterial blood gases still show poor oxygenation. As the nurse responsible for this client's care, you anticipate which physician's orders?

a. Endotracheal intubation and mechanical ventilation b. ImmediateapplicationofCPAPtoclient'snoseandmouth c. Intravenous furosemide (Lasix) 100 mg IV push stat d. CallaCODEforrespiratoryarrest. A non-rebreather mask can deliver up to 95% oxygen. When the client's oxygenation status does not respond to oxygenation at this high a concentration, this is refractory hypoxemia. Usually at this stage, the client isworking very hard to breathe and may go into respiratory arrest unless intubation and mechanical ventilation are provided to decrease the client's work of breathing.

Which of the following terms is defined as the sense of identification with a collective cultural group? a. ethnicity b. race c. cultural acquisition d. culture shock

a. Ethnicity is the sense of identification with a collective cultural group, largely based on the group's common heritage.

Which laboratory test should a nurse anticipate a physician would order when an older person is identified as high-risk for diabetes mellitus? (Select all that apply.) a. Fasting Plasma Glucose (FPG) b. Two-hour Oral Glucose Tolerance Test (OGTT) c. Glycosylated hemoglobin (HbA1C) d. Finger stick glucose three times daily

a. Fasting Plasma Glucose (FPG) b. Two-hour Oral Glucose Tolerance Test (OGTT) When an older person is identified as high-risk for diabetes, appropriate testing would include FPG and OGTT. A FPG greater than 140 mg/dL usually indicates diabetes. The OGTT is to determine how the body responds to the ingestion of carbohydrates in a meal. HbA1C evaluates long-term glucose control. A finger stick glucose three times daily spot-checks blood glucose levels.

9. For a patient on the chemotherapeutic drug vincristine (Oncovin), which of the following side effects should be reported to the physician?

a. Fatigue b. Nauseaandvomiting c. Paresthesia d. Anorexia Paresthesia is a side effect associated with some chemotherapy drugs such as vincristine (Oncovin). Ideally, chemotherapy drugs should be given by nurses who have received additional training in how to safely prepare and deliver the drugs and protect themselves from exposure. The other options are a concern but the general principles of drug administration apply.

Reye's syndrome

acute encephalopathy that follows a viral illness and is characterized pathologically by cerebral edema and fatty changes in the liver

The period of immobolization and elevation of an autograft does what?

allows the autograft time to adhere & attach to the would bed & the elevation minimizes edema.

14.A client is being tapered off opioids and the nurse is watchful for signs of withdrawal. What is one of the first signs of withdrawal?

a. Fever b. Nausea c. Diaphoresis d. Abdominalcramps Diaphoresis is one of the early signs that occur between 6 and 12 hours. Fever, nausea, and abdominal cramps are late signs that occur between 48 and 72 hours.

A nurse assesses an oral temperature for a pateint as 38.5 C (101.3 F) what term would the nurse use to report this temperature? a. fever b. hypothermia c. hypertension d. afebrile

a. Fever is an elevation in body temperature

20.When assessing a client with cervical cancer who had a total abdominal hysterectomy yesterday, you obtain the following data. Which information has the most immediate implications for planning the client's care?

a. Finecracklesareaudibleatthelungbases. b. Client'srightcalfisswollenandtender. c. Client is using the PCA every 15 minutes. d. Urineinthecollectionbagisamberandclear. 20.Answer B - Right calf swelling indicates the possible presence of deep vein thrombosis. This will change the plan of care, since the client should be placed on bedrest, while the usual plan is to ambulate the client as soon as possible after surgery.

26.Place the steps in correct order for performing colostomy care.

a. Fitthepouchsnuglyaroundthestoma. b. Assessthecolorandappearanceofthestoma. c. Wash the skin with mild soap and rinse with warm water. d. Applyaskinbarriertoprotecttheperistomalskin. e. Drytheskincarefully. f. Don a pair of clean gloves. _____, _____, _____, _____, _____, _____ 26.ANSWERS F, B, C, E, D & A

12.You are teaching the client and family how to do colostomy irrigation. Place the information in the correct order.

a. Hangthecontainerataboutshoulderheight. b. Allowthesolutiontoflowslowlyandsteadilyfor5-10minutes. c. Put 500 - 100 mL of lukewarm water in the container. d. Allow30-45minutesforevacuation. e. Lubricatethestomaconeandgentlyinsertthetubingtipintothestoma. f. Clean, rinse, and dry skin, and apply a new drainage pouch. _____, _____, _____, _____, _____, _____ 12.ANSWERS C, A, G, E, B, D, F - Prepare the warm water (cold water can cause cramping) and hang the container at shoulder height (hanging the container too high or too low will alter the rate of flow). Put on a pair of clean gloves to protect your hands from colostomy secretions. Lubricating the stoma and gently inserting will allow the water t flow into the stoma. A slow and steady flow prevents cramps and spillage. Adequate time allows for complete evacuation. Careful attention to the skin prevents breakdown.

16.While transferring a dirty laundry bag, a nursing assistant sustains a puncture would to the finger from a contaminated needle. The unit has several clients with hepatitis and AIDS; the source is unknown. Prioritize the instructions that you, as charge nurse, should give to the assistant.

a. Havebloodtest(s)drawnprotocol. b. Completeandfileanincidentreport. c. Perform a thorough aseptic Handwashing. d. Reporttotheoccupationalhealthnurse. e. Followupforresultsandcounseling. f. Begin prophylactic drug therapy. _____, _____, _____, _____, _____, _____ 16.ANSWERS C, D, B, A, F & E -

Which factor in the patient's past medical history dictates that the nurse exercise caution when administering acetaminophen (Tylenol) a. Hepatitis B b. Occasional alcohol use c. allergy to aspirin d. gastric limitation with bleeding

a. Hepatitis B

12.Before giving a beta-adrenergic blocker glaucoma agent, you would notify the physician if the client discloses a history of what condition?

a. Hypertension b. Tachycardia c. Rheumatoid arthritis d. Bradycardia ANSWER D - All beta-adrenergic blockers are contraindicated in bradycardia. Alpha-adrenergic agents can cause tachycardia and hypertension. Carbonic anhydrase inhibitors should not be given to clients with rheumatoid arthritis who are taking high doses of aspirin

28.When assessing a 22-year-old client who was admitted 3 days ago with multiple rib fractures and pulmonary contusions after a motor vehicle accident, you find that the client has shallow respirations at a rate of 38. He says he feels "dizzy and scared." His oxygen saturation is 90% with the oxygen at 6 L/minute per nasal cannula. Which action is most appropriate?

a. Increases the flow rate on the oxygen to 10 L/minute and reassess the client after about 10 minutes. b. Assist the client to se the incentive spirometer and splint his chest using a pillow while he coughs. c. Administer the ordered morphine sulfate to the client to decrease his anxiety and reduce the hyperventilation. d. Place the client on a non-breather mask at 95%-100% F102 and call the physician to discuss the client's status. The client's history and symptoms suggest that he may be developing ARDS, which will require intubation and mechanical ventilation. The maximum oxygen delivery with a nasal cannula is an FiO2 of 40%. This is achieved with the O2 flow at 6 L/min, so increasing the flow to 10 L/min will not be helpful.

22.A 59-year-old woman who had a total abdominal hysterectomy and bilateral salpingo-oophorectomy 3 days ago is complaining of flank pain and a burning sensation with urination. Her total urine output during the previous 8 hours was 210 mL. The client's temperature is 101.30 F. You call the physician to report this information and receive these orders. Which will you implement first?

a. InsertstraightcatheterPRNforoutputlessthan300mL/8hours. b. Administeracetaminophen(Tylenol)650mgorally. c. Send urine specimen to laboratory for culture and sensitivity. d. Administerceftizoxime(Cefizox)1gIVevery12hours. 22.Answer A - The client has symptoms of a urinary tract infection. Inserting a straight catheter will enable you to obtain an uncontaminated urine specimen for culture and susceptibility testing before the antibiotic is started.

After having a transverse colostomy constructed for colon cancer, discharge planning for home care would include teaching about the ostomy appliance. Information appropriate for this intervention would include: a. Instructing the client to report redness, swelling, fever, or pain at the site to the physician for evaluation of infection b. Nothing can be done about the concerns of odor with the appliance. c. Ordering appliances through the client's health care provider d. The appliance will not be needed when traveling.

a. Instructing the client to report redness, swelling, fever, or pain at the site to the physician for evaluation of infection Signs and symptoms for monitoring infection at the ostomy site are a priority evaluation for clients with new ostomies. The remaining actions are not appropriate. There are supplies avaliable for clients to help control odor that may be incurred because of the ostomy. Although a prescription for ostomy supplies is needed, you can order the supplies from any medical supplier. Dependent on the location and trainability of the ostomy, appliances are almost always worn throughout the day and when traveling

9. The day after having a radical prostatectomy, your client has many blood clots in the urinary catheter and states he has frequent bladder spasms. You notice occasional urine leakage around the catheter at the urinary meatus. The client says that his right calf is sore and complains that he feels short of breath. Which action will you take first?

a. Irrigatethecatheterwith50mLofsterilesaline. b. Administeroxybutynin(Ditropan)5mgorally. c. Dorsiflex the foot to check for Homan's sign. d. Obtainanoxygensaturationusingpulseoximetry. Answer D - It is important to assess oxygenation because the client's shortness of breath may indicate a pulmonary embolus, a serious complication of TURP. Dorsiflexion of the foot should not be done if a deep vein thrombosis is suspected, since this may dislodge thrombus. The other activities are appropriate, but are not as high a priority as ensuring that oxygenation is adequate

16.At 10:00 AM, a patient receives a new order for transesophageal echocardiography (TEE) ass soon as possible. Which action will you take first?

a. MakethepatientNPO. b. Teachthepatientabouttheprocedure. c. Start an intravenous line. d. Attachthepatienttoacardiacmonitor

4. The disease progress of cancers, such as cervical or Hodgkin's, can be classified according to a clinical staging system. Place the description of stages 0-IV in the correct order.

a. Metastasis b. Limitedlocalspread c. Cancer in situ d. Tumorlimitedtotissueoforigin e. Extensivelocalandregionalspread _____, _____, _____, _____, _____ ANSWER C, D, B, E, A - This classification system is based on the extent of the disease rather than the histological changes, Stage 0: cancer in situ, stage I: tumor limited to tissue of origin, stage II: limited local spread, stage III: extensive local and regional spread, stage IV: metastasis.

18.When developing the plan of care for a home health client who has been discharged after a radical prostatectomy, which activities will you delegate to the home health aide? (Choose all that apply.)

a. Monitortheclientforsymptomsofurinarytractinfection. b. Helptheclienttoconnectthecathetertothelegbag. c. Assess the client's incision for appropriate wound healing. d. Assisttheclienttoambulateforincreasingdistances. e. Helptheclientshoweratleasteveryotherday. 18.Answer B, D, E - Assisting with catheter care, ambulation, and hygiene is included in home health aide education and would be expected activities for this staff member.

Mr. Wright is recovering from abdominal surgery. When the nurse assists him to walk, she observes that he grimaces, moves stiffly, and becomes pail. She is aware that he has consistently refused his pain medication. Which of the following would be a priority nursing diagnosis? a. Acute pain related to fear of taking prescribed post operative medication b. Impaired physical mobility related to surgical procedure c. anxiety related to outcome of surgery d. Risk for infection related to surgical incision

a. Mr. Wrights immediate problem is pain that is unrelieved because he refuses to take his pain medication for an unknown reason. The other nursing diagnoses are plausible but not a priority in this situation.

23.An 86-year-old woman had an anterior and posterior colporrhaphy (A and P repair) several days ago. The client has been unwilling to ambulate or cough effectively. Her retention catheter was discontinued 8 hours ago. Which information obtained during your assessment has the most immediate implications for her care?

a. Oral temperature is 100.70 F. b. Abdomenisfirmandtendertopalpationabovethesymphysispubis. c. Breath sounds are decreased with fine crackles audible at both bases. d. Apicalpulseis86andslightlyirregular. 23.Answer B - After an A and P repair, it is essential that the bladder be empty to avoid putting pressure on the suture lines. The abdominal firmness and tenderness indicate that the client's bladder is distended

10.The patient is scheduled for endoscopic carpal tunnel release surgery in the morning. What key point will you be sure to teach the patient?

a. Painandnumbnesswillbeexperiencedforseveraldaystoweeks. b. Immediatelyaftersurgery,thepatientwillnolongerneedassistance. c. After surgery, the dressing will be large with dots of drainage d. Aftersurgery,thepainandparesthesiawillnolongerbepresent. 10.ANSWER A - Post-operative pain and numbness occur for a longer period of time with endoscopic carpal tunnel release than with the open procedure. Patients often need assistance post-operatively, even after they are discharged. The dressing from the endoscopic procedure is usually very small and there should not be a lot of drainage

16.Which physical assessment finding should be reported to the physician?

a. Pearlygrayorpinktympanicmembrane b. Dense,whitishringatthecircumferenceofthetympanum c. Bulging red or blue tympanic membrane d. Aconeoflightattheinnermostpartofthetympanum 16.ANSWER C - A bulging red or blue tympanic membrane is a possible sign of otitis media or perforation.

A client has just had surgery for colon cancer. Which of the following disorders might the client develop? a. Peritonitis b. Diverticulosis c. Partial bowel obstruction d. Complete bowel obstruction

a. Peritonitis Bowel spillage could occur during surgery, resulting in peritonitis. Complete or partial bowel obstruction may occur before bowel resection. Diverticulosis doesn't result from surgery or colon cancer.

Which of the following is accurate pertaining to physical exercise and type 1 diabetes mellitus? a. Physical exercise can slow the progression of diabetes mellitus. b. Strenuous exercise is beneficial when the blood glucose is high. c. Patients who take insulin and engage in strenuous physical exercise might experience hyperglycemia. d. Adjusting insulin regimen allows for safe participation in all forms of exercise.

a. Physical exercise can slow the progression of diabetes mellitus. Physical exercise slows the progression of diabetes mellitus, because exercise has beneficial effects on carbohydrate metabolism and insulin sensitivity. Strenuous exercise can cause retinal damage, and can cause hypoglycemia. Insulin and foods both must be adjusted to allow safe participation in exercise.

Which one of the following methods/techniques will the nurse use when giving insulin to a thin person? [Hint] A. Pinch the skin up and use a 90 degree angle B. Use a 45 degree angle with the skin pinched up C. Massage the area of injection after injecting the insulin D. Warm the skin with a warmed towel or washcloth prior to the injection

a. Pinch the skin up and use a 90 degree angle The best angle for a thin person is 90 degrees with the skin pinched up. The area is not massaged and it is not necessary to warm it.

5. You are supervising an RN who has pulled from the medical-surgical floor to the emergency department (ED). The nurse is providing care for a client admitted with anterior epistaxis (nosebleed). Which of these directions will you clearly provide to the RN? (Choose all that apply.)

a. Positiontheclientsupineandturnedonhisside. b. Applydirectlateralpressuretothenosefor5minutes. c. Maintain universal body substance precautions. d. Applyiceorcoolcompressestothenose. e. Instructtheclientnottoblowthenoseforseveralhours. B, C, D, E - The correct position for a client with an anterior nosebleed is upright and leaning forward to prevent blood from entering the stomach and prevent possible aspiration

25.A client is admitted through the emergency department for a strangulated intestinal obstruction with perforation. What interventions do you anticipate for this emergency condition? (Choose all that apply.)

a. Preparationforsurgery b. Bariumenema c. NG tube insertion d. Abdominalx-ray e. IVfluids 25.ANSWERS A, C, D & E - Strangulated intestinal obstruction is a surgical emergency. NG tube is for decompression of the intestine. Abdominal x-ray is the most useful diagnostic aid. IV fluids are needed to maintain fluid and electrolyte balance and delivery of medication. Barium enema is not ordered if perforation is suspected.

A patient who has fallen and injured his wrist carefully cradles it with the other hand. The patient is demonstrating which of the following responses to pain? a. behavioral b. affective c. pysiologic d. involuntary

a. Protecting or guarding a painful area is a behavioral response. Affective responses are psychological ones, and examples of physiologic or invuluntary response would be increased blood pressure or dialation of the pupils.

9. The experienced LPN/LVN reports that a client's blood pressure and heart rate have decreased and that when the face is assessed, one side twitches. What action should you take at this time?

a. Reassesstheclient'sbloodpressureandheartrate b. Reviewtheclient'smorningcalciumlevel c. Request a neurologic consult today d. Checktheclient'spapillaryreactiontolight A positive Chvostek's sign (facial twitching of one side of the mouth, nose, and cheek in response to tapping the face just below and in front of the ear) is a neurologic manifestation of hypocalcemia. The LPN/LVN is experienced and possesses the skills to take accurate vital signs.

Physician's orders for a client with acute pancreatitis include the following: strict NPO, NG tube to low intermittent suction. The nurse recognizes that these interventions will: a. Reduce the secretion of pancreatic enzymes b. Decrease the client's need for insulin c. Prevent secretion of gastric acid d. Eliminate the need for analgesia

a. Reduce the secretion of pancreatic enzymes

Radiation therapy is used to treat colon cancer before surgery for which of the following reasons? a. Reducing the size of the tumor b. Eliminating the malignant cells c. Curing the cancer d. Helping the bowel heal after surgery

a. Reducing the size of the tumor Radiation therapy is used to treat colon cancer before surgery to reduce the size of the tumor, making it easier to be resected. Radiation therapy isn't curative, can't eliminate the malignant cells (though it helps define tumor margins), can could slow postoperative healing.

20.Place the steps for removal of a foreign body from the ear canal in the correct order.

a. Referfortreatmentofexternalotitis. b. Inspectthetympanicmembranefortrauma. c. Obtain history for type of object. d. Chooseappropriatefluidforirrigationorinstillation. e. Assess,forpossibilityofperforation. _____, _____, _____, _____, _____ 20.ANSWERS C, E, D, B & A - The type of foreign body (e.g. insect, bean, bead) will determine the next steps. If there is a live insect, instill oil. Vegetable or insect matter will swell if water is used for irrigation. Tightly wedged objects like beads are difficult to flush. If perforation is suspected or if the object is not easily removed, the nurse should not attempt irrigation or instillation. Check for trauma after the object is removed. If trauma occurred, the client should be referred for antibiotics to prevent infection.

Who are the largest group of healthcare providers in the United States? a. Registered Nurses b. Physicians c. Physical therapists d. Social Workers

a. Registered nurses are the largest group of healthcare providers in the United States

3. The nursing care plan for the client with dehydration includes interventions for oral health. Which interventions are within the scope of practice for the LPN/LVN being supervised by the nurse? (Choose all that apply.)

a. Remindclienttoavoidcommercialmouthwashes. b. Encourage mouth rinsing with warm saline. c. Assess lips, tongue, and mucous membranes d. Providemouthcareevery2hourswhileclientisawake e. Seekdietaryconsulttoincreasefluidsonmealtrays.

19.You are working in the ED when a client with possible toxic shock syndrome (TSS) is admitted. The physician has given all of these orders. Which one will you implement first?

a. Removeclient'stampon. b. Obtainbloodculturesfromtwosites. c. Give O2 at 6 L/minute. d. Infusenafcillin(Unipen)500mgIV. 19.Answer A - Because the most likely source of the bacteria causing the TSS is the client's tampon, it is essential to remove it first. The other actions should be implemented in the following order: administer oxygen (essential to maximize O2 delivery to tissues), obtain blood cultures (best obtained prior to initiating antibiotic therapy to obtain accurate culture and susceptibility results), and infuse nafcillin (rapid initiation of antibiotic therapy will decrease bacterial release of toxins).

13.You are caring for a client with a nasogastric (NG) tube. Which task can be delegated to the experienced nursing assistant?

a. RemovetheNGtubeperphysicianorder. b. Securethetapeiftheclientaccidentallydislodgesthetube. c. Disconnect the suction to allow ambulation to the toilet. d. Reconnectthesuctionaftertheclienthasambulated. 13.ANSWER C - Disconnecting the tube from suction is an appropriate task to delegate. Suction should be reconnected by the nurse, so that correct pressure is checked. If the nursing assistant is permitted to reconnect the tube, the RN is still responsible for checking that the pressure setting is correct. During removal of the tube, there is a potential for aspiration, so the nurse should perform this task. If the tube is dislodged, the nurse should recheck placement before it is secured

1. You give an intradermal injection of allergen to a patient who is undergoing skin testing for allergies. A few minutes later, the patient complains about feeling anxious, short of breath, and dizzy. You notice that the patient has reddened blotches on the face and arms. All of these therapies are available on your emergency cart. Which action should you take first?

a. Startoxygenat4L/minusinganasalcannula. b. ObtainIVaccesswithalarge-boreIVcatheter. c. Administer epinephrine (Adrenalin) 0.3 mL subcutaneously d. Givealbuterol(Proventil)withanebulizer Epinephrine given rapidly at the onset of an anaphylactic reaction may prevent or reverse cardiovascular collapse as well as airway narrowing caused by bronchospasm and inflammation. Oxygen use is also appropriate, but generally is administered using a non-rebreather mask at 90%-100%

14.In monitoring patients who are at risk for spinal cord compression related to tumor growth, what is the most likely early manifestation?

a. Sudden-onsetbackpain b. Motorloss c. Constipation d. Urinaryhesitancy Back pain is an early sign occurring in 95% of patients. The other symptoms are later

Which nursing organization was the first international organization of professional women? a. ICN b. ANA c. NLN d. NSNA

a. The ICN, founded in 1899, was the first international organization of professional women.

Two nurses are taking an apical-radial pulse and note two differences in the pulse rate of 8 bpm. The nurse would document this difference as which of the following? a. pulse deficit b. pulse amplitude c. ventricual rythm d. hearth arrhythmia

a. The difference between the apical and raidal pusle rate is called the pulse deficit

24.While you are orienting a new RN to the medical-surgical unit, you observe the orientee accomplishing all of the following actions while caring for a client with severe pelvic inflammatory disease (PID), who has been admitted to the hospital for administration of IV antibiotics. Which one will require that you intervene most quickly?

a. The new RN tells the client she should avoid using tampons in the future. b. ThenewRNofferstheclientanicepacktodecreaseherabdominalpain. c. The new RN positions the client flat in bed while helping her take a bath. d. The new RN teaches the client she should not have intercourse for 2 months. 24.Answer C - The client should be positioned in a semi-Fowler's position to minimize the risk of abscess development higher in the abdomen. The other actions also require correction, but not as rapidly. Tampon use is not contraindicated after an episode of PID, although some sources recommend not using tampons during the acute infection. Heat application to the abdomen and pelvis is used for pain relief. Intercourse is safe a few weeks after effective treatment for PID

A pateint complains of sever abdominal pain. When assessing the vital signs, the nurse would not be surprised to find what assessment? a. an increase in the pulse rate b. a decrease in body temperature c. a decrease in blood pressure d. an increase in repiratory depth

a. The pulse often increases when an individual is experiencing pain. Pain does not affect body temperature and may increase blood pressure.

29.During the initial post-operative assessment of a patient who has just transferred to the post-anesthesia care unit (PACU) after repair of an abdominal aortic aneurysm, you obtain all of these data. Which has the most immediate implications for the patient's care?

a. Thearteriallineindicatesabloodpressureof190/112. b. ThemonitorshowssinusrhythmwithfrequentPACs. c. The patient does not respond to verbal stimulation d. Thepatient'surineoutputis100mLofamberurine. Elevated blood pressure in the immediate post-operative period puts stress on the graft rupture and/or hemorrhage, so it is important to lower the blood pressure quickly.

4. You are evaluating an HIV-positive patient who is receiving IV pentamidine (Pentam) as a treatment for Pneumocystis carinii pneumonia. Which information is most important to communicate to the physician?

a. Thebloodpressuredecreasedto104/76duringadministration. b. Thepatientiscomplainingofpainatthesiteoftheinfusion. c. The patient is not taking in an adequate amount of oral fluids. d. Bloodglucoseis55mg/dLafterthemedicationadministration. Pentamidine can cause fatal hypoglycemia, so the low blood glucose level indicates a need for a change in therapy. The low blood pressure suggests that the IV infusion rate may need to be slowed.

19.A patient with systemic lupus erythematosus (SLE) is admitted to the hospital for evaluation and management of acute joint inflammation. Which information obtained in the admission laboratory testing concerns you most?

a. Thebloodureanitrogen(BUN)leveliselevated. b. TheC-reactiveprotein(CRP)levelisincreased. c. The anti-nuclear antibody (ANA) test is positive. d. Thelupuserythematosus(LE)cellprepispositive. A high number of patients with SLE develop nephropathy

19.An outpatient seen in the clinic for follow-up after being diagnosed with contact dermatitis caused by poison ivy has been taking prednisone (Deltasone) 30 mg daily. You evaluate the client for adverse medication effects. Which information is of most concern?

a. Theclient'sbloodglucoseis136mg/dL b. Theclientstates,"Iameatingallthetime." c. The client complains of epigastric pain. d. Theclient'sbloodpressureis148/84 19.ANSWER C - Epigastric pain may indicate that the client is developing peptic ulcers, which require collaborative interventions such as the use of antacids, H2 receptor blockers (e.g., famotidine [Pepcid]), or proton pump inhibitors (e.g., esomeprazole [Nexium]

What would make serum amylase elevated?

acute pancreatitis nL levels 60-160 somogi U/dL

18.You are caring for a post-operative cholecystectomy client. What should be reported immediately to the physician?

a. Theclientcannotvoid4hourspost-operatively. b. Theclientreportsshoulderpain. c. The client reports severe RUQ tenderness. d. Output does not equal input for the first few hours. 18.ANSWER C - RUQ is a sign of hemorrhage or bile leak. Ability to void should return within 6 hours post-operatively. Right shoulder pain is related to unabsorbed CO2 and will resolve by placing the client in the Sims' position. Output that does not equal input after surgery for the first several hours is expected.

17.You obtain the following data about a client admitted with multiple myeloma. Which information has the most immediate implications for the client's care?

a. Theclientcomplainsofchronicbonepain. b. Theblooduricacidlevelisveryelevated. c. The 240hour urine shows Bence-Jones protein. d. Theclientisunabletoplantarflexthefeet. The lack of plantar flexion may indicate spinal cord compression, which should be evaluated and treated immediately by the physician

5. You obtain all of these assessment data about your client with continuous bladder irrigation (CBI) after a transurethral resection of the prostate (TURP). Which information indicates the most immediate need for nursing intervention?

a. Theclientstateshefeelsacontinuousurgetovoid. b. Thecatheterdrainageislightpinkwithoccasionalclots. c. The catheter is pulled taut and taped to the client's thigh. d. Theclientcomplainsofpainfulbladderspasms. Answer D - The bladder spasms may indicate that blood clots are obstructing the catheter, which would indicate the need for irrigation of the catheter with 30 - 50 mL of saline using a piston syringe.

3. You patient with Paget's disease. Which finding indicates that the physician should be notified?

a. Thepatienthasbowingofbothlegsandthekneesareasymmetric. b. Thebaseofthepatient'sskullisinvaginated(platybasia). c. The patient is only 5 feet tall and weighs 120 pounds. d. Thepatient'sskullissoft,thick,andlargerthannormal. ANSWER B - Platybasia (basilar skull invagination) causes brain stem manifestations that threaten life. Patients with Paget's disease are usually short and often have bowing of the long bones that results in asymmetric knees or elbow deformities. Their skull is typically soft, thick and enlarged.

16.You assess a 24-year-old with RA who is considering using methotrexate (Rheumatrex) for treatment. Which information is most important to communicate with the physician?

a. Thepatienthasmanyconcernsaboutthesafetyofthedrug. b. Thepatienthasbeentryingtogetpregnant. c. The patient takes a daily multivitamin tablet. d. Thepatientsaysthatshehastakenmethotrexateinthepast.

8. You incontinence. For which patient will a bladder-training program be an appropriate intervention?

a. Thepatientwithfunctionalincontinenceduetomentalstatuschanges b. The patient with stress incontinence due to weakened bladder neck support c. The patient with urge incontinence and abnormal detrusor muscle contractions d. The patient with transient incontinence due to inability to get to toileting facilities ANSWER C - As long as they are alert, aware, and able to resist the urge to urinate, patients with urge incontinence can be taught to control their bladder by starting a schedule for voiding, then increasing the intervals between voids. Patients with functional incontinence related to mental status changes or loss of cognitive function will not be able to follow the bladder-training program. The patient with stress incontinence is better treated with exercises such as pelvic kegal

11.A client is admitted to the intensive car unit (ICU) with disseminated intravascular coagulation (DIC) associated with a gram-negative infection. Which assessment information has the most immediate implications for the client's care?

a. Thereisnopalpableradialorpedalpulse. b. Theclientcomplainsofchestpain. c. The client's oxygen saturation is 87% d. Thereismottlingofthehandsandfeet. Because the decrease in oxygen saturation will have the greatest immediate effect on all body systems, improvement in oxygenation should be the priority goal of care

3. You are preparing to administer TPN through a central line. Place the steps for administration in the correct order.

a. Useaseptictechniquewhenhandlingtheinjectioncap. b. ThreadtheINtubingthroughaninfusionpump. c. Check the solution for cloudiness or turbidity. d. Connectthetubingtothecentralline. e. Selectthecorrecttubingandfilter. f. Set infusion pump at prescribed rate. _____, _____, _____, _____, _____, _____ ANSWERS C, E, B, A, D & F

17.You are taking histories from several clients who report vertigo. Which client report concerns you the most?

a. Vertigowithhearingloss b. Episodicvertigo c. Vertigo without hearing loss d. "Merry-go-round"vertigo ANSWER C - Vertigo without hearing loss should be further assessed for nonvestibular causes, such as cardiovascular or metabolic.

Which of the following diets is most commonly associated with colon cancer? a. low fiber, high fat b. low fat high fiber c. low protein, high carb d. low carb, high protein

a. a. low fiber, high fat low fiber, high fat diet reduced motility and increases the chance of constipation. The metabolic end products of this type of diet are carcinogenic. A LOW FAT HIGH FIBER diet is recommended to help avoid colon cancer. Carbohydrates and protein aren't necessarily associated with colon cancer.

Which pain management task can the nurse safely delegate to nursing assistive personnel? a. asking about pain during vital signs b. evaluating the effectiveness of pain medication c. developing a plan of care involving nonpharmacologic interventions d. administering over-the-counter pain medications

a. asking about pain during vital signs Rationale: The nurse can delegate the task of asking about pain when nursing assistive personnel obtain vital signs. The NAP must be instructed to report findings to the nurse without delay. The nurse should evaluate the effectiveness of pain medications and develop the plan of care. Administering over-the-counter and prescription medications is the responsibility of the RN or LPN

A patient newly diagnosed with Type I DM is being seen by the home health nurse. The doctors orders include: 1200 calorie ADA diet, 15 units NPH insulin before breakfast, and check blood sugar qid. When the nurse visits the patient at 5 pm, the nurse observes the man performing blood sugar analysis. The result is 50 mg/dL. The nurse would expect the patient to be a. confused with cold, clammy skin an pulse of 110 b. lethargic with hot dry dkin and rapid deep respirations c. alert and cooperative with BP of 130/80 and respirations of 12 d. short of breath, with distended neck veins and bounding pulse of 96.

a. confused with cold, clammy skin an pulse of 110 hypoglycemia

Which of the following would the nurse expect to assess as a physiologic response to moderate pain? a. pupil dialation b. Restlessness c. Decreased pulse rate d. Protection of the painful area

a. dilation of pupils is a physiologic or involuntary response to moderate pain, whereas decreased pulse rate, also a physiologic or involuntary response, occurs when pain is severe and deep. Restlessness and protection of the painful area are behavioral responses.

Where do individuals learn their health beliefs and values? a. in the family b. in school c. from school nurses d. from peers

a. healthcare activities, health beliefs, and values are learned within one's family.

What group is the largest subculture of the healthcare system? a. nurses b. physicians c. social workers d. physical therapists

a. nurses are the largest subculture of the healthcare system

Which of the following levels of basic human needs is most basic? a. physiologic b. safety and security c. love and belonging d. self-actualization

a. physiologic

What are sx of serotonin syndrome?

abd pain, fever, swetaing, tachy, hTN delirium, myoclonus, irritability and mood changes can result in death, so contact MD immediately

A nurse is assessing for correct placement of a NG tube. The nurse aspirates the stomach content & checks pH. The nurse verifies correct tube placement if the pH is?

acidic and pH lower than 3.5

Kawasaki disease (AKA mucocutaneous lymph node syndrome)

acute systemic inflammatory disease Acute Stage: fever, conjunctival hyperemia, red throat, swollen hands, rash, and enlargement of cervical lymph nodes Subacute Stage: cracking lips ad fissures, desquamation of the skin on the tips of the fingers and toes, joint pain, cardiac manifestations, thrombocytosis cardiac involvement is the most serious complication; aneurysms can develop. monitor heart sounds, rate and rhythm

Anosognosia

after brain attack/stroke, client exhibits neglect of the affected side of the body nurse plans activities to increase client's awareness of affected side

What is presbycusis?

age-related hearing loss, high frequencies lost first

bubbling of water in the water seal chamber indicates?

air drainage from the pt and occurs during exhalation, coughing, or sneezing

excessive bubbling in the water seal chamber may indicate?

air leak

Negative pressure airflow used for?

airborne precautions from small droplet infections like measles, chickenpox, or TB

What causes urinary retention (r/t periop)?

anesthetics, opioid analgesics

If a pt has muscle weakness, drowsiness, and absent DTR, what should be assessed?

antacid or OTC use b/c it can contain Mg that could cause hyperMg (> 2.5)

What are cross reactive foods with a latex allergy?

apple, apricot, avocado banana, carrot, celery, cherry, chestnut fig, grape, kiwi, melon nectarine, passion fruit, papaya, peach, pear, pineapple, plum, potato tomato

Sengstaken-Blakemore tube

asserts pressure on esophageal varices to stop bleeding

What should urine output be?

at least 30 mL/hr

How often should a RN monitor a pt's IV site when receiving a continuous IV infusion?

at least once an hour

Potassium (K)- rich foods

avocado, bananas, cantaloupe, carrots, fish, mushrooms, oranges, potatoes, pork, beef, veal, raisins, spinach, strawberries, and tomatoes, honey-dew melon

What is appropriate for a 24 hr urine collection?

avoid strenuous activity & high protein meals before test b/c may inc creatine clearance able to drink as much fluid as you want before & during the test save urine and keep in fridge for 24 hr

tetracycline

avoid sun and dairy products milk, decreases obsorbation

Site selection for a newborn

axillary initially

Site selection for a client immediately following mandibular surgery

axillary or tympanic

Site selection for a client with stuffy nose

axillary or tympanic

Which phrase best describes the science of nursing? a) The skilled application of knowledge b) The knowledge base for care c) Hands-on care, such as giving bath. d) Respect or each individual patient

b) The knowledge base for care p.21

Falsely high blood pressure readings can be caused by which of the following? a) lying in the supine position b) a blood pressure cuff that is too small c) a blood pressure cuff that is too large d) the arm positioned below the level ofthe heart

b) a blood pressure cuff that is too small

Which of the following phrases best defines culture? a. A dominant group within a society b. A shared system of beliefs, values, and behaviors c. One's values are replaced by the values of the dominant culture d. Categories are based on specific physical characteristics

b. Culture may be defined as a shared system of beliefs, values, and behavioral expectations that provide social structure for daily living

he goal for pre-prandial blood glucose for those with Type 1 diabetes mellitus is: a. <80 mg/dl b. < 130 mg/dl c. <180 mg/dl d. <6%

b. < 130 mg/dl

Which of the following symptoms is a client with colon cancer most likely to exhibit? a. A change in appetite b. A change in bowel habits c. An increase in body weight d. An increase in body temperature

b. A change in bowel habits The most common complaint of the client with colon cancer is a change in bowel habits. The client may have anorexia, secondary abdominal distention, or weight loss. Fever isn't associated with colon cancer.

Which of the following is one element of a healthy community? a. meets all the needs of its inhabitants b. offers access to healthcare services c. has mixed residential and industrial areas d. is little concerned with air and water quality

b. A healthy community offers access to healthcare services to treat illness and to promote health.

Your patient has developed a low-grade fever and states that she has felt very tired lately. You interpret these findings as indicating which stage of infections? a. incubation period b. prodromal stage c. full stage of illness d. convalescent period

b. During the prodromal stage, the person has vague signs and symptoms, such as fatigue and a low-grade fever.

A school nurse is teaching a class of junior-high students about the effects of smoking. This educational program will meet which of the aims of nursing? a. promoting health b. preventing illness c. restoring health d. facilitating coping with disability or death

b. Educational programs can reduce the risk of illness by teaching good health habits

A patient is having dyspnea. What would the nurse do first? a. remove pillows from under the head b. elevate the head of the bed c. elevate the foot of the bed d. take the blood pressure

b. Elavating the head of the bed allows the abdominal organs to descend, giving the diaphragm greater room for expansion and facilitating lung expansion

12.You are preparing the post-operative CTS patient for discharge. Which information is important to provide to this patient?

b. Handmovementswillberestrictedfor4-6weeksaftersurgery.

The nurse is planning dietary changes for a client following an episode of pancreatitis. Which diet is suitable for the client? a. Low calorie, low carbohydrate b. High calorie, low fat c. High protein, high fat d. Low protein, high carbohydrate

b. High calorie, low fat

the nurse teaches a patient at home to use clean technique when changing a wound dressing. This is which of the following? a. The nurse's preference b. Safe for the home setting c. Unethical behavior d. grossly negligent

b. In the home setting, where the patient's environment is more controlled, medical asepsis is usually recommended.

When assessing a temperature rectally, the nurse would use exterme care when inserting the thermometer to preevnt which of the following? a. an incrase in heart rate b. a decrase in heart rate c. a decrease in blood pressure d. an increase in respirations

b. Insertion of a rectal thermometer may stimulate the vagus nerve, which, inturn, would decrease heart rate. The may potentially be harmful for patients with cardiac problems.

While preparing the client for a colonoscopy, the nurse's responsibilities include: a. Explaining the risks and benefits of the exam b. Instructing the client about the bowel preparation prior to the test c, Instructing the client about medication that will be used to sedate the client d. Explaining the results of the exam

b. Instructing the client about the bowel preparation prior to the test The nurse is responsible for instructing the client about the bowel preparation prior to the test. Answers 1, 3, 4 are the physician's responsibility.

A young hispanic mother comes to the local clinic because her baby is sick. She speaks only Spanish and the nurse speaks only English. What should the nurse do? a. Use short words and talk more loudly? b. Ask an interpreter for help c. Explain why care can't be provided d. Provide instructions in writing.

b. Many agencies have a qualified interpreter who understands the healthcare system and can reliably provide assistance.

A nurse is providing care based on Maslow's hierarch of basic human needs. For which of the following nursing activities is the approach useful? a. making accurate nursing diagnoses b. establishing priorities of care c. communicating concerns more concisely d. integrating science into nursing care

b. Maslow's hierarch of basic human needs is useful for establishing priorities of care.

Which of these laboratory values noted by the nurse when reviewing the chart of a diabetic patient indicates the need for further assessment of the patient? a. Fasting blood glucose of 130 mg/dl b. Noon blood glucose of 52 mg/dl c. Glycosylated hemoglobin of 6.9% d. Hemoglobin A1C of 5.8%

b. Noon blood glucose of 52 mg/dl The nurse should assess the patient with a blood glucose level of 52 mg/dl for symptoms of hypoglycemia, and give the patient some carbohydrate-containing beverage such as orange juice. The other values are within an acceptable range for a diabetic patient.

Applying the gate control theory of pain, which of the following would be an effective nursing intervention for a patient with lower back pain? a. Encouraging regular use of analgesics b. applying a moist heating pad to the area at prescribed intervals c. Reviewing the pain experience with the patient d. Ambulating the patient after administering medication

b. Nursing measures such as applying warmth to the lower back stimulate the large nerve fibers to close the gate and block the pain. The other choices involve attempts to stimulate nerve fibers that interfere with pain transmission as explained by the gate control theory.

One of the benefits of Glargine (Lantus) insulin is its ability to: a. Release insulin rapidly throughout the day to help control basal glucose. b. Release insulin evenly throughout the day and control basal glucose levels. c. Simplify the dosing and better control blood glucose levels during the day. d. Cause hypoglycemia with other manifestation of other adverse reactions.

b. Release insulin evenly throughout the day and control basal glucose levels. Glargine (Lantus) insulin is designed to release insulin evenly throughout the day and control basal glucose levels.

A patient complains of abdominal pain that is difficult to localize. The nurse categorically interprets this as which type of pain? a. causalgia b. visceral c. superficial d. pyschogenic

b. Teh patient's pain would be categrozied as visceral pain, which is poorly loclized and can originate in body organs in the abdomen. It is not complex regional pain syndrome (causalgia), which is pain that occurs in the area of injured peripheral nerves, cutaneous pain, which is superfical and usually involves the skin or subcutanteous tissue, or phychogenic pain, in which a physical cuase for the pain cannot be identified.

What is the purpose of the ANA's Scope and Standards of Practice? a. To describe the ethical responsibility of nurses b. To define the activities that are special and unique to nursing c. To establish nursing as an independent and free standing profession d. To regulate the practice of nursing

b. The ANA's Scope and Standards of Practice define the activities of nurses that are specific and unique to nursing.

A patient has intravenous fluids infusing int he right arm. When taking a blood pressure on this patient, what would the nurse do in this situation? a. take the blood pressure in the right arm b. Take the blood pressure in the left arm c. Use the smalles possible cuff d. Report the inability to take the blood pressure

b. The blood pressure should be taken in the arm opposite the one with the infusion.

Before assessing respirations, the nurse reviews normal rates for adults. Which rate would the nurse identify as normal? a. 1-6 breaths/min b. 12 to 20 breaths/min c. 60-80 breaths/min d. 100-120 breaths/min

b. The normal repiratory rate for adults is 12-20 breaths per minute

The guidelines for Carbohydrate Counting as medical nutrition therapy for diabetes mellitus includes all of the following EXCEPT: a. Flexibility in types and amounts of foods consumed b. Unlimited intake of total fat, saturated fat and cholesterol c. Including adequate servings of fruits, vegetables and the dairy group d. Applicable to with either Type 1 or Type 2 diabetes mellitus

b. Unlimited intake of total fat, saturated fat and cholesterol

What are the smallest infectious agents capable of causing an infection? a. bacteria b. viruses c. molds d. yeast

b. a virus is the smallest of all microorganisms and can be seen only with a special microscope.

A student nurse is learning to assess blood pressure. What does the blood pressure measure? a. flow of blod through the circulation b. force of blood against arterial walls c. force of blood against venous walls d. flow of blood through the heart

b. blood pressuer is the measurement of the force of blood against arterial walls.

A 1200-calorie diet and exercise are prescribed for a patient with newly diagnosed type 2 diabetes. The patient tells the nurse, "I hate to exercise! Can't I just follow the diet to keep my glucose under control?" The nurse teaches the patient that the major purpose of exercise for diabetics is to a. increase energy and sense of well-being, which will help with body image. b. facilitate weight loss, which will decrease peripheral insulin resistance. c. improve cardiovascular endurance, which is important for diabetics. d. set a successful pattern, which will help in making other needed changes.

b. facilitate weight loss, which will decrease peripheral insulin resistance. Rationale: Exercise is essential to decrease insulin resistance and improve blood glucose control. Increased energy, improved cardiovascular endurance, and setting a pattern of success are secondary benefits of exercise, but they are not the major reason.

When assessing pain in a child, the nurse needs to be aware of which of the following considerations: a. Immature neurologic development results in reduced sensation of pain. b. Inadequate or inconsistent relief of pain is widespread. c. Reliable assessment tools are currently unavailable. d. Narcotic analgesic use should be avoided

b. healthcare personnel are only now becoming aware of pain relief as a priority for children in pain. The evidence supports the fact that children do indeed feel pain and reliable assessment tools are available specifically for use with children. Opioid analgesics may be safely used with children as long as they are carefully monitored.

Which of the following descriptions does the nurse use to best describe intractable pain? a. intermittent in nature b. Resistant to treatment c. Excruciating d. Widespread

b. intractable pain is sever pain that is resistant to relief measures. The other terms do not describe this resistance to treatment.

A patient who is admitted with acute hepatic encephalopathy and ascites receives instructions about appropriate diet. The nurse determines that the teaching has been effective when the patient's choice of foods from the menu includes a. an omelet with cheese and mushrooms and milk. b. pancakes with butter and honey and orange juice. c. baked beans with ham, cornbread, potatoes, and coffee. d. baked chicken with french-fries, low-fiber bread, and tea.

b. pancakes with butter and honey and orange juice. B Rationale: The patient with acute hepatic encephalopathy is placed on a LOW-protein diet to decrease ammonia levels. The other choices are all higher in protein and would not be as appropriate for this patient. In addition, the patient's ascites indicate that a low-sodium diet is needed and the other choices are all high in sodium.

A nurse wants to acquire knowledge of a specific culture. What could be done first? a. talk to coworkers b. review literature c. talk to family members of the patient d. ask others with more experience for help

b. reviewing literature about a specific culture can provide the nurse with a starting point for information about cultural values, dietary practices, family lines of authority, and helth and illness beliefs and practices.

Practicing careful hand hygiene and using sterile techniques are ways in which nurses meet which basic human need? a. physiologic b. safety and security c. self esteem d. love and belonging

b. safety and security

When assessing a patient receiving a continuous opioid infusion, the nurse immediately notifies the physician when the patient has which of the following findings? a. a respiratory rate of 10/min with normal deapth b. a sedation level of 4 c. Mild confusion d. Reported constipation

b. sedation level is more indicative or respiratory depression because it usually precedes it. A sedation level of 4 calls for immediate action because that patient has minimal or no response to stimuli.

Which phrase best describes the science of nursing? a. The skilled application of knowledge b. The knowledge base for care c. Hands-on care, such as giving a bath d. Respect for each individual patient

b. the science of nursing is the knowledge base for care that is provided. In contrast, the skilled application of that knowledge is the art of nursing.

In terms of kidney fx, why are ACEs, better than CCBs?

b/c ACES work on efferent arterioles to dilate kidney arteries resulting in lower glomerular pressure vs. CCBS that work on afferent arterioles inc pressure

If the child is 3 years of age or younger, the pinna of the ear is pulled?

back and down

Several units of blood are to be administered, the nurse asks for ____to reduce the risk of cardiac dysrhythmias?

blood warmer

Cherry Angioma

bright red lesions that occur with aging and have no clinical significance

At which age does a child begin to accept that he or she will someday die: c) 9-12 years old d) 12-18 years old

c) 9-12 years old

A client with diabetes who needs to learn to inject his own insulin states, "Ive had a good night's sleep, so let's tackle that syringe." The client if showing: a) Feedback c) Readiness

c) Readiness

To assess a patient's BP, the nurse will do which of the following? a) use only the diaphragm of the stethoscope b) inflate the cuff to between 180 and 200 mm Hg c) inflate the cuff to 30 mm Hg higher than the point he last palpated a pulse d) talk to the patient in a calm, reassuring voice while auscultating

c) inflate the cuff to 30 mm Hg higher than the point he last palpated a pulse

A client is brought to the emergency room in an unresponsive state, and a diagnosis of hyperglycemic hyperosmolar nonketotic syndrome is made. The nurse would immediately prepare to initiate which of the following anticipated physician's orders? a) endotracheal intubation b) 100 units of NPH insulin c) intravenous infusion of normal saline d) intravenous infusion of sodium bicarbonate

c) intravenous infusion of normal saline The primary goal of treatment in hyperglycemic hyperosmolar nonketotic syndrome (HHNS) is to rehydrate the client to restore fluid volume and to correct electrolyte deficiency. Intravenous fluid replacement is similar to that administered in diabetic ketoacidosis (DKA) and begins with IV infusion of normal saline. Regular insulin, not NPH insulin, would be administered. The use of sodium bicarbonate to correct acidosis is avoided because it can precipitate a further drop in serum potassium levels. Intubation and mechanical ventilation are not required to treat HHNS.

A nurse is assessing viatl signs on several hospitalized children. The nurse would plan to use the oral route to assess temperature for which patient? a. 6 month old infant b. patient receiving oxygen therapy by mask c. 15 year old healthy adolescent d. unconcious patient

c. A healthy adolescent would be an appropriate patient for assessing temperature by the oral route.

Efforts by healthcare facilities to reduce the incidence of HAI's include an awareness of which of the following? a. The CDC requires all states to report HAIrates from

c. According to the Joint Commission, death or serious injury casued by an infection-related event is considered a sentinel event and must be reported. ...

Which of the following persons would most likely be diagnosed with diabetes mellitus? A 44-year-old: a. Caucasian woman. b. Asian woman. c. African-American woman. d. Hispanic male.

c. African-American woman. Age-specific prevalence of diagnosed diabetes mellitus (DM) is higher for African-Americans and Hispanics than for Caucasians. Among those younger than 75, black women had the highest incidence.

A patient develops a urinary tract infection after an indwelling urinary catheter has been inserted. This would most accurately be termed as which type of infection? a. a viral infection b. a chronic infection c. an iatrongenic infection d. an oppotunistic infection

c. An iatrogenic infection develops as a result of the insertion of an indwelling catheter. Because this infection just developed, it is not chronic, nor did it occur becuase of any altered physiology that may give an opportunistic organism a chance to cause infection. Urinary infections are bacterial, not viral.

The nurse is caring for the client diagnosed with ascites from hepatic cirrhosis. What information should the nurse report to the health-care provider? a. A decrease in the client's daily weight of one (1) pound. b. An increase in urine output after administration of a diuretic. c. An increase in abdominal girth of two (2) inches. d. A decrease in the serum direct bilirubin to 0.6 mg/dL.

c. An increase in abdominal girth of two (2) inches. Rationale: An increase in abdominal girth would indicate that the ascites is increasing, meaning that the client's condition is becoming more serious and should be reported to the health-care provider.

A nurse is interviewing a newly admitted patient. Which question would be considered culturally sensitive? a. do you think you will be able to eat the food we have here? b. Do you understand that we can't prepare special meals? c. What types of food do you eat for meals? d. Why cant you just eat our food while you are here?

c. Asking patients what types of foods they eat for meals is culturally sensitive

Which nonsteroidal anti-inflammatory drug might be administered to inhibit platelet aggregation in a patient at risk for thrombophlebitis a. ibuprofren b celecoxib c aspirin indomethacin

c. Aspirin Rationale: Aspirin is a unique NSAID that inhibits platelet aggregation. Low dose aspirin therapy is commonly administered to decrease the risk of thrombophlebitis, MI, and stroke. Ibuprofren, celecoxib, and indomethacin are NSAIDS, but they do not inhibit platelet aggregation

Which action should the nurse take before administering morphine 4.0 intravenously to a patient complaining of incisional pain? a. Assess the patient's incision b. Clarify the order with the prescriber c. assess the patient's respiratory status d. monitor the patient's heart rate

c. Assess the patient's respiratory status

Which of the following diabetes drugs acts by decreasing the amount of glucose produced by the liver? a. Sulfonylureas b. Meglitinides c. Biguanides d. Alpha-glucosidase inhibitors

c. Biguanides Biguanides, such as metformin, lower blood glucose by reducing the amount of glucose produced by the liver. Sulfonylureas and Meglitinides stimulate the beta cells of the pancreas to produce more insulin. Alpha-glucosidase inhibitors block the breakdown of starches and some sugars, which helps to reduce blood glucose levels

A nurse states, I know I am cleaner than most of my patients. What does this statement indicate? a. cultural assimilation b. racism c. ethnocentrism d. sterotyping

c. Ethnocentrism occurs when one believes that one's own ideas and practices are superior to those of others.

The nurse is caring for a patient with a diagnosis of hypothyroidism. Which nursing diagnosis should the nurse most seriously consider when analyzing the needs of the patient? a. High risk for aspiration related to severe vomiting b. Diarrhea related to increased peristalsis c. Hypothermia related to slowed metabolic rate d. Oral mucous membrane, altered related to disease process

c. Hypothermia related to slowed metabolic rate Thyroid hormone deficiency results in reduction in the metabolic rate, resulting in hypothermia, and does predispose the older adult to a host of other health-related issues. One quarter of affected elderly experience constipation.

Although all of the following are important to culturally competent nursing care, which one is the most basic? a. learning another language b. having signifigant information c. treating each person as an individual d. recognizing the importance of family

c. In all aspects of nursing, it is important to treat each patient as an individual. This is also true in providing culturally competent care.

The nurse is taking an apical pulse. What equipment will he take into the patients room? a. sphyogmomanometer b. electronic thermometer c. stethoscope d. doppler apparatus

c. The apical pulse can only be assessed by listening with a stehosocope

During treatment of a patient with a Minnesota balloon tamponade for bleeding esophageal varices, which nursing action will be included in the plan of care? a. Encourage the patient to cough and deep breathe. b. Insert the tube and verify its position q4hr. c. Monitor the patient for shortness of breath. d. Deflate the gastric balloon q8-12hr.

c. Monitor the patient for shortness of breath. Rationale: The most common complication of balloon tamponade is aspiration pneumonia. In addition, if the gastric balloon ruptures, the esophageal balloon may slip upward and occlude the airway. Coughing increases the pressure on the varices and increases the risk for bleeding. The health care provider inserts the tube and verifies the position. The esophageal balloon is deflated every 8 to 12 hours to avoid necrosis, but if the gastric balloon is deflated, the esophageal balloon may occlude the airway.

What type of authority regulates the practice of nursing? a. International standards and codes b. Federal guidelines and regulations c. State nurse practice acts d. Institutional policies

c. Nurse practice act are established in each state to regulate the practice of nursing

A nurse cares for a client following a liver biopsy. Which nursing care plan reflects proper care? a. Position in a dorsal recumbent position, with one pillow under the head b. Bed rest for 24 hours, with a pressure dressing over the biopsy site c. Position to a right side-lying position, with a pillow under the biopsy site d. Neurological checks of lower extremities every hour

c. Position to a right side-lying position, with a pillow under the biopsy site Positioning the client in a right side-lying position with a pillow under the biopsy site reflects proper care. Answer 1 does not permit the necessary pressure applied to the biopsy site. B ed rest is only required for several hours. There is no reason to do neurological checks.

Minority groups living within a dominant culture may lose the cultural characteristics that made them different. What is this process called? a. cultural diversity b. cultural imposition c. cultural assimilation d. ethnocentrism

c. When minority groups live within a dominant group, many members lose the cultural characteristics that once made them different

What historic event in the 20th century led to an increased emphasis on nursing and broadened the role of nurses? a. religious reform b. crimean war c. world war II d. Vietnam War

c. World War II

The nurse is having difficulty obtaining a capillary blood sample from a client's finger to measure blood glucose using a blood glucose monitor. Which procedure will increase the blood flow to the area to ensure an adequate specimen? a. Raise the hand on a pillow to increase venous flow. b. Pierce the skin with the lancet in the middle of the finger pad. c. Wrap the finger in a warm cloth for 30-60 seconds. d. Pierce the skin at a 45-degree angle.

c. Wrap the finger in a warm cloth for 30-60 seconds. The hand is lowered to increase venous flow. The finger is pierced lateral to the middle of the pad perpendicular to the skin surface.

John and Mary, each parents of one child, are both divorced. When they marry, the family structure that is formed will be described as which of the following: a. nuclear family b. extended family c. blended family d. cohabitating family

c. a blended family is formed when parents bring unrelated children from previous relationships together to form a new family

when caring for a patient with latex allergy, the nurse creates a latex-safe environment by doing which of the following? a. carefully cleaning the wall-mounted blood pressure device before using it. b. donning latex gloves outside the room to limit powder dispersal c. Using a latex-free pharmacy protocol d. placing the patient in a semiprivate room

c. a latex-free pharmacy protocol is a vital component when creating a safe environement for this patient.

Mr. L. has a seven-year history of hepatic cirrhosis. He was brought to the emergency room because he began vomiting large amounts of dark-red blood. An Esophageal Balloon Tamponade tube was inserted to tamponade the bleeding esophageal varices. While the balloon tamponade is in place, the nurse caring for Mr. L. gives the highest priority to a. assessing his stools for occult blood. b. evaluating capillary refill in extremities. c. auscultating breath sounds. d. performing frequent mouth care.

c. auscultating breath sounds. Rationale: Airway obstruction and aspiration of gastric contents are potential serious complications of balloon tamponade. Frequent assessment of the client's respiratory status is the priority.

A patient develops food poisoing from conaminated potato salad. what is the means of transmission for the infection organism? a. direct contact b. vector c. vehicle d. airborne

c. contaminated food is a vehicle for transmitting an infection.

A patient received 6 units of REGULAR INSULIN 3 hours ago. The nurse would be MOST concerned if which of the following was observed? a. kussmaul respirations and diaphoresis b. anorexia and lethargy c. diaphoresis and trembling d. headache and polyuria

c. diaphoresis and trembling indicates hypoglycemia

Which of the following lists the recommended sequence for removing soiled personal protective equipment when the nurse prepares to leave the patient's room? a. gown, goggles, mask, gloves, and exit the room b. gloves, wash hands, remove gown, mask, and goggles c. gloves, goggles, gown, mask, and wash hands d. goggles, mask, gloves, gown, and wash hands

c. gloves are always removed first becuase they are most liekly to be contaminated, and hands should be washed thoroughly after the equipment has been removed and before leaving the room.

For a nurse under with unsoiled hands, effective hand hygiene between patients requires which of the following? a. at least a 15 second scrub with plain soap and water b. at least a 23 minute scrub with an antimicrobial soap c. use of an alcohol-based antiseptic handrub d. that a mask be worn when scrubbing

c. hands that are not visibly soiled can be effeftively cleaned with an alchohol-based hand rub

Which hospitalized patient is most at risk for developing a helthcare-associated infection? a. Mr. Y, a 60 year old patient who smokes two packs of cigarettes daily b. Mrs. J, a 40 year old patient who has a white blood cell count of 6,000/mm3 c. Mr. L, a 65 year old patient who has indwelling urinary catheter in place d. Mrs. M, a 60 yaer old patient who is a vegetarian and slightly underweight

c. indwelling urinary catheters have been implicated in most health care associated infections.

A patient with acute pancreatitis has a nasogastric (NG) tube to suction and is NPO. The nurse explains to the patient that the major purpose of this treatment is a. control of fluid and electrolyte imbalance. b. relief from nausea and vomiting. c. reduction of pancreatic enzymes. d. removal of the precipitating irritants.

c. reduction of pancreatic enzymes. Rationale: Pancreatic enzymes are released when the patient eats. NG suction and NPO status decrease the release of these enzymes. Fluid and electrolyte imbalances will be caused by NG suction and require that the patient receive IV fluids to prevent this. The patient's nausea and vomiting may decrease, but this is not the major reason for these treatments. The pancreatic enzymes that precipitate the pancreatitis are not removed by NG suction.

Of the following statements, which one is true of self actualization? a. Humans are born with fully developed self-actualization b. self-actualization needs are met by having confidence and indepence c. The self-actualization process continues throughout life. d. loneliness and isolation occur when self-actualization needs are unmet

c. self actualization, or reaching one's full potential is a process that continues through life

When developing the plan of care for a patient with chronic pain, the nurse plans interventions based on the knowledge that chronic pain is most effectively relieved when analgesics are administered in what matter? a. on a p.r.n. basis b. Conservatively c. Around the clock (ATC) d. Intramuscularly

c. the prn protocol is totally inadequate for patients experiencing chronic pain. ATC doses of analgesics are more effective, wheras conservative pain management for whatever reason may also prove ineffective.

A nurse is documenting a blood pressuer of 120/80 mm Hg. The nurse itermprets the 120 to represent which of the following? a. pulse rate b. diastolic pressure c. systolic pressure d. pulse deficit

c. the systolic pressure is 120 mm Hg. The diastolic pressure is 80 mm Hg

Which of the following descriptions is true os using a placebo for pain control without the patients consent? a. widespread practice b. consistently effective c. deceptive d. justified to determine whether the pain is real

c. using a placebo to control pain creates distrust in the nurse-patient relationship and is considered unethical

A nurse sees smoke emerging from the suction equipment being used. Which is the greatest priority in the event of a fire? a.Report the fire. b. Extinguish the fire. c. Protect the clients. d. Contain the fire.

c.Protect the clients. Rationale: In the event of a fire, the nurse's priority responsibility is to rescue or protect the clients under his or her care. The next priorities are to report or alert the fire department, contain or confine, and extinguish the fire

What is an approp assessment for Raynaud phenomenon?

can you tolerate pressure on your hand

What are white patches on buccal mucosa indicative of?

candida albicans

genital warts infection

cauliflower like growth or growths that are soft & fleshy

an absence of fluctuation may indicate?

chest tube obstruction or the lung has reexpanded & that no more air is leaking into the pleural space

A nurse is preparing to administer meds through a nasogastric tube that is connected to suction. To administer the med, the nurse would?

clamp the NG tube for 30 to 60 min following admin of the meds

What do you do after administering the preop meds?

client in bed side rails up call light w/in reach instruct client not to get out of bed

What is scleral buckling?

compresses the retina to repair detached retina wear eye sheild avoid straining, vomiting, nausea, bearing down, coughing or sneezing

What can cause outflow probs when undergoing peritoneal dialysis?

constipation, assess for last BM

Glucose tolerance test

consume high-carb diet (200-300 g per day) for at least 3 days prior discontinue oral contraceptives, corticosteroids, salicylates, and thiazide derivatives 3 days prior discontinue insulin or oral hypoglycemic on day of test fasting required from midnight before test alcohol, coffee, and tea avoided for 12 hours before test

bacterial conjunctivitis is highly?

contagious

What breath sounds indicate pulmonary edema?

crackles, rhonchi

Latex allergy

cross rxn with bananas, kiwis, pineapples, tropical fruits, grapes, avocados, potatoes, hazelnuts, water chestnuts

Which of the following phrases decribes one of the purposes of the ANA's Nursing Social Policy Statement? a) To describe the nurse as a dependent caregiver b) To provide standards for nursing edu. programs c) To regulate nursing research d) To describe nursing's values & social responsibilty

d

fluctuation of water in the tube in the water seal chamber during inhalation and exhalation is?

expected

What is the normal respiratory rate for an adult? a) 30-40 breaths per minute b) 20-30 breaths per minute c) 10-15 breaths per minute d) 12-20 breaths per minute

d) 12-20 breaths per minute

Your patient has just finished a cup of hot tea. How long will you wait before taking her temperature? a) 5 minutes b) 10 minutes c) 15 minutes d) 30 minutes

d) 30 minutes

The nurse receives change-of-shift report about the following four patients. Which patient will the nurse assess first: a) A patient who has malnutrition associated with 4+generalized pitting edema b) A patient whose potential nutrition has 10 mL of solution left in the infusion bag d) A patient who is receiving continuous internal feedings and has new onset crackles throughout the lungs

d) A patient who is receiving continuous internal feedings and has new onset crackles throughout the lungs

During which stage of NREM sleep would you expect a client to be most difficult to arouse: c) Stage III d) Stage IV

d) Stage IV

A patient who has recieved an opioid analgesic will most likely exhibit which of the following vital sign changes? a) tachycardia b) bradycardia c) increased blood pressure d) decreased respiratory rate

d) decreased respiratory rate

Which of the following are considered defense mechanisms: b) denial c) Sublimation

d) denial

To help alleviate spiritual distress effectively, the nurse must: b) offer to pray with the client d) find out what the client perceives his/her spiritual needs to be

d) find out what the client perceives his/her spiritual needs to be

client newly diagnosed with diabetes mellitus has been stabilized with daily insulin injections. A nurse prepares a discharge teaching plan regarding the insulin and plans to reinforce which of the following concepts? a) always keep insulin vials refrigerated b) ketones in the urine signify a need for less insulin c) increase the amount of insulin before unusual exercise d) systematically rotate insulin injections within one anatomic site

d) systematically rotate insulin injections within one anatomic site Insulin doses should not be adjusted nor increased before unusual exercise. If ketones are found in the urine, it possibly may indicate the need for additional insulin. To minimize the discomfort associated with insulin injections, insulin should be administered at room temperature. Injection sites should be rotated systematically within one anatomic site.

Which of the following nursing degrees prepares a nurse for advanced practice as a clinical specialist or nurse practitioner? a. LPN b. ADN c. BSN d. Master's

d. A Master's degree prepares advanced practice nurses.

When working in the community, the nurse will recommend routine screening for diabetes when the person has one or more of seven risk criteria. Which of the following persons that the nurse comes in contact with most needs to be screened for diabetes based on the seven risk criteria? a. A woman who is at 90% of standard body weight after delivering an eight-pound baby b. A middle-aged Caucasian male c. An older client who is hypotensive d. A client with an HDL cholesterol level of 40 mg/dl and a triglyceride level of 300 mg/dl

d. A client with an HDL cholesterol level of 40 mg/dl and a triglyceride level of 300 mg/dl The seven risk criteria include: greater than 120% of standard body weight, Certain races but not including Caucasian, delivery of a baby weighing more than 9 pounds or a diagnosis of gestational diabetes, hypertensive, HDL greater than 35 mg/dl or triglyceride level greater than 250 or a triglyceride level of greater than 250 mg/dl, and, lastly, impaired glucose tolerance or impaired fasting glucose on prior testing.

While taking an adult patients pulse, a nurse finds the rate to be 140 bpm. What should the urse do next? a. check the pulse again in 2 hours b. check the blood pressure c. record the information d. report the rate

d. A rate of 140 bpm in an adult is abnormal pulse and should be reported to the isntructor of the nurse in charge or the patient.

Which of the following diagnostic tests should be performed annually over age 50 to screen for colon cancer? a. Abdominal CT scan b. Abdominal x-ray c. Colonoscopy d. Fecal occult blood test

d. Fecal occult blood test Surface blood vessels of polyps and cancers are fragile and often bleed with the passage of stools. Abdominal x-ray and CT scan can help establish tumor size and metastasis. A colonoscopy can help locate a tumor as well as polyps, which can be removed before they become malignant.

Which nurse in history is credited with establishing nursing education? a. clara barton b. lilian wald c. lavinia dock d. florence nightingale

d. Florence Nightingale established nursing education

ADH works on which part of the nephron?

distal tubule and collecting duct to reabsorb water

The nurse working in the physician's office is reviewing lab results on the clients seen that day. One of the clients who has classic diabetic symptoms had an eight-hour fasting plasma glucose test done. The nurse realizes that diagnostic criteria developed by the American Diabetes Association for diabetes include classic diabetic symptoms plus which of the following fasting plasma glucose levels? a. Greater than 106 mg/dl b. Greater than 126 mg/dl c. Higher than 140 mg/dl d. Higher than 160 mg/dl

d. Higher than 160 mg/dl

A frail elderly patient with a diagnosis of type 2 diabetes mellitus has been ill with pneumonia. The client's intake has been very poor, and she is admitted to the hospital for observation and management as needed. What is the most likely problem with this patient? a. Insulin resistance has developed. b. Diabetic ketoacidosis is occurring. c. Hypoglycemia unawareness is developing. d. Hyperglycemic hyperosmolar non-ketotic coma

d. Hyperglycemic hyperosmolar non-ketotic coma Illness, especially with the frail elderly patient whose appetite is poor, can result in dehydration and HHNC. Insulin resistance usually is indicated by a daily insulin requirement of 200 units or more. Diabetic ketoacidosis, an acute metabolic condition, usually is caused by absent or markedly decreased amounts of insulin.

To help relive her pain, Ann concentrates on a favorite vacation setting. The nurse interprets this technique as which of the following? a. distraction b. relaxation c. Recall d. Imagery

d. Imagery is a mind-body interaction that decreases pain sensation by focusing on pleasurable images.

Proliferative retinopathy is often treated using: a. Tonometry b. Fluorescein angiogram c. Antibiotics d. Laser surgery

d. Laser surgery Scatter laser treatment is used to shrink abnormal blood vessels in an effort to preserve vision. When there is significant bleeding in the eye, it is removed in a procedure known as vitrectomy. Tonometry is a diagnostic test that measures pressure inside the eye. A fluorescein angiogram is a diagnostic test that traces the flow of dye through the blood vessels in the retina; it is used to detect macular edema.

Of which of the following symptoms might an older woman with diabetes mellitus complain? a. Anorexia b. Pain intolerance c. Weight loss d. Perineal itching

d. Perineal itching

A patient complains of pain in a site that is different from where it originates. The nurse documents this as what type of pain? a. Transient pain b. superficial pain c. phantom pain d. referred pain

d. Referred pain is perceived in an area distant from its point of origin, where as transient pain is brief but passes quickly.

Which of the following phrases describes one of the purposes of the ANA's nursing's social policy statement? a. to describe the nurse as a dependent caregiver b. To provide standards for nursing educational programs c. to regulate nursing research d. to describe nursing's values and social responsiblity

d. The nursing's social policy statement describes the values and social responsibility of nursing

A nurse is caring for and obese 62 year old patient with arthritis who has developed an open reddened area over his sacrum. which of the following is a priority nursing diagnosis? a. imbalanced nutrition: more than body requirements related to immobility b. impaired physical mobility related to pain and discomfort c. Chronic Pain related to immobility d. risk for infection related to altered skin integrity

d. The priority diagnosis in this situation is the possibility of an infection developing in the open skin area.

A nurse knows that the blood pressure is often higher in oder adults based on the understanding that which of the folowing occurs with age? a. loss of muscle mass b. changes in exercise level c. decreased peripheral resistance d. decreased elasticity in arterail walls

d. With aging, elasticity in arterial walls is decreased, contributing to an elevated blood pressure reading.

Which of the following is one of the developmental tasks of the older adult family? a. maintain a supportive home base b. prepare for retirement c. cope with loss of energy and privacy d. adjust to loss of a spouse

d. a developmental task of an older individual is to cope with the loss of a spouse.

What is the best broad definition of family? a. a father, a mother, and children b. a gruop whose members are biologically related c. a unit that includes aunts, uncles, and cousins d. a group of people who live together

d. although all of the responses may be true, the best definition is a group of people who live together

Mrs. Young is receiving ATC medication for treatment of terminal cancer. She has recently reported several episodes of breakthrough pain. What treatment is most effective to manage these sudden flare-ups of pain? a. Increasing the dose of her ATC medication for treatment of terminal cancer. b. Restricting her physical activity c. Doing nothing more since her cancer is terminal d. Supplementing with doses of a short-acting opioid

d. breakthrough pain is best addressed by administering a short-acting opioid similar to her ATC medication. Increasing the dose of her ATC medication also increases her risk for developing side effects. All pain can be treated effectively, and limiting physical activity will not affect her breakthrough pain but may negatively affect her current lifestyle and self esteem.

A college student who has type 1 diabetes normally walks each evening as part of an exercise regimen. The student now plans to take a swimming class every day at 1:00 PM. The clinic nurse teaches the patient to a. delay eating the noon meal until after the swimming class. b. increase the morning dose of neutral protamine Hagedorn (NPH) insulin on days of the swimming class. c. time the morning insulin injection so that the peak occurs while swimming. d. check glucose level before, during, and after swimming.

d. check glucose level before, during, and after swimming. Rationale: The change in exercise will affect blood glucose, and the patient will need to monitor glucose carefully to determine the need for changes in diet and insulin administration. Because exercise tends to decrease blood glucose, patients are advised to eat before exercising. Increasing the morning NPH or timing the insulin to peak during exercise may lead to hypoglycemia, especially with the increased exercise.

The nurse has opened the sterile supplies and donned two sterile gloves to complete a sterile dressing change, a procedure that requires surgiacal asepsis. The nurse must do which of the following? a. keep splashes on the sterile field to a minimum b. cover the nose and mouth with gloved hands if a sneeze is imminent c. Use forceps soaked in a disinfectant d. consider the outer 1 inch of the field as contaminated

d. considering the outter inch of a sterile field as contaminated is a principle of surgical asepis.

When lactulose (Cephulac) 30 ml QID is ordered for a patient with advanced cirrhosis, the patient complains that it causes diarrhea. The nurse explains to the patient that it is still important to take the drug because the lactulose will a. promote fluid loss. b. prevent constipation. c. prevent gastrointestinal (GI) bleeding. d. improve nervous system function.

d. improve nervous system function. Rationale: The purpose for lactulose in the patient with cirrhosis is to lower ammonia levels and prevent encephalopathy. Although the medication may promote fluid loss through the stool, prevent constipation, and prevent bearing down during bowel movements (which could lead to esophageal bleeding), the medication is not ordered for these purposes for this patient.

The nurse administers codeine sulfate 30mg orally to a patient who underwent craniotomy 3 days ago for a brain tumor. How soon after administration should the nurse reassess the patient's pain? a. Immediately b. in 10 minutes c. in 15 minutes d. in 60 minutes

d. in 60 minutes Rationale: Codeine administered by the oral route reaches peak concentration in 60 minutes; therefore the nurse should reassess the patient's pain 60 min after administering. The nurse should reassess pain after 10 min when administering codeine by IM or SC routes. Drug administered by the IV routes are effective almost immediately; however codeine is NOT recommended for IV administration.

When planning strategies for pain control in older patients, the nurse should be aware of which of the following? a. pain is a natural outcome of the aging process. b. Sensitivity to pain increases with age c. Narcotic use should be avoided d. Denial of pain may occur

d. older people frequently deny pain because they view it as an ominous sign that may interfere with their independence. Pain sensitivity may decrease with age, but even this assumption is unsafe. Pain is not a natural outcome of the again process. Opioid medications can be used if the older patients response is carefully monitored ad evaluated.

Of all physiologic needs, which one is the most essential? a. food b. water c. elimination d. oxygen

d. oxygen

The CDC standard precaution recommendations apply to which of the following? a. only patients with diagnosed infections b. only blood and body fluids with visible blood c. all body fluids including sweat d. all pateints receiving care in hospitals

d. standard precautions apply to all patients receiving care in hospitals, regardless of their diganosis or possible infection status

A nurse states, that woman is 78 years old-too old to learn how to change a dressing. What is the nurse demonstrating? a. cultural imposition b. clustering c. cultural competency d. stereotyping

d. stereotyping is assuming that all members of a group are alike.

The nurse is performing discharge teaching for a patient with Addison's disease. It is MOST important for the nurse to instruct the patient about: a. signs and symptoms of infection b. fluid and electrolyte balance c. seizure precautions d. steroid replacement

d. steroid replacement steroid replacement is the most important information the client needs to know.

Which organizations initially developed the guidelines for minimum protection standards for infection prevention and control? a. OSHA b. Individual healthcare facilities C. the state governing body d. the CDC

d. the CDC established the initial minimum requirements for infection prevention and control.

The nurse provides vigilant monitoring of a patient receiving epidural analgesia to prevent the occurrence of which of the following? a. pruritus b. urinary retention c. vomiting d. respiratory depression

d. too much of an opioid drug given by way of an epidural catheter or a displaced catheter may result in the occurrence of respiratory depression. Pruritus, urinary retention, and vomiting may occur but are not life threatening

Colon cancer is most closely associated with which of the following conditions? a. appendicitis b. hemorroids c. hiatal hernia d. ulcerative colitis

d. ulcerative colitis Chronic ulcerative colitis, granulomas, and familial polyps seem to increase a person's chance of developing colon cancer. The other conditions listed have no known effect on the colon cancer risk.

What would you do for someone that has hyperMg?

d/c Mg use support ventilation loop diuretic/IV calcium teach about OTC meds w/Mg

the height of a cane should be ?

even with the greater trochanter with elbows flexed at 15 to 30 degrees of flexion.

How do you prevent atelectasis?

deep breathe, ambulate

What are imp client preop client teaching?

deep breathing and coughing exercises incentive spirometry leg and foot exercises splinting incision

What is myasthenia gravis?

def of ACh at myoneuronal jxn sx include muscle weakness produced by repeated movements that clear up with rest

What do you do if you suspect shock with spinal anesthesia?

do not elevate legs any higher than placing a pillow otherwise diaphragm muscles impaired

fibrinolytic therapy

done to reverse tissue damage after onset of chest pain, but tissue is reversable only if therapy is started within 6 hrs of chest pain. if chest pain lasts more than 6 hrs than damage will be irreversable and fibrinolytic theraphy is useless

What should a pt know about hip prothesis?

dont ever cross legs do not maintain hip in flexed position during sex don't sleep on affected side walking is a great exercise resume full fxn by 3 months w/exception of strenuous sports

meningitis is transmitted by?

droplet infection

precaution for mycoplasmal pneumonia

droplet precautions

post abdominal hysterectoy ambulate asap

dvt changes plan of care

How often should a pt deep breath?

every 1 to 2 hours

How often should you admin oral care?

every 2 hrs

How often should you do Passive ROM if pt is confined to bedrest?

every 2 hrs

What do you do if shock develops?

elevate legs treat cause

Autografts placed over joints or on lower extremities are ______ and ____ following surgery for ___ to ___ days.

elevated; immobolized; 3 to 7 days

What position will the pt with DVT be placed in?

elevation of the affected leg to facilitate blood flow by the force of gravity which will relieve edema and pain

What sign is indicative of preeclamp?

epigastric pain

scabies

erythematous, papular eruptions

deep partial-thickness burn

extends into the skin dermis and the wound is red and dry, with white areas in deeper parts; can convert to a full-thickness burn if tissue damage increases with infection, hypoxia, or ischemia

What indicates constipation postop (once solid diet resumed)?

failure to pass stool within 48 hrs

What are GERD r/f?

female > 45 fat white smoker NG tube

Signs of transfusion reaction

fever, chills, tachycardia, tachypnea, dyspnea, hives or skin rash, flushing, backache, and decreased blood pressure.

What is most indicative of severe brain damage?

flaccid muscles b/c even if twitching, that means there is still brain power

asterixis

flapping tremor early sign of liver encephalopathy

classic symptoms of TSS

high fever (temperature of 101 F or higher), vomiting, severe diarrhea

What drugs if given together can cause ototoxicity?

furosemide and indocin-both are otototoxic [NSAIDS and loop diuretics]

____ bubbling not____ bubbling should be noted int he suction control chamber.

gentle; vigorous

Roferon

given for treatment of Hep C. - Nausea and vomiting are common adverse effects of interferon alfa-2a, but continued vomiting should be reported to the physician because dehydration may occur. The medication may be given by either the subcutaneous or intramuscular route. Flu-like symptoms such as a mild temperature elevation, headache, muscle aches, and anorexia are common after initiating therapy but tend to decrease over time.

A nurse provides colostomy care for a pt with MRSA, contact precautions include wearing?

gloves, gown,goggles, face shield

What is the first thing to do if a pt is drooling, anxious, sore throat, and febrile?

go the the ER b/c probably acute epiglottis

positioning the drainage system below the pts chest allows ?

gravity to drain the pleural space

What should a pts resp rate be?

greater than 10, less than 30 breaths/min

Anterior Pituitary Hormones

growth hormone, FSH, and LH

Where is the fundus at 16 weeks?

halfway btw the symphysus pubis and umbilicus

A RN is preparing to change the parenteral nutrition solution bag & tubing, what should the RN ask the client to do during the tubing changes?

have the patient take a deep breathe and hold it during tubing changes.

what are the 2 positions for a pt during a thoracentesis?

have the pt sitting at the edge of the bed leaning over the bedside table or lying in bed on the unaffected side with the head of the bed elevated 30 to 45 degrees

the pt has had a craniotomy, what positions can this pt be in?

head of bed elevated 30 degrees; head in midline neutral position; place pt on the nonoperative side

urinary tract infection clinical manifestations

hematuria, edema, flank pain, & headache

What does Chevosks sign test for?

hypocalcemia, tap side of face to see if it twitches

What are celexa (an SRRI, benzo) SE?

hypomania, mania, insomnia, impotence, headache, dry mouth

0.45% normal saline is what type of solution?

hypotonic

What do you start with once oral fluids are permitted?

ice chips, water

A nurse is inserting a NG tube in an adult client. During the procedure, the client begins to cough & has difficulty breathing. what should the nurse do?

if the pt experiences difficulty breathing or any respiratory distress, withdraw the tube slighty, stop the advancement, & wait until the distress subsides.

What is a concern in the elderly with medications, particularily Tagment?

inc r/f CNS effects like drowsiness/confusion/memory loss esp in renal pt take medication at bedtime and WITH meals inc fiber in diet b/c can cause constipation

What are signs of dislocated prothesis?

increased hip pain shortening of the leg soft popping sound heard when affected leg is moved

Crohn's disease

inflammatory disease that can occur anywhere in GI tract, typical terminal ileum... leads to thickening and scarring, narrowing of lumen of intestine characterized by exascerbations and remissions nonbloody diarrhea, increasing in frequency, duration, and severity over time cramp-like colicky pain after meals

Full Thickness Burn

injured area appears waxy white, deep red, yellow, brown, or black; injured surface appears dry. Edema is present under eschar. tan or fawn color with skin that is hard, dry, and inelastic. little or no pain (nerve endings have been damaged)

Superficial-thickness Burn

injury to epidermis. mild to severe erythema is noted, and the skin blanches with pressure. Painful

postpartum endometritis is frequently associated with?

invasion of bacteria that may arise from the GI tract or from the lower genital tract.

What is urinary retention?

involuntary accumulation of urine in the bladder as a result of low muscle tone

Deep Full Thickness Burn

involves injury to the muscle, bone, and tendons. appears black and eschar is hard and inelastic

family members who have been in close contact with a pt w/tuberculosis are placed on prophylactic therapy with ___ for ___ to ___.

isoniazid; 6 to 12 months

When a pt is on heparin, why would a MD order warfain?

it takes 4-5 days for coumadin to kick in, and pt needs a PO med to go home on before d/c heparin

What are two isotonic IV solutions?

lactated ringers and 0.9% normal saline

Cushing's reflex

late sign of increased ICP widening pulse pressure (sys. rises faster than diastolic) bradycardia

Incontinence, assess in client:

laughing leakage

the RN is preparing to administer an enema. The RN positions the pt in what position?

left lateral position with right leg acutely flexed

the sigmoid and descending colon are located on what side of the body?

left side

For administering an enema, the pt is placed in what position?

left sims so that the enema solution can flow by gravity in the natural direction of the colon.

What do you do with dehiscense or evisceration?

low folwers knees bent to prevent abdo tension cover wound with sterile normal saline dressing notify physician

Hypotonic solutions have a ______ osmolality than do body fluids.

lower

What foods exacerbate GERD?

mint, carbonated, high fat, spicy, lemonade, chocolate, oil, tomatoes

What is stage I of renal failure?

more peeing b/c kidney not able to concentrate urine so will see polyuria and nocturia

superficial partial-thickness burn

mottled red base and broken epidermis and a wet shiny and weeping surface are present. large blisters can be seen covering an extensive area. skin is edematous and painful

continuous gentle bubbling in the suction control of chest tube is considered?

normal findings

Bell's Palsy

not a brain attack/ stroke symptoms often disappear spontaneously in 3-5 weeks

genital herpes

one or more vesicles that rupture & heal

Site selection for a healthy 60 year old woman

oral

Site selection for a client with hemorrhoids

oral, tympanic, or axillary

Site selection for a client with low platelet count due to chemotherapy

oral, tympanic, or axillary

Posterior Pituitary Hormones

oxytocin and ADH

syphilis infection

painless & indurated

Assessment: Vomiting postop. Abdominal distension. Absence of flatus.

paralytic ileus

A nurse is assisting a MD with the removal of a chest tube. The nurse should instruct the pt to?

perform the valsalva maneuver or take a deep breath and hold the breathe

What is intermediate postop stage?

period of 4 to 24 hours after surgery

What can untx impetigo lead two interms of sx?

periorbital edema untx impetigo can lead to acute glomerulonephritis which periorbital edema is a sign

Client experiencing dyspnea

place in LATERAL position

how do you measure the length of the nasogastric tube?

place the tube at the tip of the pt's nose, extend the tube to the earlobe & down to the xiphoid process

How to best maintain mobility of joints in a pt w/OA?

place them in functional position

After the first 24 hours, a pt with a right above the knee amputation of the right leg is placed in what position?

placed flat on the bed to reduce hip contracture

Albumin is used as ?

plasma expander

the chest tube insertion site is covered with an occlusive airtight dressing to?

prevent air from entering the pleural space.

Why are fluids important in a pt who has sickle cell disease?

prevents dehydration

type of prevention crisis: keeping the crisis from occuring

primary prevention

precautions for airborne disease like tuberculosis?

private negative airflow pressure rooms & personal respiratory protection devices

precautions for meningitis?

private room, cohort client, or use of a standard precaution mask

What is wound evisceration?

protrusion of intl organs through an incision

What do you do if hemorrhaging?

provide pressure! IV fluids, blood

positive airflow used for?

pts with stem cell transplants

After a liver biopsy, the pt is placed?

right side lying position with a small pillow or folded towel under the puncture site for 3 hours

As a result of fluid & blood loss, fresh frozen plasma is often used for?

rapid volume expansion

What are sx of an acute gout reaction?

redness & pain in big toe subside in 3-4 days; give colchcine and NSAIDS

Steps of handling the situation of a fire

rescue the client; activate the alarm; confine the fire; extinguish the fire

What is mechanical way to promote voiding?

run warm water over the perineum

Glasgow Coma Scale

scale of 1-15 (lower than 8 indicates that coma is present) Motor Response Points: Obeys a simple response = 6 Localizes painful stimuli = 5 Normal flexion (withdrawal) = 4 Abnormal flexion (decorticate posturing) = 3 Extensor response (decerebrate posturing) = 2 No motor response to pain = 1 Verbal Response Points: Oriented = 5 Confused conversation = 4 Inappropriate words = 3 Responds with incomprehensible sounds = 2 No verbal response = 1 Eye-Opening Points: Spontaneous = 4 In response to sound = 3 In response to pain = 2 No response, even to painful stimuli = 1

The Sengstaken-Blakemore tube is a triple lumen gastric tube that may be used to treat bleeding esophageal varices. The nurse knows that which item must be kept at the bedside at all times?

scissors

type of prevention crisis: focuses on reducing the intensity & duration of the crisis during the crisis itself

secondary prevention

why would you place a pt w/ pelvic inflammatory disease in _____ position?

semi fowlers; allows gravity to aid in drainage of the abdominal cavity & helps prevent the formation of abscesses high in the abdomen.

What is wound dehiscence?

separation of the wound edges

What can GBS cause? (group beta strep)

sepsis in preggar- should give intrapartum abx for prophylaxis (only 1 in 4,000 infants will be infected then)

A RN is preparing to insert a nasogastric tube into a client. The nurse prepares to place the client into what position?

sitting or high fowlers to prevent aspiration

With lipid emulsion, the RN would ask about allergies to?

soybean oil

a pt is HIV positive what type of precaution will he be on?

standard precaution; HIV pts do not warrent a special precaution

What is the extended postop stage?

the period of at least 1 to 4 days after surgery

How is rotavirus spread?

thought fecal-oral req contact precautions if incontinent/diapered

Instructions for client after treatment of cervical cancer with radiation (cesium) implant

use vaginal dialator to prevent vaginal narrowing and stenosis sexual activity can be resumed in about 3 weeks may need to douche twice daily for as long as discharge and odor persist

candida infection clinical manifestations

vaginal pain, itching, & a thick white vaginal discharge

post surgery voiding and bowl sounds

voiding comes back after 6 hrs absent bowl sounds for 24-48 hrs

When is the pt expected to void postop?

w/in 6 to 8 hrs, should be at least 200 mL

two point gate with crutches resembles?

walking

mouthwash for chemotherapy client with mucositis

weak salt and sodium bicarbonate mouth wash (acidic promotes bacterial growth and other solutions are harmful to oral tissue)

Gestational week: heart stage

week 3: two parallel tubes week 5: double heart chambers visible by ultrasound week 12: heartbeat heard on Doppler week 20: heard on fetoscope

two point gate is used when ?

weight bearing is allowed on both feet

Why would a pt's extremities feel cold when in traction?

when pt are in the recumbant position, it inc the heart's workload, and if heart cannot handle it the periphery will get cool need to assess for edema and peripheral pulses

How does a person with cataracts see?

with opacity that causes obj to be distorted and blurred

the weakened leg moves ____the cane or ______, in ambulating or in going down the stairs.

with; right after it;

SHould you try to wake the postop pt?

yes. make freq attempts to awaken the client until he awakens


Related study sets

Macroeconomics Problem Set and Quiz Review

View Set

Cardiovascular, Hematologic, and Lymphatic Systems

View Set

Chapter 26: Ancient Greece - The Rise of Democracy - Practice Test Questions

View Set

Diabetes Mellitus Med-Surg book chapter 48

View Set

Old Testament Survey Study Guide Exam 1, Old Testament Survey Unit 3 Quiz Answers

View Set

RAD 101, Chapter 8: Vertebral Column

View Set